Internal Medicine (PA easy)

अब Quizwiz के साथ अपने होमवर्क और परीक्षाओं को एस करें!

Q 118.17: A 75-year-old woman with a history of hypertension is seen for preoperative evaluation prior to bladder suspension. On auscultation, a crescendo-decrescendo systolic ejection murmur is heard at the upper right sternal border, radiating to the carotids bilaterally. Given the patient's physical exam findings, which of the following is the most likely diagnosis? A Aortic stenosis B Aortic regurgitation/insufficiency C Mitral stenosis D Tricuspid regurgitation/insufficiency

A Choice A is correct, as the murmur of aortic stenosis is usually described as a crescendo-decrescendo or systolic ejection murmur that is heard best at the right upper sternal border. In addition, the murmur of aortic stenosis is frequently transmitted to the carotid arteries. Choice B is incorrect, as the murmur of aortic regurgitation is usually described as a high-frequency decrescendo early diastolic murmur that is heard best at the left upper sternal border or at the right upper sternal border. Choice C is incorrect, as the murmur of mitral stenosis is described as a low-frequency rumbling diastolic murmur that is decrescendo in early diastole, but may become crescendo up to the first heart sound, with moderately severe mitral stenosis and sinus rhythm. Choice D is incorrect, as the murmur of tricuspid regurgitation is described as a holosystolic descrescendo murmur. (Crawford et al., 2009, Chapter 1)

Q 118.14: You are reviewing the labs of a 78-year-old male with a 4-month history of fatigue. You suspect primary hypothyroidism as your diagnosis. What laboratory findings would support your conclusions? A elevated thyroid-stimulating hormone (TSH) B low thyroid-stimulating hormone (TSH) C low parathyroid hormone (PTH) D high total triiodothyronine (T3) E high free triiodothyronine (FT4)

A The correct answer is (A). An elevated TSH and low FT 4 is characteristic of primary hypothyroidism. A low TSH and elevated T 3 or FT 4 would suggest hyperthyroidism. Low PTH suggests hypoparathyroidism. (McPhee and Papadakis, 2011, Chapter 26)

Q 116.19: You are evaluating a 69-year-old female who complains of an intermittent sensation of hot flashes, flushing of her face/chest, and pruritus after starting a new medication for her cholesterol. Which of the following medications is the most likely cause of her symptoms? A niacin B lovastatin C gemfibrozil D ezetimibe E fenofibrate

A The correct answer is (A). Niacin has a characteristic side effect of hot flashes, flushing, and pruritus. These symptoms can be reduced by addition of ASA or a nonsteroidal anti-inflammatory drug (NSAID) if there are no contraindications. The other choices are unlikely to cause this combination of symptoms. (McPhee and Papadakis, 2011, Chapter 28)

Q 117.14: You are evaluating an 80-year-old female for the first time. She has a history of mild Alzheimer's disease, for which she takes Aricept. She states that she feels fine but her daughter feels she is depressed and has been complaining of not feeling well. Her daughter admits that the patient has a history of primary hyperparathyroidism. What laboratory results would be most consistent with her diagnosis of hyperparathyroidism? A high serum calcium B low intact PTH C low cortisol D low urine calcium E high cortisol

A The correct answer is (A). The hallmark of primary hyperparathyroidism is a high serum calcium and high intact PTH. A low intact PTH is consistent with hypoparathyroidism. The urine serum calcium is usually high in primary hyperparathyroidism. Cortisol is related to endocrine conditions affecting the adrenal cortex. (McPhee and Papadakis, 2011, Chapter 26)

Q 119.6: A 55-year-old female presents to the emergency department with acute tetany and carpal pedal spasms. She has a positive Trousseau's phenomenon on exam. Her calcium and PTH is low. What is the most appropriate treatment at this time? A calcium gluconate IV B thiamine C magnesium oxide D vitamin D E calcium carbonate

A The correct answer is (A). The patient has symptoms of acute hypocalcemia of hypoparathyroidism, with a low calcium and PTH. Acute treatment of hypocalcemic tetany due to hypoparathyroidism is calcium gluconate IV to maintain the calcium between 8-9 mg/dl. Once the patient is stable p.o. calcium such as calcium carbonate and p.o. vitamin D is added. If magnesium is low, it should also be corrected using magnesium sulfate IV. Once corrected, the patient may need to continue supplementation with p.o. magnesium oxide. Thiamine is not indicated at this time for treatment of acute tetany, due to hypoparathyroidism. (McPhee and Papadakis, 2011, Chapter 26)

Q 97.1: A 55-year-old generally healthy male returns from a vacation to Mexico with a 5-day history of nausea, vomiting, fever of 100.5°F, and vague RUQ abdominal discomfort. Which of the following physical examination findings would be most consistent with your suspected diagnosis? A jaundice B ascites C macular diffuse rash D white tonsillar exudate E petechial rash extremities

A The correct answer is (A). The patient's history is most consistent with a diagnosis of hepatitis A. physical examination findings in hepatitis A include jaundice and hepatomegaly. Ascites, choice (B), is likely to occur with chronic hepatitis, not acute hepatitis A. Exudative tonsillitis, choice (D), is more indicative of strep A as a cause, but is not consistent with the history above. A macular or petechial rash, choices (C) and (E) would be unlikely as a direct result of hepatitis A. (Tintinalli, et al., 2010, Chapter 83)

Q 108.6: A 78-year-old Caucasian female has a 3-year history of stiffness and achiness of bilateral shoulders and hips. She has been tested for rheumatoid arthritis in the past and has been found negative. Multiple radiographs of her hips and shoulders are unremarkable. She admits that she was placed on prednisone for an allergic reaction and noted a temporary resolution of her symptoms. For the past two weeks she complains of increasing symptoms now involving her neck and pain in her jaw with chewing. Today she noticed that her scalp is sore when she brushed her hair on the right side. What is the most feared complication of this condition that may be prevented with prompt diagnosis and treatment? A blindness B costovertebral angle (CVA) C aneurysm D arm claudication E polymyalgia rheumatica (PMR)

A The correct answer is (A). This patient has long standing symptoms of PMR with current symptoms suggestive of giant cell (temporal) arteritis. Visual loss is the most feared complication of temporal arteritis, but it can be prevented by prompt initiation of high-dose prednisone. PMR often occurs with or prior to development of temporal arteritis and is not considered a complication. Large vessel involvement--which may result in choices (B), (C), and (D)--is less common than temporal artery involvement in GCA. The patient does not have symptoms of large vessel involvement. (Imboden et al., 2007, Chapter 31), (Fauci, et al, 2008, Chapter 319)

Q 120.10: A generally healthy, 20-year-old male complains of malaise, mild flank discomfort, and cola-colored urine for the past 24 hours. He denies any history of kidney stones. He does admit to having a sore throat and fever about a week ago that has since resolved. His BP is 160/94 P-92 and T is 97.2˚F. His urinalysis is positive for protein, RBCs, leukocytes, and RBC casts. His BUN (blood urea nitrogen) and CR are also elevated. A post-streptococcus glomerulonephritis B infectious mononucleosis C urinary tract infection (UTI) caused by resistant e-coli D pyelonephritis E metabolic syndrome

A The correct answer is (A). This patient's history of sore throat and acute nephritic presentation lead to the most likely diagnosis of post-streptococcus glomerulonephritis . Infectious mononucleosis can cause a sore throat, but is not typically associated with the nephritic presentation above. UTIs and pyelonephritis may present with pyuria and hematuria, but would unlikely cause hypertension and elevated BUN/CR. Pyelonephritis would likely present with fever. Metabolic syndrome is not associated with a sore throat or any of these renal symptoms. (Fauci et al., 2008, Chapter 277)

Q 99.10: A 40-year-old female presents to your office with symptoms of weight gain, hirsuitism, and easy bruising. Past medical and surgical history is noncontributory. She drinks one glass of wine on weekends and does not smoke cigarettes. She takes one multivitamin daily. Upon physical exam, you note facial fullness, central obesity, and thin skin. Which of the following is a valuable biochemical screening test for this patient that will aide in the diagnosis? A Dexamethasone suppression test B Radioactive iodine uptake C Glucose tolerance test D Cosyntropin stimulating test E Plasma fractionated free metanephrines

A The correct choice is A, dexamethasone suppression test. This patient is presenting with classic signs and symptoms of Cushing's syndrome. The dexamethasone suppression test is a simple test of the hypothalamic-pituitary-adrenal axis, and requires ingestion of oral dexamethasone at nighttime and a blood test in the morning hour, to measure the amount of plasma cortisol. Most patients with Cushing's syndrome demonstrate a lack of normal axis suppression and present with a morning plasma cortisol level >5 mcg/dL. Choice B, radioactive iodine uptake, is used in patients with suspected thyroid disorders. Choice C, glucose tolerance test, is used in patients with suspected diabetes mellitus and in prenatal testing, to investigate gestational diabetes. Choice D, cosyntropin stimulating test, is used to investigate possible adrenal insufficiency. Choice E, plasma fractionated free metanephrines, is used in the diagnostic workup of pheochromocytoma. (Aron et al., 2007, Chapter 10) (Fitzgerald et al., 2011, Chapter 26)

Q 80.3: Which of the following sets of lab values is most consistent with the diagnosis of Hashimoto's thyroiditis? A High serum TSH, low serum total T4, and high thyroidal peroxidase antibodies B Low serum TRH, low serum TSH, and low serum free T3 levels C Normal TSH, normal serum total T4, and normal radioactive iodine uptake D Low serum TSH, high serum total T4, and high thyroid stimulating antibodies E High serum TSH, high radioactive iodine uptake, and high serum free T3

A The correct choice is A, high serum TSH, low serum total T 4 , and high thyroidal peroxidase antibodies. Hashimoto's thyroiditis is the most common cause of primary hypothyroidism and is autoimmune in nature. The serum thyroid hormone levels are low, secondary to the destruction occurring in the thyroid gland. The negative feedback loop causes the pituitary to respond by increasing production and secretion of TSH. Thyroidal peroxidase, thyroglobulin, and TSH receptor blocking autoantibodies can be found in these patients. Choice B, low serum TRH, low serum TSH, and low serum free T 3 levels, is seen in patients with secondary hypothyroidism relating to pathology in the hypothalamus. Choice C, normal TSH, normal serum total T 4 , and normal radioactive iodine uptake, is seen in patients who are euthyroid. Choice D, low serum TSH, high serum total T 4 , and high thyroid stimulating antibodies, are findings in patients with primary hyperthyroidism (e.g. Graves' disease). Choice E, high serum TSH, high radioactive iodine uptake, and high serum free T 3 can be seen in patients with secondary hyperthyroidism, as a result of anterior pituitary pathology. (Bauer et al., 2010, Chapter 20)

Q 109.9: A patient was prompted to visit his health care provider after his wife started to notice that he was not interested in eating, has lost weight, and has been suffering from nausea for the last few weeks. The practitioner notes hyperpigmentation of the patient's skin, although the patient denies any recent sun or tanning salon exposure. Routine non-fasting blood work reveals the following: Sodium = 130 meq/L Potassium = 5.2 meq/L Chloride = 105 meq/L Glucose = 135 mg/dL Hemoglobin = 13.0 g/dL Hematocrit = 39.0 WBC count = 8,000/mm 3 Which of the following physical exam findings would you expect to see in this patient? A Orthostatic hypotension B Wide, purple striae C Central obesity D Full facial features E Exophthalmous

A The correct choice is A, orthostatic hypotension. The first step in the discussion of this patient is the suspected diagnosis of adrenal insufficiency. Patients with this disorder will have an excess of ACTH, which will act like melanocyte stimulating factor on the skin and cause hyperpigmentation. In adrenal insufficiency, aldosterone is deficient, thereby causing a decrease in sodium retention and potassium excretion. Hypotension is found in approximately 90% of these patients, sometimes associated with syncope as well. Choices B, C, and D are found in patients with cortisol excess. Choice E can be found in patients with Graves' disease. (Aron et al., 2007, Chapter 10)

Q 89.2: A 40-year-old female presents to your office with symptoms of weight gain, hirsuitism, and easy bruising. Past medical and surgical history is noncontributory. She drinks one glass of wine on weekends and does not smoke cigarettes. She takes one multivitamin daily. Upon physical exam, you note facial fullness, central obesity, and thin skin. Which of the following is the most common cause of her symptoms? A Pituitary adenoma B Iatrogenic C C Adrenal micronodular hyperplasia D Adrenocortical adenoma E Ectopic ACTH syndrome

A The correct choice is A, pituitary adenoma. This patient's clinical presentation is typical in Cushing's syndrome. The most common cause of Cushing's syndrome (other than ingestion of oral steroid medications) is Cushing's disease. This disease is caused by a benign, ACTH secreting pituitary adenoma. Choice B, iatrogenic, refers to the ingestion of prescribed (or non-prescribed) oral corticosteroid medications. This is frequently seen in patients requiring long-term oral steroid medications. This patient does not have this type of history. Choice C, adrenal micronodular hyperplasia, and choice D, adrenocortical adenoma, can cause Cushing's syndrome at less frequent incidence. Choice E, ectopic ACTH syndrome, presents more commonly in males with extremely elevated levels of plasma cortical and ACTH. These patients commonly have a positive history of an ectopic source of the ACTH, such as in small cell carcinoma of the lung. (Aron et al., 2007, Chapter 10) (Else et al., 2010, Chapter 21)

Q 85.2: A young man presents with difficulty breathing at times. Upon exam you note evidence of a firm, fixed thyroid nodule with extension toward the trachea and surrounding muscles. The patient has a family history of thyroid cancer. You are concerned that the patient may have medullary thyroid cancer. Which of the following lab tests would you monitor in this patient after treatment? A Serum calcitonin B Plasma alkaline phosphatase C Serum anti-thyroglobulin antibodies D Serum T3 resin uptake E Urine CA-125 levels

A The correct choice is A, serum calcitonin. Both calcitonin and CEA are secreted by medullary thyroid cancer cells, and are used both in diagnosis and monitoring of patients after treatment. Choice B, alkaline phosphatase, is elevated in disorders of the bone and biliary tract. Choice C, serum anti-thyroglobulin antibodies, are most commonly associated with autoimmune disorders of the thyroid, such as Hashimoto's thyroiditis. Choice D, serum T 3 resin uptake, is an indirect test of thyroid function. It measures the amount of unoccupied binding sites for thyroid hormone. Choice E, urine CA-125 levels, are not commonly used for monitoring any disorder. Serum CA-125 has been used to investigate and follow patients with malignancies, such as ovarian cancer. (Greenspan et al., 2007, Chapter 8)

Q 111.7: A 40-year-old female presents to the clinic with symptoms of a 10 pound unintentional weight loss, diarrhea, and palpitations. She has a positive family history for thyroid disease and diabetes mellitus. Her screening TSH level is 0.15mIU/L, and her fasting plasma glucose is 105mg/dL. Which physical exam finding would you expect to find in this patient? A Tremor B Thin hair C Puffy face D Delayed reflexes E Bradycardia

A The correct choice is A, tremor. This question first requires the identification of hyperthyroidism in the patient. The symptoms of weight loss, diarrhea, and palpitations, with a low TSH level, are characteristic of the disorder. Hyperthyroidism also presents most commonly in women in the third and fourth decade, as well as in the elderly. Common physical exam findings of hyperthyroidism include tachycardia, warm moist skin, tremor, goiter, muscle weakness, lid retraction or lag, and gynecomastia. Choices B, thin hair, C, puffy face, D, delayed reflexes and E, bradycardia, are all signs of hypothyroidism. (Jameson et al., 2008, Chapter 335) (Fitzgerald et al., 2011, Chapter 26)

Q 117.19: A 24-year-old man presenting to the clinic 1 week ago was diagnosed with depression and subsequently prescribed 10 mg/day of fluoxetine. He unexpectedly shows up today and states that he is not experiencing any improvement since starting the medication. What is the best treatment option at this time? A double the dose of fluoxetine to 20 mg/day B maintain the current dose of fluoxetine and comfort the patient that the medication may still take at least 1 to 2 more weeks to work C discontinue the fluoxetine and start sertraline D discontinue the fluoxetine and start amitriptyline E maintain the current dose of fluoxetine and add phenelzine to the medication regimen

Alleviation of symptoms associated with depression is typically slow in onset following initiation with SSRIs. Fluoxetine, for instance, can take anywhere between 2 to 6 weeks to achieve substantial benefit when used for depression. After just 1 week of therapy, there is little justification to increase the current dose or switch to another SSRI such as sertraline. Switching the patient to a TCA such as amitriptyline at this point would further delay symptom relief, as TCAs can take several weeks to produce improvement. Compared to SSRIs, TCAs are also more likely to create unwanted side effects such as weight gain, orthostatic hypotension, and constipation. Combining an SSRI with a monoamine oxidase inhibitor (MAOI) such as phenelzine can cause serotonin syndrome that can be lethal. In order to avoid interaction between SSRIs and MAOIs, it is recommended that at least 4 to 5 weeks pass after discontinuing one and starting the other. (Eisendrath and Lichtmacher, 2008, pp. 923-924) Eisendrath SJ , Lichtmacher JE. Psychiatric disorders. In: Tierney LM Jr , McPhee SJ , Papadakis MA, eds. Current Medical Diagnosis & Treatment. 47th ed. New York: McGraw-Hill; 2008.

Q 92.10: You are examining a 48-year-old male with a history of liver cirrhosis from chronic hepatitis C. Which of the following physical examination findings is a result of the effects of portal hypertension? A gynecomastia B caput medusae C testicular atrophy D jaundice E ecchymosis

B The correct answer is (B). Caput medusa, or dilated abdominal veins, is a direct result of portal hypertension. Other findings could include splenomegaly, hemorrhoids, and esophageal varices. The other choices are effects of liver cell failure. (McPhee and Papadakis, 2011, Chapter 16) Source: (McPhee and Papadakis, 2011, Chapter 16)

Q 95.9: Your patient comes to the office for a follow up of her atrial fibrillation and hypertension. She has noted that she have been more tired than usual. Laboratory findings include a thyroid-stimulating hormone (TSH) < 0.05 mU/L. What medication is the most likely cause of her laboratory findings? A cardizem B amiodarone C warfarin D dotalol E labetalol

B The correct answer is (B). Amiodarone is an antiarrhythmic medication containing iodine that is commonly used in treatment of atrial fibrillation. The use of amiodarone can cause thyrotoxicosis by several mechanisms and may also cause hypothyroidism. In this case the patients suppressed TSH would suggest the presence of amiodarone induced thyrotoxicosis. A high T 3 and FT 4 would support your diagnosis. All the other choices used in the treatment of atrial fibrillation would not cause thyroid dysfunction. (McPhee and Papadakis, 2011, Chapter 26)

Q 120.6: Your patient is taking atorvastatin 40 mg daily for her history of hyperlipidemia. Based on the potential side effects of this medication what labs are indicated for periodic monitoring? A HGB/HCT B AST/ALT C WBC count D TSH/T4 E B12

B The correct answer is (B). Atorvastatin is an HMG-CoA reductase inhibitor, which has a potential to cause liver injury. It is recommended that liver enzymes are monitored regularly due to the potential for liver injury. The remaining choices are incorrect because statins are not known to cause changes in these laboratory values and monitoring based on potential adverse reactions to atorvastatin is not recommended. (McPhee and Papadakis, 2011, Chapter 28)

Q 120.19: A 72-year-old female is a new patient in your office who is not taking any medications other than cranberry pills. She has been healthy most of her life but recently has had several episodes of kidney stones. Furthermore, in the past 6 months she has felt somewhat tired and depressed but can't figure out why. She had a bone density during a health screening and was told she has osteoporosis but is not taking anything for this at this time. She wonders if osteoporosis would cause her joint and bone pains. She has also had some abdominal discomfort and constipation recently, which does not seem to be attributed to what she eats. Her labs in the office today reveal serum calcium of 11.4 mg/dl with normal renal function. What is the most likely cause of her symptoms? A Cushing's syndrome B hyperparathyroidism C hypothyroidism D hypoparathyroidism E hypopituitarism

B The correct answer is (B). The classic symptoms of "bone, stones, abdominal groans, psychotic moans, and fatigue overtones" fits this patient's history. These symptoms, combined with an elevated serum calcium over 10.5 mg/dl, primary indicate that hyperparathyroidism is the most likely diagnosis of those listed. An elevated PTH would confirm the diagnosis. The other choices do not explain all the symptoms and the elevated calcium. (McPhee and Papadakis, 2011, Chapter 26)

Q 55.4: A 63-year-old female 3-day's postoperative thyroidectomy complains of tingling around her mouth and feet, and muscle spasms. You note the following finding on exam. What is the most likely cause of her symptoms and physical examination findings? Source: (McPhee and Papadakis, 2011, Chapter 26) A hypothyroidism B hypoparathyroidism C hyperkalemia D hyperparathyroidism E hypocalcemia

B The correct answer is (B). The patient's recent history of thyroidectomy, together with her symptoms, are suggestive of hypocalcemia due to hypoparathyroidism. Hypoparathyroidism is most commonly caused by removal of the parathyroids during thyroidectomy. The picture above is known as Chvostek's sign, which is suggestive of hypocalcemia. The other choices listed in this question would not cause Chvostek's sign. Hyperparathyroidism causes hypercalcemia.

Q 105.10: A 39-year-old male patient, who underwent a kidney transplant four months ago, reports to the clinic complaining of pain on swallowing and difficulty swallowing solids. On physical exam, you note a few white plaques present on his tongue. You order an upper endoscopy, which reveals small, yellow-white plaques with surrounding erythema present along the esophagus. What is your treatment of choice for this patient? A Amphotericin lozenges B Diflucan (fluconazole) C Septra (TMP/SMX) D Nexium (esomeprazole) E Nystatin swish and swallow

B The correct answer is Diflucan (fluconazole). This patient has candidal esophagitis, and the recommended initial therapy is with fluconazole, 100mg/d for 14 to 21 days. Amphotericin lozenges can be used for oropharyngeal candidiasis, but not esophageal. Septra is an antibiotic and will not treat candidal infections. Nexium is a proton pump inhibitor and will not treat candidal infections. *Nystatin can be used for oropharyngeal candidiasis, but not esophageal.* (McPhee SJ, Papadakis MA. Current Medical Diagnosis & Treatment, 2010, p. 535)

Q 121.19: A 50-year-old male presents to your office with symptoms of weight gain and easy bruising. He is currently being treated for hypertension. No surgical history is noted and family history is noncontributory. He does not drink alcohol. Upon questioning, he also notes some difficulty with climbing stairs. He has had to use the railing more often than in the past. A photo of the patient is shown below. Which of the following blood hormones would most likely be found to be in excess? A Renin B Cortisol C Testosterone D Antidiuretic hormone E Thyroid stimulating hormone

B The correct choice is B, Cortisol. The reader must first consider Cushing's syndrome as the diagnosis for this patient. The clinical manifestations of Cushing's syndrome are noted in this patient, and include easy bruising, proximal muscle weakness, pinkish-purple wide striae, weight gain, and central obesity. These findings are related to the excess Cortisol present in the plasma of these patients. In addition, hypertension is found in 75 to 85% of patients with Cushing's syndrome. Choices A, rennin, and C, testosterone, are noted to be low or normal in patients with Cushing's syndrome. Choice D, Antidiuretic hormone, is lacking in patients with diabetes insipidus. Choice E, thyroid stimulating hormone, is an anterior pituitary hormone that is essential in thyroid stimulation. (Else et al., 2010, Chapter 21)

Q 115.5: A 60-year-old female patient living in New York City presents for a routine office visit prior to travelling to Europe with her husband for a second honeymoon. Her medical history includes pernicious anemia, for which she is being treated with vitamin B12 supplementation. She uses no other medications. Past surgical history includes an appendectomy at age five. Upon reviewing her symptoms, you find that she has been trying to self treat for chronic constipation, without positive results. She also notes that she is easily tired and has gained 10 pounds in the last few months. On exam you note bradycardia and cool, dry skin. What is the most likely cause of her recent symptoms? A Major depression B Hashimoto's thyroiditis C Hypothalamic destruction D Toxic thyroid nodule E Iodine deficiency

B The correct choice is B, Hashimoto's thyroiditis. This autoimmune disorder is by far the most common cause of hypothyroidism in adults. This patient is presenting with classic signs and symptoms of hypothyroidism including bradycardia, cool, dry skin, and modest weight gain. She also has a history of another autoimmune disorder, pernicious anemia, which is commonly associated with hypothyroidism. Choice A, major depression, seems unlikely since the patient is planning a second honeymoon. Although many of her symptoms can be attributed to depression, her cool, dry skin is not characteristic of the disorder. Choice C, hypothalamic destruction, can cause hypothyroidism, but it is rare. Patients with choice D, toxic thyroid nodule, would typically present with symptoms of hyperthyroidism. Choice E, iodine deficiency, is a cause of hypothyroidism, but it is unlikely in patients living in developed countries with access to a variety of food sources. (Cooper et al., 2007, Chapter 8)

Q 110.10: A patient is admitted to the hospital for an acute adrenal crisis. He has a history of chronic adrenal insufficiency and was admitted with severe weakness, nausea and vomiting while fighting a pulmonary infection. Which of the following suggested plans might this patient have forgotten or been unable to do? A Restrict fluid intake during times of metabolic stress B Increase the daily dose of hydrocortisone during times of metabolic stress C Hold the daily dose of hydrocortisone during times of metabolic stress D Add levothyroxine to the daily dose of hydrocortisone during times of metabolic stress E Increase the daily ingestion of proteins during times of metabolic stress

B The correct choice is B, increase the daily dose of hydrocortisone during times of metabolic stress. The cortisol dose should be increased to between 60 and 80 mg/day, to mimic the normal physiologic response of the body. Increased mineralocorticoid therapy is generally not required. Choice A, restrict fluid, would aggravate the potential for the development of dehydration in this patient. Choice C would lead to further cortisol depletion. Choices D and E would not benefit this patient. (Aron et al., 2007, Chapter 10)

Q 110.2: A 25-year-old woman is seen today in your office for vague abdominal pain, nausea, anorexia, weight loss, anxiety, and dizziness. Her past medical history is significant for type 1 diabetes mellitus, and her family history is significant for hypothyroidism in several family members. A review of systems reveal a history of amenorrhea. Upon exam, you note hyperpigmentation of her skin and areas of vitiligo, but no mucocutaneous candidiasis. You are not surprised to find that her serum ACTH level is elevated and her serum cotisol is low. Which of the following syndromes should be investigated in this patient? A Type 1 polyglandular autoimmune syndrome type 1 B Type 2 polyglandular autoimmune syndrome type 2 C Multiple endocrine neoplasia type 2A D Multiple endocrine neoplasia type 1 E Metabolic syndrome

B The correct choice is B, type 2 polyglandular autoimmune syndrome. This patient is presenting with signs and symptoms of adrenal insufficiency including abdominal pain, nausea, anorexia, vomiting, weight loss, anxiety, and hyperpigmentation. Type 2 polyglandular autoimmune syndrome presents most commonly in young women between 20 and 40 years old, with evidence of adrenal insufficiency, type 1 diabetes mellitus, and autoimmune thyroid disease. Because of the strong family history of thyroid disease, it would be prudent to conduct an investigation into this disorder, starting with a serum TSH level. Primary ovarian failure and vitiligo may be symptoms of the autoimmune polyglandular syndrome as well. Choice A, type 1 autoimmune polyglandular syndrome, presents more commonly in childhood with adrenal insufficiency, hypoparathyroidism, and mucocutaneous candidiasis. Choices C and D are inherited disorders, characterized by the development of several different types of endocrine organ neoplasias. Choice E, metabolic syndrome, includes a constellation of several metabolic disorders that increase the risk of cardiovascular disease and diabetes mellitus in the patient. (Fitzgerald et al., 2011, Chapter 26)

Q 105.3: Your supervising physician asks you to advise him which finding is least likely to be suggestive of a thyroid malignancy in your 49-year-old female with a small palpable thyroid nodule. Which of the following choices would be least likely to suggest malignancy in this patient? A ultrasound showing lesion with microcalcifications B ultrasound showing a lesion of > 1 cm C hot nodule on 123I uptake scan D ultrasound showing a solid lesion E cold nodule on 123I uptake scan

C The correct answer is (C). A hot nodule, which is a hyperfunctioning thyroid nodule, suggests a benign etiology. The other choices, including ultrasound findings of microcalcifications, solid lesions, and lesions > 1 cm, should increase your index of suspicion for possible malignancy. Cold nodules are nonfunctioning thyroid nodules, which should increase your suspicion, especially in combination with suspicious ultrasound and/or clinical examination findings. (Fauci et al., 2008, Chapter 335), (McPhee and Papadakis, 2011, Chapter 26)

Q 99.5: Your patient states she has been gaining weight for no apparent reason over the past year and is concerned that she might have hypothyroidism. What other historical information would support a diagnosis of hypothyroidism? A anxiety B diarrhea C depression D palpitations E heat intolerance

C The correct answer is (C). Depression is a common presenting symptom of hypothyroidism. Weight gain can occur with hypothyroidism. Other symptoms may be weakness, fatigue, and menorrhagia. Hoarseness may also be a presenting symptom. The other choices are common symptoms of hyperthyroidism. (McPhee and Papadakis, 2011, Chapter 26)

Q 117.20: Your patient is a 77-year-old male with a history of hypertension. For about the past 50 years has smoked a pipe daily. He feels great but admits that his cholesterol was elevated for the past 5 years, and has declined treatment. His best friend died of an myocardial infarction last week and the patient now agrees to treatment for his hyperlipidemia. His LDL is 285 mg/dl and HDL is 30 mg/dl. You decide to put him on simvastatin 80 mg QD. Prior to initiation, you advise the patient to notify you about which of the following potential side effects of this medication? A cough B double vision C myalgias D elevated blood pressure E restlessness

C The correct answer is (C). Myalgias are common side effects of statins, which may result in a patient discontinuing the medication. If the patient develops significant myalgias a CPK may be ordered to rule out myositis and if elevated the statin may need to be discontinued. Elevated liver enzymes may occur, which may result in discontinuation of the medication. The other options are unlikely side effects of statins. (McPhee and Papadakis, 2011, Chapter 28)

Q 115.13: The patient below comes to your office for a routine physical exam. She has no complaints. You note the following physical examination findings. There is an absence of tenderness with palpation. What is the most likely diagnosis? Source: (Wolff and Johnson, 2009, Section 15) A gout B cellulitis C hyperlipidemia D rheumatoid arthritis E lacrimal duct obstruction

C The correct answer is (C). The following finding represents xanthelasma, which is commonly associated with hyperlipidemia. In some familial hyperlipidemias, skin eruptions or nodules may form (xanthomas). Gout and rheumatoid arthritis are types of inflammatory arthritis and manifestations are more likely to be found around joints. Gouty tophi may also be found around the helix of the ear and are usually whitish in color in contrast to the yellow xanthelasma. There is no evidence of erythema consistent with cellulitis and the patient is asymptomatic. Lacrimal duct obstruction would likely be symptomatic and there would be tender swelling over the lacrimal duct.

Q 121.5: A 41-year-old female comes in for her annual exam. She generally feels well and has no complaints other than some general fatigue. On examination you note a normal size thyroid with a palpable firm 1.5 cm nodule in the left lobe of her thyroid. What is the best test to order that will provide a definitive diagnosis? A thyroid ultrasound B thyroid uptake scan C fine needle biopsy of nodule D MRI thyroid E TSH

C The correct answer is (C). The patient has a solitary, > 1cm, firm nodule, which is suspicious for a thyroid malignancy. A fine needle biopsy of the nodule would provide you with a definitive diagnosis. A thyroid ultrasound, thyroid uptake scan, and TSH would be helpful in determining the etiology of the nodule but will not provide you with a definitive diagnosis. Since this patient has a suspicious clinical examination a biopsy is indicated. MRI of the thyroid is not a preferred test. (McPhee and Papadakis, 2011, Chapter 26)

Q 80.13: 42-year-old female complains of weight gain (especially in her abdomen) over the past 8 months. She also has noted that her skin bruises easily. Her husband has noted she seems to be very moody lately and she is worried about their relationship. Furthermore, her hair seems to be getting thinner and she is now getting acne like she had in her teenage years. She wonders if this is due to menopause since her periods have stopped suddenly about a year ago. On physical examination her BP = 170/50, P = 82, T = 98.2˚F. You note the following findings on examination (see picture). What diagnostic test is indicated initially to confirm your suspected diagnosis? A cosyntropin stimulation test B MRI pituitary C dexamethasone suppression test D 24-hour urine for protein E serum protein electrophoresis

C The correct answer is (C). The patient's symptoms are consistent with a diagnosis of Cushing's syndrome (or disease). Her physical examination findings of hypertension and abdominal obesity with the classic purple striae also support the diagnosis. The initial diagnostic test of choice would be the dexamethasone suppression test. If the test is positive, further confirmatory testing is done which would also help to identify the cause. An MRI of the pituitary is appropriate if further testing suggests the possibility of a pituitary adenoma as the cause of the Cushing's syndrome, but is not used as an initial diagnostic test for Cushing's disease. A cosyntropin stimulation test, choice (A), is indicated for the diagnosis of Addison's disease. (McPhee and Papadakis, 2011, Chapter 26) Source: (McPhee and Papadakis, 2011, Chapter 26)

Q 63.3: You are evaluating a 67-year-old male with known cirrhosis of the liver secondary to alcoholic liver disease, although he has been sober for the past year. He is brought in to the emergency department by his daughter, who notes that for the past few days he has seemed to be more confused. On examination you note the patient to be mildly confused but alert to person and place. He has noticeable asterixis. He is not currently taking any medications and his blood alcohol level is undetectable. What is the treatment of choice in this case based on your physical examination findings? A amoxicillin B prednisone C lactulose D folic acid E thiamine

C The correct answer is (C). This patient most likely has hepatic encephalopathy due to end-stage liver disease. Asterixis indicates an increase in serum ammonia. The treatment of choice is lactulose. Both folic acid and thiamine are used in the treatment of alcoholic liver disease, but do not treat elevated ammonia levels. Antibiotics may be used secondarily in patients nonresponsive to lactulose, but amoxicillin is not preferred. Prednisone is not a treatment for hepatic encephalopathy.

Q 119.3: A 42-year-old male presents with a three year history of progressive dysphagia, weight loss, and nocturnal aspiration. An upper GI study (barium swallow) shows a moderately dilated esophagus with a smooth, tapered, distal "bird's beak" deformity. What is the most likely diagnosis? A Diffuse esophageal spasm B Barrett's esophagus C Achalasia D Gastroesophageal reflux disease E Esophagitis

C The correct answer is achalasia. Diffuse esophageal spasm typically presents with dysphagia and chest pain, and has a corkscrew or nutcracker appearance on barium swallow. Barrett's esophagus does not provoke particular symptoms and does not have a characteristic appearance on a barium swallow. Gastroesophageal reflux typically presents with heartburn and regurgitation, and a barium swallow is not used to diagnose this condition. Esophagitis presents with odynophagia, dysphagia, and chest pain, and a barium swallow is not used to diagnose this condition. (McPhee SJ, Papadakis MA. Current Medical Diagnosis & Treatment, 2010, p. 541)

Q 121.17: Release of which of the following substances is triggered by pituitary growth hormone and promotes growth of other tissues in the body? A C-pepide B IL-I C IGF-I D Thyroxine E Catecholamines

C The correct choice is C, IGF-I or insulin like growth factor I. This growth factor leads to increased DNA, RNA, and protein synthesis, which leads to overgrowth of bone, soft tissue, and cartilage. Choice A, c-peptide, is a part of the prohormone of insulin. Choice B, IL-I or interleukin I, is an important cytokine that promotes cell activation. Choice D, thyroxine, potentiates the actions of growth hormone on tissues. (Fitzgerald et al., 2011, Chapter 26) (Aron et al., 2007, Chapter 5)

Q 105.4: A new patient is seen in your internal medicine office today. She is coming in to request the removal of several skin tags. She is a 55-year-old woman with a history of untreated acromegaly. A health maintenance plan is set up with the patient, and includes a colonoscopy. This patient is at increased risk for which of the following findings on colonoscopy? A Anal fissures B Ulcerative colitis C Colon polyps D Pseudomembranous colitis E Colonic fistulas

C The correct choice is C, colon polyps. Approximately 30% of patients with acromegaly have been found to have colon polyps. These patients also have an increased risk of colon cancer. Patients with acromegaly have not been found to be at increased risk for the other response choices listed here. (Melmed et al., 2008, Chapter 333)

Q 82.7: Which of the following cardiac medications is known to cause clinically significant hypothyroidism? A Furosemide B Captopril C Amiodarone D Digoxin E Dopamine

C The correct choice is C; Amiodarone is an antiarrythmic medication used to treat patients with recurrent ventricular tachycardia or fibrillation. Its structure is similar to thyroid hormone, and its metabolites antagonize thyroid hormone function in approximately 13% of patients treated with amiodrone in the United States. Choice A, furosemide, choice B, captopril, choice D, digoxin, and choice E, dopamine, have no effect on thyroid function. (Fitzgerald et al., 2011, Chapter 26) (Katzung et al., 2009, Chapter 13) (Roden et al., 2006, Chapter 34; 2008, Chapter 335)

Q 121.7: An 88-year-old female patient has been advised by her primary care physican that she needs a computed tomography (CT) scan of her abdomen and pelvis due to persistent abdominal pain, bloating, and weight loss. She was told that she needs to hold one of her medications the day of the procedure and that she may resume the medication 48 hours later. She can't remember which medication she should discontinue. Which medication listed below should she discontinue temporarily as specified above due to the diagnostic test ordered? A glyburide B glipizide C pioglytizone D metformin E acarbose

D The correct answer is (D). A CT of the abdomen and pelvis requires p.o. and IV iodinated contrast unless ordered specifically without IV contrast. There is an increased risk of acute renal failure with IV iodinated contrast. The risk to the patient may increase with metformin and therefore should ideally be held prior to and for 48 hours after any radiocontrast IV study to avoid the added possibility of lactic acidosis. The other treatments for non-insulin-dependent diabetes mellitus (NIDDM) do not require discontinuation with IV contrast studies for these time periods. However, the am dose of a sulfonylurea (glyburide or glipizide) may be held until after the study that day when the patient resumes eating to avoid hypoglycemia in some patients. (McPhee and Papadakis, 2011, Chapter 27)

Q 121.2: A 44-year-old female complains of nonproductive cough for the past 6 months. She denies rhinorrhea, wheezing, dyspnea, chest pain, or hemoptysis. Her medical problems include hypertension. Medications include benazepril 10 QD, Amlodipine 5 mg QD, and HCTZ 25 mg QD. She is a nonsmoker and denies any foreign travel. The following CXR PA/LAT is taken. What is the most likely cause of her cough? Source: (Hanley and Welsh, 2003, Chapter 3) A amlodipine B pneumonia C bronchiectasis D benazepril E asthmatic bronchitis

D The correct answer is (D). ACE inhibitors such as benazepril have a potential adverse reaction of a chronic cough. Discontinuing the ACE inhibitor is appropriate in this case while substituting this for another antihypertensive. The CXR is normal and there are no findings suggestive of pneumonia, bronchiectasis, or asthmatic bronchitis. A chronic cough is not a significant side effect of amlodipine. (McPhee and Papadakis, 2011, Chapter 11)

Q 91.5: A 32-year-old female returns to the office following an emergency department visit for abdominal pain that has since resolved. A computed tomography (CT) scan performed in the emergency department reveals a 1 cm cavernous hemangioma in the right lobe of the liver. What are your recommendations to the patient regarding the CT findings? A immediate referral to oncologist B referral to GI for liver biopsy C referral to surgeon for resection D reassurance that no treatment is necessary E repeat CT scan in 3 months

D The correct answer is (D). Cavernous hemangiomas are common benign lesions of the liver that rarely require surgical intervention unless they are very large (>10 cm) and symptomatic. No further imaging follow-up is necessary. (Greenberger et al, 2009, Chapter 49)

Q 116.8: Your patient is a 44-year-old female who was advised by her endocrinologist that she has thyroid cancer and seeks your opinion. You inquire about the type of thyroid cancer but the patient is unsure. She does state that she was advised that it is the most common and she has a good prognosis. What is the most likely cause of her thyroid malignancy? A lymphoma B follicular thyroid carcinoma C anaplastic thyroid carcinoma D papillary thyroid carcinoma E medullary thyroid carcinoma

D The correct answer is (D). Papillary thyroid carcinoma is the most common and least aggressive of the thyroid carcinomas. The usual age of presentation is in the early 40s. This is followed in frequency by follicular thyroid carcinoma. Anaplastic thyroid cancer is very aggressive and has a high mortality rate. Lymphoma affecting the thyroid can occur, but is not common, and usually presents in the elderly. (McPhee and Papadakis, 2011, Chapter 26)

Q 105.16: A 42-year-old male comes to the office to discuss his total cholesterol of 215 mg/dl, which was obtained last week at a health fair. He is generally healthy, but smokes one pack of cigarettes per day for the past 10 years. He is unsure of his family history. What is your next step? A prescribe simvastatin B advise dietary changes C repeat the total cholesterol today, fasting D obtain a fasting lipid profile E prescribe gemfibrozil

D The correct answer is (D). The patient comes in only with total cholesterol. With this information a fasting lipid profile is indicated to evaluate the patient's cardiovascular risk and should be the next step. Repeating the total cholesterol is not very helpful. It is premature to recommend a specific treatment at this point until you fully assess the patient's risk. (McPhee and Papadakis, 2011, Chapter 28)

Q 67.3: A 38-year-old male with a history of frequent urinary tract infections (UTIs) and kidney stones is following up with the urology physician assistant. His urine today continues to reveal persistent hematuria, which he had the last few times he saw his primary care physician. His vitals are 120/80, P = 72, T = 96.7˚F. He denies any discomfort at this time. GFR is normal. You review a urogram, which reveals a paint-brush appearance of the papillae. The kidneys are normal in size. What is the most likely diagnosis? A chronic renal insufficiency B renal cell carcinoma C end stage renal disease D medullary sponge disease E polycystic kidney disease

D The correct answer is (D). The patient's history of hematuria, recurrent UTIs, and kidney stones are a common presentation of this inherited disease with medullary cysts. The findings on the urogram of a paint-brush appearance of the papillae are pathognomonic of the disease. Choices (A) and (C) are unlikely due to the normal GFR. The patient's history is not suggestive of choice (B), renal cell carcinoma. A solitary solid renal mass would suggest renal cell carcinoma. (Lerma, et al., 2008, Chapter 46)

Q 73.5: One of your patients is requesting your help. He has seen three physicians in the past 3 months and is still having symptoms. He states, "I keep having these episodes of feeling like I am going to die. Out of the blue I feel real nervous, I get a splitting headache, break out in a sweat, and even feel like I am trembling. Sometimes I feel my heart beating out of my chest but don't have any pain or shortness of breath." He has had numerous tests including a cardiac stress test, multiple EKGs, complete blood count (CBC), basic metabolic panel (BMP), thyroid tests, and computed tomography (CT) scans of his head, which were normal. His last physician told him that he had an anxiety disorder and should try medications; he declined. His only medical problem is hypertension, which has worsened recently. He is currently taking lisinopril 20 mg QD, amlodipine 10 mg QD, and HCTZ 25mg QD. His physical examination is unremarkable except a BP of 190/92, P = 74. What is the best test to order to confirm your suspected diagnosis? A dexamethasone suppression test B sleep study C renal artery ultrasound D plasma fractionated free metanephrines E thyroid uptake scan

D The correct answer is (D). The patient's symptoms and uncontrolled hypertension with a previous negative evaluation for cardiac, electrolyte, or thyroid causes highly suggests a pheochromocytoma as the possible cause. Plasma fractionated free metanephrines is a very sensitive test for the diagnosis of pheochromocytoma. Another test to consider would be evaluation of the total urinary metanephrines. A dexamethasone suppression test, choice (A), is used to r/o Cushing's syndrome, which is unlikely due to the normal physical examination findings. A sleep study, choice (B), (suggesting sleep apnea) and renal artery ultrasound, choice (C), (suggesting renal artery stenosis) are used to diagnose secondary hypertension, but would unlikely explain all the symptoms in the scenario. A thyroid uptake scan, choice (E), is not indicated since there is no indication of abnormal thyroid testing suggesting hyperthyroidism, or examination stated findings suggestive of a thyroid nodule. (McPhee and Papadakis, 2011, Chapter 26)

Q 79.3: A 32-year-old male presents with odynophagia, dysphagia, and chest pain. His past medical history consists of him being HIV positive. He is currently not taking any medications, as he cannot afford to pay for them. An endoscopy is ordered and the results show several large, deep ulcerations. Initial treatment consists of which of the following medications? A Nystatin suspension B Acyclovir C Famciclovir D Ganciclovir E Fluconazole

D The correct answer is ganciclovir. This patient has cytomegalovirus esophagitis, and the initial treatment is ganciclovir 5 mg/kg IV every 12 hours for 3 to 6 weeks. Nystatin suspension is used to treat oropharyngeal candidiasis. Acyclovir and famciclovir are used to treat herpetic esophagitis. Fluconazole is used to treat candidal esophagitis.

Q 76.10: A 30-year-old female presents to your office for a routine physical exam. She has not seen a health care provider in many years. Upon talking with the patient, you find out that she had been diagnosed with hypertension several years ago, but was unable to afford the antihypertensive medications that were prescribed to her. She has no complaints at this time. Upon exam of the head and neck, you note widened spaces between her lower incisor teeth and a large, fleshy nose. Her skin is oily and she demonstrates mild proximal muscle weakness. Her EKG reveals a left axis deviation and widened QRS. What is the most likely rationale for her clinical presentation? A Diabetes mellitus B Cushing's syndrome C Hypothyroidism D Acromegaly E Clinical depression

D The correct choice is D, acromegaly. Patients with acromegaly have an abundance of growth hormone secretion. This leads to excessive growth of many areas of the body including soft tissue. Patients with acromegaly also have an increased incidence of hypertension and left ventricular hypertrophy. None of the other choices will cause this patient's constellation of symptoms. Patients with many endocrine disorders may develop weaknesses as seen in this patient, but the large nose and widely spaced teeth are characteristic of acromegaly. (Melmed et al., 2008, Chapter 333)

Q 115.7: A 72-year-old male presents to you for a routine physical exam. He complains of increasing abdominal distention, weight loss, and night sweats over the past 2 months. He admits to having a blood transfusion as a young adult. His labs include +anti-HCV and HCV RNA. Which of the following is the most likely explanation for his history and laboratory findings? A acute hepatitis B B alcoholic hepatitis C chronic hepatitis A D Gilbert's syndrome E hepatocellular carcinoma

E The correct answer is (E). The patient's symptoms are consistent with a possible malignancy. Labs indicate chronic hepatitis C, likely longstanding with liver cirrhosis, which is associated with an increased risk of hepatocellular carcinoma. Chronic hepatitis B is also associated with an increased risk of hepatocellular carcinoma, but not acute hepatitis B, choice (A). Choices (B), (C), and (D) are not consistent with the laboratory results and do not increase the patient's risk of hepatocellular carcinoma. (Fauci et al., 2008, Chapter 298), (Greenberger et al, 2009, Chapter 49)

Q 104.9: This is a protrusion of pharyngeal mucosa that develops at the phayngoesophageal junction, between the inferior pharyngeal constrictor and the cricopharyngeus. It often presents in older patients, with symptoms of dysphagia and regurgitation that develop slowly over many years. Late in the disease process patients may note halitosis, spontaneous regurgitation of undigested food, or nocturnal choking. Which of the following choices is the correct term for this condition? A Meckel diverticulum B Hiatal hernia C Esophageal web D Achalasia E Zenker diverticulum

E The correct answer is Zenker diverticulum. Meckel's diverticulum is a pouch on the wall of the lower part of the small bowel, which is congenital. A hiatal hernia is a protrusion of a portion of the stomach into the chest through a hole in the diaphragm. An esophageal web is a thin, diaphragm-like membrane of squamous mucosa. Achalasia is a motility disorder, characterized by loss of peristalsis in the distal two-thirds of the esophagus and impaired relaxation of the LES. (McPhee SJ, Papadakis MA. Current Medical Diagnosis & Treatment, 2010, p. 538)

Q 95.4: A 47-year-old man presents with signs and symptoms of Cushing's syndrome. He has no history of depression, alcoholism, anorexia, or medication use. His past medical history is negative for any known malignancies. An MRI of the pituitary gland is not definitive for any masses. You are trying to decide if the symptoms are caused by an occult pituitary adenoma or occult malignancy in another area of the body. Which of the following is the definitive test used to differentiate pituitary from non-pituitary ACTH dependent Cushing's syndrome? A 12:00 a.m. plasma cortisol B Random plasma ACTH C 24-hour urine free cortisol D MRI of the adrenal glands E Inferior petrosal sinus ACTH

E The correct choice is E, inferior petrosal sinus ACTH. This test can definitively indicate whether or not an elevated plasma ACTH level is due to secretion from the pituitary or an ectopic source. Simultaneous measurements of ACTH from the peripheral circulation and the inferior petrosal sinus after CRH stimulation can help to locate the surge's origin. Highly accurate results can be obtained if this test is performed by a skilled interventional radiologist. Although the other choices listed are used in the evaluation of patients with hypercortisolemia, they do not detect any related ACTH surge source. (Aron et al., 2007, Chapter 10)

Q 88.6: What is the definitive treatment of choice for elderly patients diagnosed with Graves' disease? A Beta blocking agents B Levothyroxine C Methimazole D Total thyroidectomy E Radioactive iodine

E The correct choice is E, radioactive iodine. This is the treatment of choice in the elderly because it is efficient, easy to take, and inexpensive. Choice A, beta blocking agents, are useful in the treatment of symptoms of hyperthyroidism, such as palpitations, but they are not a definitive treatment for the disorder. Choice B, levothyroxine, is used for thyroid hormone supplementation in patients with hypothyroidism. Choice C, methimazole, is an anti-thyroid drug that has increased toxicity in the elderly and is more useful in younger patients with mild hyperthyroidism. Choice D, total thyroidectomy, has a limited role as a treatment for hyperthyroidism, and is associated with increased morbidity in the elderly. (Greenspan et al., 2007, Chapter 24)

Q 77.10: What range of time does it take for a PPD test to become positive as an immune response? A 2 to 10 weeks B 12 to 20 weeks C 21 to 40 weeks D >40 weeks E Immediately after exposure

The Correct Answer is: A The typical timeframe is shortly after the immune system can register the new strain into the system. False positives can occur in individuals who have had prior infections. (Chesnutt MS, Prendergast TJ. Current Medical Diagnosis and Treatment, 2011, Chapter 9, Pulmonary Disorders)

Q 109.17: A chest x-ray on an 81-year-old male with a four-day history of productive cough, dyspnea, fever, chills, and shortness of breath reveals a left sided pleural effusion. Pleural fluid analysis reveals a decreased glucose level, elevated lactate dehydrogenase, and 20,000 polymorphonuclear white blood cells/mcL. What is the most likely cause of this effusion? A Bacterial pneumonia B Congestive heart failure C Malignancy D Pulmonary embolus E Tuberculosis

The Correct Answer is: A A pleural effusion is a collection of fluid within the pleural space, due to an increased rate of fluid formation with decreased absorption. Pleural effusions are classified as transudative versus exudative, based on the underlying cause. This patient exhibits pleural fluid analysis results that are consistent with a parapneumonic effusion. Malignancy and tuberculosis also cause exudative effusions, with different fluid analysis results and various patient presentations (see Table 9-23). Congestive heart failure and pulmonary embolism are associated with transudative effusions. (McPhee SJ, Papadakis MA. Current Medical Diagnosis & Treatment 2011, Chapter 9, Pulmonary Disorders)

Q 59.9: A 33-year-old man with HIV infection is brought in by his partner for evaluation of altered mental status. The partner has noticed waxing and waning periods of confusion throughout the day, difficulty in performing tasks such as balancing a checkbook, and deterioration of handwriting. The patient reports no fever or headache. What is the most likely diagnosis? A AIDS dementia complex B central nervous system lymphoma C cryptococcal meningitis D progressive multifocal leukoencephalopathy (PML) E toxoplasmosis

The Correct Answer is: A AIDS dementia complex is the most common cause of mental status changes in patients with HIV infection. The deterioration of handwriting is often an early manifestation. Difficulty in performing cognitive tasks and diminished motor speed are typical, as is the waxing and waning of manifestations of dementia. Patients with central nervous system lymphoma and toxoplasmosis present with headache, focal neurologic deficits, seizures, and/or altered mental status. Patients with cryptococcal meningitis have, most typically, headache and fever, but fewer than 20% have meningismus. They also usually have normal mental status. Patients with PML have primarily focal neurologic deficits such as aphasia, hemiparesis, and cortical blindness.

Q 81.8: A 28-year-old male has Hemophilia A. He is in for genetic counseling on the risk of his children developing the disease. What is the likelihood that a son of his will develop Hemophilia A? A 0% B 25% C 50% D 75% E 100%

The Correct Answer is: A All daughters of a hemophilic male are carriers of hemophilia, whereas all sons are normal. Hemophilia A is one of only two sex-linked pattern-bleeding disorders, and as such the disease occurs almost exclusively in males. Sons of carriers have a 50% chance of being affected and daughters of carriers have a 50% chance of being carriers themselves. (Lichtman et al., Williams Hematology 8e, Chapter 124, Hemophilia A and Hemophilia B)

Q 97.7: A 72-year-old male is found to have an absolute lymphocyte count of 13,500 (1000 to 3500). Marrow examination demonstrates infiltration with leukemic lymphocytes, with a lacy or interstitial pattern. What is the most common physical finding that would be associated with this patient? A Cervical and supraclavicular lymphadenopathy B Proptosis and headache C Rhinitis and polyneuropathy D Splenomegaly and hepatomegaly E Weight loss and night sweats

The Correct Answer is: A All of these findings can be present in this patient with chronic lymphocytic leukemia, but the most common physical finding in these patients are lymphadenopathy with over 80% presenting with nontender adenopathy. All others are findings that may occur, but are significantly less common. (Lichtman et al., Williams Hematology 8e, Chapter 94, Chronic Lymphocytic Leukemia and Related Diseases)

Q 89.7: Which of the following exerts its action by inhibiting cell wall synthesis? A amoxicillin B ciprofloxacin C doxycycline D erythromycin E gentamicin

The Correct Answer is: A All β-lactam antibiotics, including the penicillins (eg, amoxicillin) and cephalosporins, prevent bacterial growth by inhibiting cell wall synthesis. Fluoroquinolones (eg, ciprofloxacin) block bacterial DNA synthesis. Erythromycin, doxycycline, and gentamicin all inhibit protein synthesis but via different mechanisms. (Craig and Stitzel, 2004, pp. 519, 526, 538, 544) Craig CR , Stitzel RE. Modern Pharmacology with Clinical Applications. 6th ed. Baltimore, MD: Lippincott Williams & Wilkins; 2004.

Q 116.9: A 21-year-old college student presents with headache, fever, and a stiff neck. You perform a lumbar puncture for suspected meningitis. Which of the following cerebrospinal fluid analysis results is consistent with bacterial meninigitis? A Elevated opening pressure, markedly elevated white blood cell count, decreased glucose, increased protein concentration B Elevated opening pressure, mildly elevated white blood cell count, markedly elevated red blood cell count with noted xanthochromia C Mildly elevated opening pressure, white blood cell count elevated but less than 1000/mL, normal glucose, protein slightly increased D Mildly elevated opening pressure, elevated white blood cell count, low glucose, high protein, positive spirochetes E Normal opening pressure, normal white blood cell count, elevated red blood cell count

The Correct Answer is: A Bacterial meningitis is associated with specific cerebrospinal fluid analysis results, although some overlap with other diagnoses does exist. The CSF for bacterial meningitis should reveal a markedly elevated white blood cell count, including a high percentage of polymorphonuclear leukocytes. Additional findings should include a decreased glucose concentration and elevated protein values. Gram stain analysis will reveal bacteria morphology and suggest an infectious etiology, although at times additional specialized testing may be needed, such as a fluorescent treponemal antibody absorption (FTA-ABS) test, in cases of suspected neurosyphilis. Choice B suggests subarachnoid hemorrhage, choice C suggests viral meningitis, choice D suggests syphilis, and choice E suggests a traumatic tap. (Ropper AH, Samuels MA. Adams and Victor's Principles of Neurology, 9e, Chapter 2, Special Techniques for Neurologic Diagnosis)

Q 65.2: A 53 year-old female is diagnosed with bladder cancer after an extensive work-up. Which pathology would be expected on her results, considering the most common cause of bladder cancer? A Urothelial (transitional) cell carcinoma B Squamous cell carcinoma C Adenocarcinoma D Clear cell carcinoma E Basal cell carcinoma

The Correct Answer is: A Baldder carcinoma pathology is transitional (urothelial) cell carcinoma (~90% of cases) (A); squamous cell carcinoma (B), (~7% of cases); adenocarcinoma (C), (~2% of cases). Clear cell carcinoma (D) and basal cell carcinoma (E) are not common findings in bladder cancer.

Q 73.1: Which of the following is a potential complication of acute pyelonephritis? A perinephric abscess B renal vein thrombosis C allergic interstitial nephritis D struvite stones E hepatic failure

The Correct Answer is: A Because pyelonephritis is an infectious disease, the most likely complication is a perinephric abscess, which occurs as the result of inadequate therapy. Since it is not vascular in origin, renal vein thrombosis would not occur. Allergic interstitial nephritis is caused by an antigen-antibody reaction, which does not occur with acute pyelonephritis. Struvite stones are due to chronic infection with urease-producing organisms, such as Proteus and Pseudomonas, not to an acute infection. Hepatic failure can be a complication of acute renal failure, but not acute pyelonephritis. (Stoller et al., 2009, p. 830) Stoller ML , Kane CJ , Meng MV. Urology. In: Tierney LM , McPhee SJ , Papadakis MA, eds. Current Medical Diagnosis and Treatment. 48th ed. New York, NY: McGraw-Hill; 2009.

Q 54.19: A 22-year-old recent immigrant from Vietnam, who is 28 weeks pregnant with her first child, presents to the emergency department with complaints of worsening dyspnea and lower extremity edema. She is unable to answer definitively whether or not she has a history of rheumatic fever. On physical examination, a possible opening snap, loud S 1 , and a very soft diastolic rumbling murmur is auscultated. When the patient is placed in the left lateral decubitus position, the murmur is accentuated, and heard best at the apex. With inspiration, the murmur does not increase in amplitude. On echocardiogram, mitral stenosis is noted. Which of the following is the most appropriate next step in management of this patient? A Beta blockade and support stockings B ACE inhibitor C Mitral valve replacement D Cardiac catheterization E Increased sodium intake to maintain fluid volume

The Correct Answer is: A Choice A is the most appropriate next step in the management of this pregnant patient with mitral stenosis. In pregnancy, blood volume, cardiac output, and heart rate are increased. In pregnant patients with mitral stenosis, this increases the pressure across the mitral valve and can lead to pulmonary edema. The use of appropriate beta blockade is helpful for decreasing the heart rate, and may be used in conjunction with digoxin if the patient develops atrial fibrillation, a common dysrhythmia in this patient population. Support stockings are helpful in preventing venous pooling in the lower extremities, which can lead to large fluctuations in hemodynamics. Choice B, ACE inhibitors, are contraindicated in pregnancy, secondary to the teratogenic effects. Choice C, mitral valve replacement, is indicated only if medical management is insufficient to prevent congestive heart failure, a serious consideration in this valvulopathy, which is the most likely to cause death in pregnancy. Choice D, cardiac catheterization, is used for evaluation of coronary artery disease, but is not indicated in this young patient with few risk factors for coronary artery stenosis. Choice E, increased sodium intake to maintain fluid volume, would worsen the pressure across the mitral valve, and thus is incorrect, as avoidance of fluid overload is one of the keys to management of a pregnant patient with mitral stenosis, especially in the latter stages of pregnancy. Patients who are well-controlled throughout pregnancy, with medical management, are still at risk during labor and delivery, when large fluctuations in hemodynamics will occur.

Q 97.5: A 24-year-old HIV-positive man comes to the emergency department complaining of severe left-sided chest discomfort, which radiates through to the left trapezius region. On coming into the room, you note that he is sitting up and hunched forward. On physical examination, the patient's blood pressure is 135/78, with a pulse of 85 bpm, and a pericardial friction rub is noted. Laboratory findings demonstrate elevated serum creatine kinase levels and normal serial troponin levels. His EKG demonstrates global ST segment elevation. His CXR demonstrates no acute process. Which of the following is the most likely diagnosis in this patient? A Acute pericarditis B Acute myocardial infarction C Acute bacterial endocarditis D Acute ascending aortic dissection E Acute costochondritis

The Correct Answer is: A Choice A is the most likely finding, as this patient is exhibiting signs, symptoms, and EKG findings pathognomonic for acute pericarditis, which is likely infectious in the setting of a patient with HIV. A pericardial friction rub is heard best with the patient in a seated position, during expiration, and is frequently found in patients with pericarditis. Choice B, an acute myocardial infarction, is less likely in a patient of this age, especially with normal serial troponins. Acute pericarditis can sometimes present with elevated serum creatine kinase levels when the epicardium is also involved. Choice C, acute bacterial endocarditis, is less likely in a patient with these EKG changes. Choice D, aortic dissection, would present with chest pain; however, the patient would be markedly hypotensive, less stable on presentation, and a CXR would demonstrate widening of the superior mediastinum. (Fauci et al., 2001, pp. 810-811, 1365-1366, 1431)

Q 94.2: A 60-year-old marathon runner has noticed a progressive decline in his exercise tolerance over the past year. He is now dypsneic while walking up one flight of stairs, and notices increased lower extremity edema. He experienced an episode of chest pressure while playing racquetball, and underwent a nuclear stress test, which was found to be normal. On physical exam, a grade 2/6 crescendo-decrescendo murmur is noted, radiating to the carotid arteries. An echocardiogram is ordered. Which of the following results would be expected, based on the patient's history and physical exam findings? A Bicuspid aortic valve with severe stenosis B Small patent foramen ovale C Tricuspid regurgitation D Pulmonic stenosis E Apical aneurysm

The Correct Answer is: A Choice A, a bicuspid aortic valve, would be the most likely finding on echocardiogram given the patient's history and physical exam findings. The patient is demonstrating symptoms suggestive of congestive heart failure at a relatively young age, with relatively few cardiac risk factors. A congenital bicuspid aortic valve does not usually cause symptoms until the fifth or sixth decade, after many years of calcification have led to severe stenosis. Symptom onset can be gradual or sudden. The patient's physical exam findings are pathognomonic for aortic stenosis, so choices C and D are inappropriate. Choice B, patent foramen ovale, can result in congestive heart failure if the PFO is large and if a large shunt is present; therefore, a small PFO is unlikely to be the cause of this patient's symptoms. Choice E, apical aneurysm, can also lead to congestive heart failure if large, but would not present with this patient's physical exam findings. (McPhee et al., 2011, Chapter 10)

Q 115.18: A 24-year-old HIV-positive man comes to the emergency department complaining of severe left-sided chest discomfort, which radiates through to the left trapezius region. On coming into the room, you note that he is sitting up and hunched forward. On physical examination, the patient's blood pressure is 135/78, with a pulse of 85 bpm, and a pericardial friction rub is noted. Laboratory findings demonstrate elevated serum creatine kinase levels and normal serial troponin levels. His EKG demonstrates peaked T waves. His CXR demonstrates a "water bottle" cardiac silhouette. Which of the following are serious consequences of acute pericarditis, which require careful monitoring? A Pericardial effusion B Aortic dissection C Myxedema D Chylopericardium E Acute myocardial infarction

The Correct Answer is: A Choice A, pericardial effusion, is a serious consequence of acute pericarditis, which requires careful monitoring to ensure that progression of the pericardial effusion does not lead to cardiac tamponade, which can be fatal if not treated promptly. Choice B is unlikely in a patient with acute pericarditis. Choices C, D, and E are noninfectious causes, not consequences of pericarditis. (Fauci et al., 2001, p. 1368)

Q 74.1: A 57-year-old woman with a history of rheumatic fever is seen complaining of dyspnea while vacuuming her apartment, which has been worsening over the last few months. On physical exam, a very soft systolic murmur is auscultated. With inspiration, the murmur increases, and is heard best at the left lower sternal border. A large and early v jugular venous wave is noted. Which of the following is the most likely finding on echocardiogram given this patient's physical exam findings? A Tricuspid regurgitation B Ventricular septal defect C Atrial septal defect D Aortic stenosis E Mitral stenosis

The Correct Answer is: A Choice A, tricuspid regurgitation, is the most likely finding in a patient with a history of rheumatic fever who is presenting with a systolic murmur that increases with inspiration. It is heard best at the left lower sternal border. Tricuspid stenosis is also frequently seen in association with tricuspid regurgitation. The murmur of tricuspid stenosis, however, is a diastolic murmur, which increases with inspiration, and is also heard best at the left lower sternal border. Choice B, ventricular septal defect, would present with a harsh holosystolic murmur, heard best at the third and fourth left interspaces along the sternum. Choice C, an atrial septal defect, if large, could present with similar symptoms of exertional dyspnea secondary to a large shunt, but auscultation would reveal a moderately loud systolic ejection murmur heard best in the second and third interspaces. This is secondary to increased pulmonary arterial flow. Choice D, aortic stenosis, presents with a systolic ejection murmur heard best at the right upper sternal border, frequently radiating to the carotid arteries. The murmur of mitral stenosis, choice E, is also frequently associated with a history of rheumatic fever; however, it is heard best in the left lateral decubitus position, with the bell of the stethoscope at the apex, and inspiration does not increase it. In patients with tricuspid stenosis, mitral stenosis should always be suspected. (LeBlond et al., 2009, Chapter 8)

Q 64.6: A 57-year-old woman with a history of rheumatic fever is seen complaining of dyspnea while vacuuming her apartment, which has been worsening over the last few months. On physical exam, jugular venous distension is appreciated. Auscultation of the chest reveals a possible opening snap, loud S 1 , and a very soft diastolic rumbling murmur is auscultated at the left lower sternal border. When the patient is placed in the left lateral decubitus position, the murmur is accentuated, and heard best at the apex. Both hepatomegaly and splenomegaly are noted. On transthoracic echocardiogram, which of the following findings is most likely to be seen in addition to mitral valve stenosis? A Tricuspid stenosis B Atrial septal defect C Aortic stenosis D Ventricular septal defect E Left ventricular hypertrophy

The Correct Answer is: A Choice A, tricuspid stenosis, is frequently seen in association with mitral valve stenosis in patients with a history of rheumatic heart disease (between 5% and 10% of the time). Both murmurs are similar in character, with the main difference that the murmur of tricuspid stenosis is heard best at the left lower sternal border, and the murmur of mitral stenosis is heard best in the left lateral decubitus position, with the bell at the apex. Because they are similar in nature, a high level of suspicion for tricuspid stenosis should be maintained, so that tricuspid stenosis is not overlooked. Tricuspid regurgitation may also be seen in association with tricuspid stenosis. Patients frequently present with signs and symptoms of right-sided heart failure, including hepatic congestion and splenomegaly. Choices B, C, D, and E are not seen as frequently as tricuspid stenosis in association with rheumatic heart disease and mitral valve stenosis.

Q 71.6: A patient presents to the clinic with a family member. Upon obtaining history from the patient, he responds with excessive details of his symptoms and the reason for his visit. He is unable to answer a question directly without signification elaboration. What problem does this patient have? A circumstantiality B derailment C incoherence D tangentiality

The Correct Answer is: A Circumstantiality is seen in someone who eventually gets to the point after a delay in the thought process. Tangentiality is a disturbance in thought causing the person to start a train of thought, but never getting to the point. Derailment is when a patient skips to another subject. This mainly occurs if a topic is brought up that the patient does not wish to discuss. (Nurcombe and Ebert, 2008, p. 48) Nurcombe B , Ebert MH. The psychiatric interview. In: Ebert MH , Loosen PT , Nurcombe B , Leckman JF, eds. Current Diagnosis and Treatment in Psychiatry. New York: McGraw-Hill; 2008.

Q 92.7: Which of the following is the appropriate treatment for acute Clostridium tetani infection? A tetanus immune globulin, tetanus toxoid, and metronidazole B tetanus immune globulin and penicillin C tetanus toxoid and penicillin D tetanus immune globulin, tetanus toxoid, and penicillin E tetanus immune globulin and tetanus toxoid

The Correct Answer is: A Clostridial tetani infection is a vaccine-preventable disease that results in approximately 50 cases/yr in the United States. Even with modern medical resources, one of four or one of five patients with generalized tetanus dies. Almost all cases occur in individuals who are not properly immunized. Sixty percent of cases occur in older adults for whom immunity has waned. Tetanus presents in different forms including generalized, localized, cephalad, and neonatal. Generalized is the most common and symptoms include mood changes, trismus, diaphoresis, dysphagia, and drooling. Later symptoms include painful flexion and adduction of the arms and pain with extension of the legs. Convulsions and spasms are possible, along with a variety of autonomic symptoms. Treatment includes airway protection, benzodiazepines for muscle spasm, tetanus immune globulin immediately, and three doses of tetanus toxoid given by the standard schedule. Metronidazole has been demonstrated to be the most effective antimicrobial. Labetalol may be used for catecholamine-induced hypertension but the patient must also be monitored for hypotension and bradycardia. (Bartlett, 2007, pp. 2205-2207) Bartlett JG. Clostridial infections. In: Goldman L , Ausiello D, eds. Cecil Medicine. 23rd ed. Philadelphia, PA: Saunders Elsevier; 2007.

Q 76.3: A 24-year-old male has an eight-month history of loose thought associations, social withdrawal, auditory hallucinations, and deterioration in his personal appearance and hygiene. Upon examination, he is noted to have a flat affect, perceptual distortions, and behaves like he is detached from his own actions. If chosen for treatment, which of the following medications would require weekly white blood cell count monitoring for the first six months? A Clozapine (Clozaril) B Haloperidol (Haldol) C Olanzapine (Zyprexa) D Risperidone (Risperdal) E Quetiapine (Seroquel)

The Correct Answer is: A Clozapine has a risk of agranulocytosis. While the risk is only 1%, weekly monitoring of the white blood cell count for the first six months, followed by monitoring of the white blood cell count every other week thereafter, is required. The other medications listed do not have the risk of agranulocytosis. (McPhee SJ, Papadakis MA. Current Medical Diagnosis & Treatment, 2010, p. 955)

Q 117.4: A 42 year old male presents to your office with a 5 day history of multiple episodes of intense, unilateral retroorbital pain of short duration associated with lacrimation and flushing. He describes the pain as a "hot poker" sensation with occurrences at about the same time each day, although not limited to this time. Which of the following is the most likely diagnosis? A Cluster headache B Migraine headache C Sinus headache D Temporal arteritis E Tension headache

The Correct Answer is: A Cluster headaches are typically described as explosive, with marked unilateral retroorbital pain. Patients will often note some relation to time of day, but the headaches can occur at other times as well. Cluster headache patients often have exacerbations with recurrent symptoms over a period of time. Some patients will develop refractory or prolonged episodes, which will require additional management. Cranial parasympathetic autonomic activation leads to the associated symptoms of conjuntival injection, lacrimation, rhinorrhea, and nasal congestion. Cluster headaches must be differentiated from other diagnoses, utilizing history and physical exam findings. (Fauci et al., Harrison's Principles of Internal Medicine, 17e, Chapter 15, Headache)

Q 110.20: An 18-year-old female presents with a history of large open comedones and painful abscesses in both axillae. She has been treated by her primary care provider with oral antibiotics. A thorough physical exam may show which other affected sites? A breasts, groin, and perineum B face and scalp C only the axillae are affected D upper back, ears, and lower legs

The Correct Answer is: A Common areas of involvement in hydradenitis suppurtiva include the axillae, breasts, anogenital region, perineum, and scalp. (Wolff et al., 2009, Page 16)

Q 106.18: Which of the following psychiatric conditions has the highest mortality rate? A anorexia nervosa B bulimia nervosa C depression D generalized anxiety disorder E obsessive-compulsive disorder

The Correct Answer is: A Complex biologic, psychological, and social factors determine onset and severity of eating disorders, along with mortality and morbidity. Patients with eating disorders have some of the highest rates of psychiatric comorbidities, including depression, affective disorders, anxiety disorders, obsessive-compulsive personality traits or disorder, and substance abuse. Eating disorders are risk factors for both attempted and completed suicide. Suicide attempts occur in up to 35% of patients with bulimia, but completed suicides do not appear to be elevated. *In contrast, women with anorexia have been shown to have a fifty-fold increased risk for completed suicide. Suicide is one of the leading causes of death for patients with anorexia.* (Tintinalli, et al., 2011, Chapter 286)

Q 111.4: A 28-year-old male presents with a rash on his hands that is unresponsive to topical steroids. On physical exam you notice periocular violaceous erythema and edema. He also exhibits significant proximal muscle weakness. What is the most likely diagnosis? A Dermatomyositis B Psoriasis C Seborrheic dermatitis D Systemic lupus erythematosus

The Correct Answer is: A Dermatomyositis is an autoimmune disease that targets the skin and skeletal muscle. Skin lesions usually consist of a periorbital heliotrope rash, which can have associated edema. There can also be flat topped violaceous papules (Gottron papules) located on the neck, shoulders, and knuckles. Periungual erythema with telangiectasias may also occur. Possible muscular symptoms include muscle tenderness, muscle atrophy, and progressive proximal muscle weakness. (Wolff et al., 2009, Fig. 14-17B, pg. 370-373)

Q 121.8: A 45-year-old woman comes to the office for evaluation of "yeast infection again." Review of the chart shows that she has had six episodes of vaginal candidiasis in the past 18 months. She and her husband have been mutually monogamous during their 23-year marriage; he had a vasectomy following the birth of their second child. Her two children weighed 10# 2 oz. and 10# 4 oz. at delivery. Her last menstrual period occurred one week ago and was slightly heavier than normal, but her preceding two periods were scanty and "off schedule." Her medications include a thiazide diuretic for mild hypertension and a senior multivitamin. She is 62" tall and weighs 198#. Assuming her examination demonstrates candidiasis, this woman needs most urgently to be screened for which of the following? A diabetes mellitus B hyperlipidemia C hypothyroidism D perimenopause E polycystic ovarian syndrome

The Correct Answer is: A Diabetes causes recurrent vaginal candidiasis in women with uncontrolled glucose. Type 2 diabetes is further suggested by her obesity and history of macrosomia in offspring. Furthermore, thiazide diuretics may increase glucose levels. She needs also to be screened for hyperlipidemia (B), given her weight and probable diabetes, but that is not as urgent. She may be hypothyroid (C), although the clinical picture more strongly suggests diabetes; hypothyroidism is also more likely to cause menorrhagia than scanty periods. She may also be perimenopausal (D), given irregular menses. Polycystic ovarian syndrome (E) is unlikely given no history of infertility. Masharani U, Diabetes Mellitus & Hypoglycemia, in Current Medical Diagnosis and Treatment, 52 nd ed. 2013.

Q 91.9: Which of the following antineoplastic medications is most likely to cause cardiac toxicity and precipitate heart failure? A doxorubicin B cisplatin C cyclophosphamide D tamoxifen E 6-mercaptopurine

The Correct Answer is: A Doxorubicin is a common antineoplastic drug used for a variety of cancers, including breast, bladder, ovarian, and endometrial, among many others. Unfortunately, it has a well-established, dose-dependent adverse effect on the heart that is linked to free-radical formation. (Medina and Fausel, 2008, p. 2099) Medina PJ , Fausel C. Cancer treatment and chemotherapy. In: DiPiro JT , Talbert RL , Yee GC, et al., eds. Pharmacotherapy: A Pathophysiologic Approach. 7th ed. New York: McGraw-Hill; 2008.

Q 101.1: While you are doing a funduscopic exam on an 80-year-old female with progressive vision loss, you notice drusen formations on her retinas. What is the most likely diagnosis? A Macular degeneration B Retinal detachment C Central retinal artery occlusion D Diabetic retinopathy E Central retinal vein occlusion

The Correct Answer is: A Drusen are yellow colored collagen deposits in Bruch's membrane of the retina. They can be diffuse, discrete, or confluent. Retinal pigment changes and atrophy are see in "dry" macular degeneration. "Wet" macular degeneration demonstrates choroidal neovascularization, or serous retinal pigment hemorrhages and retinal detachments. (Riordan et al., 2008, Chapter 6)

Q 63.6: Which of the following symptoms is most frequently reported in patients that have peptic ulcer disease? A Dyspepsia B Sharp abdominal pain C Relief of pain with food D Pain that awakens the patient at night E Significant vomiting

The Correct Answer is: A Dyspepsia (epigastric pain) is the hallmark of peptic ulcer disease and is present in 80 to 90% of patients. The abdominal pain is typically described as dull, gnawing, aching or "hunger like," but isn't typically described as sharp. Relief of pain with food is reported in about 50% of patients. Furthermoe, 66% of patients with duodenal ulcers and 33% of patients with gastric ulcers report nocturnal pain that awakens them at night. Significant vomiting is not typical of uncomplicated peptic ulcer disease, and is more suggestive of gastric outlet obstruction or gastric malignancy.

Q 69.1: A 45-year-old man presents to the office with the complaint of perianal pain and bleeding. Examination reveals an anal/perianal mass complex. Biopsy is taken and the results are positive for epidermoid carcinoma of the anus. Which of the following treatment options would be the most appropriate therapy for this patient? A Local resection, chemotherapy, and external beam radiation B Abdominoperineal resection C Chemotherapy only D Local resection only E Radiation only

The Correct Answer is: A Epidermoid carcinoma of the anal canal is a slow-growing tumor that often presents as an anal or perianal mass. Wide excision, followed by 5-fluorouracil, mitomycin, and external beam radiation (Nigro Protocol), typically results in a greater than 80% cure rate. The presence of inguinal lymph node metastasis is a poor prognostic indicator. Management of recurrences is typically achieved by performing an abdominoperineal resection. (Bullard, 2010, p. 1053) Bullard Dunn KM , Rothenberger DA. Colon, rectum and anus. In: Brunicardi FC, et al., eds. Schwartz's Principles of Surgery. 9th ed. New York, NY: McGraw-Hill; 2010.

Q 101.2: A 51-year-old female undergoing chemotherapy for stage IIC breast cancer presents to her primary care provider with a complaint of fatigue, becoming tired very easily, and an inability to perform any of her normal daily activities. What is the most common reversible cause of chemotherapeutic -related fatigue? A anemia B anxiety C chronic pain D neutropenia with sepsis E sleep disturbance

The Correct Answer is: A Fatigue is one of the most common symptoms in cancer patients, experienced by 70% to 100% of those receiving cancer treatment. It is most commonly related to the chemotherapeutic agent itself and will resolve when treatment is completed. In the interim, anemia is the most common cause and this can be treated with hematopoietic growth factors, erythropoietin (epoetin alfa and epoetin beta), granulocyte colony-stimulating factor (G-CSF), granulocyte-macrophage colony-stimulating factor (GM-CSF). (Kantarjian, et al., 2006, Chapter 41)

Q 101.10: What is the treatment of choice for a patient who is HIV positive and has a confirmed pneumonia caused by Pneumocystosis jiroveci (as shown above)? A trimethoprim-sulfamethoxazole B Azithromycin C Aztreonam D Cephalexin E Erythromycin

The Correct Answer is: A For a confirmed bacteria or suspected case of pneumonia caused by Pneumocystosis jiroveci, TMP-SMX is the antibiotic of choice for this patient. (Shelburne SA, Hamill RJ. Medical Diagnosis and Treatment, 2011, Chapter 36, Mycotic Infections)

Q 105.8: A patient presents with a cerumen impaction. Which of the following is true when performing the Rinne- Weber test? A Weber lateralizes to the affected ear B Weber is equal in both ears C Weber lateralizes to the unaffected ear D Air conduction > Bone conduction in the affected ear E Air conduction = Bone conduction in the affected ear

The Correct Answer is: A For conductive hearing loss, the Weber will lateralize to the affected ear and bone conduction will be greater than air conduction. Conductive hear loss prevents sound from entering the inner ear due to obstruction in the external auditor canal and middle ear. Examples would be cerumen impaction and otitis media. Sensory neural hearing loss affects the inner ear and cranial nerve VIII. Weber will lateralize to the unaffected ear (normal ear) and Rinne will reveal Air conduction > Bone conduction. (Le Blond et al., 2009, Chapter 20)

Q 111.1: A 42-year-old African American male is admitted to the hospital with heme positive urine and anemia. He recently completed a course of trimethoprim/sulfamethoxazole for a urinary tract infection. What is the most likely cause of these findings? A Glucose-6-phophate dehydrogenase deficiency B Factor VIII disorder C Idiopathic thrombocytopenic purpura D Sickle cell anemia E Thalassemia

The Correct Answer is: A G6PD deficiency is an X-linked recessive disorder affecting 10 to 15% of African American males. Hemolysis occurs as a result of oxidative stress on the red cells after exposure to infection or certain drugs, and leads to a hemolytic episode. Common drugs include sulfonamides. Factor VIII and ITP are coagulation disorders, not anemias. Sickle cell and thalassemia are not transient and do not usually present this late in life. (McPhee SJ, Papadakis MA, Tierney LM. Current Medical Diagnosis and Treatment, 2010, Chapter 13, Blood Disorders)

Q 67.2: Assuming that a patient has maintained a normal baseline creatinine of 1.0 mg/dL with a normal glomerular filtration rate (GFR) of 100 mL/min, which of the following indicates a more significant change in the GFR? A increase in creatinine from 1.0 to 2.0 mg/dL B increase in creatinine from 2.0 to 4.0 mg/dL C increase in creatinine from 4.0 to 8.0 mg/dL D increase in creatinine from 8.0 to 16.0 mg/dL

The Correct Answer is: A GFR describes the amount of blood passing through the kidneys per minute. There is an inverse relationship between GFR and serum creatinine. In a patient with normal renal function, doubling of the serum creatinine represents a loss of approximately 50% of GFR. Using this information, the loss of GFR can be estimated from changes in the serum creatinine. For example, assume normal creatinine levels of 1.0 mg/dL and normal GFR of 100 mL/min. A doubling of the serum creatinine from 1.0 mg/dL to 2.0 mg/dL represents an approximate reduction in GFR from 100 mL/min to 50 mL/min (50% of GFR has been lost). Each additional doubling of the creatinine decreases the remaining GFR by approximately one half. When renal function is severely impaired, large increases in the creatinine (ie, from 8.0 to 16.0 mg/dL) represent only small decreases in GFR (from about 12 to 6 mL/min). This example emphasizes the importance of detecting increases in serum creatinine early. However, serum creatinine level does not become abnormal until ˜25% of renal function is lost. Therefore, other methods of estimating GFR are more useful in detecting early decreases in GFR. (Levey, 1999; Stevens and Perrone, 2008) Levey AS. Bosch JP , Lewis JB , et al. A more accurate method to estimate glomerular filtration rate from serum creatinine: a new prediction equation. Ann Intern Med. 1999;130:461-470. 10075613 Stevens L , Perrone RD. Assessment of kidney function: serum creatinine; BUN; and GFR. In Rose BD, ed. 2008. http://www.uptodate.com , version 16.3

Q 82.6: A 45-year-old female presents with a rash that started on her ankles two days ago and is spreading up her leg. The lesions are asymptomatic; however, she has abdominal pain and joint pain. The lesions are palpable and non-blanchable when compressed. What is the most likely diagnosis? A Henoch-Schonlein purpura B Polyarteritis nodosa C Pityriasis rosea D Psoriasis vulgaris

The Correct Answer is: A Henoch-Schonlein Purpura (HSP) is a hypersensitivity vasculitis that occurs most frequently after an infection with group A streptococcus. This rash consists of the classic palpable purpura, and can be accompanied by abdominal pain that is worse after meals secondary, bowel ischemia, bloody diarrhea, and arthritis. (Wolff et al., 2009, Fig. 14-34A, pg. 399-400)

Q 73.4: A 27-year-old female presents to the emergency department with severe RUQ pain. Ultrasonography reveals gallstones. In preparation for a potential cholecystectomy, a CBC is obtained that reveals a normocytic anemia with an increased mean corpuscular hemoglobin concentration (MCHC). She is slightly jaundiced and you are able to palpate her spleen on examination. What is her underlying diagnosis? A Hereditary spherocytosis B Iron deficiency anemia C Sickle cell anemia D Thalassemia E Von Willebrand's disease

The Correct Answer is: A Hereditary Spherocytosis (HS) is characterized by jaundice, an enlarged spleen, and often gallstones; gallstones are more frequently seen in young people, triggering the investigation into HS. An increased MCHC is a characteristic feature of HS and is almost the only condition in which this finding is seen. Iron deficiency anemia does not have an increased MCHC. Sickle Cell has characteristic findings different than the presenting findings. Thalassemia is noted for a microcytosis and Von Willebrand's is a coagulation disorder. (Fauci, et al, Harrison's Principles of Internal Medicine, 17e, Chapter 101, Hemolytic Anemias and Anemia Due to Acute Blood Loss)

Q 113.1: An 18-year-old female presents with a history of draining abscesses in the axilla and groin, with large, open comedones. Currently, she has multiple scars in the axillae. What is the most likely diagnosis? A hidradenitis suppurtiva B nodulocystic acne C acanthosis nigricans D methicillin resistant staphylococcus aureus (MRSA)

The Correct Answer is: A Hidradenitis suppurtiva affects females more than males, and may show a family history of nodulocystic acne and/or hidradenitis suppurtiva. Skin lesions are usually tender nodules and abscesses that may spontaneously drain. Open comedones, including double comedones, are common. Eventually, sinus tracts may form. Nodulocystic acne consists of nodules and cysts, ranging in size from 1 to 4 cm in diameter. These lesions are distributed on the face, back, and chest. Acanthosis nigricans is described as a velvety, hyperpigmented plaque distributed around the neck, in the axillae, and in the groin. MRSA can have several different presentations, ranging from erythrasma to the presence of papules and pustules. Comedones are not associated with MRSA. (Wolff et al., 2009, Page 16)

Q 54.13: According to the Centers for Disease Control and Prevention (CDC), which of the following cannot be treated with a course of doxycycline 100 mg PO BID for 10 to 14 days? A babesiosis B human monocytic ehrlichiosis (HME) C Lyme disease D Rocky Mountain spotted fever E All can be treated with doxycycline.

The Correct Answer is: A Human monocytic ehrlichiosis, Lyme disease and Rocky Mountain spotted fever are all tick-borne diseases causes by spirochetes and treated with doxycycline for 10 to 14 days. Babesiosis is an intracellular parasite and may not necessarily require treatment but, when warranted, the CDC recommends atovaquone plus azithromycin.

Q 101.9: A 42-year-old male exhibits an unstable, dance-like gait, dystonia, irritability, and depression, which is limiting his ability to work. His older sister has similar symptoms. What is the most likely diagnosis? A Huntington's disease B Wilson's disease C Medication-induced chorea D Parkinson's disease E Hyperbilirubinemia

The Correct Answer is: A Huntington's disease, an inherited nervous system disorder seen in families, typically arises in adulthood. It progresses to include mental and motor disturbances. Classic findings include chorea of various severity, dyskinesia, progressive rigidity, and mental changes such as irritability, depression, antisocial personality, and suicidality. This must be differentiated from other nervous system disorders, as well as other causes of chorea and nervous system impairment. (McPhee SJ, Papadakis MA. Current Medical Diagnosis & Treatment 2011, Chapter 24, Nervous System Disorders)

Q 64.7: A 58-year-old man is in the hospital postoperative day 3 after a laparoscopic right colon resection. Your morning labs reveal a serum potassium level of 2.9 mEq/L (normal 3.5-5.0 mEq/L) despite aggressive potassium replacement during the previous shift. At this time you should check which of the following laboratory values? A Magnesium B Calcium C Phosphorous D Sodium E Albumin

The Correct Answer is: A Hypokalemia is a common electrolyte disturbance in surgical patients. It can be caused by enhanced losses, hyperaldosteronism, inappropriate replacement, and intracellular shifts caused by alkalosis. Symptoms of hypokalemia may include constipation, neuromuscular weakness, diminished tendon reflexes, paralysis, and distinctive electrocardiographic changes. Concomitant deficiencies in magnesium can contribute significantly to the development of hypokalemia as well as hypocalcemia. In the surgical patient with persistent hypokalemia refractory to potassium administration, one should check magnesium levels and correct as appropriate.

Q 105.1: What is the most common electrolyte that can effect the initiation of ventricular tachycardia? A Magnesium B Sodium C Chloride D Phosphorus E Calcium

The Correct Answer is: A Hypomagnesia and hypokalemia are the two electrolyte disorders for ventricular tachycardia. (Bashore et al., Current Medical Diagnosis and Treatment, Chapter 10)

Q 107.7: A 45-year-old woman presents with weight gain, fatigue, dry skin, constipation, and oligomenorrhea. On physical exam, bradycardia and slow deep tendon reflexes are noted. Her free T 4 is low and TSH is elevated. Which of the following medications may be responsible for her condition? A amiodarone B beta-blockers C levadopa D hydrocholorthiazide

The Correct Answer is: A Hypothyroidism is reported in up to 10% of patients taking amiodarone, an antiarrhythmic medication. With the high iodine content of the medication and the structural similarities to thyroxine, thyroid abnormalities occur. Common side effects of amiodarone include bradycardia and constipation, so laboratory evaluation for thyroid dysfunction must be used. (Fitzgerald, 2009, p. 978) Fitzgerald PA. Endocrine diseases. In: McPhee SJ , Papadakis MA, eds. Current Medical Diagnosis and Treatment. 48th ed. New York, NY: McGraw-Hill; 2009.

Q 115.4: A 24-year-old man with a recent history of a viral illness comes to the emergency room complaining of severe left-sided chest discomfort, which radiates through to the left trapezius region. On coming into the room, you note that he is sitting up and hunched forward. On physical examination, the patient's temperature is 39°C, blood pressure is 135/78, with a pulse of 85 bpm, and a pericardial friction rub is noted. Laboratory findings demonstrate elevated serum creatine kinase levels and normal serial troponin levels. Which of the following would be the most likely electrocardiographic findings? A Diffuse ST segment elevation B Peaked T waves C Inferior Q waves D Loss of R-wave amplitude E U waves

The Correct Answer is: A In a patient with these signs, symptoms, and lab findings, acute pericarditis is the most likely diagnosis. In patients with acute pericarditis, EKG changes occur secondary to inflammation of the subepicardium, leading to widespread elevation of the ST segments, often with upward concavity, which returns to normal after several days, followed by T wave inversion. No significant QRS complex changes are noted, so choice C, the development of inferior Q waves (frequently associated with an inferior myocardial infarction), is incorrect. Choice B is frequently noted with severe hyperkalemia. Loss of R-wave amplitude, choice D, is associated with myocardial infarction. Choice E, U waves, are associated with hypokalemia. (Fauci et al., 2001, pp. 1289-1270, 1366)

Q 107.1: A 57-year-old man with a history of HTN, hyperlipidemia, and chronic tobacco use presents to the emergency department with complaints of worsening chest tightness over the last 2 months. He initially noticed that every time he raked leaves, he had a few minutes of chest tightness, which was relieved within 5 minutes if he rested. He now notices that raking will precipitate severe chest discomfort, diaphoresis, and dyspnea, which lasts for 20 minutes even if he rests. Last night, while watching football, he again noticed chest tightness, which began suddenly and slowly dissipated over 15 minutes. His physical examination is normal. An EKG performed during an episode of chest discomfort demonstrates normal sinus rhythm at 90 bpm with ST-segment depression. Beta blockers, IV nitroglycerin, aspirin, and oxygen are started. Serial troponin levels are negative. A repeat EKG continues to demonstrate ST segment depression, along with t-wave inversion. The patient's chest discomfort is decreased in intensity and duration, but returns periodically. Which of the following is the most appropriate next step in the management of this patient? A Cardiac catheterization B Exercise nuclear stress test C Holter monitor D Tilt table test E Transesophageal echocardiogram

The Correct Answer is: A In patients with unstable angina pectoris, if symptoms and EKG changes are not stabilized with appropriate medical therapy, including beta blockade, aspirin, oxygen, and nitroglycerin, then choice A, cardiac catheterization, with likely percutaneous coronary intervention, would be recommended, as this patient is at high risk for acute myocardial infarction. Choice B, exercise nuclear stress testing, would provide information regarding exercise tolerance and exercise-induced dysrhythmias, as well as regarding myocardial ischemia, but in this unstable patient, it would not be recommended. In patients whose conditions stabilize with medical therapy, nuclear stress testing is a viable option. Choice C, Holter monitor, is a useful diagnostic tool for evaluation of patients with palpitations occurring on a daily basis. Choice D, tilt table testing, is utilized in the evaluation of patients suffering from near-syncope or syncope. Choice E, transesophageal echocardiogram, is helpful in more direct visualization of heart valves, especially when transthoracic echocardiogram is unclear. (Fauci et al., 2008, Chapter 238)

Q 113.7: Septic arthritis in adults younger than 30 years is usually caused by A Neisseria gonorrhea B Staphylococcus aureus C Pseudomonas aeruginosa D Streptococcus pyogenes E Salmonella species

The Correct Answer is: A In patients younger than 30 years, gonococcus is the most common cause of septic arthritis. When all patients are considered, Staphylococcus aureus is the most common cause. Patients with prevalent joint disease and intravenous drug users are especially susceptible to Staphylococcus. Pseudomonas is also a common cause of septic arthritis in intravenous drug users. Salmonella is not a common cause of joint infection. (Lange and Lederman, 2007, p. 966) Lange C , Lederman MM. Infections involving bones and joints. In: Andreoli TE , Carpenter CCJ , Griggs RC, et al., eds. Andreoli and Carpenter's Cecil Essentials of Medicine. 7th ed. Philadelphia, PA: Saunders Elsevier; 2007.

Q 74.4: A 50-year-old woman with a history of hypertension complains of chest tightness and dyspnea while walking up one flight of stairs. She recently experienced an episode of near-syncope while walking her dog. She denies a history of rheumatic fever. On auscultation, a crescendo-decrescendo systolic ejection murmur is heard at the upper right sternal border radiating to the carotids bilaterally. Troponin levels are negative at 0, 3, and 6 hours. Her EKG demonstrates evidence of left ventricular hypertrophy. A transthoracic echocardiogram reveals significant aortic stenosis secondary to congenital bicuspid aortic valve, left ventricular hypertrophy, and normal left ventricular systolic function. Which of the following is the most appropriate next step in management? A Cardiac catheterization followed by aortic valve replacement B Monitoring via repeat transthoracic echocardiogram in 6 months C Monitoring via transesophageal echocardiogram in 6 months D Treadmill exercise stress test E Automatic internal cardiac defibrillator placement

The Correct Answer is: A In symptomatic patients demonstrating significant aortic stenosis, aortic valve replacement after cardiac catheterization, to evaluate for coronary artery disease and possible concomitant coronary artery bypass surgery with aortic valve replacement, is indicated. As the patient is demonstrating the classic symptoms of severe aortic stenosis, choice A is the most appropriate next step in management. Choices B and C are thus inappropriate, as the patient is already symptomatic. If the patient were not symptomatic, choice B would be a viable choice compared to choice C, because it is less invasive than transesophageal echocardiogram. Choice D would be inappropriate, as strenuous physical activity should be avoided in patients with severe aortic stenosis. Choice E is appropriate therapy for patients at risk for ventricular tachycardia/fibrillation. (McPhee et al., 2011, Chapter 10)

Q 67.7: Assuming no contraindications, which of the following class of medications is considered the preferred long-term control therapy for asthma? A inhaled corticosteroids B leukotriene antagonists C long-acting B2 agonists D methylxanthines E muscarinic antagonists

The Correct Answer is: A Inhaled corticosteroids (eg, beclomethasone, fluticasone, triamcinolone, etc) are the preferred long-term control therapy for persistent asthma in all patients because of their potency and consistent effectiveness. Low- to medium-dose inhaled corticosteroids offer several advantages over other medications, including the ability to reduce bronchial hyper-responsiveness, improve overall lung function, and reduce severe exacerbations that often lead to emergency department visits and hospitalizations. (Chesnutt et al., 2008, p. 209; Kelly and Sorkness, 2008, pp. 485-486;) Chesnutt MS , Murray JA , Prendergast TJ. Pulmonary disorders. In: Tierney LM Jr , McPhee SJ , Papadakis MA, eds. Current Medical Diagnosis & Treatment. 47th ed. New York: McGraw-Hill; 2008. Kelly HW , Sorkness CA. Asthma. In: DiPiro JT , Talbert RL , Yee GC, et al., eds. Pharmacotherapy: A Pathophysiologic Approach. 7th ed. New York: McGraw-Hill; 2008.

Q 59.8: A 36-year-old auto mechanic presents to the emergency department after hurting his back on the job. While lifting an object, he experienced sudden pain in his lower back with radiation to the right buttock. He was initially treated for muscle strain with a nonsteroidal anti-inflammatory drug (NSAID) after x-rays of his lumbosacral spine demonstrated no pathology. He continued to complain of this low back pain now radiating posteriorly down his left leg to the mid-thigh. Physical examination is unremarkable. The most likely diagnosis is A lumbosacral strain B left S1 radiculopathy C cauda equina syndrome D L5-S1 disc herniation E lateral femoral cutaneous neuropathy

The Correct Answer is: A Low back pain is one of the more common presenting neurologic complaints to a primary care provider. Most acute pain syndromes are benign, self-limiting conditions, with pain arising from myofascial sources. Patients with back pain and normal neurologic examinations are unlikely to have any serious underlying pathology and further diagnostic testing is usually unrevealing.

Q 64.2: Which of the following is most useful in diagnosing renal artery stenosis? A magnetic resonance angiography (MRA) B computed tomography (CT) scanning C captopril renal scan D renal artery biopsy E intravenous pyelogram (IVP)

The Correct Answer is: A Magnetic resonance angiography, enhanced with gadolinium, is 99% to 100% sensitive and 71% to 96% specific for diagnosing renal artery stenosis (RAS). This study has largely replaced the captopril renal scan and contrast-enhanced arteriography in diagnosing RAS. The principle behind the captopril renal scan is that ACE inhibitors lower GFR. In a kidney with already-compromised blood flow due to RAS, administration of the ACE inhibitor further decreases GFR in the affected kidney despite maintenance of adequate plasma volume. GFR in the contralateral kidney remains normal. Subsequent injection of a radionuclide reveals delayed uptake in the compromised kidney. Although arteriography provides the most definitive diagnosis, it carries its own risks of contrast-induced injury and bleeding. MRA is a low-risk procedure due to its noninvasive nature. Renal artery biopsy would not yield this diagnosis. IVP is utilized to visualize the anatomical structure of the urinary tract in situations such as urinary tract trauma and outflow obstruction, although increasingly it too is being replaced by noninvasive testing, such as ultrasound, CT scanning, and MRI. It remains a useful test to pinpoint the location of a calculus in the urinary tract.

Q 109.18: Which of the following drugs can cause syndrome of inappropriate antidiuretic hormone (SIADH)? A carbamazepine B glyburide C lithium carbonate D metoprolol

The Correct Answer is: A Many medications can enhance the release or potentiate the effects of ADH. Carbamazepine may increase ADH release. (Andreoli, 2007, pp. 289-291)

Q 89.10: Which of the following patterns of stiffness is most characteristic of patients with rheumatoid arthritis? A morning stiffness lasting at least 1 hour B exacerbation of joint stiffness with walking C frequent, brief episodes of stiffness after inactivity D stiffness reflected by a major delay in muscle relaxation E stiffness evidenced by increased resistance to passive movement

The Correct Answer is: A Morning stiffness lasting at least 1 hour is characteristic of rheumatoid arthritis (RA). Exacerbation of joint stiffness with weight bearing (such as walking) and frequent, brief episodes of stiffness (lasting <30 minutes) after inactivity are both more characteristic of degenerative joint disease, not RA. Stiffness reflected by a major delay in relaxation after muscle contraction is seen in myotonic dystrophy. Stiffness evidenced by increased resistance to passive movement describes the "rigidity" associated with parkinsonism. (Hellmann and Imboden, 2008, pp. 720-721) Hellmann DB , Imboden JB. Arthritis and musculoskeletal disorders. In: Tierney LM , McPhee SJ , Papadakis MA, eds. 2008 Current Medical Diagnosis and Treatment. New York, NY: McGraw-Hill; 2008:703-756.

Q 116.3: A patient with myasthenia gravis would likely experience symptomatic benefit with which of the following? A acetylcholinesterase inhibitors B muscarinic antagonists C α1-blockers D β-blockers E dopamine agonists

The Correct Answer is: A Myasthenia gravis is characterized by autoantibodies directed against nicotinic cholinergic receptors at neuromuscular junctions. By inhibiting the enzyme responsible for metabolizing acetylcholine (acetylcholinesterase), the synaptic concentration of acetylcholine increases and can bind more frequently to functional nicotinic receptors yet to be affected by the disease. This can alleviate the symptoms such as limb weakness, difficulty swallowing, and difficulty chewing associated with the disease. Examples of acetylcholinesterase inhibitors include neostigmine and pyridostigmine. (Aminoff, 2008, pp. 892-893) Aminoff MJ. Nervous system disorders. In: Tierney LM Jr , McPhee SJ , Papadakis MA, eds. Current Medical Diagnosis & Treatment. 47th ed. New York: McGraw-Hill; 2008.

Q 106.11: A 47-year-old female patient is diagnosed with a duodenal ulcer. She was determined to have H. pylori infection. Which of the following antibiotics is indicated for the eradication of H. pylori in the treatment of peptic ulcer disease? A Metronidazole B Cephalexin C Trimethoprim-sulfamethoxazole D Gentamycin E Ofloxacin

The Correct Answer is: A Of the antibiotics listed, metronidazole is the only one that has the indication in the treatment of H. pylori induced peptic ulcers. (McPhee SJ, Papadakis MA. Current Medical Diagnosis & Treatment, 2010, p. 551)

Q 118.20: A 63-year-old male presents with an asymptomatic lesion in his mouth that was discovered by his dentist at a check-up. It is ill marginated with pigment ranging from medium brown to black. Parts of the lesion are raised. What is the next appropriate step in management? A Biopsy of the lesion B Inject the lesion with triamcinolone C Observe for changes over the next six months D Prescribe keflex for 10 days

The Correct Answer is: A Oropharyngeal melanoma is characterized by varying pigment occurring in an irregularly shaped lesion. Although this is a rarely occurring melanoma, a biopsy should be done and any pigmented oral lesion should be excised. Areas which are raised within the lesion usually indicate sites of invasion. (Wolff et al., 2009, Fig. 34-13, pg. 1040 & 327)

Q 71.10: A patient who resides in northern Arizona presents with signs and symptoms that are consistent with pneumonia. He is afebrile and appears non-toxic. His physical exam is unremarkable, and blood work is within normal limits. A chest x-ray is ordered and reveals bilateral upper lobe infiltrates. Based on this information, what is the best medication treatment for this patient? A Itraconazole B Erythromycin C Oseltamivir D Doxycycline E Amoxicillin

The Correct Answer is: A Patient is presenting with clinical signs and symptoms of fungal pneumonia. The presentation of being afebrile, along with upper lobe infiltrates bilaterally in a region of the country that consistently has more fungal pneumonia's would lead the clinician with the reasoning that a fungal process is present. The azole's are the best treatment for fungal pathogens. (Proia LA. Current Diagnosis and Treatment in Pulmonary Medicine, Section X, Diseases of the Pleura, Chapter 38, Fungal Pneumonias)

Q 118.18: Generally, patients who are malingering: A use illness to attain a goal B have avoidant personalities C follow prescribed treatment regimens D have a history that agrees with their physical symptoms

The Correct Answer is: A Patients who are malingerers do not want to improve until their goal is met. Goals may be financial, occupational, or legal. These patients will act differently when they think they are not being observed. They may fake their symptoms in order to be admitted to a hospital or to obtain drugs. These patients have an antisocial personality disorder. (Ford, 2008, p. 417; Sadock and Sadock, 2008, pp. 421-422) Ford, CV. Somatoform disorders. In: Ebert MH , Loosen PT , Nurcombe B , Leckman JF, eds. Current Diagnosis and Treatment in Psychiatry. New York: McGraw-Hill; 2008. Sadock BJ , Sadock VA. Concise Textbook of Clinical Psychiatry, 3rd ed. Philadelphia, PA: Lippincott, Williams & Wilkins; 2008.

Q 117.6: A phobia is an excessive fear of an object or place that leads to or can be preceded by: A panic attack B depression C hallucinations D delusions E confabulations

The Correct Answer is: A Patients who have a phobia realize it is an irrational fear and try to avoid whatever they have the fear of. In attempts to avoid the "problem," patients can develop anxiety or panic attacks. (Sadock and Sadock, 2008, p. 250; Shelton, 2008, p. 361) Sadock BJ , Sadock VA. Concise Textbook of Clinical Psychiatry, 3rd ed. Philadelphia, PA: Lippincott, Williams & Wilkins; 2008. Shelton RC. Other psychotic disorders. In: Ebert MH , Loosen PT , Nurcombe B , Leckman JF, eds. Current Diagnosis and Treatment in Psychiatry. New York: McGraw-Hill; 2008.

Q 100.1: A 65-year-old man with a history of emphysema, obstructive sleep apnea, prior inferior myocardial infarction, and pulmonary hypertension presents with complaints of increasing dyspnea over the last 6 months. He has also recently noticed increasing lower extremity edema. On physical exam, jugular venous distension is appreciated. Auscultation of the chest demonstrates a blowing holosystolic murmur along the lower left sternal border. Hepatomegaly, ascites, and lower extremity edema are noted. Which of the following would be the most likely finding on echocardiogram given the patient's physical exam findings? A Tricuspid regurgitation B Aortic stenosis C Atrial septal defect D Aortic regurgitation/insufficiency E Mitral stenosis

The Correct Answer is: A Patients with pulmonary hypertension and right heart failure frequently demonstrate right ventricular hypertrophy, which leads to tricuspid regurgitation; therefore, choice A is the most likely of the choices offered. This patient is demonstrating signs and symptoms of right heart failure, and with a prior inferior infarction is at risk for the development of tricuspid regurgitation. In addition, the patient demonstrates the blowing holosystolic murmur at the left lower sternal border, which is characteristic for tricuspid regurgitation. Choice B, aortic stenosis, presents with a systolic ejection murmur. An atrial septal defect, choice C, if large, could present with similar symptoms of exertional dyspnea secondary to a large shunt, but auscultation would reveal a moderately loud systolic ejection murmur heard best in the second and third interspaces. This is secondary to increased pulmonary arterial flow. Choice D, aortic regurgitation, presents with a soft diastolic murmur heard best at the left sternal border. (Fauci et al., 2008, Chapter 230)

Q 105.12: A 62-year-old obese woman presents with progressive numbness and tingling in her feet for the past 3 months. On physical examination, the patient is found to have decreased sensation to pinprick and vibration, absence of ankle reflexes, and difficulty with tandem walking. Which is the most common etiology of her symptoms? A diabetes mellitus B alcoholism C vitamin B12 deficiency D spinal cord tumor E rheumatoid arthritis

The Correct Answer is: A Peripheral neuropathy is a syndrome that is manifested by muscle weakness, paresthesias, decreased deep tendon reflexes, and autonomic disturbances most commonly in the hands and feet, such as coldness and sweating. There are many causes of peripheral neuropathy ranging from metabolic conditions to malignant neoplasm, rheumatoid arthritis, and drug and alcohol use. The increase in non-insulin-dependent diabetes mellitus due to obesity in the American population has increased the incidence of associated disease states. (Aminoff et al., 2005, pp. 213-214) Aminoff MJ , Greenberg DA , Simon RP. Clinical Neurology. 6th ed. New York, NY: McGraw-Hill; 2005.

Q 115.9: A 28-year-old man has a positive HIV ELISA and Western Blot but has never had an opportunistic infection. Of the following laboratory parameters, which, if present, is consistent with a diagnosis of AIDS in this man? A CD4 lymphocyte count of 175/mL B HHV-8 titer of 1:160 C HSV-2 titer of 1:80 D platelet count of 10,000/mL E total white blood cell count of 1500/mL

The Correct Answer is: A Persons with a CD4 count less than 200/mL or a CD4 percentage below 14% are now included in the Centers for Disease Control and Prevention category of "definitive AIDS diagnoses with laboratory evidence of HIV infection." Persons with HIV-AIDS may have positive herpes titers or depressed platelet or white blood cell counts, but these are not diagnostic of AIDS in the absence of symptoms. (Katz and Zolopa, 2009, p. 1177) Katz MH , Zolopa AR. HIV infection. In: McPhee SJ , Papadakis MA, eds. Current Medical Diagnosis and Treatment. 48th ed. New York, NY: McGraw-Hill; 2009.

Q 110.11: A patient presents with epistaxis from the right nares, along with direct pressure to the nares and elevation of the head. Which of the following is an appropriate initial treatment? A Phenylephrine spray and anterior packing B Triamcinolone spray and anterior packing C Azelastine spray and anterior packing D Momentasone spray and anterior packing E Cromolyn sodium spray and anterior packing

The Correct Answer is: A Phenylephrine is a topical decongestant, and acts as a vasoconstrictor to aid in stopping minor anterior septal epistaxis. Triamcinolone and momentasone are nasal steroids used for allergic rhinitis. Cromolyn sodium is a mast cell stabilizer, and azelastine is a topical H1 selective antihistamine, used for allergic rhinitis. (McPhee et al., 2011, Chapter 6)

Q 119.5: A 30-year-old woman presents to the office with polyuria, fatigue, and a chronic white vaginal discharge with vaginal pruritis. She has been having the discharge off and on for the past 6 months with recurrent treatment failures. Which of the following is the most likely diagnosis? A type 2 diabetes mellitus B hyperthyroidism C hypothyroidism D diabetes insipidus

The Correct Answer is: A Polyuria, polydipsia, and fatigue are all findings that can be consistent with both type 1 and type 2 diabetes. Any woman who presents with a chronic vaginal discharge or chronic vaginal pruritis should be screened for type 2 diabetes. (Masharami, 2009, p. 1056) Masharami U. Diabetes mellitus and hypoglycemia. In: McPhee SJ , Papadakis MA, eds. Current Medical Diagnosis and Treatment. 48th ed. New York, NY: McGraw-Hill; 2009.

Q 102.7: A 56 year-old woman develops a fever of 102.1 F on her first post-operative surgery day following cholecystectomy. Which of the following is the most likely diagnosis? A Atelectasis B Catheter-related phlebitis C Drug reaction D Urinary tract infection E Wound infection

The Correct Answer is: A Post-operative fever that occurs in the first 24 hours is most likely caused by atelectasis (A). Catheter-related phlebitis (B), drug reactions (C) and urinary tract infection (D) typically present between post-operative days 2 and 4, while wound infection (E) typically presents on or after post-operative day 5.

Q 109.1: A 57-year old male with hypovolemia and decreased cardiac output is diagnosed with acute kidney injury (acute renal failure). What is the most likely cause of acute kidney injury? A Prerenal azotemia B Ischemia-associated injury C Glomerulonephritis D Antibiotics E Postrenal acute kidney injury

The Correct Answer is: A Prerenal azotemia (A) is the most common form of acute kidney injury (AKI). Azotemia is a buildup of nitrogen waste products in the body, resulting in a rise in serum Cr or BUN concentration, due to inadequate renal plasma flow and intraglomerular hydrostatic pressure to support normal glomerular filtration. The most common clinical conditions associated with prerenal azotemia are hypovolemia, decreased cardiac output, and medications that interfere with renal autoregulatory responses such as nonsteroidal anti-inflammatory drugs (NSAIDs) and inhibitors of angiotensin II. Intrinsic AKI includes glomerulonephritis, ischemia (B), sepsis/infection, vascular injury, and nephrotoxins including antibiotics (D)/(C). Postrenal AKI (E) occurs when the normally unidirectional flow of urine is acutely blocked either partially or totally, leading to increased retrograde hydrostatic pressure and interference with glomerular filtration. Postrenal AKI includes bladder outlet obstruction. Waikar SS, Bonventre JV. Chapter 279. Acute Kidney Injury. In: Longo DL, Fauci AS, Kasper DL, Hauser SL, Jameson JL, Loscalzo J, eds. Harrison's Principles of Internal Medicine. 18th ed. New York: McGraw-Hill; 2012. http://www.accessmedicine.com/content.aspx?aID=9129805 . Accessed March 6, 2013.

Q 85.1: A middle-aged woman presents with elevated cholestatic liver enzyme levels. She is not taking any medications, does not drink alcohol, and does not complain of abdominal pain. She has not had any previous biliary tract surgery. Which of the following is the most likely diagnosis? A primary biliary cirrhosis B pancreatitis C cholecystitis D fatty liver E primary sclerosing cholangitis

The Correct Answer is: A Primary biliary cirrhosis affects women typically between ages 40 and 60. It is often discovered incidentally when the serum alkaline phosphatase level is found to be elevated. Many patients do not have pain, which is more common in cholecystitis or pancreatitis. Primary sclerosing cholangitis is more likely to occur in a patient with known inflammatory bowel disease. (Friedman, 2009, pp. 607-608, 622) Friedman LS. Liver, biliary tract, and pancreas disorders. In: McPhee SJ , Papadakis MA, eds. Current Medical Diagnosis and Treatment. 48th ed. New York, NY: McGraw-Hill; 2009.

Q 105.7: Your 65-year-old patient with a history of tobacco abuse was recently diagnosed with stage III lung cancer. He has not started treatment yet and presents to his oncologist with complaints of nausea, anorexia, and increasing fatigue over the last several days. He has been eating less than usual but has been able to maintain a normal fluid intake. His wife reports that he has been more forgetful and confused than usual. His medical history includes hypertension, for which he has been taking 25 mg of hydrochlorothiazide for 12 years, and gastro-esophageal reflux disease (GERD), for which he takes omeprazole. He has no history of significant side effects from his medications. You order labs, and the calcium level is elevated at 11.9 mg/dL. What is the most likely etiology of his hypercalcemia? A malignancy B hyperparathyroidism C thiazide diuretic use D dehydration E vitamin D toxicity

The Correct Answer is: A Primary hyperparathyroidism and malignancy account for 90% of all cases of hypercalcemia. *Ten to twenty percent of patients with cancer develop hypercalcemia, most commonly because of breast, lung, kidney, head and neck carcinomas, and multiple myeloma and lymphoma.* Given this patient's history of lung cancer, this is the most likely etiology of his hypercalcemia. Although it is possible that the patient's symptoms could be due to hyperparathyroidism, this is a relatively rare disorder, affecting only about 0.1% of the population, making malignancy a much more likely etiology. He is taking a low dose of hydrochlorothiazide, which has been stable for years; therefore, this is unlikely to be causing excessive fluid losses and dehydration with hemoconcentration. However, this medication could be exacerbating the hypercalcemia. He is not taking vitamin D, so there is nothing to suggest vitamin D toxicity. (Cho et al., 2009, pp. 778-780; Fitzgerald, 2009, pp. 1007-1009; Rugo, 2009, pp. 1483-1484) Cho K , Fukagawa M , Kurokawa K. Fluid and electrolyte disorders. In: Tierney LM , McPhee SJ , Papadakis MA, eds. Current Medical Diagnosis and Treatment. 48th ed. New York, NY: McGraw-Hill; 2009. Fitzgerald PA. Endocrine disorders. In: Tierney LM , McPhee SJ , Papadakis MA, eds. Current Medical Diagnosis and Treatment. 48th ed. New York, NY: McGraw-Hill; 2009. Rugo HS. Cancer. In: Tierney LM , McPhee SJ , Papadakis MA, eds. Current Medical Diagnosis and Treatment, 48th ed. New York, NY: McGraw-Hill; 2009.

Q 116.13: Which of the following radiographic studies is indicated for the initial evaluation of a questionable palpable mass in the area of the kidney, with no other complaints by the patient? A renal ultrasound B intravenous pyelogram (IVP) C abdominal computed tomography D magnetic resonance imaging

The Correct Answer is: A Renal masses are initially identified by ultrasound. Ultrasound will be able to distinguish between a solid mass and a cyst. It is not uncommon to find some texts state that an intravenous pyelogram is noted as the initial test. Intravenous pyelograms have limited value, especially in differentiating small tumors. Whether a mass or a cyst, these findings are usually referred to a urologist who will follow-up with their own IVP and CT. (McAninch, 2008, p. 511) McAninch JW. Disorders of the kidneys. In: Tanagho EA , McAninch JW, eds. Smith's General Urology. 17th ed. New York, NY: McGraw-Hill; 2008:506-520. McAninch JW. Disorders of the penis and male urethra. In: Tanagho EA , McAninch JW, eds. Smith's General Urology. 17th ed. New York, NY: McGraw-Hill; 2008:625-637.

Q 57.10: What is the most common blood gas abnormality in patients with a pulmonary embolism? A Respiratory alkalosis B Metabolic acidosis C Metabolic alkalosis D Respiratory acidosis E Compensated respiratory acidosis

The Correct Answer is: A Respiratory alkalosis occurs as a result of the hyperventilation.

Q 118.19: What is the most common pathogen that causes an acute bronchitis? A Viral B Bacterial C Fungal D Unknown E Spirochetal

The Correct Answer is: A Respiratory viruses are the most common cause of acute bronchitis. In clinical medicine, it is rare to obtain cultures for patients who present with bronchitis symptoms. (Tallman TA. Tintinalli's Emergency Medicine, Section 8. Pulmonary Emergencies, Chapter 67, Acute Bronchitis and Upper Respiratory Tract Infections)

Q 101.4: Taking vitamin C, E, zinc, and beta carotene, and stopping smoking, have a preventative effect on the progression of which of the following diseases? A Macular degeneration B Retinal detachment C Central retinal artery occlusion D Diabetic retinopathy E Central retinal vein occlusion

The Correct Answer is: A Smoking cessation and taking supplements, including vitamin C, E, zinc, and beta carotene, have shown an eight percent decrease in progression of late stage macular degeneration. However, smokers or previous smokers should not take beta carotene, due to its link with lung cancer in smokers. (Riordan et al., 2008, Chapter 6)

Q 112.5: Which of the following findings would be evidence of a patient who has longstanding chronic obstructive pulmonary disease (COPD)? A a decreased FEV1 on spirometry that is not fully reversible with nebulizer treatment B chest radiograph with an elevated hemidiaphragm C FEV1/FVC ratio > 0.7 D a decreased A-a-Do2 on arterial blood gas E an abnormal sweat test

The Correct Answer is: A Spirometry provides objective information about pulmonary function and assesses the results of therapy. Pulmonary function tests early in the course of COPD reveal only evidence of abnormal closing volume and reduced midexpiratory flow rate. Reductions in FEV 1 and in the ratio of forced expiratory volume to vital capacity (FEV 1 % or FEV 1 /FVC ratio) occur later. In severe disease, the FVC is markedly reduced. Lung volume measurements reveal a marked increase in residual volume (RV), an increase in total lung capacity (TLC), and an elevation of the RV/TLC ratio, indicative of air trapping, particularly in emphysema. Arterial blood gas measurements characteristically show no abnormalities early in COPD other than an increased A-a-DO2. Indeed, they are unnecessary unless (1) hypoxemia or hypercapnia is suspected, (2) the FEV 1 is < 40% of predicted, or (3) there are clinical signs of right heart failure. Radiographs of patients with chronic bronchitis typically show only nonspecific peribronchial and perivascular markings. Plain radiographs are insensitive for the diagnosis of emphysema; they show hyperinflation with flattening of the diaphragm in less than half of cases. (McPhee and Papadakis, 2011, Chapter 9)

Q 119.12: A 45-year-old woman with a known seizure disorder has been noncompliant with her anticonvulsant medication due to side effects she has been experiencing. While in your office, she starts convulsing at a frequency that does not allow consciousness. Which of the following is the most appropriate initial drug treatment? A lorazepam B phenytoin C phenobarbital D valproic acid

The Correct Answer is: A Status epilepticus, defined as a continuous seizure or repeated seizures in which interval consciousness is not obtained, is a medical emergency. An intravenous infusion of a longer acting benzodiazepine, such as lorazepam, has been shown to be effective in terminating a seizure. If the seizures persist, then other potential agents to use after the initial lorazepam infusion include fosphenytoin (phenytoin) or valproic acid (phenobarbital). (Lowenstein, 2008, p. 2511) Lowenstein DH. Seizures and epilepsy. In: Fauci AS , Braunwald E , Kasper DL, et al., eds. Harrison's Textbook of Medicine. 17th ed. New York, NY: McGraw-Hill; 2008.

Q 107.3: Which of the following types of renal calculi is associated with an infectious cause? A struvite B uric acid C calcium oxalate D cystine E calcium phosphate

The Correct Answer is: A Struvite stones form when urea-splitting organisms, such as Proteus, Klebsiella, Pseudomonas, and Staphylococcus, are present in the urinary tract. Ammonia is formed when urease breaks down urea. This results in an alkaline urine, which decreases the solubility of struvite, favoring the production of stones. Calcium stones result from hyperabsorption of calcium in the intestine, impaired renal tubular reabsorption of calcium, primary hyperparathyroidism, intestinal hyperabsorption of oxalate, and hypocitraturia. Uric acid stones are due to hyperuricosuria or a urinary pH <5.5, which causes uric acid to dissociate. They are also the only radiolucent calculi. Cystinuria, an inborn error of metabolism, results in cystine stones. (Stoller et al., 2009, pp. 833-837) Stoller ML , Kane CJ , Meng MV. Urology. In: Tierney LM , McPhee SJ , Papadakis MA, eds. Current Medical Diagnosis and Treatment. 48th ed. New York, NY: McGraw-Hill; 2009.

Q 108.3: A patient presented to your office with multiple somatic complaints. During the mental status exam, you notice that the patient loses the thread of conversation and discusses irrelevant topics based on an external stimulus. The patient never gets back to the main point he or she was trying to express. What is this thought process called? A tangentiality B circumstantiality C looseness of association D word salad E neologisms

The Correct Answer is: A Tangentiality is a disturbance in thought causing the person to start a train of thought, but never getting to the point. Circumstantiality is seen in someone who eventually gets to the point after a delay in the thought process. Word salad is a mixture of words and phrases that are incoherent. Looseness of association is when the ideas shift between subjects that are totally unrelated to each other. Neologisms are the creation of new words. (Nurcombe and Ebert, 2008, p. 48) Nurcombe B , Ebert MH. The psychiatric interview. In: Ebert MH , Loosen PT , Nurcombe B , Leckman JF, eds. Current Diagnosis and Treatment in Psychiatry. New York: McGraw-Hill; 2008.

Q 121.9: Which of the following concerning changes in nevi can be associated with melanoma? A asymmetry, irregular borders, color changes, and growth of the lesion B asymmetry, enlarged border, color uniformity, diameter <4mm C asymmetry, bleeding, color uniformity, diameter <4mm D symmetry, well defined borders, color irregularity, diameter <4mm E symmetry, sharp borders, no color, and growth of the lesion

The Correct Answer is: A The ABCDE's of moles are asymmetry, border irregularity, color change or irregularity, diameter >5mm, and evolving (changing in some way). These are all signs that the mole should be evaluated for possible dysplastic or malignant changes. (Wolff et al., 2009, Page 301)

Q 71.2: A 22 year-old male is involved in a motor vehicle crash resulting in fracture of the left femur and left ribs 3 through 6. Approximately 24 to 36 hours after admission he becomes mildly confused and his RR increases to 40. Chest x-ray reveals diffuse pulmonary opacities. ABG shows pH 7.39, PCO 2 34, PO 2 55. What is the most likely diagnosis? A ARDS B Cardiac contusion C Pleural effusion D Pneumothorax E Pulmonary thromboembolism

The Correct Answer is: A The acute onset of respiratory distress after trauma is consistent with ARDS (A). The chest x-ray findings are inconsistent with cardiac contusion (B), pleural effusion (C), pneumothorax (D) and pulmonary thromboembolism (E). (Mention by letter)

Q 109.2: A 60-year-old male presents with scaling feet for several months. The nails are spared. The patient has tried over-the-counter hydrocortisone cream with no help. KOH shows branching hyphae and spores. Which of the following should be part of an appropriate treatment regimen? A topical clotrimazole bid x 4 weeks B lotrisone cream bid x 2 weeks C fluticasone ointment bid x 2 weeks D mupirocin ointment bid x 4 weeks

The Correct Answer is: A The clinical presentation and KOH results are consistent with tinea pedis or athlete's foot. Since the nails are unaffected, topical treatment with clotrimazole is appropriate. Lotrisone contains an antifungal and a steroid; this combination medication is not appropriate for this patient. Fluticasone is a class 3 topical steroid that would worsen this patient's condition. Mupirocin is a topical antibiotic and will not help resolve a fungal infection. (Wolff et al., 2009, Pages 695-699)

Q 76.4: A 7-year-old male is suspected to have a Wilms' tumor (nephroblastoma). What is the most common symptom at presentation? A Abdominal mass B Hypertension C Hematuria D Coagulopathy E Fever

The Correct Answer is: A The diagnosis of a Wilms' tumor is most commonly made after the discovery of an asymptomatic mass, by a family member or a physician, during a routine physical examination. The most common sign is an abdominal mass (A). Other symptoms at presentation include abdominal pain and distention, anorexia, nausea and vomiting, fever (E) and hematuria. Hypertension (B) is seen in 25-60% of cases and is caused by elevated renin levels. As many as 30% of patients demonstrate hematuria (C) and coagulopathy (D) can occur in up to 10%. Konety BR, Williams RD. Chapter 21. Renal Parenchymal Neoplasms. In: Tanagho EA, McAninch JW, eds. Smith's General Urology. 17th ed. New York: McGraw-Hill; 2008. http://www.accessmedicine.com/content.aspx?aID=3128159 . Accessed March 6, 2013.

Q 70.2: What is the most common embolic source of acute arterial occlusion in the lower extremities? A Atrial fibrillation B Aortic aneurysm C Myocardial infarction D Prosthetic cardiac valve E Iliac artery thrombus

The Correct Answer is: A The heart accounts for 80% of all emboli, with atrial fibrillation making up 70% of that. Aortic aneurysms are frequently lined with thrombus but infrequently embolize; aneurysmal disease only accounts for 6% of all acute arterial occlusion. Acute myocardial infarction (especially those associated with left ventricular thrombus) accounts for 25% of cardioembolism, with peripheral embolization often the first sign of a previously "silent" MI. Prosthetic cardiac valves make up a still small but increasingly prevalent source of emboli. Peripheral arterial thrombi account for only 3% of acute occlusion.

Q 66.2: A 12-year-old male begins to sneeze, and develops itchy, watery eyes about 15 minutes after being exposed to a cat. There is no respiratory difficulty. What phase of allergic response is he in? A Humoral B Priming C Cellular D Seasonal E Perennial

The Correct Answer is: A The humoral or early phase occurs in the first 15 minutes of being exposed to an allergen. The symptoms are caused by release of histamine. The cellular phase is the late phase, and occurs after four to six hours of allergen exposure. Seasonal allergic rhinitis occurs in a regular pattern each year, corresponding to pollen exposure. Perennial rhinitis occurs year round, and may be more linked with indoor allergen exposures.

Q 66.9: A 54-year-old female presents complaining of decreased visual acuity to her right eye over the past few hours. She denies pain, and describes having wavy vision and seeing flashes of light. Her visual acuity in the affected eye is 20/200. What condition best describes the following physical finding? A Retinal detachment B Central retinal artery occlusion C Open angle glaucoma D Angle closure glaucoma E Optic neuritis

The Correct Answer is: A The image demonstrates a detached retina. The superior aspect of the retina appears wavy and flowing. A central retinal artery occlusion is characterized by a pale retina, as well as a cherry red spot on the macula. Open angle glaucoma does not cause acute vision loss. Angle closure glaucoma causes painful acute vision loss. Optic neuritis is characterized by painful visual loss and a swollen optic disc.

Q 106.10: A 33-year-old woman presents with a 3-year history of a persistent, unfluctuating depressed mood. She also notes persistent insomnia, poor concentration, and very little appetite. She denies previous similar symptoms, substance abuse, current prescriptive drug use, and has had no change in her overall life circumstances. She remains functional at work and in most relationships. On the basis of the information presented, what is the most likely diagnosis? A dysthymic disorder B premenstrual dysphoric disorder C major depressive disorder D cyclothymic disorder

The Correct Answer is: A The main historical component that points to this diagnosis is the long-term (equal to or greater than 2 years), unfluctuating symptoms without mention of manic or hypomanic symptoms that would be typical of cyclothymic disorders. No variances with menstrual cycles are mentioned. Major depressive disorder is generally associated with more intense symptoms, including suicidal ideation, and only requires a 2-week duration of symptoms to diagnose. (Loosen and Shelton, 2008, p. 328; Sadock and Sadock, 2008, p. 226) Loosen PT , Shelton RC. Mood disorders. In: Ebert MH , Loosen PT , Nurcombe B , Leckman JF, eds. Current Diagnosis and Treatment in Psychiatry. New York: McGraw-Hill; 2008. Sadock BJ , Sadock VA. Concise Textbook of Clinical Psychiatry, 3rd ed. Philadelphia, PA: Lippincott, Williams & Wilkins; 2008.

Q 115.1: A 17-year-old female (Ht: 5'6", Wt: 105 lbs) is noted to have macrocytic anemia. Her serum vitamin B12 level is 298 (normal range 203 to 339). What is the most likely diagnosis? A Folate deficiency B Hemolytic anemia C Iron deficiency anemia D Sideroblastic anemia E Vitamin B12 deficiency

The Correct Answer is: A The most common cause of folate deficiency is inadequate dietary intake. This patient is anorexic, which increases her chances of developing a folate deficiency, a common cause of macrocytic anemias. Her vitamin B12 level is in the normal range, making this diagnosis unlikely. Iron deficiency causes a microcytic anemia. Sideroblastic anemia is diagnosed by examining the bone marrow, and the MCV is usually normal. Hemolytic anemias generally do not affect MCV. (McPhee SJ, Papadakis MA, Tierney LM. Current Medical Diagnosis and Treatment, 2010, Chapter 13, Blood Disorders)

Q 105.19: A 28-year-old woman who is HIV positive presents with substernal discomfort and painful swallowing for the past week. Her physical examination is unremarkable, but on endoscopy she has extensive adherent yellowish plaques on the esophageal mucosa. What is the most likely diagnosis? A candidal esophagitis B cytomegalovirus (CMV) esophagitis C gastroesophageal reflux disease (GERD) D herpetic esophagitis

The Correct Answer is: A The most common symptoms of infectious esophagitis in immunocompromised persons are dysphagia and odynophagia. Endoscopic evaluation is highly accurate. Candidal esophagitis is characterized by the yellow-white plaques described. CMV esophagitis is characterized by a few large, shallow, superficial ulcerations while herpetic esophagitis has many small deep ones. About half of patients with GERD will have erosions or ulcers distally at the squamocolumnar junction. (Katz and Zolopa, 2009, p. 520) Katz MH , Zolopa AR. HIV infection. In: McPhee SJ , Papadakis MA, eds. Current Medical Diagnosis and Treatment. 48th ed. New York, NY: McGraw-Hill; 2009.

Q 106.12: A 48-year-old woman presents with a chief complaint of gradually progressing difficulty in climbing stairs over the past 3 months. The physical examination shows there is notable proximal muscle weakness of the upper and lower extremities. The remainder of the examination is unremarkable. The laboratory evaluation shows an elevated serum creatinine phosphokinase level, and a muscle biopsy reveals lymphoid inflammatory infiltrates. Which of the following is the appropriate initial treatment of choice in this patient? A prednisone B azathioprine C methotrexate D immunoglobulin E hydrochloroquine

The Correct Answer is: A The most likely diagnosis in this patient is polymyositis. This is supported by the finding of a gradual progressive proximal muscle weakness and elevation of creatinine phosphokinase level. The finding of lymphoid inflammatory infiltrates on muscle biopsy confirms the diagnosis. Initial treatment of choice in this condition is the use of a corticosteroid (prednisone). Patients who do not respond to prednisone may then benefit from the use of methotrexate or azathioprine. Both intravenous immune globulin and hydroxychloroquine are effective for the treatment of patients with dermatomyositis that is resistant to prednisone therapy. (Hellmann and Imboden, 2008, pp. 733-735) Hellmann DB , Imboden JB. Arthritis and musculoskeletal disorders. In: Tierney LM , McPhee SJ , Papadakis MA, eds. 2008 Current Medical Diagnosis and Treatment. New York, NY: McGraw-Hill; 2008:703-756.

Q 66.11: A 32-year-old female is brought into the emergency department by her partner. His report indicates that she had been in her usual state of good health until a couple of days ago. At that time she started to complain of feeling fatigued. She now appears jaundiced and lethargic, and is complaining of chest pain. On exam, her spleen is palpable. Hemoglobin is 6 g/dl and she is Coombs positive. What is the most likely diagnosis? A Autoimmune hemolytic anemia B Glucose-6-phosphate dehydrogenase deficiency C Hereditary spherocytosis D Pyruvate kinase deficiency E Thalassemia

The Correct Answer is: A The onset of autoimmune hemolytic anemia (AHA) is often abrupt and dramatic. Anemia can develop in days, along with jaundice and splenic enlargement. When this triad is present, the suspicion for AHA must be high. The diagnostic test for AHA is the Coombs test. If positive, it confirms the presence of the antibody on the red cells. All other diseases listed are Coombs negative hemolytic anemias. (Fauci, et al, Harrison's Principles of Internal Medicine, 17e, Chapter 101, Hemolytic Anemias and Anemia Due to Acute Blood Loss)

Q 108.5: A 25-year-old female presents with a complaint of dry, stinging hands for the past two months. She has never had any rashes or similar problems. She does not work. The patient stays home to care for her six-month-old baby. She has tried treating her hands with over-the-counter lotions, but reports that they sting upon application. What should appropriate management of this condition include? A triamcinolone 0.025% ointment bid, moisturize with petrolatum frequently, use gloves when hands in water B withhold all treatments for one week and have patient undergo patch testing to determine allergen C punch biopsy at periphery of outbreak, and treat with ketoconazole cream for two weeks D KOH in the office, treat with ketoconazole cream alternating with bactroban cream until resolved

The Correct Answer is: A The patient is experiencing an irritant contact dermatitis, secondary to having her hands in water frequently and using diaper wipes, which can be very irritating due to the alcohol content. The appropriate treatment would consist of reducing the irritant (water and wipes) by using barrier protection (gloves). A mid-potency topical steroid, such as triamcinolone 0.025% ointment twice daily, until the irritation has improved is appropriate treatment. Petrolatum and petrolatum based emollients are best for frequent moisturization. The history given is classic for irritant dermatitis, and withholding treatment for one week along with patch testing is not necessary, unless the patient does not respond to conservative therapy. A biopsy is not indicated, as this is classic irritant dermatitis. Ketoconazole cream is an antifungal medications and bactroban is a topical antibiotic. This patient presentation is not typical for a fungal infection, and should not be treated with an antifungal unless a positive KOH or fungal culture has been done. The bactroban ointment can help prevent a secondary bacterial infection if fissures are present. (Wolff et al., 2009, Page 24)

Q 94.4: A 25-year-old female presents with a complaint of dry, cracking hands for two months. She has never had any rashes or similar problems. She does not work outside the home. She is the mother of a 6-month-old healthy female. Over-the-counter lotions have been tried, but they sting with application. What is the next appropriate step? A treat with triamcinolone 2.5% ointment bid, moisturize with petrolatum, and protect hands from moisture B patch testing to determine allergen C punch biopsy at periphery D KOH test

The Correct Answer is: A The patient most likely has an irritant dermatitis, secondary to increased water exposure, from having an infant. Appropriate treatment includes a topical steroid, like triamcinolone. Ointments are better vehicles than creams, as they penetrate better and moisturize. Using petroleum based moisturizers are more effective than oil based in repairing the epidermis. There is no evidence at this point of an allergen causing the outbreak. If the disorder is resistant to treatment, then patch testing to determine the allergen may be an appropriate step. A punch biopsy is not indicated, unless there is no response to treatment. A KOH test would be indicated if the rash was suspicious for a fungal infection. If the patient does not respond to treatment with a topical steroid, KOH testing may be indicated. (Wolff et al., 2009, Page 24)

Q 86.2: A 29-year-old woman comes in for evaluation of "panic attacks." She has no history of anxiety or depression. She says that during these episodes, which have been getting more frequent and more severe over the past month, she perspires heavily, feels highly anxious ("as though the end is coming"), and as if her heart is "going to jump out of my chest." Recently she has also begun experiencing headaches during these attacks. Her best friend told her that her face gets really "blotchy" during the attacks and then "awfully red" afterwards. Her examination today is remarkable only for blood pressure of 160/100. What is the most sensitive test for diagnosing her condition? A plasma fractionated free metanephrines B serum chromogranin A C serum thyroid stimulating hormone D urine fractionated metanephrine and creatinine E urine toxicology screen

The Correct Answer is: A The plasma fractionated free metanephrine test is the most sensitive test for a pheochromocytoma which this woman's symptoms strongly suggest. Serum chromogranin A (B) is elevated in about 90% of patients and its level correlates with tumor size. Serum TSH (C) would not be appropriate since the symptoms are not as suggestive of hyperthyroidism as they are of pheochromocytoma. A positive plasma test should be followed by a urine fractionated metanephrine test and creatinine level (D). A urine toxicology screen (E) would be appropriate if cocaine use were suspected; however the symptoms more strongly suggest pheochromocytoma. Fitzgerald PA, Endocrine Disorders, in Current Medical Diagnosis and Treatment, 52 nd ed. 2013.

Q 108.2: A 46-year-old female complains of heartburn, steatorrhea, and a 20-lb weight loss. She was diagnosed with a solitary ulcer in the duodenal bulb, but it has been refractory to treatment. Which of the following laboratory findings are most likely to be present in this patient? A Increased fasting serum gastrin B Increased fasting serum glucagon C Increased fasting serum insulin D Increased fasting serum glucose E Increased fasting lipids

The Correct Answer is: A The suspected diagnosis is Zollinger-Ellison syndrome, and the most sensitive and specific method for identifying this syndrome is by the presence of an increased fasting serum gastrin level. A gastrinoma would not cause elevation of any of the other choices listed. (McPhee SJ, Papadakis MA. Current Medical Diagnosis & Treatment, 2010, p. 556)

Q 70.7: A 48-year-old African American male presents with dyspnea, 2-pillow orthopnea, and swelling to his lower legs that has developed over the last month. He also complains of fatigue and decreased exercise tolerance, stating that he has trouble climbing one flight of steps. On physical examination, his blood pressure is 178/98, pulse rate is 102, and respiratory rate is 20. There is 5 cm JVD, crackles at the bilateral lung fields, tachycardia, and an S3 is heard on cardiac auscultation. There is 2+ pitting edema to the lower extremities. His electrocardiogram reveals a sinus tachycardia at a rate of 105 and left ventricular hypertrophy. The chest x-ray reveals cardiomegaly with increased interstitial markings in all lung fields. There is a small right pleural effusion that blunts the costophrenic angle. He is on a thiazide diuretic. On repeat examination, the patient's blood pressure remains high. Which medication should be added to better control the patient's blood pressure? A Lisinopril B Diltiazem C Spironolactone D Amiodarone E Hydralazine

The Correct Answer is: A The use of an ACE inhibitor is a logical second order medication for this patient. Along with the diuretic, the ACE will further lower blood pressure. Calcium channel blockers are not an optimal choice, as they can worsen heart failure. ACE inhibitors can also have a protective effect on renal function, as well as improve morbidity and mortality in diabetics. While hydralazine can also be used for pressure control, ACE inhibitors would be a choice above hydralazine. (Bashore et al., Current Medical Diagnosis and Treatment, Chapter 10)

Q 81.2: A 27-year-old female carries a diagnosis of hereditary spherocytosis. She has been noted to have a few small gallstones that currently do not require a cholecystectomy. While her mean corpuscular hemoglobin concentration is increased, she maintains a hemoglobin level around 10. What is the best way treat her condition? A Annual examinations B Immediate splenectomy C Regular transfusions to maintain HGB above 12.0 D Splenectomy followed by anti-pneumococcal vaccination E Splenectomy with a cholecystectomy

The Correct Answer is: A There is currently no treatment recommended for patients with HS with a compensated anemia. This patient has a slight anemia that is compensated and would not require transfusions, splenectomy, or other treatment or therapy at this time. Annual examinations would be appropriate. (Fauci, et al, Harrison's Principles of Internal Medicine, 17e, Chapter 101, Hemolytic Anemias and Anemia Due to Acute Blood Loss)

Q 116.1: A 48-year-old African American male presents with dyspnea, 2-pillow orthopnea, and swelling to his lower legs that has developed over the last month. He also complains of fatigue and decreased exercise tolerance, stating that he has trouble climbing one flight of steps. On physical examination, his blood pressure is 178/98, pulse rate is 102, and respiratory rate is 20. There is 5 cm JVD, crackles at the bilateral lung fields, and tachycardia and an S 3 is heard on cardiac auscultation. There is 2+ pitting edema to the lower extremities. His electrocardiogram reveals a sinus tachycardia at a rate of 105 and left ventricular hypertrophy. The chest x-ray reveals cardiomegaly with increased interstitial markings in all lung fields. There is a small right pleural effusion that blunts the costophrenic angle. What is the initial medication of choice for treatment of this patient's edema? A Hydrochlorothiazide B Diltiazem C Amiodarone D Terazosin E Metoprolol

The Correct Answer is: A Thiazide diuretics are indicated for the initial treatment of fluid overload related to dilated cardiomyopathy. Calcium channel blockers are to be avoided, as they can worsen heart failure. Amiodarone is utilized for arrhythmic events and not purely for heart failure. (Bashore et al., Current Medical Diagnosis and Treatment, Chapter 10)

Q 121.4: A 26-year-old female presents with a history of a rash around her neck, off and on for several years. It has been treated with mid potency prescription topical steroids, only to recur again. She reports that during treatment she will discontinue wearing jewelry until the rash resolves. The patient complains of pruritis but no other symptoms. What is the most likely diagnosis? A contact dermatitis B atopic dermatitis C herpes zoster D tinea Corporis

The Correct Answer is: A This is a classic contact dermatitis, secondary to nickel allergy. Nickel is a very common metal that is contained in metals, clothing, and jewelry. It is a delayed, cell mediated hypersensitivity reaction, so it takes multiple exposures before an allergic response is exhibited. Atopic dermatitis is usually manifested prior to the age of six. The classic distribution is the flexural surfaces of the extremities. Herpes zoster presents with prodromal neuralgic pain two to three weeks prior to outbreak. The rash has vesicles and erythema in a dermatomal distribution that crust over after three to five days. The pain may last after resolution of the lesions. Presentation of tinea corporis is pruritic, annular scaling patches that enlarge with central clearing. (Wolff et al., 2009, Page 26)

Q 117.2: A 62-year-old female diabetic patient complains of a pruritic rash under her breasts. A physical exam shows an eroded red plaque with satellite papules. What is the most likely cause? A Candida albicans B Staph aureus C group A strep D herpes simplex

The Correct Answer is: A This is consistent with a yeast infection of the skin, caused by Candida albicans. Diabetic patients are particularly susceptible to these infections. A staph infection of the skin will have either honey colored crusting or inflammatory papules and pustules with erythema of the skin. A strep infection of the skin will have inflammatory papules and pustules with erythema of the skin. A herpes simplex infection will have vesicles that crust over after a few days. There is often preceding neurogenic pain prior to the outbreak. (Wolff et al., 2009, Page 720-721)

Q 107.10: A 25-year-old male presents with complaints of a transient lesion in the groin. This has been present off and on for years. He states it is mildly pruritic, and notices it after he has had headaches or mild aches for which he takes acetaminophen. The lesion occurs at the same site each time. What is the most likely diagnosis? A fixed drug eruption B herpes zoster C herpes simplex D tinea cruris

The Correct Answer is: A This lesion is the classic presentation of a fixed drug eruption (FDE). FDEs can be precipitated by numerous drugs, including acetaminophen. Herpes zoster is a unilateral vesicular rash that occurs along a dermatome. There is neurogenic pain prior to and following an outbreak. It is not precipitated by a drug. Herpes simplex is a vesicular rash that crusts over after several days. The outbreak is frequently preceded by neurogenic pain. Tinea cruris is a well demarcated red scaling rash that may have satellite papules which occur in the groin. It is not precipitated by a drug. (Wolff et al., 2009, Pages 567-568)

Q 121.16: In the emergency department, you are asked to evaluate a 77-year-old man with a history of HTN who had a syncopal episode while chasing after his dog. He admits to recent episodes of chest discomfort, also associated with activity, as well as dyspnea at lower levels of activity including walking up one flight of stairs. On physical exam, a grade III/IV crescendo-decrescendo systolic ejection murmur can be heard best over the right upper sternal border. His EKG demonstrates NSR @ 80 bpm, with evidence of left ventricular hypertrophy. His troponin levels are negative for ischemia. What is the next most appropriate test or procedure? A Echocardiography B VQ scan C CT scan of the head D Serum D-dimer levels E MRI of the heart

The Correct Answer is: A This patient exhibits all the signs of progression of aortic stenosis, thus echocardiography is the next most appropriate test. A determination of severity can then be made, with possible cardiac catheterization if severe aortic stenosis is suspected, in preparation for surgical intervention if necessary. A VQ scan is appropriate if pulmonary embolism were suspected. A CT scan of the head could be considered if a head injury was suspected, but would not be the next step in the management of this patient. Serum D-dimer levels might be used to rule out pulmonary embolism, although it is a fairly nonspecific test. An MRI of the heart is not considered standard of care for aortic stenosis. (Crawford et al., 2009, Chapter 7)

Q 86.3: A 33-year-old female presents to the emergency department for the complaint of abdominal pain that has been present for the last month. During her workup, she has an ECG completed. She denies any shortness of breath, palpitations, orthopnea, dyspnea on exertion, syncope, weakness, or headaches. On physical exam, she is alert, awake, and in no distress. Her vital signs are as follows: temperature is 98.9, pulse is 100, respiratory rate is 18, and blood pressure is 132/90. Her HEENT is within normal limits, her neck is supple with a slightly enlarged thyroid that is non-tender without nodules, her lungs are clear, and cardiac is a regularly irregular rhythm. Based on these findings, what is the best therapy for this patient? A No therapy B Ablation therapy C Beta blockers D Digoxin E Verapamil

The Correct Answer is: A This patient has asymptomatic bigeminy. In this case, the patient is otherwise healthy and has no other medical problems. Patients with this presentation require no therapy, and this is an incidental finding on her evaluation. Only monitoring of the patient is considered. (Bashore et al., Current Medical Diagnosis and Treatment, Chapter 10)

Q 95.10: A 54 year old male is undergoing treatment for chronic myelogenous leukemia. On examination today his WBC is increased from 17,000 to 67,000, Hgb decreased from 13 to 9, spleen is now palpable on examination and he complains of abdominal fullness, an increased number of marrow blasts are noted on bone marrow biopsy and he "just doesn't feel good". What is the most likely diagnosis? A Accelerated phase of chronic myelogenous leukemia B Blast phase of chronic myelogenous leukemia C Development of treatment related lymphoma D Molecular transformation phase of chronic myelogenous leukemia E Transformation of chronic myelogenous leukemia to Acute myelogenous leukemia

The Correct Answer is: A This patient has classic signs of transformation from the chronic phase of CML to the accelerated phase. Objective findings to confirm this are increased percentages of blood blasts, thrombocytopenia development, increased basophils, and new clonal cytogenetics abnormalities. Blast crisis is the most severe form of accelerated phase and would indicate the disease is transformed to overt acute leukemia. There is not a phase described as molecular transformation, and the findings are not consistent with a secondary malignancy in this patient. (Lichtman et al., Williams Hematology 8e, Chapter 90, Chronic Myelogenous Leukemia and Related Disorders)

Q 88.1: A patient is undergoing induction treatment for acute myelogenous leukemia. They received daunorubicin and cytarabine seven days ago. They present in the emergency department with a temperature of 38.4°C, HR: 90, RR: 20. They are complaining of cough, fatigue, and diarrhea. They appear pale, fatigued, and slightly tachypenic. Lung fields are clear to auscultation interrupted by cough, and bowel sounds are hyperactive. What is the next best step in caring for this patient? A Obtain blood cultures and start cefepime B Obtain chest x-ray and blood cultures C Obtain CBC, LFTs, and blood cultures D Obtain CT of the chest and admit E Obtain pulse ox and start O2 by nasal cannula

The Correct Answer is: A This patient has developed a neutropenic fever following induction chemotherapy. The most important next step is to obtain blood cultures and start IV antibiotics. Additional steps can include urine and sputum culture. Consideration should be given for both gram positive and negative coverage. The lungs are clear due to the inability to form pus with neutropenia. A chest x-ray or CT may be appropriate, but are not the next best steps. Currently, there is no reason to start oxygen and this would not be the next best step. (Lichtman et al., Williams Hematology 8e, Chapter 89, Acute Myelogenous Leukemia) (Lichtman et al., Williams Hematology 8e, Chapter 22, Treatment of Infections in the Immunocompromised Host)

Q 97.10: A 24-year-old male is brought to the clinic by his mother, who is concerned because her son believes that a local TV news anchor is in love with him. The mother states that this thought has been persistent for the last two to three months, and that he goes around town telling everyone about their relationship; however, she knows that her son does not even know the TV news anchor. The mother notes that this belief has not impaired his daily functioning, but has significantly affected his social functioning. The patient is exhibiting signs most consistent with which of the following psychiatric disorders? A Delusional disorder B Histrionic disorder C Paranoid disorder D Schizoid disorder E Schizotypal disorder

The Correct Answer is: A This patient is exhibiting signs most consistent with delusional disorder. Delusional disorder is a psychosis in which the person has persistent beliefs that are non-bizarre, such as being persecuted, being related to or loved by a well-known person, or that their partner is unfaithful. In this disorder, the delusions tend not to affect the patient's intellectual and occupational activities, but social and marital functioning are significantly affected. Clinical findings of histrionic personality disorder include being dependent, immature, seductive, egocentric, vain, and emotionally labile. Clinical findings of someone who has paranoid personality disorder would include defensiveness, being overly sensitive, secretive, suspicious, and hyper-alert, and having a limited emotional response. Schizoid personality disorder is characterized by shyness, introversion, being withdrawn, and avoiding close relationships. Schizotypal disorder is characterized by being superstitious, socially isolated, and suspicious, with limited interpersonal ability, odd speech, and eccentric behaviors. (McPhee SJ, Papadakis MA. Current Medical Diagnosis & Treatment, 2010, p. 952)

Q 115.17: A 28-year-old male smoker presents with a complaint of numbness and pain in his fingers. He notices this after being exposed to the cold. He states that his fingers appear pale or even blue at times. After warming, his fingers turn red before returning to their normal color. What should be included in appropriate management of this condition? A Counsel the patient to stop smoking B Systemic glucocorticoids C Take aspirin prior to cold exposure D Topical ketoconazole each morning

The Correct Answer is: A This patient is experiencing Raynaud phenomenon. This is digital ischemia that can occur after exposure to cold or emotional stress. It is more common in smokers or patients whose occupation involves using vibratory tools. Management includes patient education to include cold avoidance behavior and wearing loose-fitting clothing. Cessation of smoking is imperative. Drug therapy is used in patients with progressive and severe Raynaud's. (Wolff et al., 2009, Fig. 14-32, pg. 394-395)

Q 63.4: A 77-year-old female has been treated for the last two weeks for a community-acquired pneumonia. While on oral azithromycin, the patient continues to develop fevers, some as high as 103 0 F. Her oral intake has decreased, and her effort to breathe continues to be labored. On examination, the patient continues to have rhonchi and some mild rales that are best appreciated in the anterior right lung region. A follow-up chest x-ray reveals a consolidated infiltrate of the right middle lobe. A CT of the chest reveals a loculated, fluid-filled area of the right middle lobe with no evidence of a foreign body. Based on these new findings, what is the most likely pathogen causing this ongoing infection? A Staphylococcal aureus B Pseudomonas aeurginosa C Hemophilus influenza D Klebsiella pneumoniae E Chlamydia pneumoniae

The Correct Answer is: A This patient is having a history and physical exam that is consistent with an empyema. This loculated collection of fluid will harbor bacteria, the most common pathogen being Staphylococcus aureus. Intravenous antibiotics as well as surgical drainage are warranted.

Q 71.7: A patient who is suspected of having pneumonia produces a sputum sample that grows gram-positive diplococci in chains (see image). What is the best choice of antibiotics for this type of patient? A Penicillins B Cephalosporins C Fluroquinolones D Aminoglycosides E Macrolides

The Correct Answer is: A This patient is having an infection that is consistent with the bacterium Streptococcus pneumoniae. The best and most appropriate treatment for this pathogen is penicillins. (Chesnutt MS, Prendergast TJ. Current Medical Diagnosis and Treatment, 2011, Chapter 9, Pulmonary Disorders)

Q 106.6: An 18-year-old female presents to your office with the complaint of palpitations for the last 2 months. The episodes are frequent and accompanied with lightheadedness and shortness of breath. The patient's mother has taken her pulse when some of the episodes occur and states that the rate gets as high as 170 beats per minute. On exam, she is alert, awake, and oriented. Her resting pulse is 55 and her blood pressure is 122/65. Her lungs are clear throughout, and her cardiac exam revealed a regular rate and rhythm, without murmurs, rubs, or gallops. An ECG is obtained, as shown. Based on her history, physical exam, and ECG, what is the best pharmacologic treatment plan for this patient? A Flecanide B Hydrochlorothiazide C Lisinopril D Adenosine E Digoxin

The Correct Answer is: A This patient is presenting with Wolff-Parkinson-White syndrome, as evidenced by the delta waves on the ECG. These conditions will generally occur in individuals at the onset of early adulthood. Management for this condition pharmacologically includes the use of class IC drugs, such as flecanide. Other choices include procainamide, sotalol, and amiodarone. Digoxin therapy may worsen and widen the QRS complex and place the patient into a ventricular tachycardia. (Calkins H. Hurst's the Heart, Chapter 38, Supraventricular Tachycardia: AV Nodal Reentry and Wolff-Parkinson-White Syndrome)

Q 113.5: Upon examination of a 24-year-old male, you note painless lymphadenopathy along the cervical and femoral chains. He notes that he has had to change his bedding daily because he soaks his sheets from sweat. He has also lost 35 pounds without trying to over the last three months. He thinks that he has had fevers at night but has never checked. This is a classic description of which of the following? A "B" symptoms B Cat scratch fever C Neutropenia D Sickle cell crisis E Sideroblastic anemia

The Correct Answer is: A This patient is presenting with signs and symptoms of non-Hodgkin's lymphoma. Many patients with lymphoma exhibit fever, drenching night sweats, and weight loss—these symptoms are referred to as "B" symptoms. Cat scratch fever usually has painful lymphadenopathy. Sickle cell crisis and sideroblastic anemia do not produce non-painful lymphadenopathy. Neutropenia involves a low white cell count. (McPhee SJ, Papadakis MA, Tierney LM. Current Medical Diagnosis and Treatment, 2010, Chapter 13, Blood Disorders)

Q 91.6: A 59-year-old woman with a known history of rheumatoid arthritis presents with relatively severe complaints of pain, notable bony deformity of the hands with extra-articular findings of cutaneous nodules, scleritis, and pleurisy. On physical examination, the patient is found to have splenomegaly. Which of the following is the most appropriate laboratory evaluation to order to further evaluate the suspected diagnosis? A complete blood count (CBC) B uric acid C C-reactive protein D antinuclear antibodies E erythrocyte sedimentation rate

The Correct Answer is: A This patient presentation of known rheumatoid arthritis with severe deformities, extra-articular findings, and splenomegaly is most likely Felty syndrome. Felty syndrome is characterized by the triad of deforming rheumatoid arthritis, splenomegaly, and neutropenia. The appropriate laboratory test to order would be a CBC to evaluate for neutropenia. Uric acid testing is helpful in evaluating gout but is not relevant to this patient presentation. Ordering an erythrocyte sedimentation rate or C-reactive protein is not necessarily helpful in diagnosing Felty syndrome; in an acute inflammatory flare, both would most likely be elevated. Antinuclear antibodies could be present in 20% to 40% of patients but are not diagnostic of Felty syndrome. (Hellmann and Imboden, 2008, p. 721; Linker, 2008, p. 438) Hellmann DB , Imboden JB. Arthritis and musculoskeletal disorders. In: Tierney LM , McPhee SJ , Papadakis MA, eds. 2008 Current Medical Diagnosis and Treatment. New York, NY: McGraw-Hill; 2008:703-756. Linker CA. Blood disorders. In: Tierney LM , McPhee SJ , Papadakis MA, eds. 2008 Current Medical Diagnosis and Treatment. New York, NY: McGraw-Hill; 2008:422-472.

Q 110.13: A 22-year-old African American male presents to the emergency department with complaints of syncope, which occurred during intense physical exertion. He did not have symptoms prior to exercise, but states that he started having chest pain and shortness of breath right before the syncopal episode. Upon physical examination, he is afebrile, his pulse rate is 93, his respiratory rate is 16, and his blood pressure is 142/100. His lungs are clear, and a cardiovascular examination reveals a bisferiens carotid pulse and a loud S 4 . The electrocardiogram shows a normal sinus rhythm with ventricular hypertrophy, and q-waves in the septal leads. Which of the following tests is most useful to render an accurate diagnosis? A Echocardiography B Holter monitor C Exercise treadmill D Chest x-ray E Electrophysiology study

The Correct Answer is: A This patient presents with a history that is consistent with restrictive cardiomyopathy. While this is not a common diagnosis, it usually presents in younger males who experience symptoms while exerting themselves. Echocardiography is the best assessment test to determine wall motion and thickness of the myocardium. It can also assess any valvular disorders or areas of decreased wall motion abnormalities. (Bashore et al., Current Medical Diagnosis and Treatment, Chapter 10)

Q 116.16: A 48-year-old man presents to the emergency department with acute right upper quadrant tenderness, fever, and mild jaundice. Which of the following is most likely to be elevated in the blood? A bilirubin B creatinine C glucose D ketones E uric acid

The Correct Answer is: A This patient's signs and symptoms correlate with a suspected case of cholecystitis. Jaundice is associated with hyperbilirubinemia, in which the excess bilirubin can deposit in tissues such as the skin, sclera, and nails, causing a yellowish discoloration. Bilirubin is the waste product generated from the metabolism of hemoglobin. (Greenberger and Paumgartner, 2008, pp. 1995-1996; Pratt and Kaplan, 2008, p. 261) Greenberger NJ , Paumgartner G. Diseases of the gallbladder and bile ducts. In: Fauci AS , Braunwald E , Kasper DL, et al., eds. Harrison's Principles of Internal Medicine. 17th ed. New York, NY: McGraw-Hill Medical; 2008. Pratt DS , Kaplan MM. Jaundice. In: Fauci AS , Braunwald E , Kasper DL, et al. eds. Harrison's Principles of Internal Medicine. 17th ed. New York, NY: McGraw-Hill Medical; 2008.

Q 118.2: A 29-year-old woman comes to the office because she "just keeps gaining weight and can't stop." Since she was last seen in the office at age 24, she has gained nearly 100#. Physical examination is remarkable for blood pressure of 140/92. She appears depressed. Her trunk and abdomen are heavy with normal-sized extremities. Her facial hair is dark and in a "male" distribution. Large dark violaceous striae are present on her abdomen and proximal extremities. Which of the following additional findings is most consistent with this presentation? A Buffalo hump B Doughy, thickened skin C Exophthalmos D Lid lag E Thickened tongue

The Correct Answer is: A This woman appears to have Cushing Syndrome (hypercortisolism) which is characterized, in addition to the signs listed above, by plethoric facies, supraclavicular fatpads, and the so-called "buffalo hump." Doughy, thickened skin (B) and thickened tongue (E) may be found in hypothyroidism, exophthalmos (C) in Graves disease, and lid lag (D) in hyperthyroidism from any cause. Fitzgerald PA, Endocrine Disorders, in Current Medical Diagnosis and Treatment, 52 nd ed. 2013.

Q 95.1: Which of the following medications is NOT recommended by the American Neurological Association based upon Level I evidence to treat severe and unrelenting neuropathic pain because of significant side effect potential? A amitryptyline B duloxetine C gabapentin D methadone E pregabalin

The Correct Answer is: A Tricyclic antidepressants (TCAs) may be utilized to treat neuropathic pain. However, amitryptyline has relatively higher adverse events, particularly in the elderly population, especially when compared to secondary amine TCAs, such as desipramine. (Fauci et al., 2008, Chapter 12)

Q 97.9: Of the following choices, which regimen is considered first-line therapy for Helicobacter pylori-positive individuals with peptic ulcer disease? A omeprazole & clarithromycin & amoxicillin B omeprazole & ranitidine & clarithromycin C esomeprazole & clarithromycin & ampicillin D ranitidine & amoxicillin & bismuth subsalicylate E misoprostol & clarithromycin & metronidazole

The Correct Answer is: A Triple-therapy regimens consisting of a proton pump inhibitor (PPI) and two antibiotics are considered first-line therapy for the eradication of Helicobacter pylori. PPI-based regimens that combine clarithromycin and amoxicillin or clarithromycin and metronidazole have been shown to have the most effective eradication rates. There are also 4-drug regimens that include bismuth subsalicylate that have been shown to be effective as well. Because of lower eradication rates, it is not recommended that histamine receptor antagonists like ranitidine be substituted for a PPI. Misoprostol is used for reducing the risk of nonsteroidal anti-inflammatory agent (NSAIA)-induced gastric ulcer in patients at high risk of developing complications from these ulcers and in patients at high risk of developing gastric ulceration. It has no effect on H pylori eradication. (Berardi and Welage, 2008, pp. 577-578) Berardi RR , Welage LS. Peptic ulcer disease. In: DiPiro JT , Talbert RL , Yee GC, et al., eds. Pharmacotherapy: A Pathophysiologic Approach. 7th ed. New York: McGraw-Hill; 2008.

Q 114.10: A 78-year-old male presents to the office due to increasing exertional dyspnea and cough for the past week. Physical exam reveals an S3 gallop, mild JVD, and 2+ pitting edema of the bilateral lower extremities. The patient has had mild congestive heart failure in the past. A chest x-ray reveals prominent pulmonary vasculature without any additional complications. Which of the following is released by the ventricular myocardium in response to elevated ventricular pressure and overload? A B-Type natriuretic peptide (BNP) B Creatine kinase MB (CK-MB) C Plasma d-dimer D Total creatine kinase (CK) E Troponin

The Correct Answer is: A Two markers, B-type natriuretic peptide (BNP) and N-terminal pro-BNP, provide representation of the ventricular response to volume and pressure overload, and are elevated in hypervolemic states. These markers provide diagnostic information, including differentiating dyspnea causes. They may also be used to monitor CHF patient prognosis and progression. Increased levels of the markers, although sensitive, are not necessarily specific for CHF, and can be elevated in multiple disease processes and patient populations, such as the elderly and women. (McPhee SJ, Papadakis MA. Current Medical Diagnosis & Treatment 2011, Chapter 10, Heart Disease)

Q 63.10: A 25-year-old woman presents not feeling well 1 week after returning from a trip to central Africa. She has had steadily increasing fever, abdominal distention, and diarrhea. She also has rashes on her abdomen, chest, and back, which are characterized by 3-mm pink papules, which blanch with pressure. Heart rate is 60 beats/min. Blood culture is positive but final identification is pending. Most likely diagnosis is A typhoid fever B yellow fever C malaria D hepatitis E shigellosis

The Correct Answer is: A Typhoid fever is caused by Salmonella typhus. It is contracted by contaminated food or water. There are several endemic areas throughout Africa. Symptoms and signs may be nonspecific but often include blanchable, pink, papular rash over the trunk and fever that increases in stepwise fashion. Blood culture is positive in 80% of cases in the first week. Abdominal symptoms may include distention and constipation, initially, followed by diarrhea and, possibly, splenomegaly. Prevention is accomplished by multidose oral or single-dose vaccine.

Q 70.4: A 75-year-old woman, mother of four, presents to your office to establish care. Appearing healthy, she reports a past medical history positive for hypertension and denies any additional problems. However, when specifically asked she admits to having urinary incontinence for "a couple of years" and now describes symptoms that have recently worsened, with the patient experiencing the need to void almost hourly. These desires to urinate are so severe that she is now using four to five adult incontinence pads per day to manage the urine she leaks. What is the most likely diagnosis? A urge incontinence B stress incontinence C overflow incontinence D functional incontinence

The Correct Answer is: A Urinary incontinence is defined as involuntary urine loss. Urge incontinence is the result of uninhibited urge sensations that are so strong that the patient experiences an involuntary urine loss. Women particularly experience this problem with the changes associated with aging (weakened pelvic muscles secondary to childbirth as well as estrogen depletion causing weakening of the detrusor muscle). The problem may be worsened by the use of diuretics to treat hypertension. Stress incontinence is associated with increases in intra-abdominal pressure (laughing, sneezing, coughing, etc.). Overflow incontinence is associated with leaking small amounts of urine from mechanical factors that affect an already distended bladder. Functional incontinence is associated with patients who exhibit cognitive impairment (severe dementia). (Johnston et al., 2009, pp. 66-68) Johnston CB , Harper GM , Landefeld CS. Geriatric disorders. In: McPhee SJ , Papadakis MA, eds. Current Medical Diagnosis and Treatment. 48th ed. New York, NY: McGraw-Hill; 2009:56-71.

Q 63.8: What is the most common cause of restrictive cardiomyopathy? A Amyloidosis B Pericarditis C Marfans syndrome D Fatty infiltrative disease E Sarcoidosis

The Correct Answer is: A While restrictive cardiomyopathy is seen in such cases as hemochromatosis, glycogen deposition, endomyocardial fibrosis, sarcoidosis, hypereosinophilic disease, and scleroderma, amyloidosis is the most common cause among the choices provided.

Q 77.3: A 62 year old male presents with a concern regarding the numerous brown, warty looking papules on his back. They are asymptomatic but sometimes get caught on clothing which can cause irritation and bleeding. What is the most likely diagnosis? A Seborrheic keratosis B Melanocytic nevus C Verruca vulgaris D Malignant melanoma

The Correct Answer is: A Seborrheic Keratoses are described as brown warty looking papules that have a "stuck on" appearance. They are more common in patients over 50 years old. (Wolff & Johnson, pg 215) (Fig 9-49a & c, Wolff K, Johnson RA: Fitzpatrick's Color Atlas & Synopsis of Clinical Dermatology, 6 th ed: http://www/accessmedicine.com)

Q 77.6: A 34-year-old male has a one and one-half day history of fever, chills, a non-productive cough, and malaise. He is otherwise healthy with no long-standing medical history, and is taking no chronic medications. On examination, the patient has a temperature of 101.3 0 F, BP 123/63, P 78, R 18. His HEENT reveals mild rhinorrhea, moist mucous membranes, clear lung sounds, and a regular rate and rhythm. The rapid nasal viral test for influenza B is positive. Based on this information, what is the medication treatment for this patient? A Amantadine B Oseltamavir C Ramantadine D Famciclovir E Azithromycin

The Correct Answer is: B Oseltamivir is the best antiviral medication for the treatment of acute influenza. This medication is ideally started within the first onset of illness, usually within the first 24 to 36 hours. Amantadine and Ramantidine have been shown to not be effective, and there is growing resistance to the medication. (Shandera WX, Patel S. Current Medical Diagnosis and Treatment, 2011, Chapter 32, Viral and Riskettsial Infections)

Q 77.5: A 72-year-old farmer presents with multiple rough adherent scaly lesions on his scalp. They are better felt than seen. He reports mild pain when he inadvertently scratches them. What would the most effective management include for this patient? A shave excision B 5-FU cream 5% C doxycycline D no treatment is required

The Correct Answer is: B These lesions are consistent with actinic keratoses. Treatment is indicated, as 10% of these lesions progress to squamous cell carcinoma. First line treatment is 5-Fu cream 5%. Excision is not indicated for actinic keratoses. Doxycycline is an antibiotic used to treat acne and various bacterial infections, and it is not an effective treatment for actinic Keratosis. (Wolff et al., 2009, Page 268)The Correct Answer is: B These lesions are consistent with actinic keratoses. Treatment is indicated, as 10% of these lesions progress to squamous cell carcinoma. First line treatment is 5-Fu cream 5%. Excision is not indicated for actinic keratoses. Doxycycline is an antibiotic used to treat acne and various bacterial infections, and it is not an effective treatment for actinic Keratosis. (Wolff et al., 2009, Page 268)

Q 62.2: A 45-year-old female presents with a sudden onset of vertigo, nausea, and vomiting. Upon physical exam, you note that she is holding on to the rails of the bed, and her pain gets worse when you attempt any movement of her head. Neurologic exam is grossly normal. Which combination of the following medications is indicated to treat the patient's symptoms? A Hydrochlorothiazide, lorezapam, and gentamycin B Lorezapam, prochlorperazine, and diphenhydramine C Scopolamine, aspirin, and cisplatin D Metoclopromide, hydrochlorothiazide, and cyclobenzaprine E Diazepam, hydrocodone, and hydrochlorothiazide

The Correct Answer is: B *All other combinations include an ototoxic medication: furosemide, gentamycin, aspirin, and cisplatin.* Treatment of acute vertigo is more effective using a combination of vestibular suppressants (benzodiazepines), anti-emetics (prochlorperazine), and anticholinergics (diphenhydramine or scopolamine).

Q 99.1: A 45-year-old male presents with a non-tender nodule protruding from his lower eyelid. There is some surrounding erythema to the conjunctiva, but no discharge is seen. He states that it has been there for one month. He has no visual problems. What is your diagnosis? A Blepharitis B Chalazion C Dacryocyctitis D Hordeolum E Conjunctivitis

The Correct Answer is: B A chalazion is a sterile, chronic, and non-painful granulomatous nodule, caused by a previous acute infection in a meibomian gland. It can develop over a period of a few weeks. Treatment is intralesional steroids or surgical curettage. (Riordan et al., 2008, Chapter 4)

Q 110.1: In a patient who does not have thyroid disease, an elevated serum thyroid stimulating hormone (TSH) may be found under which of the following conditions? A acute corticosteroid administration B acute psychiatric admission C development of an hCG-secreting tumor D pregnancy E use of amphetamines

The Correct Answer is: B About 15% of patients who are admitted for an acute psychiatric illness will have an elevated TSH in the absence of thyroid disease. Acute corticosteroid administration (A), hCG-secreting tumors (C), pregnancy (D), and use of amphetamines (E) are all associated with low TSH levels. Fitzgerald PA, Endocrine Disorders, in Current Medical Diagnosis and Treatment, 52 nd ed. 2013.

Q 95.5: A 30-year-old man presents complaining of back pain radiating down his right leg. On examination, you note that his knee jerk reflex is absent on the right. This finding suggests compression of which spinal nerve root? A L1-L2 B L3-L4 C S1-S2 D T11-T12 E C5-C6

The Correct Answer is: B Absence of the knee jerk reflex suggests compression of the L3-L4 spinal nerve root. The four most commonly tested deep tendon reflexes are the Achilles (ankle jerk) reflex, quadriceps (knee jerk) reflex, triceps reflex, and the biceps reflex. The nerve roots that each tests in ascending order are 1 and 2, 3 and 4, 5 and 6 (biceps), and 7 and 8 (triceps). One only needs to remember that the ankle jerk is a sacral nerve root, the knee jerk is a lumbar nerve root, and the biceps and triceps are cervical nerve roots. (Aminoff et al., 2005, p. 367) Aminoff MJ , Greenberg DA , Simon RP. Clinical Neurology. 6th ed. New York, NY: McGraw-Hill; 2005.

Q 112.1: A 35-year-old female presents with persistent erythema, with telangiectasias on the central portion of her face. She also experiences exacerbations of red papules and small pustules. What is the most likely diagnosis? A acne vulgaris B acne rosacea C periorificial dermatitis D systemic lupus erythematosus

The Correct Answer is: B Acne rosacea is differentiated from acne vulgaris by the absence of comedonal lesions. The hallmark of rosacea is inflammatory lesions and telangiectasias distributed in the central face. In addition, flushing is common in response to change in temperature, spicy foods, and stress. Acne vulgaris consists of comedonal lesions and inflammatory papules and pustules. There is no association of telangiectasias with acne vulgaris. This is an inflammatory condition, with a slight preponderance in females. It consists of small, red papules located around the mouth and sometimes adjacent to the nose. The rash from systemic lupus erythematosus is described as well defined erythema in the malar distribution. There may be fine scaling, erosions, or crusts. (Wolff et al., 2009, Page 9)

Q 117.12: Which of the following is the drug of choice for treating herpes simplex virus (HSV) types 1 and 2? A amantadine B acyclovir C zidovudine D nystatin E zanamivir

The Correct Answer is: B Acyclovir is the treatment of choice for HSV disease, typically in oral doses of 200 mg five times daily or 400 mg three times daily. In situations where oral acyclovir cannot be absorbed effectively by the GI tract or tolerated by the patient, intravenous acyclovir can be administered at a rate of 15 mg/kg/day. (Knodel, 2008, p. 1926) Knodel LC. Sexually transmitted diseases. In: DiPiro JT , Talbert RL , Yee GC, et al., eds. Pharmacotherapy: A Pathophysiologic Approach. 7th ed. New York: McGraw-Hill; 2008.

Q 99.8: Which of the following are common adverse effects associated with aminoglycosides? A diarrhea and bone marrow depression B ototoxicity and nephrotoxicity C blurred vision and hyperglycemia D headache and hypoglycemia E rash and dyspepsia

The Correct Answer is: B All aminoglycosides are ototoxic and nephrotoxic. The likelihood of experiencing these toxicities occurs when treatment lasts beyond 5 days, at higher doses, in elderly patients, and those suffering from renal insufficiency. Other agents that produce either of these toxicities should not be used concurrently. (Craig and Stitzel, 2004, pp. 541-542) Craig CR , Stitzel RE. Modern Pharmacology with Clinical Applications. 6th ed. Baltimore, MD: Lippincott Williams & Wilkins; 2004.

Q 70.9: Your patient has symptoms consistent with perennial allergic rhinitis, and after performing a history and physical examination, you elect to perform an IgE-specific serum antibody test for both food and respiratory allergens. The results return and the patient does not have an IgE positive response to a single allergen tested, yet the patient's total serum IgE is elevated dramatically. What would be the most appropriate next step in the diagnosis and treatment of this patient? A Repeat the test—it is clear that an error was made or samples were switched. B Perform a more detailed history carefully looking for the offending allergen(s) and expand the serum IgE-specific antibody testing to include additional potential triggers. C Perform the more sensitive serum IgM-specific antibody testing. D Refer the patient to an allergist for definitive skin testing. E Clearly this is one of the many forms of nonallergic rhinitis; treat the patient accordingly.

The Correct Answer is: B Allergen-specific serum IgE testing is an easy and accurate method for determining the presence of atopic allergy, and with newer in vitro technology available, in vitro testing is at least equivalent to skin testing in efficacy. In vitro assays are safe, specific, cost-effective, and reproducible, and do not require the patient to be free of antihistamines and other medications that may interfere with skin testing. They are also easy and quick and are therefore preferred, especially in children and in anxious patients. Although the original in vitro assay, the RAST test (radioallergosorbent test), is no longer performed, its name is still used today to generally describe IgE-specific blood testing. However, not all in vitro assays available today are alike. The newer assays tend to be faster, more reliable, and more efficient than previous tests. The ImmunoCAP is an excellent example of this newer technology. Not using a reliable assay may affect the diagnosis of atopy and therefore the prescribing of appropriate therapy In vitro testing can be cost-effective if an appropriately chosen inhalant screening battery of 10 to 12 allergens consisting of the most prevalent pollens, molds, dust mites, and animals in the local environment is used. In children, common allergenic foods are substituted or added. No further testing is necessary if this battery is negative. If the screening battery is positive and if no immunotherapy is considered, additional allergy testing can be performed. (Lalwani, 2008, Chapter 13)

Q 109.5: Which of the following clinical manifestations is most characteristic of polymyalgia rheumatica (PMR)? A subcutaneous inflammatory lesions B pain and stiffness of proximal muscle groups C insidious onset of symmetrical joint involvement D widespread musculoskeletal pain and tender points E symmetrical weakness initially in the legs that progresses caudally

The Correct Answer is: B An abrupt onset of proximal muscle pain and stiffness in the shoulder and pelvic girdle areas, usually associated with fever, malaise, and weight loss, is characteristic of polymyalgia rheumatica. Subcutaneous inflammatory lesions denote erythema nodosum. These lesions are associated with pregnancy and several systemic disorders, such as sarcoidosis, tuberculosis (TB), and streptococcal infections. Insidious onset of symmetrical joint involvement is most commonly associated with rheumatoid arthritis. Widespread musculoskeletal pain and tender points, referred to as "trigger points," are seen with fibromyalgia syndrome. Trigger points may be found anywhere on the body but are most common in the neck, shoulders, hands, low back, and knees. Symmetrical weakness initially in the legs that progresses caudally is characteristic of Guillain-Barré syndrome. (Hellmann and Imboden, 2008, p. 739) Hellmann DB , Imboden JB. Arthritis and musculoskeletal disorders. In: Tierney LM , McPhee SJ , Papadakis MA, eds. 2008 Current Medical Diagnosis and Treatment. New York, NY: McGraw-Hill; 2008:703-756. Explanation

Q 111.8: The diagnosis of systemic lupus erythematosus (SLE) is supported by a positive initial antibody screen; however, this test is not specific. Which of the following tests is most specific in the diagnostic evaluation of SLE? A gliadin antibody B antibody to double-stranded DNA (anti-dsDNA) C antinuclear antibody (ANA) D anticentromere antibody E antiribosomal P antibody

The Correct Answer is: B Autoantibody production is the primary immunological abnormality seen in patients with systemic lupus erythematosus (SLE); the antinuclear antibody (ANA) is most characteristic of SLE and seen in 95% of patients with SLE but is not specific for the diagnosis of SLE. A positive ANA can also be found in patients with lupoid hepatitis, scleroderma, rheumatoid arthritis, Sjögren disease, dermatomyositis, and polyarteritis. ANA testing should be employed as the initial screening test in a patient suspected of having SLE. A negative total ANA test is strong evidence against the diagnosis of SLE, whereas a positive test is not confirmatory of the diagnosis. The most specific antibody tests for SLE are antibodies to double-stranded DNA (anti-dsDNAs) and anti-Smith (anti-SM). Although these tests are more specific for SLE, they are less sensitive than the ANA test. Anti-dsDNA is positive in 60% of patients with SLE and anti-SM is positive in 30% of patients. Anti-dsDNA is more likely to reflect disease activity. Gliadin antibody assay is utilized to assess patients with suspected celiac disease. Anticentromere antibody is associated with CREST (calcinosis, Raynaud phenomenon, esophageal dysmotility, sclerodactyly, and telangiectasia) syndrome in scleroderma. Antibodies to ribonucleoprotein are present in patients with a mixture of overlapping rheumatological symptoms known as "mixed connective tissue disease." (Hellmann and Imboden, 2008, pp. 725-729) Hellmann DB , Imboden JB. Arthritis and musculoskeletal disorders. In: Tierney LM , McPhee SJ , Papadakis MA, eds. 2008 Current Medical Diagnosis and Treatment. New York, NY: McGraw-Hill; 2008:703-756.

Q 120.2: A patient is found with a microcytic anemia. Further laboratory findings include a serum iron value of 26 mcg/dl (50 to 175 mcg/dl), a total iron binding capacity of 376 mcg/dl (250 to 370 mcg/dl), a lactic dehydrogenas of 143 U/L (100 to 190 U/L) and a creatinine level of 0.8 mg/dl (0.1 to 1.5 mg/dl). Which of the following is the best recommendation for therapy? A Ascorbic acid 250 mg po daily B Ferrous sulfate 325 mg po three times daily C Glucophage 500 mg po twice daily D Sodium ferric gluconate 1.5 GMs IM monthly E Vitamin B12 1000 mcg po daily

The Correct Answer is: B Based on laboratory values, this patient has iron deficiency anemia. The most important part of treatment is identifying the cause of the blood loss. Therapy consists of replacement of iron through the use of ferrous sulfate. If the patient is unable to tolerate oral iron dosing, IM sodium ferric gluconate may be used, but only after intolerance of po dosing is demonstrated. Glucophage is used for diabetes. Vitamin B12 is used in vitamin B12 anemia (a macrocytic anemia). Ascorbic acid (vitamin C) is used to increase iron absorption if needed. (McPhee SJ, Papadakis MA, Tierney LM. Current Medical Diagnosis and Treatment, 2010, Chapter 13, Blood Disorders)

Q 118.10: A 42-year-old male presents to the office after awakening with left facial droop, slurred speech, mild left ear pain, and drooling from the left side of his mouth. On physical exam, left facial droop is present, along with the inability to fully close the left eye or raise the left eyebrow. There are no other findings on physical exam. Which of the following is the most likely diagnosis? A Acute transient ischemic attack B Bell palsy C Encephalitis D Intracranial neoplasm E Temporal arteritis

The Correct Answer is: B Bell palsy is classically described as a sudden onset of symptoms involving a unilateral facial nerve, with resultant paralysis. The full face is involved on the affected side, unlike some strokes. The differential diagnoses of stroke, tumor, and various infections must be considered. (McPhee SJ, Papadakis MA. Current Medical Diagnosis & Treatment 2011, Chapter 24, Nervous System Disorders)

Q 86.7: As a diver descends for a deep water dive, at about 10 feet of depth he begins to feel nausea, severe ear pain, and develops vertigo and vomiting. What is the most likely cause of his symptoms? A Decompression sickness B Decreasing pressure in the middle ear C Benign paroxysmal positional vertigo D Increasing pressure in the middle ear E Equalization of pressure between the middle ear and eustachian tube

The Correct Answer is: B Boyle's law states that as a diver descends, the increasing external pressure causes an equal decrease in pressure in the middle ear, which must be equalized during the descent. If the middle ear pressure is not equalized, the tympanic membrane becomes severely retracted, due to the negative middle ear pressure. This can result in hemotympanum, hemorrhage, or tympanic membrane perforation. Ascent causes increased pressure in the middle ear as the external pressure is decreased. Equalization techniques must also be used to prevent a tympanic membrane perforation. Decompression sickness occurs on ascent, when nitrogen gas bubbles are forced into the middle ear, and vascular and lymphatic spaces. (Lalwani A.K., 2008, Chapter 57)

Q 117.11: A 57-year-old woman with a history of rheumatic fever is seen complaining of dyspnea while vacuuming her apartment, which has been worsening over the last few months. On physical exam, a possible opening snap, loud S 1 , and a very soft diastolic rumbling murmur is auscultated. Which of the following maneuvers would be the most appropriate to choose to increase the intensity of the murmur for better identification? A Isometric hand grip exercise B Listening with the bell at the apex, with the patient in the left lateral decubitus position C Inspiration, followed by the patient holding his/her breath D Valsalva maneuver E Having the patient lie flat with the knees bent

The Correct Answer is: B Choice B is best used when listening to the murmur of mitral stenosis, which is the murmur auscultated in this patient. Choice A, isometric hand grip exercises, increase the intensity of the murmur of mitral regurgitation by increasing arterial and left ventricular pressure, which increases the flow across the mitral valve, thereby increasing the murmur's intensity. Choice C will increase the AP diameter, making it more difficult to hear the murmur. With the Valsalva maneuver, choice D, the murmur decreases in intensity. Choice E is the best position for the abdominal exam, especially in males. (Crawford et al., 2009, Chapter 1)

Q 89.4: A 55-year-old woman with a history of emphysema, who is undergoing chemotherapy for lung cancer, is sent to see you by her oncologist regarding a sudden increase in dyspnea, with exertion and fatigue. On physical exam, pulsus paradoxus is noted. Which of the following is the most likely diagnosis given the patient's physical exam findings? A Aortic stenosis B Cardiac tamponade C Mitral regurgitation D Hypertrophic cardiomyopathy E Atrial fibrillation

The Correct Answer is: B Choice B is the best choice, as pulsus paradoxus is frequently seen in patients with a pericardial effusion, which may lead to cardiac tamponade. Patients diagnosed with cancer, particularly of the lung and breast, may accumulate fluid within the pericardial sac, leading to cardiac tamponade. The finding of pulsus paradoxus is defined as a decrease in systolic arterial pressure of greater than 10 mmHg. It is an accentuation of the normal decrease in systolic arterial pressure of less then 10mm Hg that normally accompanies inspiration. Pulsus paradoxus is not typically seen in the conditions listed in choices A, C, D, and E. (LeBlond et al., 2009, Chapter 8)

Q 110.19: A 57-year-old man with a history of HTN, hyperlipidemia, and chronic tobacco use, presents to the office with complaints of chest tightness that occurs every time he begins raking leaves. If he stops and rests, it is relieved within 5 minutes. He has no associated nausea or diaphoresis, but does admit to associated dyspnea. Which of the following is the most appropriate next step in the management of this patient? A Cardiac catheterization B Exercise nuclear stress test C Holter monitor D Tilt table test E Transesophageal echocardiogram

The Correct Answer is: B Choice B, exercise nuclear stress testing, would provide information regarding exercise tolerance and exercise-induced dysrhythmias, as well as information regarding myocardial ischemia. Choice A, cardiac catheterization, would be utilized in patients diagnosed with acute myocardial infarction or after a stress test suspicious for myocardial ischemia. Choice C, Holter monitor, is a useful diagnostic tool for the evaluation of patients with palpitations occurring on a daily basis. Choice D, tilt table testing, is utilized in evaluation of patients suffering from near-syncope or syncope. Choice E, transesophageal echocardiogram, is helpful in more direct visualization of heart valves, especially when transthoracic echocardiogram is unclear. (Fauci et al., 2008, Chapter 238)

Q 84.1: A 22-year-old woman with a history of scoliosis presents to the office with complaints of a retrosternal chest discomfort, occurring frequently at rest and lasting for several hours at a time. On physical exam, a mid-systolic click is noted. Which of the following maneuvers would be the most appropriate to choose for better identification of the murmur? A Left lateral decubitus position listening with the bell of the stethoscope B Standing C Seated leaning forward D Inspiration E Expiration

The Correct Answer is: B Choice B, standing, will cause the mid-systolic click associated with mitral valve prolapsed to move toward S1 or become more audible. Choice A, rolling the patient to the left lateral decubitus position, is most useful in identification of the murmur of mitral stenosis. Choices C, D, and E have no effect on the mid-systolic click. Inspiration increases the intensity of the murmurs associated with tricuspid stenosis and regurgitation. (Fauci et al., 2008, Chapter 230)

Q 119.2: A 66-year-old man with a history of a cardiac murmur since childhood presents with complaints of increasing dyspnea while walking up one flight of stairs and increased lower extremity edema. On physical examination, a late-peaking crescendo-decrescendo murmur, preceded by a systolic ejection click, is noted. An S4 gallop is appreciated. Hepatomegaly and splenomegaly are appreciated. An EKG demonstrates right ventricular hypertrophy, and no acute ST or T wave changes. Which of the following is the most appropriate next diagnostic study? A Chest x-ray B Transthoracic echocardiogram C Holter monitor D Treadmill exercise stress test E Cardiac catheterization

The Correct Answer is: B Choice B, transthoracic echocardiogram, is a simple, sensitive and non-invasive diagnostic tool, which can evaluate for the presence of valvulopathy. However, in patients with pulmonic stenosis, it offers limited direct visualization of the pulmonic valve, and although it is the most appropriate next diagnostic study it is usually followed by other diagnostic procedures, such as transesophageal echocardiogram, which offer better visualization of the pulmonic valve directly. Choice A might be able to give evidence of cardiomegaly, or calcification of heart valves, but would not be sensitive enough to detect the degree of valvulopathy if present. Choice C is a useful diagnostic tool for the evaluation of patients complaining of palpitations, but is incorrect for this patient who has no symptoms of palpitations. Choice D, although a useful diagnostic tool for the evaluation of exercise tolerance and in patients complaining of chest pain, does not allow direct visualization of the heart valves to evaluate the degree of pulmonic stenosis. Choice E, cardiac catheterization in patients demonstrating severe pulmonic stenosis, is not only diagnostic, but also therapeutic, as percutaneous balloon valvuloplasty is the preferred method of treatment for critical pulmonic stenosis. (Crawford et al., 2009, Chapter 28)

Q 80.5: A 24-year-old HIV-positive man comes to the emergency department complaining of severe left-sided chest discomfort, which radiates through to the left trapezius region. On coming into the room, you note that he is sitting up and hunched forward. On physical examination, the patient's blood pressure is 135/78, with a pulse of 85 bpm, and a pericardial friction rub is noted. Laboratory findings demonstrate elevated serum creatine kinase levels and normal serial troponin levels. His EKG demonstrates peaked T waves. His CXR demonstrates a "water bottle" cardiac silhouette. Which of the following diagnostic studies would be considered the most appropriate next step in management of this patient? A Cardiac catheterization B Transthoracic echocardiography C CT of the thorax D VQ scan E Lower extremity venous doppler

The Correct Answer is: B Choice B, transthoracic echocardiography, would allow for monitoring of a patient with acute pericarditis, to determine if a pericardial effusion and/or cardiac tamponade develops. Choice A, cardiac catheterization, would be appropriate in a patient with acute myocardial infarction. Choice C, CT of the thorax, is not as effective or specific as transthoracic echocardiography for the development of a pericardial effusion and/or tamponade. Choices D and E would be appropriate diagnostic studies if pulmonary embolism is suspected, but not as the next step in management of this patient with pericarditis. (Fauci et al., 2001, pp. 1365-1366)

Q 69.6: What is the most common pulmonary complaint of a patient with tuberculosis? A Hemoptysis B Chronic cough C Wheezing D Dyspnea E Chest pain

The Correct Answer is: B Chronic cough is the most common complaint. Hemoptysis can occur but is rare, and a productive cough becomes more prevalent after the chronic cough has been established for a while. (Chesnutt MS, Prendergast TJ. Current Medical Diagnosis and Treatment, 2011, Chapter 9, Pulmonary Disorders)

Q 110.6: Your 25-year-old female patient is a smoker, takes oral contraceptives, and complains of shortness of breath and wheezing, which forced her to stop smoking less than a week ago. She has no cough and her lungs are clear on your examination. Her vital signs are as follows: Pulse 72, respirations 14, blood pressure 115/70 mm&thinsp;Hg, and her pulse oximetry is 94%, and her height is 64 inches. In an effort to distinguish between various pathologies, you order spirometry followed by a beta2-agonist nebulizer treatment, and then after 10 minutes a repeat spirometry. Her repeat spirometry FEV 1 improves by 225 ml which is approximately 16% and from this you tell her that you are diagnosing her with which of the following? A acute exacerbation of chronic bronchitis B asthma C chronic obstructive pulmonary disease D hyperventillation syndrome E pulmonary embolism

The Correct Answer is: B Clinicians are able to identify airflow obstruction on examination, but they have limited ability to assess it or to predict whether it is reversible. The evaluation for asthma should include spirometry (FEV 1 , FVC, FEV 1 /FVC) before and after the administration of a short-acting bronchodilator. These measurements help determine the presence and extent of airflow obstruction and whether it is immediately reversible. Airflow obstruction is indicated by a reduced FEV 1 /FVC ratio. Significant reversibility of airflow obstruction is defined by an increase of 12% and 200 mL in FEV 1 or 15% and 200 mL in FVC after inhaling a short-acting bronchodilator. A positive bronchodilator response strongly confirms the diagnosis of asthma but a lack of responsiveness in the pulmonary function laboratory does not preclude success in a clinical trial of bronchodilator therapy. Severe airflow obstruction results in significant air trapping, with an increase in residual volume and consequent reduction in FVC, resulting in a pattern that may mimic a restrictive ventilatory defect. (McPhee and Papadakis, 2011, Chapter 9)

Q 88.2: A 48-year-old male presents with an 8-day history of productive cough, subjective fevers, and malaise. He is otherwise healthy without any active medical problems. He is a social drinker of alcohol, and denies any tobacco or drug use. On physical examination, the patient is alert and oriented. His temperature is 100.4 0 F, pulse rate is 56, respiratory rate is 18, and blood pressure is 133/64. HEENT is within normal limits and a chest exam has diffuse expiratory wheeze with decreased sounds to the right lower lung fields. Blood labs reveal WBC 14.4, Hgb 11.3, Plt 233, ALT 65, AST 102, and PO 4 2.1. A chest x-ray reveals a dense consolidation with bulging fissures. Based on these findings, what is the most likely pathogen affecting this patient? A Klebsiella pneumoniae B Legionella pneumophila C Pseudomonas aeurginosa D Mycoplasma pneumoniae E Streptococcus pneumoniae

The Correct Answer is: B Dense consolidation with bulging fissures is pneumonia consistent with Legionella pneumophila. Pleural effusions may also occur, as well as nodular irregularities in the immunocompromised host. (Sabria M, Yu VL. Harrisons Online, Part 7, Infectious Disease, Section 6, Diseases Caused by Gram-Negative Bacteria, Chapter 141, Legionella Infection)

Q 56.8: Which of the following do the majority of patients with dissociative identity disorder also meet diagnostic criteria for? A schizophrenia B post-traumatic stress disorder C bipolar II disorder D major depressive disorder

The Correct Answer is: B Dissociative identity disorder (DID), formerly known as multiple personality disorder, is classified as a trauma spectrum disorder due to the strong link with early childhood trauma and/or maltreatment. As such, approximately 70% of DID patients also meet criteria for PTSD.

Q 68.2: Mr. Smith leaves home and does not return nor does he go to work. A friend of Mr. Smith sees him in another state while on vacation. When he approaches Mr. Smith, he does not recognize him and has a total different demeanor. What type of disorder does Mr. Smith have? A amnesia B dissociative fugue C schizophrenia D dissociative identity disorder E depersonalization

The Correct Answer is: B Dissociative or psychogenic fugue is precipitated by a stressful event that causes the patient to develop amnesia, leave home, and assume another identity. (Sadock and Sadock, 2008, p. 297; Eisendrath and Lichtmacher, 2009, p. 915) Eisendrath SJ , Lichtmacher JE. Psychiatric disorders. In: McPhee SJ , Papdakis MA, eds. Current Medical Diagnosis and Treatment, 48th ed. New York: McGraw-Hill; 2009. Sadock BJ , Sadock VA. Concise Textbook of Clinical Psychiatry, 3rd ed. Philadelphia, PA: Lippincott, Williams & Wilkins; 2008.

Q 72.10: In Western society, diverticulosis most often occurs in which portion of the colon? A transverse B sigmoid C descending D ascending E equally common in all parts of the colon

The Correct Answer is: B Diverticulosis may arise anywhere in the large intestine, from the cecum to the end of the sigmoid colon. In Western societies, diverticula most often occur in the sigmoid colon where there is greatest intraluminal pressure. (McQuaid, 2009, p. 572) McQuaid KR. Gastrointestinal disorders. In: McPhee SJ , Papadakis MA, eds. Current Medical Diagnosis and Treatment. 48th ed. New York, NY: McGraw-Hill; 2009.

Q 80.10: A patient with HIV positive status, a CD4 count of 277/mcL, and is on chronic HIV medication therapy is diagnosed with pneumonia. What is the most common pathogen that would result in pneumonia for this patient? A Pneumocystosis jiroveci B Streptococcus pneumoniae C Pseudomonas aeurginosa D Klebsiella pneumoniae E Hemophilus influenzae

The Correct Answer is: B Due to the tight control of most of the HIV population, the most common pathogen is actually the most common community-acquired pathogen in the United States, Streptococcus pneumoniae. When the patient has lower CD4 counts and higher viral loads they are more susceptible to pathogens such as pneumocystosis jiroveci. (Chesnutt MS, Prendergast TJ. Current Medical Diagnosis and Treatment, 2011, Chapter 9, Pulmonary Disorders)

Q 79.9: Which of the following viral exanthems includes a fever followed by a diffuse maculopapular rash that spares the face and resolves in about 2 days? A measles B erythema subitum C erythema infectiosum D rubella

The Correct Answer is: B Erythema subitum or roseola is caused by the human herpesvirus 6 and presents clinically as described. The fever resolves when the rash begins and the entire process is self-limiting with usual full recovery with supportive care. The key to this question is that the rash spares the face. Erythema infectiosum (fifth disease), rubella, and measles also begin with a febrile prodrome, but each of the associated rashes starts on the head or face before progressing to other parts of the body. (Kaye and Kaye, 2008, p. 122) Kaye ET , Kaye KM. Fever and rash. In: Fauci AS , Braunwald E , Kasper DL, et al., eds. Harrison's Principles of Internal Medicine. 17th ed. New York, NY: McGraw-Hill; 2008.

Q 92.9: A 64-year-old man has been experiencing intermittent left lower abdominal pain associated with alternating diarrhea and constipation. The pain has been increasing over the past 24 hours and is now associated with a fever. The abdomen is tender with evidence of peritoneal signs. Which of the following diagnostic studies is most appropriate to evaluate this patient? A Barium enema B Computed tomography (CT) C Sigmoidoscopy D Colonoscopy

The Correct Answer is: B For a patient with diverticular disease, the preferred study to evaluate complications, such as a perforation or abscesses, is a CT scan. A barium enema or endoscopic procedure is contraindicated due to increased risk of perforation during an acute exacerbation. (Su, 2006, p. 208; Otterson, 2006, p. 1133) Su LT , Fry R. The colon, rectum and anus. In: Alturi P , Karakousis GC , Porrett PM , Kaiser LR, eds. The Surgical Review: An Integrated Basic and Clinical Science Study Guide. 2nd ed. Philadelphia, PA: Lippincott Williams & Wilkins; 2006. Otterson MF , Korus GB. Diverticular disease. In: Mulholland MW , Lillemoe KD , Doherty GM , Maier RV , Upchurch GR, eds. Greenfield's Surgery: Scientific Principles and Practice. 4th ed. Philadelphia, PA: Lippincott Williams & Wilkins; 2006.

Q 91.1: A patient presents complaining of a painful rash on his lips. What is the causative pathogen for the rash shown in the image? A Herpex zoster B Herpes simplex type 1 C Staphylococcus aureus D Herpes simplex type 2 E Staphylococcus pyogenes

The Correct Answer is: B Herpes simplex type 1 causes an orolabial and gingival vesicular rash. Herpes simplex type 2 causes genital lesions and is sexually transmitted. Herpes zoster or shingles causes a dermatomal, unilateral, and painful vesicular rash. Staphylococcus aureus and beta hemolytic streptococci cause bacterial skin infections. (McPhee et al., 2011, Chapter 6)

Q 76.2: You are reviewing laboratory results on a 60-year-old male from 2 days prior and note that the patient's potassium was 5.6 mEq/L but otherwise his BMP is normal. You speak to the patient on the phone; he states he feels fine. Which of the following medications would most likely be responsible for the abnormal potassium? A clonidine B enalapril C hydralazine D nebivolol E felodipine

The Correct Answer is: B Hyperkalemia is a potential adverse reaction of ACE inhibitors such as enalapril. ACE inhibitors should be suspected as a cause of hyperkalemia and may require discontinuation. Choices (A), (C),(D), and (E) do no cause hyperkalemia. (McPhee and Papadakis, 2011, Chapter 11)

Q 59.10: A 37-year-old female presents to the labor and delivery department complaining of intermittent pain and contractions. Upon arrival, she also complains of vaginal bleeding. She is a G3P2 at 39 weeks gestation; no other prenatal complications are noted. She is a non-smoker. A physical exam reveals the following: P 90, BP 130/80, T 98.7°F, abdomen gravid, positive bowel sounds, and left lower quadrant tenderness noted. A sterile speculum exam reveals the cervix to be dilated 8, fetus is cephalic, and membranes are intact. The fetal monitor reveals heart tones in the 140s with mild, decreased variability and good quality contractions noted. Delivery is felt to be imminent, and vaginal delivery has been determined to be the best course of action. What will likely decrease bleeding and shorten time to delivery? A Increased activity level B Amniotomy C Oxytocin therapy D Epidural placement E IV sedation

The Correct Answer is: B If the fetus is mature and vaginal delivery (versus c-section) has been determined to be the best course of action, then amniotomy may diminished amnionic fluid volume. This might also allow for better spiral artery compression, and serve to both decrease bleeding from the implantation site and reduce entry of thromboplastin into the maternal circulation.

Q 68.8: Which drug can potentially lead to oropharyngeal candidiasis, and which agent can be used to treat this type of infection? A albuterol; ketoconazole B triamcinolone; fluconazole C fluticasone; amantadine D cromolyn sodium; levofloxacin E flunisolide; metronidazole

The Correct Answer is: B If they coat the mouth and throat, inhaled corticosteroids (eg, triamcinolone, fluticasone, flunisolide) can alter the local bacteria and fungal population, thereby enhancing fungal growth. In cases of oropharyngeal candidiasis (thrush), white spots on the tongue and hard palate can be visualized, and the patient usually has pain on swallowing. In the asthma patient, the utilization of a spacer with a metered dose inhaler (MDI) can help minimize the chances of oropharyngeal candidiasis, as can routine gargling and rinsing following each inhaled treatment. Fluconazole is an antifungal agent that is effective in treating oropharyngeal candidiasis. (Kelly and Sorkness, 2008 pp. 486-487; Schindler et al., 2008, p. 190; Chesnutt et al., 2008, p. 209) Kelly HW , Sorkness CA. Asthma. In: DiPiro JT , Talbert RL , Yee GC, et al., eds. Pharmacotherapy: A Pathophysiologic Approach. 7th ed. New York: McGraw-Hill; 2008. Schindler J , Lustig L , Jackler RK , et al. Ear, nose & throat. In: Tierney LM Jr , McPhee SJ , Papadakis MA, eds. Current Medical Diagnosis & Treatment. 47th ed. New York: McGraw-Hill; 2008. Chesnutt MS , Murray JA , Prendergast TJ. Pulmonary disorders. In: Tierney LM Jr , McPhee SJ , Papadakis MA, eds. Current Medical Diagnosis & Treatment. 47th ed. New York: McGraw-Hill; 2008.

Q 88.8: A 56-year-old male is diagnosed with an H. pylori associated ulcer. The ulcer was 0.8 mm in size on endoscopy. He is placed on a proton pump inhibitor, clarithromycin, and amoxicillin. If his dyspeptic symptoms resolve after starting this regimen, how many days should he take the proton pump inhibitor to complete the treatment? A 5 to 7 days B 10 to 14 days C 14 to 21 days D 4 to 6 weeks E Indefinitely

The Correct Answer is: B In an H. pylori associated ulcer that is < 1cm in size and dyspeptic symptoms resolve with therapy, 10 to 14 days is the recommended length of treatment. For patients with large or complicated ulcers, antisecretory therapy should be continued for an addition two to four weeks (duodenal ulcer) or four to six weeks (gastric ulcer) after completion of the antibiotic regimen. (McPhee SJ, Papadakis MA. Current Medical Diagnosis & Treatment, 2010, p. 551)

Q 99.4: A 70-year-old man, with a history of HTN and aortic valve replacement 3 months ago, presents with complaints of arthralgia, myalgia, anorexia, fatigue, and weight loss over the last month, with recent dyspnea on exertion and lower extremity edema. Vital signs are as follows: Temperature 38°C, BP 102/64, P 98, RR 20. On physical exam, a new high-pitched, blowing, decrescendo diastolic murmur is noted along the left lower sternal border. Two separate blood cultures are positive for S. aureus, and found to be methicillin-resistant. A transesophageal echocardiogram demonstrates a paravalvular abscess. Which of the following is the most appropriate therapy in the management of this patient? A IV vancomycin B IV vancomycin, IV gentamicin, and PO rifampin with surgical treatment C IV amphotericin plus flycytosine, and surgical treatment D Outpatient IV ceftriaxone E IV penicillin G

The Correct Answer is: B In patients with prosthetic valve infection with methicillin-resistant S. aureus, the treatment of choice is IV vancomycin for 6 to 8 weeks, plus IV or IM gentamicin for the initial 2 weeks secondary to nephrotoxicity, and PO rifampin for 6 to 8 weeks, with susceptibility to gentamicin determined before initiation of rifampin. Surgical therapy decreases mortality in patients with S. aureus endocarditis, from over 70% with medical therapy alone to 25%, and should be considered in patients with paravalvular abscesses and symptoms suggestive of moderate to severe refractory congestive heart failure. Therefore, choice B is the most appropriate next step in the management of this patient. Choice A does not offer sufficient coverage for methicillin-resistant S. aureus. Choice C is appropriate therapy for infective endocarditis when the causative organism is Candida. Outpatient antibiotic therapy is only appropriate in patients who are stable, without clinical or echocardiographic findings to suggest complications, and IV Ceftriaxone is not appropriate for the treatment of methicillin-resistant S. aureus prosthetic valve endocarditis. Choice E is appropriate therapy for pencillin-susceptible streptococci, such as S. bovis. (Fauci et al., 2001, pp. 813-815)

Q 115.8: A 48-year-old male has a positive PPD and a negative chest x-ray. Which of the following laboratory studies must be conducted before initiating standard isoniazid therapy? A Complete blood count B Hepatic function panel C Lipid profile D PT/PTT E Thyroid-stimulating hormone

The Correct Answer is: B Isoniazid has been associated with hepatotoxicity, as well as severe and sometime fatal hepatitis. Monitoring should begin at initiation and continue throughout the course of therapy. Isoniazid is also associated with peripheral neuropathy development, especially in patients with an increased predisposition for developing a neuropathy. Alteration of renal function and visual changes have also been associated with tuberculosis therapies, indicating that renal function testing and routing eye examination should also be performed. Additional laboratory testing may be utilized for monitoring overall patient health and determining if any other conditions are also associated with the patient, such as the other choices provided. (Fauci et al., Harrison's Principles of Internal Medicine, 17e, Chapter 158)

Q 117.9: Koplik spots are a differentiating diagnostic feature of which of the following viral exanthems? A rubella B measles C varicella D parvovirus E Kawasaki disease

The Correct Answer is: B Koplik's spots, white lesions on the buccal mucosa, are characteristic of measles. The rash in measles usually presents as a red-brown rash starting with the head and moving caudally. It follows a 3- to 4-day prodrome consisting of fever, nasal drainage, conjunctivitis, and cough. Varicella may also present with mucosal lesions but they are vesicular on an erythematous base. Parvovirus, rubella, and Kawasaki disease generally do not have mucosal involvement. (Shandera and Carrales-Medina, 2009, p. 1217) Shandera WX , Corrales-Medina VF. Viral & rickettsial infections. In: McPhee SJ , Papadakis MA, eds. Current Medical Diagnosis and Treatment. 48th ed. New York, NY: McGraw-Hill; 2009.

Q 54.16: A patient exhibits air hunger and labored, deep respirations due to increased stimulation of the respiratory center in the brain. Which of the following is the most likely cause? A Congestive heart failure B Metabolic acidosis C Obstructive sleep apnea D Respiratory acidosis E Traumatic brain injury

The Correct Answer is: B Kussmaul's respiration is a form of respiratory compensation, and is most commonly associated with metabolic acidosis. During early acidosis, breathing may be rapid, but when advanced the breaths become deep, slow and labored with an urge to breathe described as "air hunger." The other etiologies suggest other causes of breathing variation, such as tachypnea, apnea, and Cheyne-Stokes respiration.

Q 82.10: A 24-year-old male with thalassemia major who has received adequate transfusions, chelation therapy, and regular health checkups is in to establish care. He is 5'4" with a BMI of 17. He eats a balanced healthy diet and gets regular exercise. You know that there are complications of this disease. What is the best next step for this patient? A Increase threshold for blood transfusions B Maintain sustained reduction of body iron C Obtain a Dexa scan for osteoporosis D Obtain an ECHO to evaluate for cardiac siderosis E Obtain regular testosterone levels and treat

The Correct Answer is: B Maintaining sustained reductions in body iron has demonstrated increased overall survival rates through reductions in cardiac disease specifically due to siderosis. While these patients are at increased risk for osteoporosis and cardiac siderosis, the next best step in this patient is to maintain reduced iron levels. There is no place for increased blood transfusion or obtaining regular testosterone levels. (Lichtman et al., Williams Hematology 8e, Chapter 47, The Thalassemias: Disorders of Globin Synthesis)

Q 69.10: A 67-year-old female presents for a follow-up visit for chronic obstruction pulmonary disease (COPD). Her most recent FEV1 is <80% predicted. Her room air oxygen saturation is 94%. She is currently managed on a short acting beta-agonist as needed, and has recently been on a taper-dosed corticosteroid for an exacerbation. Which of the following is the most appropriate next step of management for this patient? A Increase the dosage of the short acting beta-agonist B Add an anticholinergic, such as tiotropium C Begin oral theophylline D Begin chronic oxygen therapy E Begin daily oral corticosteroids

The Correct Answer is: B Management of COPD patients focuses on improving symptoms and decreasing the severity of exacerbations. The initial management should focus on smoking cessation in all patients that smoke. Medications may be utilized to allow bronchodilation, but must be used appropriately, to avoid side effects and potential harm. Anticholinergic agents have been shown to improve symptoms, FEV1, and reduce exacerbations, with less side effects than high dose beta-agonists. Long-acting beta-agonists have been shown to have similar benefits, with caution being needed when using these agents in certain populations. Corticosteroids, both inhaled and systemic, have been shown to have a vital role in COPD exacerbations, but benefits regarding mortality or limiting lung function decline have not been shown, with these agents not being considered a vital part of long-term COPD management. Oral theophylline, which provides bronchodilation and anti-inflammatory properties, is a fourth-line COPD agent, based upon its narrow therapeutic index and potential for adverse side effects. Oxygen therapy has been shown to improve the progression of COPD in patients with resting hypoxemia, defined by most as a resting O 2 saturation <88% or <90% with other comorbid findings. (McPhee SJ, Papadakis MA. Current Medical Diagnosis & Treatment 2011, Chapter 9, Pulmonary Disorders)

Q 120.3: A 60-year-old male complains of right scapular pain that is sharp for the last two days. He reports that the pain was preceded by tenderness and a tingling sensation. What is the most likely diagnosis? A Contact dermatitis B Herpes zoster virus C Impetigo D Molluscum contagiosum

The Correct Answer is: B More than 2/3 of cases of the herpes zoster virus occurs in patients over the age of 50. A herpes zoster virus flare occurs unilaterally in a dermatomal distribution. The prodromal stage can consist of neuritic pain or paresthesias prior to eruption of the rash in two to three weeks. The rash consists of vesicles that eventually crust and heal. The pain of herpes zoster can continue for months or years after the rash has resolved. (Wolff et al., 2009, Fig. 27-41, pg. 837-840)

Q 106.8: A 63-year-old male with type 2 diabetes mellitus and hyperlipidemia is being seen for routine blood work to assess his renal function. Blood pressure is 130/90 and pulse is 75. His blood chemistries show hypoalbuminemia and hypoproteinemia. His urinalysis shows urine protein excretion of 3.5 grams per 24 hours, microscopically shows oval fat bodies in the urine, and Maltese crosses under polarized light. Which of the following findings would support the suspected diagnosis? A Maculopapular rash B Peripheral edema C Jaundice D Hematuria E Costovertebral angle tenderness

The Correct Answer is: B Nephrotic syndrome is diagnosed with bland urine sedimentation, urine protein excretion > 3 g per 24 hours, hypoalbuminemia of < 3g/dl, peripheral edema, hyperlipidemia and oval fat bodies in the urine. Peripheral edema (B) is a hallmark sign of nephrotic syndrome, which occurs when the serum albumin concentration is < 3 g/dL (30 g/L). Edema is most likely due to sodium retention. Initially, this presents in the dependent areas of the body subject to gravity (lower extremities); such edema can become generalized as in periorbital edema. Patients can experience dyspnea due to pulmonary edema, pleural effusions and diaphragmatic compromise with ascites. In adults, roughly one-third of patients diagnosed with nephrotic syndrome also have a concurrent systemic disease such as diabetes mellitus, amyloidosis or systemic lupus erythematosus. Serum creatinine may or may not be abnormal at the time of presentation, depending on the severity, acuity and chronicity of the disease. Only nephrotic syndrome would show oval fat bodies in his urine. Maculopapular rash (A) is seen with scarlet fever, measles, and syphilis. Jaundice (C) is usually seen with liver abnormalities. Hematuria (D) is seen with glomerulonephritis. CVA tenderness (E) is seen with pyelonephritis Watnick S, Dirkx T. Chapter 22. Kidney Disease. In: Papadakis MA, McPhee SJ, Rabow MW, eds. CURRENT Medical Diagnosis & Treatment 2013. New York: McGraw-Hill; 2013. http://www.accessmedicine.com/content.aspx?aID=11374 . Accessed March 6, 2013.

Q 72.3: A 42-year-old African American male is admitted to the hospital with heme positive urine and anemia. He recently completed a course of trimethoprim/sulfamethoxazole for a urinary tract infection. Treatment for his condition should include which of the following? A Begin vancomycin 1 GM IV q 12 for untreated infection B No treatment C Restart bactrim IV for an undertreated UTI D Transfuse packed red cells E Transfuse platelets

The Correct Answer is: B No treatment is necessary, except to avoid known oxidant drugs. The patient is not exhibiting signs of an untreated infection, so A is wrong. The anemia is transient so no transfusions are needed. (McPhee SJ, Papadakis MA, Tierney LM. Current Medical Diagnosis and Treatment, 2010, Chapter 13, Blood Disorders)

Q 62.10: A patient presents to your office with a sudden onset of headache, right eye pain, decreased visual acuity, nausea and vomiting. His intraocular pressure is 47. Which of the following classes of medications are indicated for treatment of this condition? A Alpha agonists and antihistamines B Alpha agonists and Beta blockers C Mydriatics D Cycloplegics E Angiotensin converting enzyme inhibitors

The Correct Answer is: B Ophthalmic alpha-agonists (brimonidine) and beta blockers (timolol) decrease aqueous humor production, and decrease intraocular pressure. They facilitate aqueous flow through outflow tract and the canal of Schlemm. Other acute treatments include prostaglandin analogs(latanoprost) and carbonic anhydrase inhibitors(acetazolamide). Mydriatics, cycloplegics(tropicamide) and antihistamines( diphenhydramine) can precipitate angle closure glaucoma in patients at risk. Angiotensin converting enzyme inhibitors are used for treating systemic hypertension.

Q 118.4: A 60-year-old male presents with complaints of irritation and a white plaque on his tongue. He denies pain. During physical exam you are unable to remove the white plaque from the mucosa with a tongue depressor. What is the most likely diagnosis, represented as follows? A Oral thrush B Leukoplakia C Geographic tongue D Glossitis E Lichen planus

The Correct Answer is: B Oral leukoplakia cannot be removed from the mucosa using a tongue depressor like oral thrush can. Lichen planus can mimic candidiasis, squamous cell carcinoma, or hyperkeratosis, and requires a biopsy to diagnose. Glossitis is a generalized inflammation, and loss of papillae of the tongue is caused by vitamin deficiencies, medication reactions, auto immune reactions, or psoriasis. Geographic tongue is an asymptomatic serpiginous area of atrophy and erythema of the anterior tongue. The condition is self-limiting. (Tintinalli et al., 2011, Chapter 117) (McPhee et al., 2011, Chapter 8)

Q 106.17: A 74-year-old man with end-stage renal failure is suffering from a number of bone abnormalities, including osteomalacia. Which of the following is most likely diminished in this patient? A blood urea nitrogen (BUN) B production of 1,25-dihydroxycholecalciferol C secretion of parathyroid hormone (PTH) D secretion of thyroid hormones E serum concentration of creatinine

The Correct Answer is: B Osteomalacia represents a softening of the bone due to inadequate amounts of calcium. Hypocalcemia often develops in end-stage renal failure due to the inability of the kidneys to activate vitamin D into the form known as 1,25-dihydroxycalciferol. The kidneys possess an enzyme (1-alpha-hydroxylase) that performs this activation, but in end-stage renal failure, the activity of this enzyme declines. 1,25-Dihydroxycalciferol is essential for adequate calcium absorption in the small intestine. Without 1,25-dihydroxycholecalciferol, calcium is excreted in higher amounts in the feces. The secretion of PTH is likely to be elevated (and not diminished) in patients with end-stage renal failure, as the main stimulus for PTH secretion is hypocalcemia. Thyroid hormones do not play a role in calcium homeostasis. Both the BUN and the serum creatinine level would elevate in the patient with end-stage renal failure, as the glomerular filtration rate is diminished to the point where both urea and creatinine cannot be adequately filtered from the plasma. (Costanzo, 2006, pp. 435-437) Costanzo LS. Physiology. 3rd ed. Philadelphia, PA: Saunders Elsevier; 2006.

Q 115.15: A patient presents to your office claiming that the FBI is trying to poison him. What would these types of beliefs be called? A somatic delusion B delusion of persecution C illusion D delusion of grandeur E hallucination

The Correct Answer is: B Patients who have delusions of persecution often feel that people are taking pictures and tape recording them. Patients often believe that external agencies or relatives are attempting to harm them. (Sadock and Sadock, 2008, p. 185; Shelton, 2008, p. 294) Sadock BJ , Sadock VA. Concise Textbook of Clinical Psychiatry, 3rd ed. Philadelphia, PA: Lippincott, Williams & Wilkins; 2008. Shelton RC. Other psychotic disorders. In: Ebert MH , Loosen PT , Nurcombe B , Leckman JF, eds. Current Diagnosis and Treatment in Psychiatry. New York: McGraw-Hill; 2008.

Q 118.9: Patients who suffer with acne rosacea can relate a history of which outcome? A worsening with onset of menses B worsening with exposure to hot temperatures, spicy foods, or alcoholic beverages C improvement with onset of menopause D improvement with use of steam rooms

The Correct Answer is: B Patients who suffer with acne rosacea can relate a history of their condition worsening with exposure to hot temperatures, spicy foods, or alcoholic beverages. This is in response to increased reactivity of capillaries. Acne that worsens with the onset of menses is characteristic of acne vulgaris, not acne rosacea. Acne rosacea may resolve spontaneously; however, it is usually present in some form for a lifetime. Exposure to high temperatures, such as those in a steam room, can worsen acne rosacea. (Wolff et al., 2009, Page 9)

Q 65.8: You are evaluating a 77-year-old male with stage 3 chronic kidney disease (CKD). He denies any complaints today. His renal function has been stable over the past 6 months. However, you note that his Hgb has decreased from 10 g/dl to 8.5g /dl despite erythropoietin injections. Hemoccult stool times 3 is negative. What should you do next? A Order a blood transfusion. B Order an iron profile with ferritin. C Increase his frequency of erythropoietin to daily. D Double his dose of erythropoietin weekly. E Begin dialysis.

The Correct Answer is: B Patients with CKD may develop an anemia of chronic kidney disease, which will usually respond to subcutaneous erythropoietin q2-4weeks. However, iron is necessary for appropriate response. Patients with CKD who are iron deficient with either decreased ferritin or iron saturations will need to have their iron stores replaced by either p.o. or intravenous iron. Increasing the dose of erythropoietin or increasing the frequency is not appropriate since this will be costly and increase patient risk without benefit. Also, daily dosing or doubling the dose weekly is not recommended or indicated and may be dangerous. Dialysis is not indicated at this time. (McPhee and Papadakis, 2011, Chapter 22)

Q 107.4: A 54-year-old male patient presents to your office complaining of pain to the left eye with nausea, vomiting, and a headache after being brushed in the eye with his grandchild's stuffed animal. On examination the conjunctiva is not injected, and the cornea has a steamy appearance. You cannot visualize the retina. The pupil is fixed and 4 mm. When you stain the eye you are unable to see any lesions or scratches. You suspect: A acute bacterial conjunctivitis B acute narrow angle glaucoma C allergic conjunctivitis D herpes simplex ophthalmicus E traumatic iritis

The Correct Answer is: B Patients with acute glaucoma usually seek treatment immediately because of extreme pain and blurred vision, though there are subacute cases. The blurred vision is associated with halos around lights. Nausea and abdominal pain may occur. The eye is red, the cornea steamy, and the pupil moderately dilated and nonreactive to light. Intraocular pressure is usually over 50 mm&thinsp;Hg, producing a hard eye on palpation. (McPhee and Papadakis, 2011, Chapter 7)

Q 89.6: A 65-year-old male presents to you with complaints of decreasing hearing, along with difficulty discerning words when in conversations in noisy environments, such as restaurants. His only medication is simvastatin for hyperlipidemia. The following is his audiogram. He has bilateral decreased high frequency hearing loss, and decreased speech recognition. What is the most likely diagnosis? A Vestibular schwannoma B Presbycusis C Presbystasis D Cerumen impactions E Vestibulobasilar insufficiency

The Correct Answer is: B Presbycusis is age related bilateral loss of high frequency hearing, and decreased word recognition. Presbystasis is age related balance disorder. Vestibular schwannoma (acoustic neuroma) causes unilateral hearing loss. Vestibulobasilar insufficiency results from atherosclerosis of the vertebral arteries, and can cause many symptoms including double vision, speech defects, vertigo, ataxia, and drop attacks. (Lalwani A.K., 2008, Chapter 53)

Q 68.9: A 65-year-old man presents with complaints of acute onset of pain and swelling of the right great toe. He denies recent alcohol ingestion or trauma to the area. On physical examination, the patient is afebrile, and the first metatarsophalangeal joint is erythematous, swollen, and warm to the touch. Laboratory evaluation reveals a WBC (white blood cells) count of 12,000/μL and a normal differential. Serum uric acid level is found to be 5 mg/dL. Synovial fluid analysis reveals the presence of rhomboid-shaped crystals. Which of the following is the most likely diagnosis? A acute gout B pseudogout C psoriatic arthritis D infectious arthritis E rheumatoid arthritis

The Correct Answer is: B Pseudogout presents similarly to acute gout and is best diagnosed by the finding of the rhomboid-shaped crystals of calcium pyrophosphate in joint aspirates. Joints commonly involved in pseudogout are the knees and wrists and other joints such as the metacarpophalangeals, hips, shoulders, ankles, and elbows. The diagnosis of pseudogout is further supported by the finding of a normal serum uric acid level. Acute gout would more likely be associated with an elevated serum uric acid level. Psoriatic arthritis commonly presents with asymmetrical oligoarticular involvement of two to four joints, and in a higher percentage of patients, there is known presence of the dermatological expression of psoriasis. Infectious arthritis is ruled out with the findings of an afebrile patient and WBC count of 12,000/μL. In acute infectious arthritis, the WBCs would be expected to be elevated in the range of 50,000 to 200,000/μL. Rheumatoid arthritis usually presents with symmetrical polyarticular involvement of three or more joints.

Q 119.13: Women who suffer from migraine with aura are at increased risk for which of the following? A deep vein thrombosis B cerebrovascular accidents C pulmonary emboli D thrombophilias E dissecting aortic aneurysms

The Correct Answer is: B Relatively recent studies show that women who have migraine with aura are at a small but statistically significant increased risk of stroke. Patients in this population should be treated even more aggressively with stroke risk-reduction strategies. (Fuster et al., 2008, Chapter 3)

Q 120.7: A renal ultrasound would be most beneficial for diagnosing which of the following? A nephrotic syndrome B polycystic kidney disease C glomerulonephritis D acute tubular necrosis E lupus nephritis

The Correct Answer is: B Renal ultrasound is useful for assessing kidney size and thickness of the cortex, and for the presence of masses, cysts, obstruction, and hydronephrosis. Intrinsic disease is best assessed by establishing the clinical context, analyzing the urine for protein, cells, and casts, and possibly by doing a biopsy. Loss of cortical thickness is a nonspecific finding, and ultrasound does not establish an etiology. (Bazari, 2008, pp. 810-811; Watnick and Morrison, 2009, p. 805) Bazari H. Approach to the patient with renal disease. In Goldman L , Ausiello D, eds. Cecil Medicine. 23rd ed. Philadelphia, PA: Saunders; 2008. Watnick S , Morrison G. Kidney. In: Tierney LM , McPhee SJ , Papadakis MA, eds. Current Medical Diagnosis and Treatment. 48th ed. New York, NY: McGraw-Hill; 2009.

Q 114.4: When initially screening for CKD, which of the following would be ordered? A 24-hour urine collection B blood pressure measurement, serum creatinine level, spot urine protein measurement C renal ultrasound D abdominal CT scan E renal angiogram

The Correct Answer is: B Screening for the presence of chronic kidney disease involves checking a serum creatinine level, checking blood pressure for the presence of hypertension, checking urinary protein for evidence of glomerular injury, and obtaining a history to check for the presence of risk factors, such as hypertension, diabetes mellitus, autoimmune disease, infection, or family history. Initial screening would not include a 24-hour urine collection. This is a cumbersome, inconvenient, more expensive test than the spot urinary protein reading and would not provide additional information. Renal ultrasound and abdominal CT scan would not be indicated in the initial stages of the work-up. These would be done only after laboratory studies were done and only if indicated. (NKF-K/DOQI Guidelines, 2002, p. 31) NKF-K/DOQI Clinical Practice Guidelines for Chronic Kidney Disease: Executive Summary. New York, NY: National Kidney Foundation; 2002.

Q 101.3: Which of the following best describes simple partial seizures? A Focal seizure activity with associated altered level of consciousness, and decreased command response with postictal phase B Seizure activity related to a focal region of the brain that may be motor, sensory, or autonomic in nature, and not accompanied by impaired consciousness C Sudden and brief loss of muscle tone with impaired level of consciousness, but without postictal state D Sudden lapse of consciousness with a brief period of autonomic changes or tonic, atonic, or clonic activity, with no postictal confusion E Sudden loss of consciousness with the onset of tonic muscle contraction, followed by clonus of musculature, and accompanied by impaired respiration and postictal state

The Correct Answer is: B Simple partial seizures are due to focal brain cortex stimulation. The seizure symptoms will be related to the specific region of the brain cortex involved, and may include motor, sensory, and autonomic changes. Partial seizures are differentiated as simple versus complex, based on the alteration or lack of alteration of consciousness, with simple not involving a change in consciousness. Seizures are also differentiated by the absence or presence of a postictal state. Simple partial seizures do not have an associated postictal state (for further seizure classification information, see Table 24-2). (McPhee SJ, Papadakis MA. Current Medical Diagnosis & Treatment 2011, Chapter 24, Nervous System Disorders)

Q 114.8: Which of the following patients, without laboratory evidence of HIV, meets the Centers for Disease Control and Prevention case definition for acquired immunodeficiency syndrome (AIDS)? A 29-year-old man with pulmonary tuberculosis B 32-year-old man with Kaposi sarcoma C 35-year-old woman with invasive cervical cancer D 36-year-old man with recurrent Salmonella septicemia E 40-year-old woman with recurrent pneumonia

The Correct Answer is: B The Centers for Disease Control and Prevention AIDS case definition includes the following diseases that, with or without laboratory evidence of HIV infection, constitute a definitive diagnosis of AIDS: candidiasis of the esophagus, trachea, bronchi, or lungs; extrapulmonary cryptococcosis; cryptosporidiosis with diarrhea persisting more than 1 month; cytomegalovirus disease of an organ other than liver, spleen, or lymph nodes; herpes simplex virus infection causing a mucocutaneous ulcer that persists longer than 1 month or causing bronchitis, pneumonitis, or esophagitis; Kaposi sarcoma in a patient younger than 60; lymphoma of the brain in a patient younger than 60; disseminated Mycobacterium avium complex or Mycobacterium kansasii disease; Pneumocystis jiroveci pneumonia; progressive multifocal leukoencephalopathy; or toxoplasmosis of the brain. Other conditions in the case definition require laboratory evidence of HIV infection. (Katz and Zolopa, 2009, p. 1177) Katz MH , Zolopa AR. HIV infection. In: McPhee SJ , Papadakis MA, eds. Current Medical Diagnosis and Treatment. 48th ed. New York, NY: McGraw-Hill; 2009.

Q 83.6: A 75-year-old man is involved in a motor vehicle accident and strikes his forehead on the windshield. He complains of neck pain and severe burning in his shoulders and arms. His physical examination reveals weakness of his upper extremities. What type of spinal cord injury does this patient have? A anterior cord syndrome B central cord syndrome C Brown-Séquard syndrome D complete cord transection E cauda equina syndrome

The Correct Answer is: B The central cord syndrome involves loss of motor function that is more severe in the upper extremities than in the lower extremities, and is more severe in the hands. There is typically hyperesthesia over the shoulders and arms. Anterior cord syndrome presents with paraplegia or quadriplegia, loss of lateral spinothalamic function with preservation of posterior column function. Brown-Séquard syndrome consists of weakness and loss of posterior column function on one side of the body distal to the lesion with contralateral loss of lateral spinothalamic function one to two levels below the lesion. Complete cord transection would affect motor and sensory function distal to the lesion. Cauda equina syndrome typically presents as low back pain with radiculopathy. (Hauser and Ropper, 2008, p. 2580) Beal MF , Hauser SL. Trigeminal neuralgia, Bell's palsy and other cranial nerve disorders. In: Fauci AS , Braunwald E , Kasper DL, et al., eds. Harrison's Textbook of Medicine. 17th ed. New York, NY: McGraw-Hill; 2008.

Q 121.20: A cerebrospinal fluid analysis reveals the following results: opalescent color, increased protein, decreased glucose, and increased polymorphonuclear white blood cells (WBCs). The most likely diagnosis would be A subarachnoid hemorrhage B bacterial meningitis C viral meningitis D multiple sclerosis E encephalitis

The Correct Answer is: B The cerebrospinal fluid (CSF) analysis in bacterial meningitis includes a cloudy appearance with a markedly elevated protein and white cell content. The white cells are predominantly polymorphonuclear leukocytes (polys). Bacterial utilization of CSF glucose causes it to be low. Gram stain may or may not be positive for bacteria. The diagnosis of bacterial meningitis requires a culture of the CSF. CSF pressures at the time of the lumbar puncture are elevated in 90% of cases. In viral meningitis, the CSF white count is usually 1,000/mL. The cell types are lymphocytes or monocytes but early in the disease polys may predominate. CSF glucose is normal in viral meningitis and protein is elevated. Gram stain will be negative and the culture will show no growth. The CSF in multiple sclerosis may have a mild lymphocytosis with an increased protein concentration. CSF protein electrophoresis in multiple sclerosis shows discrete bands of IgG called oligoclonal bands. These oligoclonal bands are present in 90% of patients with multiple sclerosis. The CSF in subarachnoid hemorrhage is grossly bloody. Because bleeding can be caused by a traumatic puncture, the red blood cell (RBC) count should be done on the first and last tubes and the counts compared. In subarachnoid hemorrhage, the RBC count will be the same, whereas in a traumatic lumbar puncture, the RBCs will not be present in the last tube that is collected. The CSF in subarachnoid hemorrhage may reveal xanthochromia. This is a yellow appearance in the centrifuged CSF supernatant caused by the degradation of RBCs in the CSF. The CSF becomes xanthochromic after it has been exposed to blood for several hours. (Aminoff et al., 2005, p. 12) Aminoff MJ , Greenberg DA , Simon RP. Clinical Neurology. 6th ed. New York, NY: McGraw-Hill; 2005.

Q 66.5: Which of the following signs and symptoms is associated with the abdominal pain secondary to chronic intestinal ischemia? A Guarding and rigidity B Fear of eating C Nausea and vomiting D Bloody diarrhea E Positive obturator and psoas signs

The Correct Answer is: B The clinical symptoms associated with chronic intestinal ischemia include severe epigastric pain following meals, which results in weight loss and fear of eating. Nausea, bloody diarrhea, and vomiting as well as guarding and rigidity are consistent with acute intestinal ischemia. Obturator and psoas signs are indicative of acute appendicitis. (McKinsey, 2006, p. 457; Shelton, 2006, p. 678) McKinsey JF , Lawrence PF , Gewertz BL. Diseases of the vascular system. In: Lawrence PF , Bell RM , Dayton MT , Ahmed MI, eds. Essentials of General Surgery. 4th ed. Philadelphia, PA: Lippincott Williams & Wilkins; 2006. Shelton AA , Chang G , Welton ML. Small intestine. In: Doherty GM , Way LM, eds. Current Surgical Diagnosis & Treatment. 12th ed. New York, NY: Lange Medical Books/McGraw-Hill; 2006.

Q 69.7: A 45-year-old male presents with purulent discharge from his right ear for three weeks. He states that despite being treated by his family doctor for an ear infection one month ago, the problem continues to get worse. Upon exam, you note purulent discharge in the ear canal, an erythemic tympanic membrane, and a possible perforation. What are the pathogens most likely to culture positive? A Strep pneumoniae B Pseudomonas aeroginosa C Escherichia coli D Candida albicans E Mycoplasma pheumoniae

The Correct Answer is: B The clinical vignette describes a chronic otitis media. Usually, this refers to a complication of acute otits media with perforation. Pathogens that culture from these infections are usually pseudomonas, proteus, or staphylococcus aureus. Strep pneumoniae is often seen in acute otitis media. E.coli is a urinary tract pathogen. Candida albicans is a cause of vaginitis, and mycoplasma is a respiratory pathogen. (McPhee et al., 2011, Chapter 8)

Q 102.3: A 67-year-old male presents for his annual physical in February. On examination, you note nontender cervical and axillary adenopathy. He states that he has felt those lumps for several weeks, has noted feeling more fatigued, and has decided that he had a cold. On further inspection, you note a mildly enlarged spleen, and the patient states that he hasn't been eating as much as he usually does. A CBC reveals an elevated lymphocyte count. These findings are most consistent with which diagnosis? A Acute infection lymphocytosis B Chronic lymphocytic leukemia C Rickettsiosis D Stress lymphocytosis E Systemic lupus erythematosus

The Correct Answer is: B The findings in this patient are consistent with chronic lymphocytic leukemia. Rickettsiosis and systemic lupus erythematosus tend to cause a lymphocytopenia rather than lymphocytosis. Acute infection lymphocytosis do not have either lymph node enlargement or splenomegaly, and stress lymphocytosis is short lived following trauma, surgery, or other insults to the body. (Lichtman et al., Williams Hematology 8e, Chapter 94, Chronic Lymphocytic Leukemia and Related Diseases) (Lichtman et al., Williams Hematology 8e, Chapter 81, Lymphocytosis and Lymphocytopenia)

Q 116.11: What is the chief complaint associated with bladder cancer? A pyuria B hematuria C dysuria D urinary frequency

The Correct Answer is: B The most common complaint of bladder cancer is painless hematuria, which occurs in 85% to 90% of patients. Additional symptoms of bladder irritability, and urinary frequency, urgency, and dysuria are the second most common presentation and are usually associated with invasive bladder cancer. (Rugo, 2009, p. 1461)

Q 79.1: Which of the following is a complication of Barrett esophagus? A achalasia B adenocarcinoma C diffuse spasm D varices E stricture

The Correct Answer is: B The most serious complication of Barrett esophagus is esophageal adenocarcinoma, which arises from dysplastic epithelium. Patients with Barrett esophagus have a significantly increased risk compared to those patients who do not. (Poneros, 2009, pp. 148-150) Poneros JM. Barrett esophagus. In: Greenberger NJ, ed. Current Diagnosis & Treatment: Gastroenterology, Hepatology, & Endoscopy. New York, NY: McGraw-Hill; 2009.

Q 99.9: A 63 year-old woman presents to the clinic complaining of increasing dyspnea over the last two weeks. Which of the following diagnoses is supported by the chest x-ray below? A Dissecting aortic aneurysm B Pleural Effusion C Pneumonia D Pneumothorax E Pulmonary Embolism

The Correct Answer is: B The patient has a left-sided pleural effusion (B). The mediastinum is not widened as seen in dissection aortic aneurysms (A) and although the lung markings are absent in the left lower lobe region this is due to the accumulation of fluid (B) and not air as seen in pneumothorax (D). Clear signs of pneumonia (D) or pulmonary embolism (E) aren't present on the chest x-ray, but are potential underlying causes of effusion.

Q 64.4: A 64 year-old woman with past medical history of hypertension presents to the clinic complaining of increasing cough and dyspnea over the past two weeks. Physical exam reveals dullness to percussion and decreased breath sounds at the bilateral bases. A chest x-ray is available below: Which of the following is the most appropriate intervention for this patient? A Cardiac catheterization B Diuresis with a loop diuretic C Obtain a spiral CT of the chest D Perform thoracentesis E Treat with empiric antibiotics

The Correct Answer is: B The patient has developed a pleural effusion most likely due to CHF based on the symptoms and presence of small bilateral pleural effusions. Initial interventions include diuresis and monitoring for resolution of symptoms and the effusion (B). If the effusion fails to improve or the patient develops dyspnea at rest then thoracentesis (E) is indicated. Additional therapy or intervention (A, C, E) should be considered if the patient's condition worsens and are informed by the results of thoracentesis.

Q 120.11: Your patient complains of some weight loss and anxiety over the past 6 months, which she attributes to family issues. On examination you note the following physical examination findings, shown below. What is the most likely diagnosis? Source: (Wolff et al., 2008, Chapter 152) A hypothyroidism B Grave's disease C hypoparathyroidism D subacute thyroiditis E hyperparathyroidism

The Correct Answer is: B The patient has exophthalmos, which is most commonly associated with Graves' disease. Subacute bacterial thyroiditis, choice (D), usually has an initial hyperthyroid phase followed by hypothyroidism and would unlikely cause these physical examination changes. None of the other listed conditions would cause these physical examination findings. (Wolff et al., 2008, Chapter 152)

Q 74.5: A 24-year-old female presents with hyperpigmented macules on her cheeks, nose, and upper lip. They have been present for a couple of months. Her current medications include oral LoEstrin 24 Fe, cetirizine, and a multivitamin daily. What is the most likely diagnosis? A congenital nevus B melasma C post-inflammatory hyperpigmentation D café-au-lait macule

The Correct Answer is: B The patient is experiencing melasma secondary to the use of oral contraceptives. This is a frequent cause of melasma. Melasma can also be precipitated by hormonal changes that occur during pregnancy. The condition will resolve upon discontinuation of the oral contraceptive. A congenital nevus is a nevus that presents within the first year of life. It is monitored in the same way as acquired nevi. They can be larger than acquired nevi, with only a slight increase in chance of malignant change over time. Post-inflammatory hyperpigmentation includes darker areas of pigmentation that can result after inflammation on the skin. Common causes include acne and atopic dermatitis. The hyperpigmentation will resolve over time. A Café-au-lait macule is a type of birthmark. It is usually light tan to light brown in appearance, and can vary greatly in size. They are usually benign, but can be associated with neurofibromatosis when more than six, with a diameter greater than 1.5 cm, are present. (Wolff et al., 2009, Pages 344-346)

Q 86.6: A 58-year-old female two days status-post coronary artery bypass graft (CABG) surgery is being treated with heparin to prevent thrombosis. Her routine CBC reveals a hemoglobin of 11.2 mg/dL, hematocrit of 35%, WBC count of 5.6, and platelet count of 22,000. In addition to discontinuing heparin, which of the following is the most appropriate intervention? A Administer prednisone B Administer warfarin C Bone marrow aspirate D CT of the abdomen E Observation

The Correct Answer is: B The patient most likely has developed heparin-induced thrombocytopenia (HIT), which is associated with qualitative platelet function changes that result in increased risk of thrombosis. She should begin warfarin (B) and be evaluated for thrombosis (e.g., lower extremity Dopplers). Prednisone (A) is not indicated for the treatment of HIT, and a bone marrow aspirate (C) would not aid in establishing the diagnosis. CT of the abdomen (D) would be indicated if she had symptoms consistent with thrombosis in that region, and observation (E) fails to address her increased thrombotic risk. Konkle B. Chapter 115. Disorders of Platelets and Vessel Wall. In: Longo DL, Fauci AS, Kasper DL, Hauser SL, Jameson JL, Loscalzo J, eds. Harrison's Principles of Internal Medicine. 18th ed. New York: McGraw-Hill; 2012. http://www.accessmedicine.com/content.aspx?aID=9100733 . Accessed March 22, 2013.

Q 114.7: A 53-year-old man presents to the emergency department because of fever, headache, and confusion. On physical examination, you note an obtunded man who appears acutely ill with temperature of 104°F, blood pressure of 128/76 mm Hg, pulse of 98, and respiratory rate of 20. The patient has stomatitis, nuchal rigidity, and a positive Kernig sign. CSF examination shows increased opening pressure, 80 WBC/mL (normal < 10/mL), mildly elevated protein, and normal glucose. Which of the following tests would confirm the most likely causative organism? A CT of the head B polymerase chain reaction test for herpes simplex virus C blood culture for herpes simplex virus D serum IgG for herpes simplex virus E MRI of the head

The Correct Answer is: B The patient's presentation is consistent with viral meningitis with potential encephalitis. The presence of active stomatitis indicates herpes simplex virus as the most likely causative organism. A CT of the head could be considered prior to performing a lumbar puncture and may show temporal lobe abnormalities that support a diagnosis of herpes virus encephalitis, but like an MRI will not identify the causative organism and has limited sensitivity. Of the three herpes tests described, the PCR technique is the most likely to identify the herpes simplex virus as the causative organism in the CSF due to its high sensitivity and specificity. Serum IgG indicates prior infection from herpes simplex virus but does not confirm the causative organism of the patient's encephalitis. Viral blood cultures for herpes simplex would likely show no growth even in the presence of herpes simplex virus encephalitis. (Aminoff et al., 2005, p. 30) Aminoff MJ , Greenberg DA , Simon RP. Clinical Neurology. 6th ed. New York, NY: McGraw-Hill; 2005.

Q 97.2: A 43-year-old woman presents complaining of a "pins and needles" sensation that started bilaterally in her feet 2 days ago. The sensation now extends up to her mid-thighs. On physical examination, she is noted to have mild sensory loss, weakness, and absent reflexes bilaterally in her legs. Which of the following is the most likely diagnosis? A diabetic peripheral neuropathy B Guillain-Barré syndrome C multiple sclerosis D myasthenia gravis E hypothyroidism

The Correct Answer is: B The pattern of sensory, motor, and reflex findings occurring over an acute time period is consistent with Guillain-Barré Syndrome. Diffuse diabetic peripheral neuropathy develops more insidiously than this case scenario. Multiple sclerosis presents with central nervous system (CNS) lesions that are unlikely to occur in this pattern. Myasthenia gravis causes intermittent motor symptoms without sensory involvement. Hypothyroidism may cause weakness and delayed reflexes, but is not the single best answer for this question. (Aminoff et al., 2005, p. 212) Aminoff MJ , Greenberg DA , Simon RP. Clinical Neurology. 6th ed. New York, NY: McGraw-Hill; 2005.

Q 92.4: A person with an exaggerated sense of entitlement and uniqueness and who believes they can only be understood by people of significance is described to you by a colleague. They go on to state the person is arrogant, is lacking in empathy, and can be manipulative with relationships. What personality disorder best fits this scenario? A histrionic B narcissistic C antisocial D borderline

The Correct Answer is: B The scenario represents a typical "snapshot" of this diagnosis. These persons typically have fantasies of unlimited success and have a strong need for admiration from others. They can be jealous of others but commonly assume that others are extremely jealous of them. Treatment is made difficult as they do not accept criticism or any attack on their "narcissistic supply." (Eisendrath and Lichtmacher, 2009, p. 925-926; Sadock and Sadock, 2008, p. 384) Eisendrath SJ , Lichtmacher JE. Psychiatric disorders. In: McPhee SJ , Papdakis MA, eds. Current Medical Diagnosis and Treatment, 48th ed. New York: McGraw-Hill; 2009. Sadock BJ , Sadock VA. Concise Textbook of Clinical Psychiatry, 3rd ed. Philadelphia, PA: Lippincott, Williams & Wilkins; 2008.

Q 87.1: A 49-year-old male presents to the clinic with symptoms of nausea, occasional vomiting, vague epigastric pain, fatigue and weight loss of 35 lbs. over the past few months. On exam you find a palpable abdominal mass. Which of the following laboratory findings are most consistent with the suspected diagnosis? A Elevated CEA B Iron deficiency anemia C Elevated CA 125 D Megaloblastic anemia E Elevated CRP

The Correct Answer is: B The suspected diagnosis is gastric carcinoma. The most common laboratory finding is iron deficiency anemia related to chronic blood loss. Tumor markers are of no value, and an elevated CRP is not commonly present. Megaloblastic anemia is not the type of anemia typically present in gastric carcinoma. (McPhee SJ, Papadakis MA. Current Medical Diagnosis & Treatment, 2010, p. 1471)

Q 65.4: A 24-year-old male has an eight-month history of loose thought associations, social withdrawal, auditory hallucinations, and deterioration in his personal appearance and hygiene. Upon examination, he is noted to have a flat affect and perceptual distortions, and he behaves like he is detached from his own actions. He is started on a neuroleptic medication, and a few weeks later he is noted to pace frequently and seems to be unable to sit or stand still. What is the extrapyramidal symptom this patient is exhibiting called? A Acute dystonia B Akathisia C Drug-induced parkinsonism D Tardive dyskinesia E Verbigeration

The Correct Answer is: B The symptom this patient is exhibiting is akathisia, the most common extrapyramidal symptom of the neuroleptic medications. Acute dystonias from neuroleptic medications consist of bizarre muscle spasms of the head, neck, and tongue. Drug-induced parkinsonism consists of the same symptoms as idiopathic parkinsonism, including signs of reduced facial and arm movements, festinating gait, rigidity, and pill-rolling tremor. Tardive dyskinesia usually appears months or years after starting neuroleptic medication, and consists of involuntary stereotyped movements of the face, mouth, tongue, trunk, and limbs. Verbigeration is a symptom of schizophrenia and other psychotic disorders that consists of repetition of senseless words or phrases, but is not a side effect of neuroleptic medication.

Q 65.3: A 56-year-old woman is currently being treated with daily warfarin for thrombophlebitis. She has contracted a serious lower respiratory tract infection and is admitted to the hospital. The patient is started on ciprofloxacin upon admission, and after 3 days of treatment, her INR increases from 2.7 to 7.4. She also reports a nosebleed on the third night in the hospital. Her lower respiratory function has improved slightly, but the infection has still not resolved. Which of the following is the most likely explanation for the increase in the patient's INR? A decreased warfarin absorption in the small intestine B decreased warfarin metabolism by the liver C increased plasma protein binding of warfarin D increased warfarin absorption in the small intestine E increased warfarin metabolism by the liver

The Correct Answer is: B There are several clinically important warfarin drug interactions, with most of them causing an increase in the drug's anticoagulant effect (ie, increasing the INR). Warfarin metabolism occurs via hepatic cytochrome P450 enzymes that can be inhibited by a large number of drugs, including the fluoroquinolones. When this inhibition occurs, plasma levels of warfarin rise, thereby enhancing the anticoagulant effect.

Q 121.11: A 66-year-old male with a history of hypertension, diabetes mellitus, and hypercholesterolemia presents by emergency medical services (EMS) to the emergency department complaining of severe chest pain with radiation into his back. The patient states that he was feeling well in the morning, but while performing some light activity he felt a "ripping" sensation in his back, which he initially thought was a pulled muscle. The pain continued and the patient started to have chest pain, shortness of breath, and lightheadedness. On initial examination the patient is still in pain, pale, diaphoretic, and has a blood pressure of 85/40. His chest is clear to auscultation, and he has a 3/6 diastolic murmur best appreciated at the base of the heart. Given this clinical scenario, what is the most likely diagnosis? A Pneumothorax B Dissecting thoracic aneurysm C Acute myocardial infarction D Pulmonary embolus E Esophageal perforation

The Correct Answer is: B This patient is exhibiting a history and physical examination that is consistent with a thoracic aneurysm. The patient's history of hypertension, along with the "ripping" sensation in his back and hypotension give a clinical presentation that is suggestive of a thoracic aneurysm dissection (B). A pneumothorax (A) would have more pleuritic characteristics, and chest pains without the ripping sensation or loud diastolic murmur would be more likely in a situation of myocardial infarction (C). Pulmonary embolus (D) and esophageal perforation (E) would typically not present with the above complaints or physical exam findings. Source: http://www.accessmedicine.com/content.aspx?aID=3651494

Q 63.7: A 29-year-old female complains of a two-month history of easy bruising. She describes that the bruising is located primarily on her shins, but has noted them on other areas as well. She also describes red freckles on her lower extremities. On exam, you note non-blanching and non-palpable purpura to both legs and petechiae. She denies recent illnesses and states that she has essentially been feeling fine. The PE is normal other than the skin findings. Which diagnostic study would be most helpful as a first choice? A Antiplatelet antibody test B Complete blood count C Direct antiglobulin test D Reticulocyte count E Thyroid function

The Correct Answer is: B This patient is exhibiting signs of Idiopathic (Immune) thrombocytopenic purpura. The diagnosis is based on history, physical examination, blood count, and blood film. The American Society of Hematology guidelines recommend no further diagnostic studies. The other studies may be useful if you believe another underlying disease is causing the symptoms and thrombocytopenia, but a complete blood count should be sufficient.

Q 66.6: An 18-year-old male presents with a rash consisting of erythematous target-like lesions on his arms. Physical exam shows a healing cold sore on his lips, but no other skin or mucosal lesions are present. What is the most likely cause? A Borrelia burgdorferi B Herpes simplex virus C Rickettsia rickettsia D Staphylococcus aureus

The Correct Answer is: B This patient is exhibiting the classic rash of erythema multiforme minor. Etiology of this disorder can be traced to herpes simplex outbreaks or other viral or bacterial infections. Recurrent disease is most often associated with herpes simplex outbreaks. (Kane et al., 2002, Fig. 15-5, pg. 328-329)

Q 72.9: A 62-year-old man with a history of hypertension, diabetes mellitus type 2, hyperlipidemia, and chronic tobacco use presents to the office with complaints of a retrosternal chest pressure, associated with diaphoresis, nausea, and dyspnea, radiating down his left arm for the last 45 minutes after mowing his lawn. The patient's vital signs are stable, and on physical examination a new systolic murmur is appreciated. His EKG demonstrates evidence of acute anterolateral myocardial infarction on EKG, with ST segment elevation across the precordial leads, indicative of left anterior descending coronary artery stenosis. Which of the following cardiac markers would be expected to remain elevated one week later? A CK-MB B Troponin I C BNP D Creatine kinase E Myoglobin

The Correct Answer is: B Troponin elevation in acute myocardial infarction may be noted within two hours after myocardial infarction. It is usually elevated within 6 to 10 hours, peaks at 12 hours, and may remain elevated for 7 to 10 days; thus, choice B is the answer. Troponin elevation is rapidly replacing CK-MB as the diagnostic cardiac marker of choice for AMI. Choice A, CK-MB, peaks earlier than creatine kinase, and is cleared within two days. Choice C, BNP, is a cardiac marker followed in patients with congestive heart failure, and unless the patient develops heart failure in the next 7 days, is unlikely to be elevated. Creatine kinase becomes elevated within 4 to 8 hours, peaks within 12 to 24 hours, and returns to normal within 3 to 4 days. Serum myoglobin levels rise within 3 hours of symptoms and are elevated at 6 to 8 hours. Myoglobin peaks at 4 to 9 hours, and, with normal kidney function, returns to baseline within 24 hours. (Tintinalli et al., 2011, Chapter 52)

Q 91.4: A 46-year-old African American male is seen in the emergency department with upper right quadrant pain that radiates to the right infrascapular area. The pain is colicky and was precipitated by a meal of fried fish and French fries. Which of the following diagnostic studies is the initial study of choice for this patient? A Plain abdominal x-ray B Ultrasonography C Radionuclide scan (HIDA scan) D Computed tomography (CT)

The Correct Answer is: B Ultrasonography is the first-line study in the evaluation of patients presenting with signs and symptoms of biliary disease. The sensitivity and specificity is 95%. It can detect stones, dilation of biliary ducts, thickening of the gallbladder, and pericolic collections of fluid and can also provide information pertaining to associated liver or pancreatic pathology. (Danziger, 2006, p. 339) Danzinger RG , Nauta R , Park J. Biliary tract. In: Lawrence PF, ed. Essentials of General Surgery. 4th ed. Philadelphia, PA: Lippincott Williams & Wilkins; 2006.

Q 89.9: The best initial diagnostic modality to diagnose cholelithiasis is which one of the following? A CT scan of the abdomen B ultrasound of the abdomen C oral cholecystogram D abdominal plain film E MRI of the abdomen

The Correct Answer is: B Ultrasound has replaced oral cholecystograms as the test of choice for diagnosing cholelithiasis. CT is useful in the evaluation of the acute abdomen but the sensitivity for viewing the gallstones is poor. KUB is also not a sensitive study for cholelithiasis. MRI is expensive and not recommended as an initial screening exam for gallstones but can be used if ultrasound is equivocal. (Paumgartner, 2009, pp. 541-542) Paumgartner G , Greenberger NJ. Gallstone disease. In: Greenberger NJ, ed. Current Diagnosis & Treatment: Gastroenterology, Hepatology, & Endoscopy. New York, NY: McGraw-Hill; 2009.

Q 57.1: In a person with normal marrow function what is the mean life span of platelets? A 3 to 6 days B 7 to 10 days C 11 to 14 days D 15 to 18 days E 19 to 22 days

The Correct Answer is: B Under normal conditions, human platelets have a mean life span in circulation of 7 to 10 days. In order to confirm that the thrombocytopenia is only related to the surgery, waiting at least a week for full platelet conversion is needed.

Q 61.7: Your 24-year-old female patient has a longstanding history of hypercoaguability disorders and is on lifelong anticoagulation due to prior pulmonary emboli. She advises you that she is attempting to get pregnant. Which of the following medications should be utilized during her pregnancy? A aspirin B enoxaparen C heparin D warfarin E none of the above

The Correct Answer is: B Warfarin therapy is contraindicated in the first trimester due to its association with fetal chondrodysplasia punctata. In the second and third trimesters, warfarin may cause fetal optic atrophy and mental retardation. In general, all diagnostic and therapeutic modalities afforded the nonpregnant patient should be utilized in pregnancy. Outpatient anticoagulant therapy, with low-molecular-weight heparin (LMWH) such as enoxaparen, is indicated in pregnant women at high risk for deep venous thrombosis (DVT). LMWH may be associated with an increased risk of epidural hematoma in women receiving an epidural anesthetic in labor. One approach to this problem is to switch from LMWH to unfractionated heparin before the anticipated delivery date.

Q 109.15: A 74-year-old male has been diagnosed with aplastic anemia. His temperature is 37.4°C, heart rate is 68, respiratory rate is 20, and blood pressure is 130/86. Mild petechiae are noted to the extremities and there is some scattered bruising. The patient is currently taking Captopril, HCTZ, Lipitor, and Flo-max. Laboratory findings include: WBC: 2.9 HGB: 10.4 PLT: 54,000 Initial management for this patient should include which of the following? A Broad spectrum antibiotics B Discontinuation of Captopril C Immediate assessment for allogeneic stem cell transplant D Irradiated and leukocyte depleted red cell transfusion E Platelet transfusion

The Correct Answer is: B While the laboratory findings demonstrate a pancytopenia, levels are not low enough or physical findings are not suggestive of the need for transfusions at this time. Given the patients age, he is not eligible for an allogeneic stem cell transplant. He has no physical findings suggestive of a systemic infection requiring antibiotics. Captopril has a low risk of causing pancytopenias. It is best to discontinue this drug first as a possible cause of the pancytopenia, while continue to treat this patient symptomatically. (Lichtman et al., Williams Hematology 8e, Chapter 34, Aplastic Anemia: Acquired and Inherited)

Q 105.13: All other things being equal, the level of coding allowed in the outpatient setting is most dependent upon which of the following? A chronicity of the patient complaint B complexity of medical decision-making and/or time spent with the patient C number of items detailed within each section during the review of systems D patient insurance E patient acuity

The Correct Answer is: B With all other things being equal, the complexity of the medical decision-making is usually what determines coding levels. A patient visit may also be coded based upon the time spent with the patient. A minimum number of items must be documented in the History of Present Illness, the physical examination, and the review of systems, along with other factors, but medical decision making cannot be impacted positively by completing what is ostensibly unnecessary information. (LeBlond et al., 2009, Chapter 2)

Q 117.7: Myasthenia gravis symptomology is due to an autoimmune attack on which of the following receptors? A Alpha-adrenergic B Acetylcholine C Calcium D Dopamine E Norepinephrine

The Correct Answer is: B Within normal neuromuscular junctions, the release of acetylcholine and stimulation of the acetylcholine receptors result in muscle contraction. In myasthenia gravis, the available acetylcholine receptors are decreased due to autoimmune attack. This leads to muscle weakness and decreased functionality. Alpha-adrenergic receptors are the receptors for catecholamines, such as norepinephrine. The functions are diverse, based upon subtype and location, but may include vasoconstriction of arteries and veins and decreased gastrointestinal smooth muscle motility. Dopamine, a neurotransmitter and form of catecholamine, has unique receptors. The calcium-sensing receptor in the parathyroid regulates calcium homeostasis, which is unrelated to myasthenia gravis. (Fauci et al., Harrison's Principles of Internal Medicine, 17e, Chapter 381, Myasthenia Gravis and Other Diseases of the Neuromuscular Junction)

Q 80.8: The source of most cases of epistaxis comes from what anatomic location? A Posterior nasal septum B Frontal sinuses C Anterior nasal septum D Anterior palate E Maxillary sinuses

The Correct Answer is: C 95 percent of epistaxis come from Kesselbach's plexus, which is a superficial, fragile group of arterioles and veins that are the most likely cause of nosebleeds. Five percent are posterior bleeds that originate along the sphenopalentine artery. (Tintinalli et al., 2011, Chapter 239)

Q 65.10: A patient has a 2-day history of an itchy red left eye and marked tearing. There is no history of injury. On physical exam the conjunctiva appear markedly erythemic. What physical finding would help most in differentiating this as a viral conjunctivitis? A Copious discharge B Subconjunctival hemorrhage C Pre-auricular lymph node D Vesicular rash around the eye E Punctate keratitis

The Correct Answer is: C A palpable pre-auricular lymph node is most often seen with viral conjunctivitis and rarely seen in bacterial conjunctivitis.

Q 81.7: Which pharmacologic agents have phototoxicity as a side effect? A Tetracycline, penicillin, and metoprolol B Penicillin, ketoconazole, and metrogel C Doxycycline, hydrochlorothiazide, and naproxen D Cephalexin, acetaminophen, and metoprolol

The Correct Answer is: C A phototoxic reaction is one that results from exposure to the drug and exposure to UV light. The reaction occurs in exposed sites. (Wolff & Johnson, Table 10-4, pg 239) (Fig, 10-22, Wolff K, Johnson RA: Fitzpatrick's Color Atlas & Synopsis of Clinical Dermatology, 6 th Ed: http://www/accessmedicine.com )

Q 107.5: A 29-year-old female has a long history of supraventricular tachycardia, for which she has been treated with long-term flecanide, as well as prior therapy with verapamil. She continues to have repeated episodes, sometimes two to three times a week, along with shortness of breath and at times hypotension that has been recorded. What is the next best therapy for this patient? A Synchronized cardioversion B Cardiac catheterization C Ablation therapy D Pacemaker insertion E Long-term telemetry monitoring

The Correct Answer is: C After exhaustion of non-invasive therapies, ablation therapy can be used to try to negate the aberrant pathway for SVT. Pacemakers will not allow for an override of the pathway, and cardioversion is only a temporary solution to an acute event. Implantable telemetry monitoring is only diagnostic and not therapeutic to treat. (Calkins H. Hurst's the Heart, Chapter 38, Supraventricular Tachycardia: AV Nodal Reentry and Wolff-Parkinson-White Syndrome)

Q 102.6: Which nonnicotine related agent has been approved by the FDA as a first-line medication in the treatment of smoking cessation? A nortriptyline B clonidine C bupropion D fluoxetine

The Correct Answer is: C Bupropion SR has been approved by the U.S. Food and Drug Administration (FDA) for smoking cessation. The drug has been successful in doubling cessation rates. Side effects include dry mouth, agitation, insomnia, and headache. (Rigotti, 2008, p. 166) Rigotti NA. Smoking. In: Feldman MD , Christensen JF, eds. Behavioral Medicine in Primary Care: A Practical Guide, 3rd ed. New York: McGraw-Hill; 2008.

Q 110.16: During a spirometry test, a patient is asked to forcibly expel as much air from the lungs as possible. Which of the following represents the amount of air that remains in the patient's lungs following this maximal forced expiration? A expiratory reserve volume B functional residual capacity C residual volume D tidal volume E vital capacity

The Correct Answer is: C By definition, residual volume is the volume of air remaining in the lungs after a maximal forced expiration. The residual volume is important physiologically, as it prevents total collapse of the alveoli and minimizes the pressure and energy required to inflate the lungs during inspiration. (Boulpaep, 2009, pp. 625-626; Costanzo, 2006, pp. 185-187) Boulpaep EL. The microcirculation. In: Boron WF , Boulpaep EL, eds. Medical Physiology. 2nd ed. Philadelphia, PA: Saunders Elsevier; 2009. Costanzo LS. Physiology. 3rd ed. Philadelphia, PA: Saunders Elsevier; 2006.

Q 106.1: A 55-year-old woman with a history of emphysema, who is undergoing chemotherapy for lung cancer, is sent to see you by her oncologist regarding a sudden increase in dyspnea, with exertion and fatigue. Which of the following physical exam findings would predict a cardiac etiology for her dyspnea instead of a pulmonary etiology? A Crackles at the lung bases bilaterally B Tachycardia C Pulsus paradoxus D Soft S1, S2 E Wheezing

The Correct Answer is: C Choice C is the best choice, as pulsus paradoxus is frequently seen in patients with cardiac tamponade. Patients diagnosed with cancer, particularly of the lung and breast, may accumulate fluid within the pericardial sac, leading to cardiac tamponade. The finding of pulsus paradoxus is defined as a decrease in systolic arterial pressure of greater than 10 mmHg. It is an accentuation of the normal decrease in systolic arterial pressure of less than 10mm Hg that normally accompanies inspiration. Symptoms of dyspnea and fatigue in a patient with lung cancer and emphysema are not uncommon. The other choices are nonspecific to a cardiac etiology. Choice A, crackles at the bases, could be in pulmonary etiology. Choice B, tachycardia, is also nonspecific, and can be found in patients who are febrile, or with lung disease. Choice D, a soft S1, S2, may be found in patients with emphysema and an increased AP diameter. Choice E is likely pulmonary. (LeBlond et al., 2009, Chapter 8)

Q 68.10: A 70-year-old man with a history of congestive heart failure, CABG x 2, HTN, and diabetes mellitus, type 2, presents to the office with complaints of increasing dyspnea while walking across the room, 3 pillow orthopnea, and worsening lower extremity edema since he returned from a cruise. He admits to taking all of his medications regularly, including lisinopril 20 mg daily, carvedilol 12.5 mg twice daily, spironolactone 25 mg daily, furosemide 20 mg daily, and aspirin 81 mg daily. On physical exam, his vital signs are stable, although he has gained 10 lbs compared to his last office visit three months ago. He is also demonstrating jugular venous distention to the angle of the jaw, at 90 degrees of truncal elevation. There are crackles at the lung bases bilaterally, and 1+ pitting edemas to the knees bilaterally. His EKG is unchanged from previously (see Figure 7). Which of the following is the most likely etiology for this patient's acute exacerbation of congestive heart failure? A Labile hypertension B Acute myocardial infarction C Dietary noncompliance D Acute chordae tendinae rupture E Severe anemia

The Correct Answer is: C Choice C is the most likely etiology for this patient's acute exacerbation of congestive heart failure. Although choices A, B, D, and E are all possible causes, a more common, though mundane cause of acute exacerbation of congestive heart failure, is dietary noncompliance, especially related to sodium restriction. Patients with congestive heart failure should maintain low sodium diets, which is difficult in everyday settings, and nearly impossible aboard a cruise ship while on vacation. There is no evidence to support labile hypertension as the cause of this patient's symptoms, as his blood pressure is currently well-controlled. Choice B is also less likely than choice C, given that the patient is not demonstrating chest discomfort and no EKG changes are noted. Choice d is less likely than choice C, as on physical exam there is no new murmur noted. Choice E is less likely than choice C in this patient, as no history or physical exam findings are noted that would raise the suspicion for severe anemia. (McPhee et al., 2011, Chapter 10)

Q 119.14: A 57-year-old man with a history of HTN, hyperlipidemia, and chronic tobacco use presents to the office with complaints of chest tightness that occurs every time he begins raking leaves. If he stops and rests, it is relieved within 5 minutes. He has no associated nausea or diaphoresis, but does admit to associated dyspnea. Which of the following is the most likely diagnosis? A Pericarditis B Acute myocardial infarction C Stable angina pectoris D Prinzmetal angina E Myocarditis

The Correct Answer is: C Choice C, stable angina pectoris, is chest or arm discomfort that is reliably precipitated by activity and/or emotional distress, and relieved with rest or sublingual nitroglycerin. Choice A, pericarditis, would present with chest discomfort that is worse while supine and improves while sitting up, as well as a pericardial friction rub. Choice B, acute myocardial infarction, requires troponin elevation to establish the diagnosis. Choice D, prinzmetal angina, or variant angina pectoris, is defined as coronary artery spasm associated with ST-segment elevation, usually occurring at rest, and frequently at the same time of the day. Choice E, myocarditis, is usually preceded by a viral prodrome. (Fauci et al., 2008, Chapter 238)

Q 66.7: A 57-year-old woman with a history of rheumatic fever is seen complaining of dyspnea while vacuuming her apartment, which has been worsening over the last few months. On physical exam, jugular venous distension is appreciated. Auscultation of the chest reveals a possible opening snap, loud S 1 , and a very soft diastolic rumbling murmur is auscultated at the left lower sternal border. When the patient is placed in the left lateral decubitus position, the murmur is accentuated, and heard best at the apex. Both hepatomegaly and splenomegaly are noted. On EKG, no evidence of right ventricular hypertrophy is noted, despite the obvious signs and symptoms of right heart failure. Which of the following valvulopathies should be suspected given this patient's history and physical exam findings? A Mitral stenosis and aortic stenosis B Mitral stenosis and aortic regurgitation C Mitral stenosis and tricuspid stenosis D Aortic stenosis and mitral regurgitation E Mitral stenosis and pulmonic stenosis

The Correct Answer is: C Choice C, tricuspid stenosis, is frequently seen in association with mitral valve stenosis in patients with a history of rheumatic heart disease (between 5% and 10% of the time). Both murmurs are similar in character, with the main difference that the murmur of tricuspid stenosis is heard best at the left lower sternal border, and the murmur of mitral stenosis is heard best in the left lateral decubitus position with the bell at the apex. Because they are similar in nature, a high level of suspicion for tricuspid stenosis should be maintained, so that tricuspid stenosis is not overlooked. Tricuspid regurgitation is also frequently seen in association with tricuspid stenosis. Patients frequently present with signs and symptoms of right-sided heart failure, including hepatic congestion and splenomegaly. As tricuspid stenosis and regurgitation are present, right ventricular hypertrophy may not occur, and thus would not be demonstrated on EKG. Choices A, B, D, and E are not seen as frequently as tricuspid stenosis in association with rheumatic heart disease and mitral valve stenosis. (Fauci et al., 2008, Chapter 230)

Q 120.15: A 23-year-old female presents with a history of rigid thought patterns and a need for control. She sees herself as a perfectionist. She discloses that she feels a need to check the locks on her doors at home once every 30 minutes. She is consumed with these thoughts about locking the doors. What is the most likely classification for her personality disorder? A Histrionic B Narcissistic C Obsessive-compulsive D Paranoid E Schizotypal

The Correct Answer is: C Clinical findings of obsessive-compulsive disorder include being a perfectionist, egocentric, and indecisive, with rigid thought patterns and need for control. Clinical findings of histrionic personality disorder include being dependent, immature, seductive, egocentric, vain, and emotionally labile. Narcissistic personality disorder presents with the clinical findings of grandiosity, a preoccupation with power, lacking interest in others, and excessive demands for attention. Clinical findings of someone who has paranoid personality disorder would include defensiveness, being overly sensitive, secretive, suspicious, hyper-alert, and with a limited emotional response. Schizotypal clinical findings include being superstitious, socially isolated, and suspicious, and having limited personality ability, odd speech, and eccentric behaviors. (McPhee SJ, Papadakis MA. Current Medical Diagnosis & Treatment, 2010, p. 951)

Q 74.3: A 45-year-old man with a history of NSTEMI, CABG X 3, HTN, and hyperlipidemia presents to your office with complaints of progressive dyspnea over the last three weeks, to the point that he is now dyspneic while walking across the room. In the last few days, he has noticed bilateral lower extremity edema. Which of the following findings on physical exam would meet the criteria for a diagnosis of congestive heart failure, according to the modified Framingham clinical criteria for the diagnosis of heart failure? A Nocturnal cough B Tachycardia C Third heart sound D Pleural effusion E Hepatomegaly

The Correct Answer is: C Diagnosis of heart failure requires that the findings of two major criteria, or one major and two minor criteria, cannot be attributed to another medical condition. The patient demonstrates two minor criteria: bilateral lower extremity edema and dyspnea on ordinary exertion. Only choice C (third heart sound) falls under the heading of major criteria. Choice A, B, D, and E all fall under the heading of minor criteria. A third heart sound, or S3, is representative of early rapid filling of the left ventricle, and can occur in any condition in which there is an increased left ventricular volume, such as congestive heart failure. (Senni et al., 1998, 98:2282)

Q 112.3: A 48-year-old African American male presents with dyspnea, 2-pillow orthopnea, and swelling to his lower legs that has developed over the last month. He also complains of fatigue and decreased exercise tolerance, stating that he has trouble climbing one flight of steps. On physical examination, his blood pressure is 178/98, pulse rate is 102, and respiratory rate is 20. There is 5 cm JVD, crackles at the bilateral lung fields, and tachycardia and an S 3 is heard on cardiac auscultation. There is 2+ pitting edema to the lower extremities. His electrocardiogram reveals a sinus tachycardia at a rate of 105 and left ventricular hypertrophy. The chest x-ray reveals cardiomegaly with increased interstitial markings in all lung fields. There is a small right pleural effusion that blunts the costophrenic angle. Which initial diagnostic test gives you the best information regarding this patient's pathology? A Exercise stress test B Pulmonary function test C Echocardiogram D Ventilation/perfusion scan E CT scan of chest

The Correct Answer is: C Echocardiogram is indicated in determining the extent of the cardiomyopathy. This test will examine aspects of wall motion and valvular competency, as well as estimate an ejection fraction. Exercise stress tests are utilized more for evaluation of ischemia than function of the myocardium. (Bashore et al., Current Medical Diagnosis and Treatment, Chapter 10)

Q 110.5: A 36-year-old male returns to the office for follow up of his heartburn symptoms. He continues to have heartburn three to four times a week, even though he has been compliant with his proton pump inhibitor medication since it was prescribed three months ago. He underwent an endoscopy three weeks ago, which was normal. He denies dysphagia, early satiety, or weight loss. Which of the following tests is the most appropriate next step in evaluating this patient? A Repeat endoscopy B Barium esophagography C Esophageal manometry D Abdominal CT scan E Urea breath test

The Correct Answer is: C Esophageal monometry is the answer, and this is useful in patients who have persistent symptoms, despite PPI therapy and individuals who have had normal findings on endoscopy. There is no reason to repeat the endoscopy, as the first one was normal. Barium esophagography plays a limited role in GERD, and could be helpful if the patient complained of dysphagia. Abdominal CT is not indicated in this case. Urea breath test is used in the detection of H. pylori, which is an uncommon cause of dyspepsia in the absence of peptic ulcer disease. (McPhee SJ, Papadakis MA. Current Medical Diagnosis & Treatment, 2010, p. 531)

Q 79.6: Anticoagulation for atrial fibrillation in a 65-year-old with hypertension aims for an international normalized ratio (INR) of: A < 1.0 B 1.0 to 2.0 C 2.0 to 3.0 D 3.0 to 4.0 E > 5.0

The Correct Answer is: C For patients with atrial fibrillation, most clinicians use a CHAD-score to determine if anti-coagulation is necessary. For patients with no contraindication to anticoagulation and at least one risk factor for stroke, anti-coagulation is recommended. Once that determination is made, anticoagulation with warfarin to an INR target of 2.0 to 3.0 should be established and maintained indefinitely. (McPhee and Papadakis, 2011, Chapter 10)

Q 115.16: Use of which of the following medications can result in hearing loss? A Cefalexin B Erythomycin C Gentamycin D Ciprofloxacin E Hydrochlorothiazide

The Correct Answer is: C Gentamycin is an aminoglycoside, and can cause ototoxicity. Peak and trough levels must be drawn to determine the lowest effective dose. The remaining medications do not interfere with vestibular function. (Lalwani A.K., 2008, Chapter 53)

Q 111.10: Which of the following viruses is rodent-borne and the cause of hemorrhagic fever and a pulmonary syndrome, which begins with a fever and may rapidly progress to shock and adult respiratory distress syndrome? A human T-cell lymphotropic virus (HTLV) B Flavivirus C Hantavirus D Filovirus E coronavirus

The Correct Answer is: C Hantavirus has a rodent vector and usually manifests in either hemorrhagic fever or Hantavirus pulmonary syndrome, which can be fatal. In the United States, outbreaks are usually in the southwest. There have been 300 cases since 1993. HTLV is a lymphotropic oncovirus associated with lymphoma. Dengue and yellow fever are both caused by Flaviviridae, which is carried by mosquitoes. Filoviruses cause Ebola fever and Marburg fever. The vector is unknown. Coronavirus is the etiologic agent in severe acute respiratory syndrome. During the 2002-2003 epidemic that began in Southeast Asia, it was postulated that it was carried by the masked palm civet. (Shandera and Corrales-Medina, 2009, pp. 1233-1234) Shandera WX , Corrales-Medina VF. Viral & rickettsial infections. In: McPhee SJ , Papadakis MA, eds. Current Medical Diagnosis and Treatment. 48th ed. New York, NY: McGraw-Hill; 2009.

Q 109.12: A 28-year-old male presents with a diagnosis of Hemophilia A. Which of the following would he be deficient in? A Christmas factor B Factor VII C Factor VIII D Factor IX E Von Wildebrand's factor

The Correct Answer is: C Hemophilia A is caused by a defective synthesis of Factor VIII. Factor IX and Christmas factor are deficiencies in Hemophilia B. Deficiency of factor VII can exacerbate bleeding issues, but are not typically seen in Hemophilia A. Von Willebrand's factor is a loss of this platelet-binding multimer, and most patients will have enough factor VIII but not the ability to bind factor VIII. In very severe cases of vWF, factor VIII may become low enough to produce symptoms similar to those found in factor VIII deficiency. (Lichtman et al., Williams Hematology 8e, Chapter 124, Hemophilia A and Hemophilia B)

Q 64.3: A 37-year-old female, who is two weeks post caesarean section, develops acute dyspnea and chest pain. A chest CT reveals a left segmental perfusion defect. Which of the following is the next step in the management of this patient? A Aspirin B Embolectomy C Heparin D Pulmonary angiography E Streptokinase

The Correct Answer is: C Heparin is indicated as initial therapy for acute pulmonary thromboembolism, followed by oral anticoagulation with warfarin. Heparin promotes the effect of antithrombin, which inhibits factors Xa, IXa, Xia, and XIIa, and has been shown to decrease mortality and recurrent pulmonary embolism. Streptokinase, a thrombolytic agent, is recommended for hemodynamically unstable patients being treated with heparin, but with continued risk of death. Embolectomy, although associated with increased mortality, is another alternative for these patients. Pulmonary angiography, the gold standard for pulmonary embolus diagnosis, is being replaced with helical contrasted CT, due to angiography's invasiveness, time involvement, and cost. Aspirin, an antithrombotic agent, inhibits platelet aggregation and is effective for preventing platelet thrombosis. It also has a role in thrombosis prevention. However, anticoagulation with heparin remains the mainstay of therapy for pulmonary embolus.

Q 118.1: Which of the following is a potential adverse effect associated with unfractionated heparin (UFH)? A hyperglycemia B hypothyroidism C thrombocytopenia D excessive cough E muscle cramps

The Correct Answer is: C Heparin-induced thrombocytopenia (HIT) is a potentially serious complication of unfractionated heparin therapy, usually occurring within 4 to 10 days after heparin treatment has started. Fortunately, current estimates show that it is infrequent, occurring in approximately 0.3% to 3.0% of patients receiving UFH for more than 4 days. HIT should immediately be suspected in a patient who develops deep vein thrombosis or pulmonary embolism while receiving UFH. (Whitby and Johns, 2008, pp. 1710-1712) Whitby DH , Johns TE. Drug-induced hematologic disorders. In: DiPiro JT , Talbert RL , Yee GC, et al., eds. Pharmacotherapy: A Pathophysiologic Approach. 7th ed. New York: McGraw-Hill; 2008.

Q 67.6: A 42-year-old male presents with a history of low grade fever, cough, and myalgias for five days. He states that these symptoms began after a cave exploration trip along the Ohio River two weeks ago, and have since worsened. An x-ray reveals focal consolidation, and you suspect Histoplasmosis pneumonia. Which of the following is the first line treatment of choice? A Azithromycin B Doxycycline C Itraconazole D Terbinafine E Trimethoprim-sulfamethoxazole

The Correct Answer is: C Histoplasmosis is caused by a dimorphic fungus, most commonly Histoplasma capsulatum (although other species exist). Within the United States, endemic areas include the Ohio and Mississippi river valleys. Additional areas of risk include other parts of North, South, and Central America, Africa, Mexico, and Central Asia. Large amounts of bird and bat droppings within specific soils promote the growth of the fungus, and exposure typically occurs during activities that disrupt the soil and aerosolize the spores. Depending upon the length and intensity of exposure, and the patient's immune system and previous lung history, infections may range from asymptomatic to severe. Treatment is based upon the patient's clinical picture, with mild to moderate disease being treated with oral antifungal agents. (McPhee SJ, Papadakis MA. Current Medical Diagnosis & Treatment 2011, Chapter 36, Mycotic Infections)

Q 99.2: A 30-year-old patient presents 2 months postthyroidectomy. The patient has had symptoms of increased irritability, muscle spasms, and hair loss for the past month. On physical examination, a positive Chovstek sign is noted. Which of the following is the most likely diagnosis? A hypothyroidism B hypopituitarism C hypoparathyroidism D hypogonadism

The Correct Answer is: C Hypoparathyroidism commonly presents following thyroidectomy surgery. This patient has classic signs and symptoms of a low calcium level and hypoparathyroidism. Chovestek sign is a physical exam finding that is positive after tapping in front of the ear in the facial nerve region. When doing this, the muscle contracts. When the calcium level is low, this occurs. Hypothyroidism can occur following a thyroidectomy but the symptoms are not the same. (Fitzgerald, 2009, pp. 1004-1005) Fitzgerald PA. Endocrine diseases. In: McPhee SJ , Papadakis MA, eds. Current Medical Diagnosis and Treatment. 48th ed. New York, NY: McGraw-Hill; 2009.

Q 106.19: A 68-year-old male patient presents to your office complaining of difficulty urinating, nocturia that is interfering with his sleep to the point where he is fatigued, along with lower back pain. On physical examination, his prostate is enlarged, irregular, and slightly tender. His U/A is normal. Of the following items that may be in your differential diagnosis must you pursue fully? A benign prostatic hypertrophy B prostatitis C prostate cancer D testicular torsion E urinary retention

The Correct Answer is: C Incidental or stage A (T1) carcinoma of the prostate presents no physical signs (it is nonpalpable) and is only diagnosed by the pathologist when prostate tissue is removed as treatment for symptomatic bladder outlet obstruction presumed to be caused by benign prostatic hyperplasia or is found by an elevated PSA (T1c). Patients with stage B (T2) or higher disease have a hard nodule on the prostate that can be felt during rectal examination. Previously, 50% of patients presented with evidence of metastases, including weight loss, anemia, bone pain (commonly in the lumbosacral area), or acute neurologic deficit in the lower limbs. Today, however, fewer than 20% of patients present in this way because of earlier diagnosis due to wide use of PSA screening (stage migration). (Doherty, 2010, Chapter 38)

Q 104.1: Which of the following is indicated to confirm the diagnosis of celiac sprue in a patient with positive serologic testing? A stool for fecal fat B barium enema C intestinal biopsy D antimitochondrial antibodies E food challenge

The Correct Answer is: C Intestinal biopsy is the most specific test in establishing the diagnosis of celiac sprue in a patient who has a positive test for IgA endomysial antibody. Classic symptoms of malabsorption are more common in infants but less common in adults. Stool for fecal fat would be a nonspecific finding. Antimitochondrial antibodies are seen in patients with primary biliary cirrhosis. (McQuaid, 2009, pp. 543-544) McQuaid KR. Gastrointestinal disorders. In: McPhee SJ , Papadakis MA, eds. Current Medical Diagnosis and Treatment. 48th ed. New York, NY: McGraw-Hill; 2009.

Q 106.14: Which of the following drugs block the actions of leukotrienes and can be used for long-term control of mild persistent asthma? A cromolyn sodium B omalizumab C zafirlukast D nedocromil sodium E ipratropium bromide

The Correct Answer is: C Leukotrienes are inflammatory mediators that are generated within the lungs. When they bind to specific receptors, they induce a variety of responses, including bronchospasm and mucus production. Zafirlukast (and also montelukast) are leukotriene receptor antagonists that block these effects in the lungs and improve asthma symptoms. Zafirlukast is considered an alternative therapy for long-term control of asthma, as it has been shown to be less effective than inhaled corticosteroids. Both cromolyn and nedocromil are mast cell stabilizers and can also be used as an alternative treatment to inhaled corticosteroids. Omalizumab is an anti-IgE antibody, whereas ipratropium bromide is a muscarinic receptor antagonist. (Kelly, 2008, pp. 478, 488-490) Kelly HW , Sorkness CA. Asthma. In: DiPiro JT , Talbert RL , Yee GC, et al., eds. Pharmacotherapy: A Pathophysiologic Approach. 7th ed. New York: McGraw-Hill; 2008.

Q 110.12: Over a period of several months, a 62-year-old man has developed erectile dysfunction. He has no history of neurologic, kidney, or cardiovascular disease or diabetes mellitus. He takes a multivitamin and an occasional ibuprofen for aches and pains. He has never smoked cigarettes, drinks 1-2 glasses of wine with dinner on weekends, and uses no mind-altering drugs. Physical examination is remarkable only for bilateral gynecomastia. What is the most likely diagnosis? A breast cancer B depression C prolactinoma D steroid abuse E testicular cancer

The Correct Answer is: C Men with prolactinomas may experience erectile dysfunction, infertility, and, less commonly, gynecomastia. Breast cancer in men (A) presents as a usually as a unilateral mass. Men with depression (B) may have erectile dysfunction, but not gynecomastia. Steroid abuse (D) is associated with gynecomastia, but the patient would likely be showing other signs and symptoms. Testicular cancer (E), specifically germ cell cancer, is associated with gynecomastia in 5% of cases but this man has no testicular mass or swelling. Fitzgerald PA, Endocrine Disorders, in Current Medical Diagnosis and Treatment, 52 nd ed. 2013.

Q 76.7: A 43-year-old obese man presents for a health maintenance visit. On physical exam, it is noted that his waist circumference is 106 cm and blood pressure is 148/92 mm Hg. Which of the following fasting laboratory levels would suggest a diagnosis of metabolic syndrome (syndrome X) in this patient? A HDL of 45 mg/dL B LDL of 180 mg/dL C triglyceride of 190 mg/dL D glucose of 100 mg/dL

The Correct Answer is: C Metabolic syndrome is found in approximately 25% of Americans. It is defined as three or more of the following findings: waist circumference of greater than 102 cm in men or greater than 88 cm in women; serum triglyceride level of at least 150 mg/dL, HDL level of less than 40 mg/dL in men or less than 50 mg/dL in women; blood pressure of at least 130/85 mm Hg; and serum glucose level of at least 110 mg/dL. (Friedman and Herman-Bonert, 2009, p. 1105) Friedman TC , Herman-Bonert VS. Thyroid gland. In: McPhee SJ , Papadakis MA, eds. Current Medical Diagnosis and Treatment. 48th ed. New York, NY: McGraw-Hill; 2009.

Q 106.7: A 33-year-old woman treated with trifluoperazine for the past 3 months is seen in the emergency department because of recent-onset fever, stiffness and tremor, as reported by her accompanying sister. The patient also appears to be mildly confused when asked about location, day, and time. Her temperature is 104.5°F, and her serum creatine kinase (CK) level is markedly elevated. Which of the following has most likely occurred? A a delayed allergic reaction has occurred with trifluoperazine B tardive dyskinesia has begun to develop in the patient C the patient has developed neuroleptic malignant syndrome D the patient has developed serotonin syndrome E the patient has overdosed on trifluoperazine

The Correct Answer is: C Neuroleptic malignant syndrome is an uncommon but serious complication with therapeutic doses of antipsychotic drug therapy, particularly the first-generation (typical) class. Cardinal signs and symptoms include a body temperature above 100.4°F, altered state of consciousness, autonomic dysfunction, and rigidity. (Crismon et al., 2008, pp. 1113-1114; Eisendrath and Lichtmacher, 2008, pp. 917-919) Crismon ML , Argo TR , Buckley PF. Schizophrenia. In: DiPiro JT , Talbert RL , Yee GC, et al. eds. Pharmacotherapy: A Pathophysiologic Approach. 7th ed. New York: McGraw-Hill; 2008. Eisendrath SJ , Lichtmacher JE. Psychiatric disorders. In: Tierney LM Jr , McPhee SJ , Papadakis MA, eds. Current Medical Diagnosis & Treatment. 47th ed. New York: McGraw-Hill; 2008.

Q 71.11: A 43-year-old pilot is interested in quitting his 20-year habit of smoking. His medical history includes type 2 diabetes mellitus diagnosed 6 years ago for which he is currently taking metformin. Which of the following would be most appropriate to recommend to this patient? A alprazolam B clonidine C nicotine replacement therapy D nortriptyline E varenicline

The Correct Answer is: C Nicotine replacement therapy is relatively safe in the majority of patients and comes in many forms (transdermal patches, gums, sprays, and inhalers). Both clonidine and nortriptyline are considered second-line smoking cessation agents because of their many side effects. Neither has been approved by the FDA for smoking cessation. Alprazolam is also not indicated, and there is currently no evidence that it aids in smoking cessation. *Varenicline is a relatively new agent for smoking cessation and is a partial agonist to α 4 -β 2 nicotinic acetylcholine receptors. It has been approved by the FDA; however, varenicline is banned from use by pilots and air traffic controllers as per the Federal Aviation Administration (FAA) *(Doering et al., 2008, pp. 1090-1094) Doering PL , Kennedy WK , Boothby LA. Substance-related disorders: Alcohol, nicotine and caffeine. In: DiPiro JT , Talbert RL , Yee GC, et al., eds. Pharmacotherapy: A Pathophysiologic Approach. 7th ed. New York: McGraw-Hill; 2008.

Q 121.18: During the physical exam of a patient with a suspected pleural effusion, you ask the patient to make the sound "eee." You note on auscultation that the transmission is auscultated as "ay," suggestive of resonance through fluid. What is the name of this exam technique? A Bronchophony B Diaphragmatic excursion C Egophony D Tactile fremitus E Whispered pectoriloquy

The Correct Answer is: C Normal lungs transmit spoken sounds faintly and with indistinct syllables, except over main bronchi. An area of fluid, such as a pleural effusion, consolidation or atelectasis, and areas of fibrosis will increase sound transmission and alter the distinction of the sound. This occurs for both whispered and spoken sounds. The utilization of the spoken sound "eee," auscultating for a change to "ay" due to fluid within the lung fields, is termed egophony. The use of whispered sounds and generally spoken sounds to determine lung changes are termed whispered pectoriloquy and bronchophony respectively. Diaphragmatic excursion is performed to determine the thoracic diaphragmatic movement during respiration. Tactile fremitus assesses chest vibration during vocalization, with changes being noted in the presence of consolidation (increases fremitus) and pleural effusion (decreased or absent fremitus). (LeBlond RF, Brown DD, DeGowin RL. DeGowin's Diagnostic Examination, 9e, Chapter 8, The Chest: Chest Wall, Pulmonary, and Cardiovascular Systems; The Breasts)

Q 97.8: A patient presents with abdominal distension associated with nausea and vomiting. Which of the following findings is consistent with a paralytic ileus? A Crampy abdominal pain B Hyperactive bowel sounds C Obstipation and failure to pass flatus D Gas in small intestine only on KUB (kidney, ureter, bladder)

The Correct Answer is: C Obstipation and failure to pass flatus are actually symptoms of both paralytic ileus and a small bowel obstruction (SBO). However, patients with a paralytic ileus usually have minimal abdominal pain and hypoactive or absent bowel sounds due to hypomotility. Patients with an SBO will have crampy abdominal pain and increased bowel sounds with high-pitched sounds and rushes due to increased peristalsis. Plain films in paralytic ileus will show gas throughout the small and the large bowel on plain films as opposed to air confined to the small intestine only in SBO. (Mellinger, 2006, pp. 297-299) Mellinger JD , Macfadyen BV , Mercer DW , et al. Small intestine and appendix. In: Lawrence PF, ed. Essentials of General Surgery. 4th ed. Philadelphia, PA: Lippincott Williams & Wilkins; 2006.

Q 66.3: A 52-year-old female is taking daily ibuprofen due to severe degenerative joint disease. She is concerned about developing an ulcer, but refuses to stop the ibuprofen. Which of the following medications is approved for the prevention of NSAID-induced gastric and duodenal ulcers, and should be recommended to this patient? A Ranitidine B Ondansetron C Omeprazole D Sucralfate E Dicyclomine

The Correct Answer is: C Omeprazole is a proton pump inhibitor, and is the only medication listed that is indicated for the prevention of NSAID induced ulcers. Ranitidine is an H2 antagonist and is indicated for active duodenal or benign gastric ulcers and GERD. Ondansetron is a 5-HT3 receptor antagonist and is indicated in the prevention of nausea and vomiting. Sucralfate is approved for treatment of an active duodenal ulcer and maintaining healed ulcers. Dicyclomine is an anticholinergic, and is indicated for the treatment of irritable bowel syndrome.

Q 106.2: Point of care (POC) testing has significant advantages and disadvantages. Which of the following is most correct regarding point of care testing? A It has not been proven to streamline workflow and reduce patient call-backs. B It has not been proven to decrease global costs. C It has been proven that more patients reach target "goals" and therefore improve patient outcomes with POC testing. D It has not been proven to reduce untoward outcomes. E Costs of POC testing generally preclude its use except in academic medical centers.

The Correct Answer is: C POC testing has a multitude of advantages including streamlining of work flow, immediate results precluding the need for patient call-backs to discuss results, allowing immediate interpretation, communication to patients, and patient education. This results in more patients reaching target goals and timely patient education with improved outcomes. Although more expensive, POC testing can reduce global costs if implemented correctly. (Kost, 2002, pp. 3-12)

Q 69.2: A 63-year-old white male is seen in the ambulatory outpatient clinic with complaints of midepigastric pain, weight loss, and jaundice. On examination, he is jaundiced and his sclerae are icteric. On palpation of the abdomen, you find a distended nontender gallbladder. Which of the following is the most likely diagnosis? A Gastric carcinoma B Chronic pancreatitis C Pancreatic carcinoma D Choledocholithiasis

The Correct Answer is: C Pancreatic carcinoma presents with weight loss, jaundice, and midepigastric pain. A palpable, nontender gallbladder (Courvoisier sign) is more often associated with a pancreatic malignancy than cholelithiasis, especially if the tumor is in the head of the pancreas. In acute cholecystitis, the obstruction in the cystic duct is associated with inflammation, resulting in a tender gallbladder on palpation of the right upper quadrant (Murphy sign); obstruction of the common bile duct in choledocholithiasis will result in jaundice but not weight loss. Gastric carcinoma will present with midepigastric pain and weight loss but not jaundice or a palpable gallbladder. Midepigastric pain is the most common symptom seen in chronic pancreatitis, and weight loss may be seen in association with malabsorption secondary to exocrine insufficiency. (Danziger, 2006, pp. 343-345; Paige, 2006, p. 264; Sharp, 2006, pp. 361-362) Danzinger RG , Nauta R , Park J. Biliary tract. In: Lawrence PF, ed. Essentials of General Surgery. 4th ed. Philadelphia, PA: Lippincott Williams & Wilkins; 2006. Paige J , O'Leary JP. Stomach and duodenum. In: Lawrence PF, ed. Essentials of General Surgery. 4th ed. Philadelphia, PA: Lippincott Williams & Wilkins; 2006. Sharp KW , Goldin SB , Lomis KD. Pancreas. In: Lawrence PF, ed. Essentials of General Surgery. 4th ed. Philadelphia, PA: Lippincott Williams & Wilkins; 2006.

Q 118.6: A 58-year-old man with a medical history of gouty arthritis presents with a red, swollen joint at the base of the great toe. His diet for the past 7 to 10 days consisted of large quantities of organ meats and fresh seafood. The increased metabolism of which of the following most likely contributed to the patient's symptoms? A amino acids B polysaccharides C purines D pyrimidines E triglycerides

The Correct Answer is: C Purines are normally metabolized into uric acid by the liver and can be found in high amounts in several foods, including organ meats, seafood, beans, peas, and many others. Higher levels of meat and seafood consumption are associated with an increased risk of gout because of the hyperuricemia that can occur via purine metabolism. (Hellman and Imboden, 2008, pp. 706-708) Hellman DB , Imboden JB Jr. Arthritis and musculoskeletal disorders. In: McPhee SJ , Papadakis MA, eds. Current Medical Diagnosis & Treatment. New York, NY: McGraw-Hill Medical; 2008.

Q 54.17: A 24-year-old male has an eight-month history of loose thought associations, social withdrawal, auditory hallucinations and deterioration in personal appearance and hygiene. Upon examination, he is noted to have a flat affect and perceptual distortions, and he behaves like he is detached from his own actions. Which of the following medications, if chosen for treatment, is known to be associated with lens changes and would require an eye examination to detect cataracts every six months? A Chlorpromazine (Thorazine) B Haloperidol (Haldol) C Quetiapine (Seroquel) D Risperidone (Risperdal) E Ziprasidone (Geodon)

The Correct Answer is: C Quetiapine is the only medication listed that is associated with lens changes, and an eye examination every six months, starting at the initiation of treatment, is recommended.

Q 71.5: What is the most common cause of pneumothorax in a healthy patient? A Traumatic B Infectious C Ruptured bleb D Surfactant abnormality E Malignancy

The Correct Answer is: C Rupture of a bleb is thought to be more relevant to young, thin men, and also patients who have a family history of them, and smoking. (Chesnutt MS, Prendergast TJ. Current Medical Diagnosis and Treatment, 2011, Chapter 9, Pulmonary Disorders)

Q 73.6: What is the most common bacterial pathogen isolated in otitis externa? A Group A streptococcus B Strep pneumonae C Pseudomonas aeruginosa D Hemophilus influenza E Aspergillus

The Correct Answer is: C Staphylococcus aureus and pseudomonas are the most common pathogens isolated in otitis externa. Proteus, diphtheroids, and E. coli are also causes. (Lalwani A.K., 2008, Chapter 47)

Q 120.18: Which of the following lists the common adverse effects caused by nitroglycerin when administered sublingually at high doses? A constipation, blurred vision, tinnitus B dyspepsia, abdominal distention, vomiting C elevated pulse, facial flushing, headache D photophobia, excessive salivation, excessive tearing E wheezing, cough, heartburn

The Correct Answer is: C Sublingual nitroglycerin produces venodilation and vasodilation, which causes secondary responses of flushing and headache. The elevated pulse or tachycardia is reflexive in nature, and the heart tries to compensate for the drop in blood pressure by raising its rate. (Talbert, 2008, pp. 237-239)

Q 109.8: Which lobe is most affected by infection of tuberculosis? A Left lower lobe B Left upper lobe C Right upper lobe D Right lower lobe E Right middle lobe

The Correct Answer is: C The apical sections of the lung fields are the most typical areas where tuberculosis occurs. The right side is more prevalent than the left. Other findings on chest x-rays can include pleural effusions, Gohn lesions (calcified primary focus), and cavitation. (Chesnutt MS, Prendergast TJ. Current Medical Diagnosis and Treatment, 2011, Chapter 9, Pulmonary Disorders)

Q 115.20: A 38-year-old female is at a follow-up visit for hypertension. She is accompanied by her husband. During the clinical visit the patient is noted to be passive, letting her husband do most of the talking. She also appears to lack confidence and self-esteem. The husband ends up making the decisions, and she is over-accepting of his dominance. What is the most likely classification of this personality disorder? A Avoidant B Borderline C Dependent D Histrionic E Schizotypal

The Correct Answer is: C The clinical findings of dependent personality disorder are the most consistent with the given clinical scenario. Avoidant personality disorder presents clinically as someone who fears rejection, overreacts to rejection and failure, has poor social endeavors, and low self-esteem. Borderline personality disorder clinical findings include impulsiveness, unstable and intense interpersonal relationships, lack of self control, suicidal ideations, aggressive behavior, and a high drug abuse rate. Clinical findings of histrionic personality disorder include being dependent, immature, seductive, egocentric, vain, and emotionally labile. Schizotypal clinical findings include being superstitious, socially isolated, and suspicious, and having limited personality ability, odd speech and eccentric behaviors. (McPhee SJ, Papadakis MA. Current Medical Diagnosis & Treatment, 2010, p. 951)

Q 81.5: A 48-year-old woman presents with new-onset headache that she describes as nonspecific, worse on awakening, intermittent throughout the day but can worsen with bending over or coughing. Her husband reports that she has not been herself since the headaches started about 4 to 6 weeks ago. Which of the following tests would be best for determining the etiology of her presenting symptoms? A noncontrast head CT scan B lumbar puncture C contrast-enhanced brain MRI D noncontrast brain MRI E cerebral angiography

The Correct Answer is: C This patient has an intracranial mass until proven otherwise. Headaches starting later in life and accompanied by other neurologic or cognitive problems should raise a high suspicion of a tumor. Obtaining a contrast-enhanced MRI of the brain will demonstrate an intracranial mass lesion. The contrast will follow blood flow distribution and help in determining possible tumor type. Some lesions are difficult to see without contrast enhancement. MRI scans have a much higher resolution for soft tissue over CT scans and are preferred for looking at brain parenchyma. Prior to the advent of CT and MRI, cerebral angiography was used to look for intracranial masses. Vascular tumors have characteristic blush patterns, and if a mass effect is present, it will distort the position of the blood vessels. (Aminoff et al., 2005, pp. 82-83) Aminoff MJ , Greenberg DA , Simon RP. Clinical Neurology. 6th ed. New York, NY: McGraw-Hill; 2005.

Q 111.2: A 24-year-old female presents with low self-esteem and lack of confidence. While obtaining the history, you learn that she has a long history of recurrent maladaptive behavior. She has a tendency to blame others and has almost no introspective ability. She also has significant difficulties with interpersonal relationships. What is the most likely diagnosis? A Delusional disorder B Dysthymia C Personality disorder D Schizoaffective disorder E Schizophrenic disorder

The Correct Answer is: C This patient is exhibiting all of the features of a personality disorder. Delusional disorder is characterized by symptoms of persistent, non-bizarre delusions that include minimal impairment of daily functioning. Dysthymia is characterized by a chronic depressive disturbance that is present for at least two years and includes symptoms of sadness, loss of interest, and withdrawal from activities. Schizoaffective disorder is characterized by affective symptoms that develop alongside psychotic manifestations. These cases do not fit easily within the schizophrenic or affective categories. Schizophrenic disorder is characterized by a severe disruption of thinking, mood, and overall behavior, as well as the inability to properly filter stimuli. (McPhee SJ, Papadakis MA. Current Medical Diagnosis & Treatment, 2010, p. 952)

Q 80.1: A 36-year-old female has just been diagnosed with hypertension. During the office visit, she exhibits an excessive emotional reaction. She is seen frequently for follow-up of her hypertension over the next three months, and during that time she is noted to exhibit signs of immaturity, seductiveness, egocentricity, and dependency. What is the most appropriate classification of the suspected personality disorder? A Antisocial B Borderline C Histrionic D Obsessive compulsive E Paranoid

The Correct Answer is: C This patient is exhibiting the signs most consistent with histrionic personality disorder. Antisocial personality disorder is characterized by selfishness, callousness, promiscuousness, and impulsive behavior, and an inability to learn from experience and legal problems. The clinical findings of borderline personality disorder include impulsiveness, unstable and intense interpersonal relationships, lack of self control, suicidal ideations, aggressive behavior, and a high drug abuse rate. Clinical findings of obsessive-compulsive disorder include being a perfectionist, egocentric, indecisive, and having rigid thought patterns and a need for control. Clinical findings of someone who has paranoid personality disorder would include defensiveness, being overly sensitive, secretive, suspicious, and hyper-alert, and having a limited emotional response. (McPhee SJ, Papadakis MA. Current Medical Diagnosis & Treatment, 2010, p. 951)

Q 92.1: A 45-year-old patient presents 2 days postoperatively with a partial thyroidectomy. She has been experiencing vomiting with diarrhea. On physical exam, her temperature is 101°F and jaundice is noted. Her heart rate is irregularly irregular with a rate of 200 bpm. What would be the most appropriate pharmacological intervention? A radioactive iodine (131I) B propranolol 80 mg C PTU 600 mg D iopanoic acid 500 mg

The Correct Answer is: C This patient is in a thyrotoxic crisis or thyroid storm. She needs to be admitted for monitoring and supportive care. The initial treatment would be PTU 600 mg loading dose followed by 200 to 300 mg every 6 hours given either by nasogastric tube or rectally. (Fitzgerald, 2009, p. 988) Fitzgerald PA. Endocrine diseases. In: McPhee SJ , Papadakis MA, eds. Current Medical Diagnosis and Treatment. 48th ed. New York, NY: McGraw-Hill; 2009.

Q 115.6: A 30-year-old obese female presents with thick darkened skin in the bilateral axillae and around her armpit. She has tried over-the-counter hydrocortisone cream, with no help. There are no associated symptoms. Which of the following labs should be ordered? A complete blood cell count (cbc) with diff and liver function test (LFT) B LFT and basic metabolic panel (BMP) fasting C fasting insulin, glucose, and lipid panel D BMP and cbc with diff

The Correct Answer is: C This patient presents with acanthosis nigricans, which is most commonly associated with insulin resistance and metabolic syndrome. The most helpful labs for patients presenting in this manner are fasting insulin, glucose, and lipid panels. No helpful information will be gained from a cbc with diff and liver function test (LFT). The BMP will provide a glucose level, but not enough helpful information will be gained from these labs. (Wolff et al., 2009, Page 89)

Q 95.7: A 27-year-old man presents to the emergency department with a five-day complaint of substernal pleuritic chest pain, which worsens while lying supine. He is in no distress. A friction rub is noted over the precordium. The patient's vital signs are as follows: temperature is 100.4˚F, pulse rate is 94, respiratory rate is 20, and blood pressure is 136/84. An ECG reveals widespread diffuse ST elevations with PR interval depressions. He was recently treated for a viral respiratory infection. Which of the following is the most appropriate initial management for this patient? A Administer a broad spectrum antibiotic B Administer intravenous tissue plasminogen activator (T-PA) C Begin a nonsteroidal anti-inflammatory agent D Perform needle thoracentesis E Refer for immediate cardiac catheterization

The Correct Answer is: C This patient's diagnosis is acute inflammatory pericarditis. Viral infections are the most common cause of acute pericarditis, and males are the most commonly affected. A pericardial friction rub and EKG changes are characteristic of this diagnosis. Treatment is focused on the underlying inflammation, with NSAIDS being first-line and short course corticosteroids also being appropriate. Antibiotics are not indicated unless a bacterial etiology is confirmed or there are significant risk factors. Choices B, D, and E are not appropriate for this condition and could be harmful. (McPhee SJ, Papadakis MA. Current Medical Diagnosis & Treatment 2011, Chapter 10, Heart Disease)

Q 81.1: Which of the following examinations is a major component of routine monitoring of chronic, open angle glaucoma? A Pupillary response B Corneal reflex testing C Visual field testing D Accommodation E Visual acuity

The Correct Answer is: C Tonometry, gonioscopy, monitoring of the disc-to-cup ratio, and visual field examination are the routine exams done when monitoring primary open angle glaucoma. (Riordan et al., 2008, Chapter 11)

Q 91.3: A 52-year-old male presents to the office with a complaint of abdominal pain. He describes the pain as epigastric, dull, and achy. Antacids do help the pain for a few hours, but it then returns. He also notes that the pain often wakes him from sleep. He denies significant vomiting, dysphagia, heartburn, or weight loss. Which of the following diagnostic tests is the procedure of choice in this patient? A Abdominal CT B Barium upper GI series C Upper endoscopy D Colonoscopy E Esophageal manometry

The Correct Answer is: C Upper endoscopy is the procedure of choice for diagnosing peptic ulcer disease, which is the concern in this patient. Abdominal CT is obtained in patients suspected of having complications from an ulcer. Barium upper GI series is not as sensitive as upper endoscopy in detecting ulcer disease. Colonsocopy is used to evaluate the colon, and this patient's symptoms do not suggest a problem in that area. Esophageal manometry is used to evaluate dysphagia, which this patient denies having. (McPhee SJ, Papadakis MA. Current Medical Diagnosis & Treatment, 2010, p. 546)

Q 77.2: A 48-year-old African American male presents with dyspnea, 2-pillow orthopnea, and swelling to his lower legs that has developed over the last month. He also complains of fatigue and decreased exercise tolerance, stating that he has trouble climbing one flight of steps. On physical examination, his blood pressure is 178/98, pulse rate is 102, and respiratory rate is 20. There is 5 cm JVD, crackles at the bilateral lung fields, and tachycardia and an S 3 is heard on cardiac auscultation. There is 2+ pitting edema to the lower extremities. His electrocardiogram reveals a sinus tachycardia at a rate of 105 and left ventricular hypertrophy. The chest x-ray reveals cardiomegaly with increased interstitial markings in all lung fields. There is a small right pleural effusion that blunts the costophrenic angle. Which medication is the treatment of choice for controlling this patient's heart rate? A Amlopidine B Minoxidil C Isosorbide mononitrate D Metoprolol E Atropine

The Correct Answer is: D The use of beta-blockers is indicated for heart rate control. The other choices are not indicated for rate control and have no primary action on rate, but rather on blood pressure. (Bashore et al., Current Medical Diagnosis and Treatment, Chapter 10)

Q 76.1: A 30-year-old male presents to your office complaining of sinus and facial pain, congestion, and purulent nasal discharge for one month. He has been treated with two courses of different antibiotics by another provider, and does not feel any improvement in his symptoms. What diagnostic test is indicated? A Plain sinus radiographs B MRI C Aspiration and culture of maxillary sinuses D CT scan E Ultrasound of sinuses

The Correct Answer is: D A CT scan is the current preferred method for sinus imaging of chronic sinusitis. CT imaging has better visualization of mucosal thickening air-fluid levels and bone structures. Plain radiographs and CT scans are of limited use in acute sinusitis, because viral pathogens that cause sinus abnormalities are indistinguishable from bacterial causes. (Lalwani A.K., 2008, Chapter 14)

Q 119.17: A 45-year-old presents with a markedly tender nodule protruding from the edge of his upper eyelid. He states that this has been present for 12 hours. No discharge is seen. He denies visual problems. What is the most likely diagnosis? A Blepharitis B Chalazion C Dacryocyctitis D Hordeolum E Conjunctivitis

The Correct Answer is: D A hordeolum (sty) is caused by an acute infection of the Zeis or Moll's glands of the eyelid. Symptoms include pain and tenderness. An "internal hordeolum" points to the inner conjunctiva of the lid and an "external hordeolum" points to the skin surface of the eyelid. (Riordan et al., 2008, Chapter 4)

Q 120.1: A nonpenetrating tear of the gastroesophageal junction in association with a history of vomiting is known as which of the following? A Boerhaave syndrome B Plummer-Vinson syndrome C Peutz-Jeghers syndrome D Mallory-Weiss syndrome E Zollinger-Ellison syndrome

The Correct Answer is: D A mucosal tear of the gastroesophageal junction with a history of prolonged vomiting is known as Mallory-Weiss tear syndrome. Plummer-Vinson is a congenital syndrome associated with anemia and webbing of the esophagus. Boerhaave syndrome is a rare life-threatening problem characterized by a full-thickness tear of the esophageal wall. Zollinger-Ellison syndrome is caused by gastrin-secreting neuroendocrine tumors resulting in acid hypersecretion. (McQuaid, 2009, pp. 489, 521-522, 541, 576) McQuaid KR. Gastrointestinal disorders. In: McPhee SJ , Papadakis MA, eds. Current Medical Diagnosis and Treatment. 48th ed. New York, NY: McGraw-Hill; 2009.

Q 110.17: Which of the following represents a positive tuberculin skin test result? A A college student with 4mm of superficial erythema at site B An HIV-positive patient with 2mm induration C A low-risk individual with a pre-employment test result of 6mm induration D A nursing home resident with 12mm induration E A recent immigrant from Mexico with 8mm induration

The Correct Answer is: D A tuberculin purified protein derivative (also known as a TB test or PPD), is utilized to screen for latent Mycobacterium tuberculosis infection. Guidelines for interpreting test results, based upon induration, patient risk, and patient medical status, are published by the Centers for Disease Control and Prevention (summarized in Table 9-10 below). False-negative reactions may occur in immunosuppresed patients and those with extensive infection. False-positive and false-negative reactions can occur for various reasons, including previous vaccination with bacillus Calmette-Guirein (BCG), which may cause a false-positive. http://www.cdc.gov/TB/publications/factsheets/testing/skintesting.pdf

Q 116.17: According to the American Heart Association's most recent guidelines regarding infective endocarditis, which of the following patients requires infective endocarditis prophylaxis? A A 65-year-old man with a history of rheumatic fever prior to colonoscopy B A 29-year-old woman with a history of bicuspid aortic valve prior to vaginal hysterectomy C A 42-year-old man with a history of mitral valve regurgitation prior to vasectomy D A 22-year-old man with a history of mitral valve replacement prior to tooth extraction E A 44-year-old woman with a history of mitral valve prolapse prior to open cholecystectomy

The Correct Answer is: D According to the American Heart Association's most recent guidelines regarding infective endocarditis prophylaxis, only those patients with prosthetic heart valves, prior history of bacterial endocarditis, unrepaired or incompletely repaired cyanotic congenital heart disease (including those with palliative shunts and conduits), completely repaired congenital heart disease during the first 6 months after surgery if a prosthetic material or device was used, whether placed surgically or via catheter, repaired congenital heart disease that has residual defects at or adjacent to the site of repair, and heart transplant recipients with valvulopathy, and then in these populations of patients, only with certain procedures including tooth extraction. Therefore, the only patient among the choices offered who requires infective endocarditis prophylaxis is choice D. Choices A, B, C, and E are not considered at significant risk for infective endocarditis, regardless of the procedure. (American Heart Association, Inc., 2011, http://www.americanheart.org/presenter.jhtml?identifier=3047051)

Q 106.13: A 24-year-old woman comes to your office complaining of anxiety. The patient had witnessed a traumatic event 3 days earlier that made her feel fearful. She has not been able to tell her family about this experience. She now feels like she is numb and in a dazed, dreamlike state with poor concentration, and difficulty sleeping. She experienced a flashback of the event yesterday. What is the most likely diagnosis? A post-traumatic stress disorder B dissociative fugue C psychosis D acute stress disorder E depersonalization

The Correct Answer is: D Acute stress disorder is characterized by experiencing or witnessing a traumatic event where the person felt threatened by death or injury or the people they witnessed. The person feels fearful and helpless. Symptoms usually occur within a month of the event, last 2 days, and resolve in a month. The person feels numb, has lack of awareness of surroundings, and sees everything in a dreamlike state. Sometimes they develop amnesia. Flashbacks or recurrent images can occur with acute stress disorder. Difficulty sleeping, poor concentration, anhedonia, irritability, and despair are associated with this disorder. If not treated at the early stages, the patient is at risk of developing PTSD. (Johnson et al., 2008, pp. 377-378; Sadock and Sadock, 2008, p. 260) Johnson DC , Krystal JH , Southwick SM. Posttraumatic stress disorder and acute stress disorder. In: Ebert MH , Loosen PT , Nurcombe B , Leckman JF, eds. Current Diagnosis and Treatment in Psychiatry. New York: McGraw-Hill; 2008. Sadock BJ , Sadock VA. Concise Textbook of Clinical Psychiatry, 3rd ed. Philadelphia, PA: Lippincott, Williams & Wilkins; 2008.

Q 121.6: A 65-year-old female presents with a red irritation in her right eye. She states that this has been occurring intermittently for about two years. She also states that her eyelids are "droopy," and that she needs plastic surgery. On physical exam you notice a diffusely injected conjunctiva and an outwardly tilted lower eyelid. What is the most likely diagnosis for the abnormal physical finding? A Conjunctivitis B Dacryoadenitis C Entropion D Ectropion E Exophthalmos

The Correct Answer is: D Ageing causes a relaxation of the obicularis oris muscle, and will cause the lower eyelid to sag outwardly. This prevents the lower lid from protecting the eye, and frequently results in exposure conjunctivitis and keratitis. Treatment is surgical. (Riordan et al., 2008, Chapter 4)

Q 72.2: An 18-year-old female presents with two weeks of severe sore throat and fatigue. Her exam shows an exudative tonsillitis. A mono-spot test is positive, and a rapid strep test is positive. Which of the following medications should be avoided? A Erythromycin B Clindamycin C Cephalexin D Ampicillin E Prednisone

The Correct Answer is: D Ampicillin should be avoided, because a high percentage of mononucleosis patients develop a fine, non-allergic maculopapular rash when given ampicillin class drugs. The remaining antibiotics are appropriate for treating group A strep. Prednisone is used to reduce the pain and inflammation associated with severe tonsillitis. (McPhee et al., 2011, Chapter 8)

Q 63.9: Which of the following statements about anemia associated with CKD is TRUE? A Iron and folic acid by mouth are the most effective treatments. B Transfusion of packed red blood cells monthly is the most effective treatment. C IM erythropoietin given monthly is the most effective treatment. D It is due to the inability of the kidney to transform erythropoietin into its physiologically active form. E It occurs early in the course of CKD.

The Correct Answer is: D Anemia associated with CKD is the result of inadequate erythropoietin synthesis by the kidneys. This hormone signals the bone marrow to synthesize red blood cells. A deficiency will result in anemia. In the absence of erythropoietin, iron would not be of use since red blood cell synthesis is inadequate. Folic acid would also not be of use and does not play a role in the etiology of this type of anemia. Transfusion is a tempering measure only, used to increase oxygen-carrying capacity in the case of symptomatic ischemia. Anemia due to erythropoietin deficiency generally does not occur until the GFR decreases to <60 mL/min, or approximately 50% of normal. Intramuscular administration of erythropoietin is the only effective treatment to induce red blood cell production. Depending on the formulation used, this can be given once a week or once every 2 weeks. Oral iron supplementation is needed to produce adequate hemoglobin for the increased de novo red cell production.

Q 116.2: A 22-year-old man presents with an insidious onset of low back pain over the last 6 months. He describes the pain as dull and has difficulty localizing the pain. The pain often radiates to his thighs. The pain is worse in the morning and associated with stiffening that lessens during the day. The patient notes that there is no history of trauma. The initial laboratory evaluation shows an elevated erythrocyte sedimentation rate, positive HLA-B27, and a negative rheumatoid factor. Plain films of the lumbar spine reveal bilateral blurring of the sacroiliac joints. Which of the following is the most likely diagnosis? A systemic lupus B lumbar disc disease C rheumatoid arthritis D ankylosing spondylitis E polymyalgia rheumatica

The Correct Answer is: D Ankylosing spondylosis is the most likely diagnosis in this patient. This condition is a chronic inflammatory disorder of the joints of the axial skeleton and commonly presents in the late teens or twenties. Male patients have a higher incidence than do female patients. A common presentation is pain in the lower back with radiation to the thighs and associated limitation of movement that may lessen during the day. Laboratory findings include an elevated erythrocyte sedimentation rate and positive HLA-B27. The HLA-B27 is not a specific test for ankylosing spondylitis; a small percentage of the normal population has a positive finding of this antigen. The earliest radiographic findings occur in the sacroiliac joints, with the detection of erosion and blurring of the joint space. Systemic lupus commonly affects women of childbearing years and presents with exacerbations and remissions of arthritis, rash, fatigue, and the potential for organ system involvement. Lumbar disc disease is usually seen in the age group of 35 to 45 years and is more likely to be associated with trauma. Rheumatoid arthritis does have the potential to affect this age group, but it would more likely be associated with smaller joints of the hands, along with a positive rheumatoid factor. Polymyalgia rheumatica more commonly affects patients older than 50 years and is associated with fatigue, malaise, chronic pain, and stiffness of the proximal muscles, shoulders, neck, and pelvic girdle. (Hellmann and Imboden, 2008, pp. 746-747) Hellmann DB , Imboden JB. Arthritis and musculoskeletal disorders. In: Tierney LM , McPhee SJ , Papadakis MA, eds. 2008 Current Medical Diagnosis and Treatment. New York, NY: McGraw-Hill; 2008:703-756.

Q 73.2: The organism shown in Figure 8-1 usually enters the body from infected soil through a break in the skin of the feet. It then is carried to the lungs, travels to the mouth, and is swallowed. Once in the gastrointestinal (GI) tract, it attaches to the wall and induces bleeding, leading to an iron deficiency anemia. Associated GI symptoms are uncommon. Additional symptoms include swelling and intense itching at the site in which the larva penetrates the skin. Which of the following organisms best fits this clinical picture and the organism shown in Figure 8-1? (Courtesy of Centers for Disease Control and Prevention, National Center for Infectious Diseases, Division of Parasitic Diseases.) A Strongyloides B whipworms C pinworms D hookworms

The Correct Answer is: D As described, hookworms generally enter through the skin and travel to the lungs. There they migrate to the mouth, are swallowed, and reproduce in the gut. Females lay thousands of eggs, which are subsequently excreted in the feces and mature in soil. The adult worms attach to the intestinal wall, causing bleeding and a subsequent anemia. They can also affect absorption, leading to nutritional deficiencies. Diagnosis is made through microscopic evaluation of a stool specimen and treatment is with albendazole. Whipworms (Trichuris trichiura) also lay eggs in the stool, which then reside in the soil. Many whipworm infections are asymptomatic. Strongyloides is different from other nematodes in that it can reproduce inside the intestine and persist for years. Pinworms (enterobiasis) are very small worms that exit the anus at night to lay eggs, causing intense itching and promoting the fecal-oral transmission. (Weller and Nutman, 2008, pp. 1322-1324; Kazura, 2007, pp. 2425-2431) Weller PF. Protozoal intestinal infections and trichomoniasis. In: Fauci AS , Braunwald E , Kasper DL, et al., eds. Harrison's Principles of Internal Medicine. 17th ed. New York, NY: McGraw-Hill; 2008. Kazura JW. Nematode infections. In: Goldman L , Ausiello D, eds. Cecil Medicine. 23rd ed. Philadelphia, PA: Saunders Elsevier; 2007.

Q 112.7: A 20-year-old male presents with cough, nasal congestion, and a low grade fever for one week. His cough seems to be getting worse, which is the reason for his visit. His past medical history includes asthma and nasal polyps. On physical exam, his temperature is 101˚F, his pharynx is erythemic, and there is grey nasal discharge with a few nasal polyps seen using a nasal speculum. His lungs have a few expiratory wheezes bilaterally. What medication is to be avoided in this patient? A Penicillin B Acetaminophen C Erythomycin D Aspirin E Ciprofloxacin

The Correct Answer is: D Aspirin should be avoided in patients with asthma and nasal polyps. Aspirin can precipitate bronchospasm in these patients, due to immunologic salicylate sensitivity. (McPhee et al., 2011, Chapter 8)

Q 115.12: A 14-year-old male presents for his asthma follow up. He states that he has symptoms three to four days per week and awakens from sleep three times a month, requiring the use of his inhaler. He occasionally has to sit out of gym class due to his symptoms, but overall he functions well. He currently uses a short-acting β2-agonist as needed. What is the preferred pharmacologic agent to add to this patient's regimen? A Inhaled long-acting β2-agonist B Leukotriene receptor antagonist C Long-acting mediator inhibitor D Low-dose inhaled corticosteroid E Low-dose systemic corticosteroid

The Correct Answer is: D Asthma management follows a stepwise approach, which is based upon medication action, disease progression, and patient compliance. It also involves determining the asthma classification of the patient, as well as previous response to medications. This approach also incorporates patient education, environmental control, and comorbidity management (see Figure 9-2). (McPhee SJ, Papadakis MA. Current Medical Diagnosis & Treatment 2011, Chapter 9, Pulmonary Disorders) Stepwise approach to managing asthma.

Q 79.8: A 48-year-old female complains of ear fullness, episodes of tinnitus, and vertigo. She also complains that her hearing is not as good as it used to be. She states that this has occurred sporadically over the past year. What is the most likely diagnosis? A Benign paroxysmal positional vertigo (BPPV) B Labyrinthitis C Vestibular neuronitis D Meniere's syndrome E Presbycusis

The Correct Answer is: D BPPV is characterized by sudden vertigo, made worse with head position change, and accompanied by nausea and vomiting. Meniere syndrome is characterized by episodic severe vertigo, fluctuating sensorineural hearing loss, tinnitus, and ear "fullness." Pathologically, there is distention of the endolymphatic system throughout the inner ear, presumably due to dysfunction of the endolymphatic sac. Labyrinthitis is characterized by severe vertigo and hearing loss, and is likely a result of a viral inner ear infection. Vestibular neuronitis is also a result of a viral inner ear infection, with symptoms of severe vertigo, nausea, and vomiting, without hearing loss. Both labyrinthitis and vestibular neuronitis resolve in one to two weeks. Presbycusis is age related hearing loss. (Lalwani A.K., 2008, Chapter 53)

Q 101.8: A 21-year-old male with benign essential tremor asks to be placed on medication, to decrease his tremor for upcoming graduate school interviews. You discuss the potential benefits and side effects of medication and he is agreeable. Which of the following is the first-line agent of choice? A Alprazolam B Botulinum toxin C Primidone D Propanolol E Topiramate

The Correct Answer is: D Benign essential tremor is often worse during times of increased stress and decreased sleep. Management should include addressing these situations and providing nonpharmacologic therapy information. In specific cases such as this patient, beta-blocker therapy has been shown to effectively reduce tremor activity, with intermittent therapy being acceptable. Other medications, such as the others listed in the answer choices, may be used as alternatives to beta blockers, with each having individual risks and side effect profiles. (McPhee SJ, Papadakis MA. Current Medical Diagnosis & Treatment 2011, Chapter 24, Nervous System Disorders)

Q 116.4: A 24-year-old HIV-positive man comes to the emergency department complaining of severe left-sided chest discomfort, which radiates through to the left trapezius region. On coming into the room, you note that he is sitting up and hunched forward. Prior to examining him, you have reviewed his chart. Laboratory findings demonstrate troponins x 3, which are negative for myocardial ischemia. His EKG demonstrates diffuse ST segment elevations throughout. Which of the following physical exam findings would be most likely in this patient? A Roth spots B Splenic enlargement C Janeway lesions D Pericardial friction rub E Splinter hemorrhages

The Correct Answer is: D Choice D is the most likely finding, as this patient is exhibiting signs, symptoms, and EKG findings pathognomonic for acute pericarditis, which is likely infectious in the setting of a patient with HIV. A pericardial friction rub is heard best with the patient in a seated position, during expiration, and is frequently found in patients with pericarditis. Choice A B, C, and E are physical exam findings seen in acute bacterial endocarditis. (Fauci et al., 2001, pp. 1366-1367)

Q 80.11: A 42-year-old woman with a history of migraine cephalgia and Raynaud's phenomenon comes to the emergency department with complaints of severe chest discomfort that occurs at rest every morning (at approximately 10 AM). An EKG performed during an episode of chest discomfort demonstrates transient ST segment elevation, which is relieved with sublingual nitroglycerin. There is no troponin elevation. Cardiac catheterization is performed, and reveals coronary artery spasm, which corresponds with ST segment elevation, and no significant coronary artery stenosis. Which of the following is the most appropriate treatment regimen? A 24-hour nitroglycerin dermal patch B Thiazide diuretics C Loop diuretics D Calcium channel blockers E Aspirin

The Correct Answer is: D Choice D, calcium channel blockers, and long-acting nitrate therapy have been proven to be effective for preventing recurrences of episodes of Prinzmetal angina, with short-acting sublingual or IV nitroglycerin useful for relieving acute episodes. Choice A, 24-hour nitroglycerin dermal patch, is inappropriate, as patients can develop nitrate tolerance, and thus need a 12-hour nitrate-free period every day. Choices B and C, thiazide and loop diuretics, have no proven benefit in patients with Prinzmetal angina. Aspirin, choice E, may worsen episodes of prinzmetal angina, and thus is not recommended. (Fauci et al., 2008, Chapter 238)

Q 105.18: A 58-year-old man with a 20-year history of gastroesophageal reflux disease (GERD) presents with progressive dysphagia for 5 months associated with a 20-lb weight loss. Results from a barium swallow are pictured below (Figure 18-1). Which of the following is the most likely diagnosis? (Reproduced, with permission, from Fauci AS, Braunwald E, Kasper DL, et al. Harrison's Principles of Internal Medicine, 17th ed. New York: McGraw-Hill, 2008:1849.) A Achalasia B Esophageal leiomyoma C Uncomplicated reflux esophagitis D Esophageal carcinoma

The Correct Answer is: D Dysphagia on a background of GERD is an alarm signal for cancer, since GERD is related to increased risk for esophageal adenocarcinoma. Esophageal cancer is associated with a progressive course of dysphagia, first to bulky foods, then softer foods, and then liquids as the tumor invades the esophagus; significant weight loss is almost universal at the time of presentation. Barium swallow demonstrates narrowing at the tumor site with normal appearance of the remainder of the esophagus. Achalasia is a motor disorder characterized by dysphagia to both liquids and solids as well as regurgitation of food. Patients with achalasia typically drink large amounts of liquids to force their food down and have problems with aspiration pneumonia. Barium swallow in achalasia typically shows a dilated esophagus with a narrowing at the lower esophageal sphincter (bird's beak). Leiomyomas are generally asymptomatic. Patients with reflux esophagitis will complain of epigastric or substernal pain that is worse when supine or leaning forward. (Patti, 2006, pp. 469-471; Goyal, 2008, pp. 1848-1850) Patti MG , Pietro T , Way LW. Esophagus and diaphragm. In: Doherty GM , Way LM, eds. Current Surgical Diagnosis & Treatment. 12th ed. New York, NY: Lange Medical Books/McGraw-Hill; 2006. Goyal RK. Diseases of the esophagus. In: Fauci AS , Kasper DL , Longo DL , Braunwald E , Hauser SL , Jameson JL , Loscalzo J, eds. Harrison's Principles of Internal Medicine. 17th ed. New York, NY: McGraw-Hill; 2008.

Q 88.7: When used for advanced carcinoma of the prostate, chronic administration of leuprolide inhibits the synthesis of androgens by _____________________ A blocking gonadotropin-releasing hormone (GnRH) receptors at the anterior pituitary B blocking luteinizing hormone (LH) receptors on interstitial (Leydig) cells of the testes C increasing the secretion of GnRH from the hypothalamus D inhibiting pulsatile secretion of gonadotropins from the anterior pituitary E upregulation of the number of GnRH receptors at the anterior pituitary

The Correct Answer is: D Leuprolide is a GnRH (LHRH) agonist that suppresses the pulsatile secretion of follicle stimulating hormone (FSH) and LH (gonadotropins) from the anterior pituitary when given chronically. Continuous administration of a GnRH agonist causes down-regulation of GnRH receptors on gonadotropes, which, in turn suppresses gonadotropin release and gonadal function. Decreased amounts of LH, in particular, lead to diminished production of androgens by the testes (especially DHT), which support prostate growth. It is believed that by interrupting the hormonal pathways that modulate prostatic growth, tumor development and metastasis is slowed. (Kolesar, 2008, p. 2212; Jones, 2009, pp. 1151-1152) Kolesar JM. Prostate cancer. In: DiPiro JT , Talbert RL , Yee GC, et al., eds. Pharmacotherapy: A Pathophysiologic Approach. 7th ed. New York: McGraw-Hill; 2008. Jones EE. The female reproductive system. In: Boron WF , Boulpaep EL, eds. Medical Physiology. 2nd. Philadelphia, PA; Saunders Elsevier; 2009.

Q 107.9: A 53-year-old man is taking a proton pump inhibitor for GERD symptoms, a beta blocker and a thiazide diuretic for hypertension, an SSRI for depression, and an over-the-counter NSAID as needed for aches and pains. He has developed gynecomastia and laboratory studies reveal an elevated prolactin level. If his hyperprolactinemai is due to one of his medications, which is the most likely cause? A the beta blocker B the NSAID C the proton pump inhibitor D the SSRI E the thiazide diuretic

The Correct Answer is: D Many medications cause hyperprolactinemia, including SSRIs, tricyclic antidepressants, and antipsychotics. Hydralazine and methyldopa, but not beta blockers (A), may also raise prolactin levels; likewise opioids, but not NSAIDS (B). Cimetidine and ranitidine, but not proton pump inhibitors (C) are included among possible pharmaceutical causes. Thiazide diuretics (E) are not know to raise prolactin levels.

Q 102.8: A 34-year-old man presents to your office with the complaint of pain and fatigue in his right wrist. He states that the pain can sometimes wake him up at night and feels as if his thumb is falling asleep. He reports the problem started since he has been writing a book. On physical examination, you note a positive Tinel sign but no response to a Phalen maneuver. While sending him for a nerve conduction study, you tell him the most likely diagnosis is A ulnar nerve compression B radial nerve compression C thoracic outlet syndrome D median nerve compression E peroneal nerve compression

The Correct Answer is: D Median nerve compression can be precipitated by repetitive use of the wrist or hand or by compression of the median nerve within the carpal tunnel at the wrist from inflammation or trauma. Characteristic fatigue and pain, especially at night, may be accompanied by paresthesia in the median nerve distribution. The pain may be reproduced on examination by performing either the Tinel or Phalen maneuver but electrophysiology studies will usually confirm the entrapment. (Aminoff et al., 2005, pp. 221-222) Aminoff MJ , Greenberg DA , Simon RP. Clinical Neurology. 6th ed. New York, NY: McGraw-Hill; 2005.

Q 68.5: Which of the following conditions is a cause for central vertigo? A Meniere syndrome B Labyrinthitis C Vestibular neuronitis D Acoutic neuroma E Perilymphatic fistula

The Correct Answer is: D Meniere syndrome, labyrinthitis, vestibular neuronitis, and perilymphatic fistula are causes of peripheral vertigo. Acoustic neuroma, or eight cranial nerve schwannomas, are among the most common intracranial tumors, and a cause for central vertigo. (McPhee et al., 2011, Chapter 8)

Q 94.3: Your 45-year-old diabetic on a statin develops myalgias and you elect to check a creatine phosphokinase (CPK). While a normal CPK is 150 U/L, your patient has an elevated CPK. At what point must you discontinue the use of the statin? A > 150 U/L B > 300 U/L C > 450 U/L D > 1500 U/L E none of the above

The Correct Answer is: D Minor increases in creatine kinase (CK) activity in plasma are observed in some patients receiving reductase inhibitors, frequently associated with heavy physical activity. Rarely, patients may have marked elevations in CK activity, often accompanied by generalized discomfort or weakness in skeletal muscles. In all patients, CK should be measured at baseline. If muscle pain, tenderness, or weakness appears, CK should be measured immediately and the drug discontinued if activity is elevated significantly over baseline (> 1500 U/L). The myopathy usually reverses promptly upon cessation of therapy. If the association is unclear, the patient can be re-challenged under close surveillance. Myopathy in the absence of elevated CK has been reported. (Brunton, et al., 2006, Chapter 35)

Q 104.10: A 28-year-old woman who was born and brought up just outside of Washington, DC, comes in for evaluation of vague "problems with swallowing." She has no other symptoms except "my neck is bigger than it used to be." Examination reveals only a diffuse, somewhat irregular, nontender enlargement of the thyroid gland with distinct masses palpable within it. What is the most likely diagnosis? A endemic goiter B Graves disease C Hashimoto thyroiditis D multinodular goiter E thyroid carcinoma

The Correct Answer is: D Multinodular goiter is the most likely in a woman with these findings in the United States. It may be nontoxic as in this case or toxic, i.e., producing excessive thyroid hormones which cause symptoms of hyperthyroidism. Endemic goiter (A), which may present as a simple enlargement of the thyroid or as a multinodular one, is found almost entirely in iodine-deficient areas of the world and is extremely rare in the U.S. In Graves disease (B), the thyroid is enlarged and may exhibit a thrill and a bruit. In addition, the patient would have other signs of hyperthyroidism. The thyroid in Hashimoto thyroiditis (C) is diffusely enlarged and firm with fine nodules. A thyroid carcinoma (E) usually presents as a firm, nontender nodule in the gland. Fitzgeral PA, Endocrine Disorders, in Current Medical Diagnosis and Treatment, 52 nd ed. 2013.

Q 117.16: A 63-year-old male with type 2 diabetes mellitus and hyperlipidemia is being seen for routine blood work to assess his renal function. Blood pressure is 130/90 and pulse is 75. His blood chemistries show hypoalbuminemia and hypoproteinemia. His urinalysis shows urine protein excretion of 3.5 grams per 24 hours and microscopically shows oval fat bodies in the urine. The following lab results are within normal range: WBC, BUN, and creatinine. What is the suspected diagnosis? A Pyelonephritis B Glomerulonephritis C Acute renal failure D Nephrotic syndrome E Wilms tumor

The Correct Answer is: D Nephrotic syndrome (D) is diagnosed with bland urine sedimentation, urine protein excretion > 3 g per 24 hours, hypoalbuminemia of < 3g/dl, peripheral edema, hyperlipidemia, and oval fat bodies in the urine. In adults, roughly one-third of patients diagnosed with nephrotic syndrome also have a concurrent systemic disease such as diabetes mellitus, amyloidosis, or systemic lupus erythematosus. Serum creatinine may or may not be abnormal at the time of presentation, depending on the severity, acuity, and chronicity of the disease. Only nephrotic syndrome would show oval fat bodies in his urine. Glomerulonephritis (B) would should hypertension. Pyelonephritis (A) would show an increased WBC count. Acute renal failure (C) would show a change in BUN/creatinine. Wilms' tumor (E) is seen in children and does not have this presentation. Watnick S, Dirkx T. Chapter 22. Kidney Disease. In: Papadakis MA, McPhee SJ, Rabow MW, eds. CURRENT Medical Diagnosis & Treatment 2013. New York: McGraw-Hill; 2013. http://www.accessmedicine.com/content.aspx?aID=11374 . Accessed March 6, 2013.

Q 102.4: Which of the following is diagnostic of nephrotic syndrome? A hypoalbuminemia, hypolipidemia, proteinuria >10 g/24 h B hypoalbuminemia, hyperlipidemia, proteinuria >1 g/24 h C hypoalbuminemia, hyperlipidemia, proteinuria >2 g/24 h D hypoalbuminemia, hyperlipidemia, proteinuria >3.5 g/24 h E normal albumin, hyperlipidemia, proteinuria >10 g/24 h

The Correct Answer is: D Nephrotic syndrome is defined as proteinuria >3.5 g/24 h resulting in hypoalbuminemia (<3.0 g/dL), hyperlipidemia (total cholesterol >250 mg/dL), and edema, probably due to increased renal tubule permeability. Causes include diabetic nephropathy, HIV nephropathy, chronic hepatitis B and C, amyloidosis, systemic lupus erythematosus, constrictive pericarditis, Hodgkin's disease, minimal change disease, and many medications, including phenytoin and NSAIDs. (Watnick and Morrison, 2009, pp. 815-817) Watnick S , Morrison G. Kidney. In: Tierney LM , McPhee SJ , Papadakis MA, eds. Current Medical Diagnosis and Treatment. 48th ed. New York, NY: McGraw-Hill; 2009.

Q 69.4: A 62-year-old female complains of headaches and swelling in her legs for the past 3 months. Her current medications included in the choices below. Which of these medications is most likely responsible for both of her symptoms? A acetaminophen B hydrochlorothiazide (HCTZ) C olmesartan D nifedipine E atorvastatin

The Correct Answer is: D Nifedipine, a calcium channel blocker, is the most likely cause for the patient's complaints. Calcium channel blockers, especially those in the dihydropyridine class, are commonly associated with side effects of headaches and peripheral edema. Olmesartan, an ARB, is not commonly associated with headaches and peripheral edema. HCTZ is a diuretic that would more likely improve edema than cause it. Atorvastatin is a statin used for treatment of hyperlipidemia and is more likely to be associated with myalgias. Acetaminophen is unlikely to cause edema and may actually be used to treat headaches. (McPhee and Papadakis, 2011, Chapter 11)

Q 64.8: At what stage of hospitalization can active tuberculosis patients be placed in a non-negative pressure room? A When patient is afebrile B When patient is on a minimum of three days of antibiotics C When there is a clear chest x-ray D When there is a clear sputum gram stain E When patient is absent of leukocytosis

The Correct Answer is: D Once the patient has had a documented clear sputum gram stain that shows no evidence of the tuberculin bacteria, then the patient can be cleared. Chest x-rays can lag on clearing and would not be an effective measure of clearance. Leukocytosis does not rule in or rule out the infection and is not a reliable indicator.

Q 95.6: A 60-year-old man newly diagnosed with type 2 diabetes mellitus is most likely to have which of the following on ophthalmologic evaluation? A diabetic cataracts B glaucoma C nonproliferative retinopathy D normal fundi E proliferative retinopathy

The Correct Answer is: D Only about 20% of patients with type 2 diabetes have retinopathy at the time they are diagnosed. All patients with diabetes are at increased risk for premature development of cataracts (A) and glaucoma (B) occurs eventually in about 6% of diabetics. Nonproliferative retinopathy (C) is present in just under 2/3 of patients who have had type 2 diabetes for 16 years, and proliferative retinopathy (E) is far more common in patients with type 1. Masharani U, Diabetes Mellitus & Hypoglycemia, in Current Medical Diagnosis and Treatment, 52 nd ed. 2013.

Q 109.20: A 28-year-old man presents with a complaint of new-onset headache. The pain awakens him early in the morning and is described as a sharp, lancinating pain around his right eye, which is 9 out of 10. When he looks in the mirror he notices tearing of his right eye as well as redness and a different sized pupil compared to the left. The pain lasts only for a few minutes but can recur later in the morning. This has happened for the past several days. The patient has a history of recurrent headaches that follow this pattern and usually last for 5 to 7 days. Prior to this occurrence it has been 4 years since his last episode. Which of the following is the most appropriate preventive treatment the patient should be offered at this time? A sumatriptan (Imitrex) B dihydroergotamine C verapamil (Calan) D oral corticosteroids E oxygen

The Correct Answer is: D Oral corticosteroids started immediately will often force a cluster cycle into remission and prevent future headaches. The infrequent recurrence pattern of the cluster headaches does not support the chronic use of verapamil. Sumatriptan, dihydroergotamine, and oxygen are very useful abortive agents but do not work on stopping the cluster cycle. Both sumatriptan and dihydroergotamine have maximum daily doses and may not be used for all attacks during the day as some cluster patients may have multiple attacks throughout the day. (Aminoff et al., 2005, pp. 90-91) Aminoff MJ , Greenberg DA , Simon RP. Clinical Neurology. 6th ed. New York, NY: McGraw-Hill; 2005.

Q 106.3: A 54-year-old man complains of persistent midepigastric abdominal pain 2 weeks following the diagnosis of acute pancreatitis. The patient also complains of anorexia but no fever or chills. There is a palpable mass in the midepigastrium; bowel sounds are normal in all four quadrants. Which of the following is most likely diagnosis? A Adynamic ileus B Pancreatic carcinoma C Infected pancreatic necrosis D Pancreatic pseudocyst

The Correct Answer is: D Pancreatic pseudocysts are the most common complication associated with acute pancreatitis. A pseudocyst should be suspected for a patient who has continued abdominal pain, the development of an abdominal mass, and continued elevations of amylase or lipase levels following an episode of acute pancreatitis. An adynamic ileus would be associated with abdominal distension and changes in bowel sounds; an infected area of necrosis within the pancreatic gland would be associated with fever. Pancreatic cancer may be seen in conjunction with chronic pancreatitis. (Sharp, 2006, p. 359) Sharp KW , Goldin SB , Lomis KD. Pancreas. In: Lawrence PF, ed. Essentials of General Surgery. 4th ed. Philadelphia, PA: Lippincott Williams & Wilkins; 2006.

Q 110.7: A 60-year-old man presented with a mass in the left lobe of the thyroid. Fine needle aspiration was consistent with papillary carcinoma. There was no evidence of locally invasive or metastatic disease. Which of the following treatments is recommended for this patient? A Chemotherapy B External beam radiation C Preoperative radioiodine ablation D Total thyroidectomy

The Correct Answer is: D Papillary carcinoma is the most common type of thyroid malignancy. Treatment includes a thyroid lobectomy and isthmusectomy or total thyroidectomy. The decision regarding the extent of the surgery is based on the extent of the disease, the tumor size, and histiologic grade. A poor prognosis is seen in males, patients older than 50 years of age, primary tumors greater than 4 cm in size, tumors that are less well differentiated, or evidence of locally invasive or metastatic disease. Accordingly, the recommended treatment for this patient is a total thyroidectomy. Radioiodine ablation is recommended postoperatively. (Coe, 2006, pp. 407-408) Coe NPW. Surgical endocrinology: thyroid gland. In: Lawrence PF, ed. Essentials of General Surgery. 4th ed. Philadelphia, PA: Lippincott Williams & Wilkins; 2006.

Q 105.14: Which of the following is the bacteria that is most concerning regarding a plantar puncture wound to the sole of the foot through a rubber soled shoe and virtually unique to that specific skin injury? A bartonella henselae B methicillin-resistant Staphylococcus aureus C methicillin-sensitive Staphylococcus aureus D pseudomonas aeruginosa E streptococcal species

The Correct Answer is: D Pseudomonas aeruginosa is a bacteria frequently cultured from the inside of rubber-soled foot wear, and these bacteria are more frequently pushed deep into plantar puncture wounds when made through a rubber-soled shoe. Treatment of plantar puncture wounds through rubber-soled shoes, especially deep puncture wounds, should factor this into antibiotic consideration. (Skinner, 2006, Chapter 11)

Q 104.8: A patient with no history of treatment for primary syphilis presents with symptoms and signs consistent with secondary syphilis. The most common sign of secondary syphilis is A generalized lymphadenopathy B aseptic meningitis C alopecia D generalized maculopapular rash E superficial painless gummas

The Correct Answer is: D Secondary syphilis generally manifests itself a month or two after appearance of the primary chancre. Patients will complain of headache, fever, sore throat, and malaise and will exhibit generalized lymphadenopathy along with a maculopapular rash that begins at the sides of the trunk and later spreads over the rest of the body. The skin lesions may coalesce in warm moist areas, such as the perineum, and form large, flat-topped, pale papules termed condyloma lata. Skin and mucosal lesions are the most common signs of secondary syphilis. Aseptic meningitis and alopecia may also occur in secondary syphilis. Formation of granulomatous nodules (gummas) is not a feature of secondary disease, but rather is the hallmark of tertiary syphilis. (Philip and Jacobs, 2009, pp. 1301-1302) Philip SS , Jacobs RA. Spirochetal infections. In: McPhee SJ , Papadakis MA, eds. Current Medical Diagnosis and Treatment. 48th ed. New York, NY: McGraw-Hill, 2009.

Q 114.2: A 25-year-old asymptomatic, non-smoking male presents with a 9mm solitary pulmonary nodule, with dense central calcification on routine chest x-ray. The nodule's appearance is smooth and calcified with a well-defined edge. This same lesion was present on a work physical x-ray two years ago. Which of the following is the most appropriate management step for this patient? A Positron emission tomograph to rule out malignancy B Resection of the pulmonary nodule C Thoracic magnetic resonance imaging (MRI) D Watchful waiting with serial imaging E Pulmonary function tests

The Correct Answer is: D Solitary pulmonary nodules are categorized based upon the probability of malignancy. This patient is under the age of 30, with previous documentation of the lesion and without evidence of growth or change, which decreases the probability of malignancy. In this case, watchful waiting is indicated. Risk factors, such as advanced age, smoking, lesion growth or appearance change, and history of prior malignancy increase malignancy probability, and indicate a need for more defined evaluation. (McPhee SJ, Papadakis MA. Current Medical Diagnosis & Treatment 2011, Chapter 9, Pulmonary Disorders)

Q 107.2: A 32-year-old male presents to your office with the complaint of low back pain for 7 months. The patient states he was initially injured on the job while trying to lift a 50-pound barrel off a truck. He denies any paresthesias or bowel/bladder problems associated with the low back pain. The patient states that he had been given NSAIDs and a muscle relaxer, followed by physical therapy treatments. X-rays that were taken 5 months ago were reported as normal. He was placed on light duty at that time. The patient has seen many practitioners who have "not helped him." Another person who works with this patient was at the clinic and stated the patient has had problems with one of his other coworkers. You consider trying the patient on an antidepressant first and then possibly sending him to a pain clinic if there is no success. What is the most likely diagnosis? A conversion disorder B hypochondriasis C drug addiction D somatoform pain disorder E schizophreniform

The Correct Answer is: D Somatoform pain disorder is a focus on pain for greater than 6 months. The subjective findings outweigh the objective findings. Pain in the neck, pelvic, or low back areas are frequent sites, as well as headaches. The disorder may be precipitated by an injury. The patient will have a history of seeing multiple providers and possibly many medical and surgical treatments. The patient is unresponsive to treatment. Stressors can aggravate or precipitate the pain. There may be an expectation of secondary gains. Age of onset is around 30s and 40s. Treatment consists of placing the patient on an antidepressant and sending the patient to a pain clinic. (Ford, 2008, pp. 407-408; Sadock and Sadock, 2008, pp. 284-285) Ford, CV. Somatoform disorders. In: Ebert MH , Loosen PT , Nurcombe B , Leckman JF, eds. Current Diagnosis and Treatment in Psychiatry. New York: McGraw-Hill; 2008. Sadock BJ , Sadock VA. Concise Textbook of Clinical Psychiatry, 3rd ed. Philadelphia, PA: Lippincott, Williams & Wilkins; 2008.

Q 89.8: You evaluate a 72-year-old male for a pre-operative physical examination prior to total left-knee replacement. He has a 20-year history of hypertension and is currently taking lisinopril 10 mg QD and HCTZ 25 mg QD. He denies any complaints. You obtain the EKG below. What finding on this patient's EKG is a potential complication of sustained hypertension? Source: (Knoop et al., 2010, Chapter 23) A acute STEMI B atrial flutter C atrial fibrillation D left ventricular hypertrophy E congestive heart failure

The Correct Answer is: D The EKG represents left ventricular hypertrophy (LVH) with strain pattern. A finding of LVH on an EKG in a patient with hypertension suggests an increased cardiovascular risk. The EKG does not show ST elevations consistent with a STEMI and the patient is asymptomatic. Congestive heart failure (CHF) cannot be diagnosed by this EKG alone, but patients with CHF may have LVH. The patient has a sinus rhythm with distinguishable P waves, which are not consistent with atrial fibrillation or flutter. (McPhee and Papadakis, 2011, Chapter 11)

Q 105.15: A 56-year-old woman is being seen for regular assessment and monitoring of her type 2 diabetes mellitus. She has been following a strict diet and exercise plan for 2 years with the addition of metformin 6 months ago for an increased HgA 1c level. Her HgA 1c at today's visit is 7.1. What is the appropriate management for this patient? A add exenatide to her current therapy B change her oral therapy to rosiglitazone C add insulin to her current therapy D maintain her current therapy and recheck in 6 months

The Correct Answer is: D The HgA 1c goal for this patient is less than 6.5, with action at a level of greater than 8.0. The appropriate action at this time is to continue her current therapy and reassess in 6 months. (Barnett et al, pp. 683-694) Barnett PS , Braunstein GD. Diabetes mellitus. In: Andreoli TA , Carpenter CC , Griggs RC , et al. , Cecil Essentials of Medicine. 7th ed. Philadelphia, PA: WB Saunders; 2007.

Q 109.11: A 25-year-old man presents for evaluation of diarrhea. He is generally healthy and reports he finished a 4-day hike about 2 weeks ago. He does mention that he ran out of water on day 3 and did not have a filter with him. Today he reports that he does not feel too badly but he has had 24 hours of abdominal bloating, increased flatulence, and loose stools. He denies melena or hematochezia. His physical examination is unremarkable but stool ova and parasite examination reveal ova and trophozoites. The most appropriate antimicrobial agent for this patient is A no medicine needed; this is self-limiting and will resolve in 24 hours B ciprofloxacin C amphotericin D metronidazole

The Correct Answer is: D The clinical picture is consistent with infection with Giardia lamblia, a parasite that can be picked up from contaminated water and infects the small intestine. It can be difficult to diagnose because it may be clinically asymptomatic. Nevertheless, an infected person passes the cysts in the stool and they can survive for weeks in cold water. Infection can also be transmitted by direct fecal-oral route, especially among small children and their caregivers. Clinical symptoms vary but often include bloating, loose stools or diarrhea, belching, and possibly weight loss. The course can be episodic. Diagnosis is made with stool specimen examined for ova and parasites or by stool antigen immunoassay. First-line therapy is metronidazole 250 mg po tid for 5 days. Tinidazole is an effective alternative. (Weller, 2008, pp. 1311-1313) Weller PF. Protozoal intestinal infections and trichomoniasis. In: Fauci AS , Braunwald E , Kasper DL, et al., eds. Harrison's Principles of Internal Medicine. 17th ed. New York, NY: McGraw-Hill; 2008.

Q 117.13: In a 73-year-old male, laboratory and radiologic findings that demonstrate marrow plasmacytosis with >30% plasma cells, a monoclonal globulin spike present, and lytic bone lesions is most consistent with which of the following? A Acute myelogenous leukemia B Chronic myelogenous leukemia C Hodgkin lymphoma D Multiple myeloma E Non-Hodgkin lymphoma

The Correct Answer is: D The diagnosis of plasma cell myeloma is confirmed when at least one major and one minor criterion, or three minor criteria, are documented in a patient. This patient has one major and two minor criteria. AML and CML are not associated with lytic lesions. Hodgkin and Non-Hodgkin lymphoma typically do not have lytic lesions or monoclonal globulin spikes. (Lichtman et al., Williams Hematology 8e, Chapter 109, Myeloma)

Q 67.5: A 67-year-old man presents with pain and stiffness in his shoulders and hips lasting for several weeks with no history of trauma. He also has complaints of headache, throat pain, and jaw claudication. It is imperative to diagnose this patient promptly in order to prevent which of the following complications? A anemia B cerebral aneurysms C mononeuritis multiplex D ischemic optic neuropathy E respiratory tract complications

The Correct Answer is: D The most urgent need for diagnosis of a patient with symptoms of polymyalgia rheumatica (PMR) and giant cell arteritis is to prevent blindness caused by ischemic optic neuropathy as a result of occlusive arteritis of the ophthalmic artery. Early diagnosis is imperative as the neurological damage to the optic nerve is not reversible. Most patients with this diagnosis will have a normochromic-normocytic anemia, but this does not create urgency in treatment. Cerebral aneurysms are not common findings with PMR; large vessels such as the subclavian and aorta may be involved in giant cell arthritis in 15% of patients. Mononeuritis multiplex commonly presents with painful paralysis of a shoulder, and respiratory tract complications are more nonclassic findings with the presentation of PMR. (Hellmann and Imboden, 2008, pp. 739-740) Hellmann DB , Imboden JB. Arthritis and musculoskeletal disorders. In: Tierney LM , McPhee SJ , Papadakis MA, eds. 2008 Current Medical Diagnosis and Treatment. New York, NY: McGraw-Hill; 2008:703-756.

Q 116.5: A 45-year-old male with asthma and diabetes is diagnosed with influenza B by nasal swab. He has been ill for one and a half days. Which of the following is indicated for treating this patient? A Acyclovir B Amantadine C Nevirapine D Oseltamivir E Zanamivir

The Correct Answer is: D The neuraminidase inhibitors, including oseltamivir and zanamivir, are associated with a reduction in duration of illness, and secondary complications for both influenza A and B viral strains. However, zanamivir, due to the oral inhalation delivery route, is relatively contraindicated in this patient, due to his history of asthma and an associated increased risk of bronchospasm. Amantadine is inactive against influenza B, as well as certain influenza A strains. Nevirapine is an antiretroviral agent used in the treatment regimen for HIV. Acyclovir is an antiviral agent, but is not indicated for influenza. (Fauci et al., Harrison's Principles of Internal Medicine, 17e, Chapter 180, Influenza)

Q 108.7: A 58-year-old man presents with the acute onset of abdominal pain associated with fever and shaking chills. The patient is hypotensive and febrile with a temperature of 102.2°F. Although he is confused and disoriented, he complains of right upper quadrant pain during palpation of the abdomen. His sclerae are icteric and the skin is jaundiced. Which of the following is the most likely diagnosis? A Acute cholecystitis B Choledocholithiasis C Acute pancreatitis D Ascending cholangitis

The Correct Answer is: D The presenting symptoms associated with ascending cholangitis include fever, chills, right upper quadrant pain, and jaundice (Charcot's triad); the symptoms are secondary to an infected obstruction of the common bile duct. With spread of the infection, the patient may also develop hypotension and mental status changes; these additional symptoms in conjunction with Charcot's triad are known as Reynolds' pentad. Additional symptoms of common bile duct obstruction include light-colored stools and dark, tea-colored urine. (Danziger, 2006, pp. 344-345) Danzinger RG , Nauta R , Park J. Biliary tract. In: Lawrence PF, ed. Essentials of General Surgery. 4th ed. Philadelphia, PA: Lippincott Williams & Wilkins; 2006.

Q 68.3: A 67-year-old male with a history of dilated cardiomyopathy, 3-vessel coronary artery disease, and an ejection fraction of 20% has a history of palpitations for the last two weeks. A 24-hour Holter monitor is ordered and reveals >1000 episodes of the attached rhythm strip. His blood pressure on examination is 102/62, and his resting pulse is 52. Based on these findings, what is the best therapy to treat the patient's symptoms? A Medical management B Coronary bypass surgery C Ablation therapy D Biventricular pacing E No therapy needed

The Correct Answer is: D The rhythm strip reveals episodes of non-sustained ventricular tachycardia. Due to the patient having a low ejection fraction with the dilated cardiomyopathy, primary prevention for this patient at a minimum would be to provide him with a biventricular pacer. (Bashore et al., Current Medical Diagnosis and Treatment, Chapter 10)

Q 115.10: A 54 year old male presents with dark thickened skin and soft pedunculated papules around his neck. He states that the lesions are asymptomatic. What disease process are the findings commonly associated with? A Hypertension B Malignant melanoma C Fungal infection D Metabolic syndrome

The Correct Answer is: D The thick dark plaque around the patient's neck is acanthosis nigricans. The papules are acrochordons (or skin tags). Both conditions are associated with metabolic syndrome. (Wolff & Johnson, p231)

Q 71.4: A 67-year-old female with a history of oxygen dependent emphysema presents with a 4-hour history of increasing shortness of breath and pleuritic chest pain on the right side. Her resting oxygen saturation rate is 90%, and she is having pain on inspiration. On examination, the patient has decreased lung sounds with wheeze on the left and absent sounds on the right. There is also tympany to percussion on the right. Based on these findings, what is the best therapy for this patient? A Needle insertion to right chest wall B Supportive care C Increased oxygen delivery D Chest tube insertion E Nebulized albuterol

The Correct Answer is: D The treatment for this patient, who has a pneumothorax, is chest tube insertion and reinflation of the lung. Once the air leak has been eliminated and the lung appears reinflated on serial chest x-rays, the chest tube may be removed. (Chesnutt MS, Prendergast TJ. Current Medical Diagnosis and Treatment, 2011, Chapter 9, Pulmonary Disorders)

Q 86.4: Your patient with type 2 diabetes mellitus (T2DM) who is already on metformin and pioglitazone but continues to have elevated fasting and postprandial blood sugars. Together, you elect to begin insulin glargine at 10 U QHS initially and have the patient self manage their upwards titration adding 1 IU SQ in the evening every day that their morning fasting glucose is above 100 mg/dL. You caution the patient that she should expect to take approximately how long to achieve well-controlled sugars? A immediately B about one week C about two weeks D about one month E about three months

The Correct Answer is: D There are many regimens to begin and titrate basal insulins. Regardless, most patients require approximately 45 units of glargine to reach therapeutic goals. If they began glargine at 10 IU and add 1 IU daily until they reach 45 IU, this should take approximately 35 more days, or approximately one month. (Fauci et al., 2008, Chapter 338)

Q 121.1: A 22-year-old woman, with no previous medical problems, suddenly cried out, fell to the ground, extended her legs, flexed her arms, and jerked her extremities for 30 seconds. There was associated tongue biting and urinary incontinence. She awoke slowly over a 10-minute period and recalled nothing about the episode. She remained lethargic for several hours but the rest of her neurologic examination was normal. What is the most likely etiology for this episode? A epilepsy B hyperventilation C cardiac arrhythmia D seizure E stroke

The Correct Answer is: D This event represents a well-demarcated episode affecting some combination of consciousness, motor, and/or sensory function consequent to abnormal electrical discharges in the brain. This is consistent with the definition of a seizure. Epilepsy refers to multiple, recurrent seizures. This history is not consistent with hyperventilation, stroke, or cardiac arrhythmia, which would typically include chest pain, shortness of breath, dyspnea on exertion, or focal neurological deficits. (Aminoff et al., 2005, p. 265) Aminoff MJ , Greenberg DA , Simon RP. Clinical Neurology. 6th ed. New York, NY: McGraw-Hill; 2005.

Q 80.6: A 24-year-old male is brought to the emergency department by his girlfriend. She states that he began having a seizure in the car approximately seven minutes ago. She is not sure of his medications, but states he has a known seizure history and has seizures a few times a year. The patient is currently actively seizing. Which of the following is the first-line agent to give this patient? A Carbamazepine B Gabapentin C Levetiracetam D Lorazepam E Phenytoin

The Correct Answer is: D This patient has a known history of seizures, with current ongoing seizure activity and the concern of status epilepticus, a true emergency. Prolonged seizure activity is associated with hyperthermia, metabolic disturbances, cardiopulmonary dysfunction, and irreversible damage. Lorazepam, a benzodiazepine, increases the activity of gamma-aminobutyric acid (GABA) in the brain, thereby calming the excessive electrical nerve activity related to the seizure and slowing the central nervous system. It is the preferred first line agent for temporary control of seizure activity due to the duration of action of approximately 12 to 24 hours. This allows additional therapeutic agents to be administered while gaining control of the seizure activity. The other agents are antiepileptic medications, which are used for long-term seizure control. They are differentiated by efficacy, convenience, side effects, and drug-drug interactions. (Fauci et al., Harrison's Principles of Internal Medicine, 17e, Chapter 363, Seizures and Epilepsy)

Q 69.3: Your 48-year-old male diabetic patient presents for a new-patient visit and a complete physical examination. You elicit a history approximately one month ago of some vague complaints of left shoulder discomfort that lasted about 30 minutes and occurred during strenuous exercise. At the time, he broke out in a sweat. The patient has had no symptoms whatsoever since. The physical examination is normal, as are his vital signs, and his ECG is below. His ECG below is unchanged from previous ECGs. What is the most appropriate plan of action for this patient? A Give the patient a full-dose aspirin and call 911. B Draw labs including CBC, CMP, a fasting lipid panel, hs-CRP, HbA1C, TSH, and treat accordingly when the labs return with a follow-up visit in two weeks. C Draw labs including a CK-MB and troponin, and treat accordingly when the labs return with a follow-up visit in two weeks. D Draw labs including a CBC, CMP, fasting lipid panel, HS-CRP, HbA1C, TSH, and treat accordingly when the labs return with a follow-up visit in two weeks; until that time start the patient on an 81 mg enteric-coated aspirin, metoprolol ER 25 mg once daily and schedule an urgent appointment within the next 72 hours with a cardiologist for a nuclear stress test insuring that the patient knows to call 911 if the symptoms re-occur. E Treat the patient with a proton-pump inhibitor and begin life style changes.

The Correct Answer is: D This patient has a normal ECG and is asymptomatic for many days, but may have had a precursor episode of an acute coronary syndrome. Acute coronary syndromes (ACS) comprise the spectrum of unstable cardiac ischemia from unstable angina to acute myocardial infarction. Rather than the traditional nomenclature of unstable angina, non-Q wave and Q wave myocardial infarction, acute coronary syndromes are now classified based on the presenting ECG as either "ST elevation" or "non-ST elevation." This allows for immediate classification and guides determination of whether patients should be considered for acute reperfusion therapy. ACS represent a dynamic state in which patients frequently shift from one category to another. This warrants protecting the patient with both beta-blockers and aspirin (unless contra-indicated), as well as a cardiology consultation including a stress test, but does not escalate to the point of sending the patient to the emergency department. (McPhee and Papadakis, 2011, Chapter 10)

Q 104.2: A 54-year-old male complains of easy fatigability, loss of sense of wellbeing, anorexia, weight loss, and excessive sweating that has been going for approximately three months. On physical exam he has pallor and splenomegaly. White count is 98,000 (4 to 12,000). The marrow is hypercellular and contains the Ph chromosome. What is the most likely diagnosis? A Acute lymphocytic leukemia B Acute myelogenous leukemia C Chronic lymphoblastic leukemia D Chronic myelogenous leukemia E Multiple Myeloma

The Correct Answer is: D This patient has chronic myelogenous leukemia. 90% of patients presenting with CML are Ph chromosomes positive, and have vague symptoms and complaints that have been going on for weeks to months. Acute lymphocytic leukemia on CBC tends to have an eosinophilia. Acute myelogenous leukemia is not associated with the Ph chromosome. Chronic lymphoblastic leukemia marrow examination notes lymphocytic infiltrates. Multiple myeloma is noted for light chains and not ph chromosome. (Lichtman et al., Williams Hematology 8e, Chapter 90, Chronic Myelogenous Leukemia and Related Disorders)

Q 67.10: A 22-year-old African American male presents to the emergency department with shortness of breath, which started 2 hours prior to arrival. He does not have a history of pulmonary disease that he is aware of, and he states that in the past at random events he has had similar episodes. He does nothing to get the episodes to stop, and he also states that he feels his chest pounding at the same time of the shortness of breath. He has no medical history that he is aware of, and he takes no medications or any illicit drugs. On examination he is alert, awake, and oriented. His vital signs show T 99.0, P 142, R 18, and BP 132/82. His chest x-ray is negative for any acute cardiopulmonary disease, and his electrocardiogram has an irregularly irregular rate of 142 with visible delta waves. Based on the information provided, what anatomical pathway is responsible for the above findings of the delta waves? A Intranodal pathway B Bachmann's bundle C Perkinje fibers D Bundle of Kent E Bundle of His

The Correct Answer is: D This patient is exhibiting Wolff-Parkinson-White (WPW) syndrome by evidence of the delta waves on the electrocardiogram. The anatomic pathway that is known to be the causative abnormality in WPW is the Bundle of Kent (D). Intranodal pathway (A), Bachmann's bundle (B), Perkinje (C), and His bundles (E) are not the anatomic pathways found with WPW. Source: http://www.accessmedicine.com/content.aspx?aID=6357250&searchStr=kent+bundles#6357250

Q 120.9: A 66-year-old male with a history of hypertension, diabetes mellitus, and hypercholesterolemia presents by emergency medical services (EMS) to the emergency department complaining of severe chest pain with radiation into his back. The patient states that he was feeling well in the morning, but while performing some light activity he felt a "ripping" sensation in his back, which he initially thought was a pulled muscle. The pain continued and the patient started to have chest pain, shortness of breath, and lightheadedness. On initial examination the patient is still in pain, pale, diaphoretic, and has a blood pressure of 85/40. His chest is clear to auscultation, and he has a 3/6 diastolic murmur best appreciated at the base of the heart. Given this clinical scenario, what would be the expected finding on chest x-ray? A Normal B Pleural effusion C Decreased lung volume D Widened mediastinum E Cardiomegaly

The Correct Answer is: D This patient is exhibiting a history and physical examination that is consistent with a thoracic aneurysm. The patient's history of hypertension, along with the "ripping" sensation in his back and hypotension give a clinical presentation that is suggestive of a thoracic aneurysm dissection. In this instance it would be expected that the patient would have evidence of a widened mediastinum (D) on chest radiography. While pleural effusions (B), decreased lung volumes (C), and cardiomegaly (E) can all be present on a chest film with an aneurysm, the widened mediastinum would be the expected finding if you suspect an aneurysm. It is very unlikely that a thoracic aneurysm with hypotension would have a normal chest radiograph finding (A). Source: http://www.accessmedicine.com/content.aspx?aID=3651494

Q 99.7: A 22-year-old male presents with his girlfriend. She is concerned because of her boyfriend's behavior and she feels that he needs help. During the visit, you learn that the behaviors of concern include excessive demands for attention and grandiosity. He is also preoccupied with power and shows little interest in others. What is the most likely classification of this personality disorder? A Antisocial B Avoidant C Histrionic D Narcissistic E Schizoid

The Correct Answer is: D This patient is exhibiting all of the characteristics of narcissistic personality disorder. Antisocial personality disorder is characterized by selfishness, callousness, promiscuousness, and impulsive behavior, and an inability to learn from experience and legal problems. Avoidant personality disorder presents clinically as someone who fears rejection, overreacts to rejection and failure, and has poor social endeavors and low self-esteem. Clinical findings of histrionic personality disorder include being dependent, immature, seductive, egocentric, vain, and emotionally labile. Schizoid personality disorder is characterized by shyness, introversion, being withdrawn, and avoiding close relationships. (McPhee SJ, Papadakis MA. Current Medical Diagnosis & Treatment, 2010, p. 951)

Q 107.8: A 28-year-old male presents with complaints of pain along the cervical chain of lymph nodes after ingesting alcohol. In further discussion with him, he mentions having occasional drenching night sweats that have been occurring more frequently over the last month. He has also had a low-grade temperature of 38C on and off over the last several months, but attributed both to the summer weather. He also has lost weight over the last six months without trying. Recently, he has noted a generalized pruritus as well. He has palpable cervical chain lymph nodes. Histologic examination of a lymph node will most likely show which of the following? A Auer rods B IgG lyte chains C Philadelphia chromosome D Reed-Sternberg cells E VZV inclusion bodies

The Correct Answer is: D This patient is presenting with findings consistent with Hodgkin Lymphoma. Pain in involved lymph nodes immediately after the ingestion of alcohol is nearly specific to this disease. On histologic examination of lymph tissue, Reed-Sternberg cells can be seen. Auer rods are seen in AML. Philadelphia chromosome is typically seen in CML. IgG lyte chains are seen in multiple myeloma. VZV inclusion bodies would not be seen in this setting. (Lichtman et al., Williams Hematology 8e, Chapter 99, Hodgkin Lymphoma)

Q 109.10: A 29-year-old male who appears healthy was injured playing basketball and received an x-ray of his spinal column. A mass was noted on the film and referred to the hematologist. He denies shortness of breath or chest pain. He does note drenching night sweats that he has had for several weeks, but assumed it had to do with the hot weather. On exam he has painless lymphadenopathy and is underweight for his height. A CBC is normal and LDH is elevated. What is the most likely diagnosis? A Acute myelogenous leukemia B Cat scratch disease C Infectious mononucleosis D Non-Hodgkin lymphoma E Sarcoidosis

The Correct Answer is: D This patient presents with classic signs of Non-Hodgkin lymphoma, consisting of mediastinal mass, "B" symptoms, and painless lymphadenopathy. AML would not have a normal CBC. Cat scratch disease would have painful lymphadenopathy if present. Infectious mononucleosis would have lymphadenopathy and fever, but not drenching night sweats or mediastinal mass. Sarcoidosis would have granulomatous disease noted on a chest x-ray, not a large mediastinal mass. (McPhee SJ, Papadakis MA, Tierney LM. Current Medical Diagnosis and Treatment, 2010, Chapter 13, Blood Disorders)

Q 105.11: A 63-year-old woman presents with shortness of breath, cough, and proximal muscle weakness of 1-month duration. On clinical exam, she is noted to have a blood pressure of 156/102 mm Hg, facial flushing, mild hirsutism, truncal obesity, marked proximal muscle weakness of both the upper and lower extremity, and hyperpigmentation over the palms and back of the neck. Laboratory exam reveals hypercortisolism and increased ACTH. Which of the following would be the most likely primary diagnosis in this patient? A lymphoma B ovarian cancer C renal cell carcinoma D small cell lung carcinoma

The Correct Answer is: D Tumor cells may secrete hormones that have the same biologic actions as the normal hormone. This patient's symptoms are consistent with adrenocorticoid hyperfunction. The most common cause of ectopic ACTH syndrome is small cell lung carcinoma. This should be suspected in any patient with risk factors for lung cancer. (Friedman, 2007, pp. 663, 591-592) Friedman TC. Adrenal Gland. In: Andreoli TA , Carpenter CC , Griggs RC, et al., eds. Cecil Essentials of Medicine. 7th ed. Philadelphia, PA: WB Saunders; 2007.

Q 88.10: A 62-year-old man presents to the office concerned about an abdominal aortic aneurysm (AAA). He has had no symptoms but states that his father died from an aortic dissection at the age of 50 and his brother was diagnosed last week with an AAA. What is the most appropriate screening tool in this situation? A Abdominal radiograph B Computed tomographic angiography (CTA) C Palpation D Ultrasound E Aortography

The Correct Answer is: D Ultrasonography is cost-effective and is the most commonly utilized screening modality for AAAs. It can be utilized for initial detection of a nonruptured AAA and for monitoring of progression. Anteroposterior and lateral abdominal radiographs may reveal calcification of an AAA as an incidental finding but are nonsensitive/nonspecific. CTA scans are used in monitoring for progression of AAA size and preoperatively to assess aortic anatomy. Abdominal palpation on physical examination may be reliable in thin patients but cannot accurately provide information about the presence or absence of aneurysms in all patients. Aortography is an expensive and invasive procedure traditionally used in preoperatively planning of AAA repair; however, it has largely been supplanted by multiplanar CTA scans. (Rapp, 2006, pp. 810-811) Rapp JH , MacTaggart J. Arteries. In: Doherty GM , Way LM, eds. Current Surgical Diagnosis & Treatment. 12th ed. New York, NY: Lange Medical Books/McGraw-Hill; 2006.

Q 76.5: A 38-year-old male presents with red-brown urine in the mornings. The discoloration has come and gone several times over the last several months. He has also noted hard, painful areas on his skin, which seem to be located over veins, which have occurred intermittently. Laboratories reveal nonspherocytic red cells and a Coombs negative intravascular hemolysis. What is the most likely diagnosis? A Acute Myelogenous Leukemia B Aplastic Anemia C Iron Deficiency Anemia D Paroxysmal Nocturnal Hemoglobinuria E Von Willebrand's Disease

The Correct Answer is: D Urine that appears to be bloody, primarily in the morning and episodic in nature, along with thrombosis of dermal veins, nonspherocytic red cells and Coombs negative intravascular hemolysis are classic signs of PNH. PNH may be a symptom of aplastic anemia, but this patient doesn't have a pancytopenia. He also does not have a microcytic anemia, which rules out iron deficiency. Von Willebrand's is a coagulation disorder. (Lichtman et al., Williams Hematology 8e, Chapter 40, Paroxysmal Nocturnal Hemoglobinuria)

Q 101.5: You are asked to see a diabetic patient with retinopathy and hypertension. On examination, the patient's blood pressure is noted to be 180/90 mm Hg. Urinalysis shows microalbumin of 300 mg/dL. Labs: blood urea nitrogen 22 mg/dL, creatinine 1.5 mg/dL. Which of the following classes of antihypertensive medications would be best to prescribe in this setting? A calcium channel blocker B loop diuretic C alpha blocker D thiazide diuretic E ACE inhibitor

The Correct Answer is: E ACE inhibitors are the drug of choice in this setting. Control of systemic blood pressure can reduce renal vascular damage. In diabetic patients, ACE inhibitors are especially beneficial because of the added effect of reducing intraglomerular pressure and decreasing proteinuria. Current target blood pressure in patients with diabetic nephropathy is <130/80 mm Hg. Calcium channel blockers and diuretics do not offer renoprotective benefits but may be used to control hypertension. (American Diabetes Association, 2009, p. S28; JNC 7 Report, 2003; NKF-K/DOQI Guidelines, 2002, pp. 79-80; Watnick and Morrison, 2009, p. 819) American Diabetes Association . Diabetes care . Diabetes Care. 2009;32:S13-S61. The seventh report of the Joint National Committee on Prevention, Detection, Evaluation, and Treatment of High Blood Pressure: the JNC 7 Report . JAMA. 2003;289:2560-2572. NKF-K/DOQI Clinical Practice Guidelines for Chronic Kidney Disease: Executive Summary. New York, NY: National Kidney Foundation; 2002. Watnick S , Morrison G. Kidney. In: Tierney LM , McPhee SJ , Papadakis MA, eds. Current Medical Diagnosis and Treatment. 48th ed. New York, NY: McGraw-Hill; 2009.

Q 83.9: A 57-year-old male is being monitored for Binet Stage A CLL. He is emergently seen in the clinic with rapid lymph node enlargement, fever, weight loss, and hepatosplenomegaly. On laboratory examination, he is found to have an elevated serum lactate dehydrogenase and a monoclonal gammopathy on serum protein electrophoresis. A retroperitoneal ultrasound reveals bulky adenopathy. What is the most likely diagnosis? A Acute lymphadenitis secondary to HSV B Epstein-Barr viral infection C Hodgkin lymphoma D Rapidly advancing CLL E Richter transformation

The Correct Answer is: E All findings are classic for Richter transformation of CLL to an aggressive large B-cell, high-grade lymphoma. While HSV cannot be totally ruled out without a node biopsy, given the patient history and findings the most likely diagnosis is Richter transformation. Rapidly advancing CLL does not usually develop retroperitoneal adenopathy. Occasionally, Richter syndrome with Hodgkin lymphoma features is seen, but accounts for less than one-fifth of all cases of Richter transformation. EBV may play a role in this later syndrome. (Lichtman et al., Williams Hematology 8e, Chapter 94, Chronic Lymphocytic Leukemia and Related Diseases)

Q 70.3: Your patient is a 66-year-old female who has been dropping her coffee cup and concurrently slurring her speech. The episodes last for approximately 15 minutes. Her blood work, carotid dopplers, and MRI of the brain are normal and you suspect recurrent transient ischemic attacks (TIAs). Which of the following is NOT approved or recommended for the prevention of stroke in this patient? A aspirin B extended-release dipyridamole plus aspirin C clopidogrel D prasugrel E warfarin

The Correct Answer is: E Aspirin, aspirin plus extended-release dipyridamole, and clopidogrel are all antiplatelet agents and approved for use to reduce recurrent TIAs and ischemic cerebrovascular accidents (CVAs). Prasugrel is not FDA-approved for this indication. Warfarin is an anticoagulant and has no role in prevention of either recurrent TIA or ischemic CVA. (McPhee and Papadakis, 2011, Chapter 24)

Q 82.1: A 24-year-old female HIV-positive patient, who is not currently on medication, presents to the emergency department with acute dyspnea, tachycardia, fever, nonproductive cough, and a room air oxygen saturation of 92%. She admits feeling poorly for the past five days. A physical exam reveals bilateral basilar crackles. An x-ray reveals Pneumocystis jiroveci pneumonia. The patient has no drug allergies. Which of the following is the first-line treatment of choice? A Amphotericin B B Clarithromycin C Clindamycin D Pentamidine E Trimethoprim-sulfamethoxazole

The Correct Answer is: E Based upon current clinical data, the preferred agent for treatment of Pneumocystis jiroveci infections is oral trimethoprim-sulfamethoxazole. Second-line medications may include single agent or combination therapy, utilizing clindamycin, primaquine, dapsone, pentamadine, and/or atovaquone. Clarithromycin is the agent of choice for Mycobacterium avium complex infection, and amphotericin B would be indicated for Cryptococcal meningitis, as well as other fungal infections. (McPhee SJ, Papadakis MA. Current Medical Diagnosis & Treatment 2011, Chapter 36, Mycotic Infections)

Q 65.6: A 24-year-old man with a recent history of a viral illness comes to the emergency department complaining of severe left-sided chest discomfort, which radiates through to the left trapezius region. On coming into the room, you note that he is sitting up and hunched forward. On physical examination, the patient's temperature is 39°C, blood pressure is 135/78, with a pulse of 85 bpm, and a pericardial friction rub is noted. Laboratory findings demonstrate elevated serum creatine kinase levels and normal serial troponin levels. His EKG demonstrates peaked T waves. His CXR demonstrates no acute process. Which of the following is the most appropriate treatment for this patient? A Morphine B Enoxaparin C Nitroglycerin D Penicillin V E Indomethacin

The Correct Answer is: E Choice E, indomethacin 25-75 mg QID, and bed rest would be the most appropriate treatment in a patient with acute viral pericarditis, as a nonsteroidal anti-inflammatory agent will ameliorate the inflammatory process. Choices A and C are appropriate in a patient suspected of acute coronary syndrome. Choice B, enoxaparin, is contraindicated in patients with pericarditis, as anticoagulants could lead to worsening of pericardial effusion and cardiac tamponade, especially if it is secondary to bleeding into the pericardial space, such as with trauma or postoperatively. (Fauci et al., 2001, p. 1369)

Q 119.18: A 57-year-old woman with a history of rheumatic fever is seen complaining of dyspnea while vacuuming her apartment, which has been worsening over the last few months. On physical exam, a possible opening snap, loud S 1 , and a very soft diastolic rumbling murmur is auscultated. When the patient is placed in the left lateral decubitus position, the murmur is accentuated, and heard best at the apex. With inspiration, the murmur does not increase in amplitude. Which of the following is the most likely finding on echocardiogram? A Tricuspid regurgitation B Tricuspid stenosis C Atrial septal defect D Aortic regurgitation/insufficiency The Correct Answer is: E Choice E, mitral stenosis, is the most likely finding in a patient with a history of rheumatic fever presenting with a possible opening snap, loud S 1 , and a very soft diastolic rumbling murmur which is heard best at the cardiac apex and accentuated by placing the patient in the left lateral decubitus position. Choices A and B, tricuspid regurgitation and tricuspid stenosis, are also linked with patients with rheumatic heart disease. The murmur of tricuspid regurgitation, however, is a systolic murmur, which increases with inspiration and is heard best at the left lower sternal border. Tricuspid stenosis presents with a diastolic murmur, and with inspiration, the murmur increases. It is also heard best at the left lower sternal border. Choice C, an atrial septal defect, if large, could present with similar symptoms of exertional dyspnea secondary to a large shunt, but auscultation would reveal a moderately loud systolic ejection murmur heard best in the second and third interspaces. This is secondary to increased pulmonary arterial flow. Choice D, aortic regurgitation/insufficiency, is also a diastolic murmur; however, it is usually a diastolic decrescendo murmur heard best at the left sternal border. (LeBlond et al., 2009, Chapter 8) E Mitral stenosis

The Correct Answer is: E Choice E, mitral stenosis, is the most likely finding in a patient with a history of rheumatic fever presenting with a possible opening snap, loud S 1 , and a very soft diastolic rumbling murmur which is heard best at the cardiac apex and accentuated by placing the patient in the left lateral decubitus position. Choices A and B, tricuspid regurgitation and tricuspid stenosis, are also linked with patients with rheumatic heart disease. The murmur of tricuspid regurgitation, however, is a systolic murmur, which increases with inspiration and is heard best at the left lower sternal border. Tricuspid stenosis presents with a diastolic murmur, and with inspiration, the murmur increases. It is also heard best at the left lower sternal border. Choice C, an atrial septal defect, if large, could present with similar symptoms of exertional dyspnea secondary to a large shunt, but auscultation would reveal a moderately loud systolic ejection murmur heard best in the second and third interspaces. This is secondary to increased pulmonary arterial flow. Choice D, aortic regurgitation/insufficiency, is also a diastolic murmur; however, it is usually a diastolic decrescendo murmur heard best at the left sternal border. (LeBlond et al., 2009, Chapter 8)

Q 117.1: A 22-year-old recent immigrant from Vietnam, who is 28 weeks pregnant with her first child, presents to the emergency department with complaints of worsening dyspnea and lower extremity edema. She is unable to answer definitively whether or not she has a history of rheumatic fever. On physical examination, a possible opening snap, loud S 1 , and a very soft diastolic rumbling murmur is auscultated. When the patient is placed in the left lateral decubitus position, the murmur is accentuated, and heard best at the apex. With inspiration, the murmur does not increase in amplitude. Which of the following is the most likely finding on echocardiogram? A Tricuspid regurgitation B Tricuspid stenosis C Atrial septal defect D Aortic regurgitation/insufficiency E Mitral stenosis

The Correct Answer is: E Choice E, mitral stenosis, is the most likely finding in this patient, who presents with physical exam findings including a possible opening snap, loud S 1 , and a very soft diastolic rumbling murmur which is heard best at the cardiac apex and accentuated by placing the patient in the left lateral decubitus position. Although rheumatic fever was not positively confirmed, the patient likely did have a history, given that the majority of cases of mitral stenosis are secondary to rheumatic heart disease. Patients from Asia, Central America, and South America are exposed more frequently than their counterparts in more developed countries, where antibiotic use is more common. Choices A and B, tricuspid regurgitation and tricuspid stenosis, are also linked with patients with rheumatic heart disease. The murmur of tricuspid regurgitation, however, is a systolic murmur, which increases with inspiration and is heard best at the left lower sternal border. Tricuspid stenosis presents with a diastolic murmur, and with inspiration the murmur increases. It, too, is heard best at the left lower sternal border. Choice C, an atrial septal defect, if large, could present with similar symptoms of exertional dyspnea secondary to a large shunt, but auscultation would reveal a moderately loud systolic ejection murmur that is heard best in the second and third interspaces. This is secondary to increased pulmonary arterial flow. Choice D, aortic regurgitation/insufficiency, is also a diastolic murmur; however, it is usually a diastolic decrescendo murmur that is heard best at the left sternal border. (Fauci et al., 2008, Chapter 7; LeBlond et al., 2009, Chapter 8)

Q 69.9: A 67-year-old woman with a history of gastric ulcers is admitted with complaints of recent onset of dyspnea with exertion, 3 pillow orthopnea, lower extremity edema, and palpitations, in which she describes her heart as racing. Which of the following is likely to be the cause of her high-output congestive heart failure? A Mitral regurgitation B Aortic stenosis C Uncontrolled hypertension D Ruptured chordae tendinae E Severe anemia

The Correct Answer is: E Choice E, severe anemia, may result with progression of gastric ulcers, and is the only high-output cause of congestive heart failure among the choices offered. Other causes include severe anemia, thyrotoxicosis, and arteriovenouis shunting (for example, in hemodialysis patients). Choice A, mitral regurgitation, is a cause of excessive preload leading to heart failure. Choice D, ruptured chordate tendinae, associated with mitral regurgitation, would also be a cause of excessive preload leading to heart failure. Choices B and C, aortic stenosis and uncontrolled hypertension, are causes in which too much afterload leads to heart failure. (McPhee et al., 2011, Chapter 10)

Q 72.7: A 50-year-old woman with a history of hypertension complains of chest tightness and dyspnea while walking up one flight of stairs. She recently experienced an episode of near-syncope while walking her dog. She denies a history of rheumatic fever. On auscultation, a crescendo-decrescendo systolic ejection murmur is heard at the upper right sternal border, radiating to the carotids bilaterally. Troponin levels are negative at 0, 3, and 6 hours. Her EKG demonstrates evidence of left ventricular hypertrophy. Given the patient's physical exam findings and recent symptoms, which of the following is the most appropriate next diagnostic study? A Chest X-ray B Transesophageal echocardiogram C Holter monitor D Treadmill exercise stress test E Transthoracic echocardiogram

The Correct Answer is: E Choice E, transthoracic echocardiogram, is a simple, sensitive, and non-invasive diagnostic tool which can evaluate for the presence of valvulopathy in a patient in this age group, who is likely demonstrating severe aortic stenosis secondary to a congenital bicuspid valve. Patients with a congenital bicuspid aortic valve typically develop symptoms once the valve leaflets have become calcified and thickened, secondary to the undue stress over many years on a structurally abnormal aortic valve. Choice A might be able to give evidence of cardiomegaly or calcification of heart valves, but would not be sensitive enough to detect the degree of valvulopathy, if present. Choice B, transesophageal echocardiogram, would give information regarding valvulopathy, but is a more invasive test; therefore, choice E is more appropriate. Choice C is a useful diagnostic tool for evaluation of patients complaining of palpitations, but incorrect for this patient, who has no symptoms of palpitations. Choice D, although a useful diagnostic tool for the evaluation of exercise tolerance and in patients complaining of chest pain, does not allow direct visualization of the heart valves to evaluate the degree of aortic stenosis; as the patient is likely demonstrating severe aortic stenosis, cardiac catheterization to evaluate for coronary artery disease prior to surgery will need to be performed. (McPhee et al., 2011, Chapter 10)

Q 114.5: A 22-year-old woman with a history of scoliosis presents to the office with complaints of a retrosternal chest discomfort, occurring frequently at rest and lasting for several hours at a time. She is currently experiencing this chest discomfort during the office visit. On physical exam, a mid-systolic click is noted. With standing, the click moves closer to S1. An EKG demonstrates normal sinus rhythm at 76 bpm, with no acute ST or T wave changes. Which of the following diagnostic studies would be the most appropriate next step given this patient's physical exam findings? A Chest x-ray B Transesophageal echocardiogram C Holter monitor D Treadmill exercise stress test E Transthoracic echocardiogram

The Correct Answer is: E Choice E, transthoracic echocardiogram, is a simple, sensitive, and non-invasive diagnostic tool, which can evaluate for the presence of valvulopathy or congenital heart disease in this young patient. Choice A might be able to give evidence of cardiomegaly, but would not be sensitive enough to detect valvulopathy. Choice C is a useful diagnostic tool for evaluation of patients complaining of palpitations, but incorrect for this patient who has no symptoms of palpitations. Choice D, although a useful diagnostic tool for the evaluation of exercise tolerance and in patients complaining of chest pain, does not allow direct visualization of the heart valves to evaluate for valvulopathy. Given her relative youth and lack of other cardiac risk factors, transthoracic echocardiogram should be performed before stress testing. (Fauci et al., 2008, Chapter 230)

Q 121.10: A 57-year-old man with a history of HTN, hyperlipidemia, and chronic tobacco use presents to the office with complaints of worsening chest tightness over the last 2 months. He initially noticed that every time he raked leaves he had a few minutes of chest tightness, which was relieved within 5 minutes if he rested. He now notices that raking will precipitate severe chest discomfort, diaphoresis, and dyspnea, which lasts for 20 minutes even if he rests. Last night, while watching football, he again noticed chest tightness, which began suddenly and slowly dissipated over 15 minutes. His physical examination is normal. Which of the following is the most likely diagnosis? A Pericarditis B Non-ST-segment myocardial infarction C Stable angina pectoris D Prinzmetal angina E Unstable angina pectoris

The Correct Answer is: E Choice E, unstable angina pectoris, is based on clinical presentation, and requires chest or arm discomfort or an anginal equivalent, that either occurs at rest or with minimal activity lasting for at least 10 minutes, recent onset of severe chest discomfort, or anginal equivalent, and/or chest discomfort or anginal equivalent that has progressively been increasing in either severity, frequency, or duration. Choice A, pericarditis, would present with chest discomfort that is worse while supine and improves while sitting forward, as well as a pericardial friction rub. Choice B, acute myocardial infarction, requires troponin elevation to establish the diagnosis. Choice C, stable angina pectoris, is chest or arm discomfort that is reliably precipitated by activity and/or emotional distress, and relieved with rest or sublingual nitroglycerin. Choice D, prinzmetal angina, or variant angina pectoris, is defined as a coronary artery spasm associated with ST-segment elevation, usually occurring at rest and frequently at the same time of the day. (Fauci et al., 2008, Chapter 238)

Q 120.5: You are asked to see a patient who was admitted to the hospital. Upon attempts to obtain a history, you notice the patient states words that sound similar, but do not have the same meaning. He also does some rhyming of his words. What type of thought process would this be? A flight of ideas B circumstantiality C looseness of association D word salad E clanging

The Correct Answer is: E Clanging is a disturbance in thought in which the person selects words that are similar by sound, but do not mean the same. Sometimes the person will rhyme the words. Flight of ideas is rapid transitioning between subjects, but tends to be connected. Looseness of association is when a person changes subjects, but there is no connection between the subjects. Circumstantiality is where the person has a point and eventually gets to that point, but with delay in the thought process. Word salad is a mixture of words that have no sense. (Sadock and Sadock, 2008, pp. 23-32) Sadock BJ , Sadock VA. Concise Textbook of Clinical Psychiatry, 3rd ed. Philadelphia, PA: Lippincott, Williams & Wilkins; 2008.

Q 105.5: Which of the following medications, if started within five days of symptoms, has been shown to increase the rate of full recovery in an acute Bell's palsy patient? A Acyclovir B Aspirin C Midazolam D Nonsteroidal anti-inflammatory E Oral corticosteroid

The Correct Answer is: E Corticosteroids, used for their systemic anti-inflammatory and immune properties, have been shown to increase the proportion of Bell's palsy patients who recover completely, particularly over time. These should be considered as beneficial for the clinical outcome of the patient. Acyclovir, an antiviral medication, has not shown any additional benefit. Nonsteroidal anti-inflammatory medications may aid patient comfort, but have not been shown to impact outcome. Midazolam is not considered a first-line therapy for patients with Bell's palsy. (McPhee SJ, Papadakis MA. Current Medical Diagnosis & Treatment 2011, Chapter 24, Nervous System Disorders)

Q 83.1: A 46-year-old female with a history of poorly-controlled diabetes and grand mal seizures presents for evaluation of bilateral foot pain. She describes the pain as burning and has noticed it is worse at night. She occasionally has this pain in her feet. She denies other medical conditions, and her medications are metformin and dilantin. Which of the following medications, used for the suspected diagnosis, should be avoided in this patient? A Capsaicin B Desipramine C Gabapentin D Lidocaine patch E Tramadol

The Correct Answer is: E Diabetic peripheral neuropathy is the diagnosis, and describes any neuropathy in the diabetic patient. This patient is exhibiting a distal symmetric polyneuropathy with the classic associated symptoms, commonly called "pins and needles" by patients. When associated with pain and functional impact, pharmacologic therapy is warranted. There are many agents to choose from, with each of the answer choices being options. However, in a patient with a known seizure disorder, tramadol should be avoided, as it decreases the seizure threshold. Gabapentin, also a seizure medication, may be used, but close monitoring is suggested. (Tintinalli et al., Tintinalli's Emergency Medicine: A Comprehensive Study Guide, 7e, Chapter 166, Acute Peripheral Neurologic Lesions)

Q 71.1: What class of medications are the antibiotics of choice for adults with acute pertussis? A Cephalosporins B Penicillins C Fluroquinolones D Aminoglycosides E Macrolides

The Correct Answer is: E Erythromycin, azithromycin, and clarythromycin are all acceptable choices for the acute treatment of pertussis. Trimethoprim-sulfamethoxazole is an acceptable alternative to the macrolides for allergy or macrolide intolerant patients. (Schwartz BS. Current Medical Diagnosis and Treatment, 2011, Chapter 33, Bacterial and Chlymdial Infections)

Q 116.10: Mrs. Jones was referred for screening colonoscopy at the age of 50. She has no personal or family history of colorectal cancer. No polyps or lesions were found during the exam. She should be advised that colonoscopy should be repeated in how many years? A 1 year B 2 years C 3 years D 5 years E 10 years

The Correct Answer is: E In average-risk individuals aged 50 or greater than 50, screening colonoscopy should be repeated every 10 years following an initial normal exam. If the individual has a first-degree relative with a history of adenomas or colorectal cancer, screening should begin earlier, generally at age 40 or 10 years younger than the age at diagnosis of the youngest affected relative. (Rugo, 2009, pp. 1452-1453) Rugo HS. Cancer. In: McPhee SJ , Papadakis MA, eds. Current Medical Diagnosis and Treatment. 48th ed. New York, NY: McGraw-Hill; 2009.

Q 113.8: A 36-year-old man with a history of tobacco use notices palpitations after attending a Super Bowl party. He admits to drinking at least six bottles of beer and several mixed drinks, which is much more than he usually drinks. He denies chest discomfort or dyspnea. On physical exam, his BP is 139/82, P 136, RR 22, and Temp 37°C. On auscultation, his heart rate is tachycardic and irregular, without S3 or murmur appreciated. His echocardiogram demonstrates normal LV systolic function, normal left atrial dimensions, and normal valvular function. Based on his history, what is the most likely diagnosis regarding his palpitations? A Sinus tachycardia B Wolff-Parkinson-White syndrome C Normal sinus rhythm with PACs D Alcohol withdrawal E Holiday heart syndrome

The Correct Answer is: E In young patients without left ventricular systolic dysfunction or valvular dysfunction, binge drinking can lead to episodes of atrial fibrillation, also known as holiday heart syndrome. This is a syndrome in which atrial fibrillation is linked with excessive alcohol intake frequently higher than usual; however, in some patients, moderate alcohol intake can also lead to episodes of atrial fibrillation. Choices A, B, and C are not necessarily associated with a history of alcohol intake. Choice D is not described in this patient scenario. (Fauci et al., 2008, Chapter 387)

Q 64.5: A 63-year-old male with a longstanding history of chronic obstructive pulmonary disease and a recent exacerbation, is also found to have new onset rapid atrial fibrillation. Which of the following medications, used for atrial fibrillation, should be used cautiously as it is associated with bronchospasm and pulmonary function changes? A Amiodarone B Digoxin C Diltiazem D Ibutilide E Metoprolol

The Correct Answer is: E Initial atrial fibrillation management will depend on the stability of the patient, and may involve emergent electrocardioversion if the patient is unstable. Several types of medications are used for medication cardioversion and subsequent rate management in atrial fibrillation patients, including each of the medications listed in the answer choices. Caution should be used when administering a beta-blocker, even a cardioselective choice, to a patient with longstanding lung disease and recent exacerbation, as this may promote bronchospasm and associated dyspnea.

Q 72.5: In a patient with chronic hepatitis C infection, which of the following medical conditions would be considered a contraindication to starting the patient on interferon? A hypertension B hyperlipidemia C diabetes D migraine headaches E systemic lupus erythematosus

The Correct Answer is: E Interferon is contraindicated in patients with autoimmune disease. Interferon is also contraindicated in patients with severe liver disease and history of cardiac arrhythmia. It should be used with caution in patients with major depressive disorders, cytopenia, hyperthyroidism, and severe renal insufficiency. (Safrin, 2009, pp. 868) Safrin S. Antiviral agents. In: Katzung BG , Masters SB , Trevor AJ, eds. Basic and Clinical Pharmacology. 11th ed. New York, NY: McGraw-Hill; 2009.

Q 68.1: In patients with dilated cardiomyopathy who have multiple runs of symptomatic non-sustained ventricular tachycardia, what is the best intervention to treat this condition? A Ablation therapy B Heart transplantation C Medical therapy alone D Diet modification E Implantable defibrillator

The Correct Answer is: E It is recommended for patients with a history of dilated cardiomyopathy who have documented non-sustained ventricular tachycardia to have implantable defibrillators inserted for primary prevention of sudden cardiac death. Medical therapy alone does not protect the patient from arrhythmias, and ablation therapy is not indicated for this type of arrhythmia. (Bashore et al., Current Medical Diagnosis and Treatment, Chapter 10)

Q 92.8: A 61-year-old male presents with acute onset dyspnea, cough with thick, blood tinged sputum, and fever. He is a known, longstanding alcoholic and has Type II Diabetes, for which he takes no medication. A chest x-ray reveals a right upper lobe infiltrate with early fibrotic changes. Which of the following is an appropriate first-line medication(s) for the most likely etiologic organism? A Macrolide B Doxycycline C Second generation cephalosporin D Beta-lactam plus fluoroquinolone E Fluoroquinolone

The Correct Answer is: E Klebsiella pneumoniae, a gram negative bacteria, is associated with pneumonia in immunocompromised patients, especially alcoholics. Community-acquired pneumonia due to Klebsiellae is associated with a broad range of presentations. This may range from mild acute bronchitis or bronchopneumonia to acute onset of high fevers, chills, and cough with blood tinged, thick sputum (often referred to as "currant jelly sputum"), and aggressive chest x-ray findings, such as cavitation and empyema. Community-acquired pneumonia empiric treatment is based upon the likely causative organisms within patient subsets. This includes appropriate selections for individuals able to be handled as outpatients, but with significant past medical history and risk factors for drug-resistant bacteria, such as alcoholism. The guidelines currently indicate that these patients should be treated with a beta-lactam plus a macrolide or doxycycline, or a fluoroquinolone, or amoxillin-clavulanate. The guidelines also include treatment recommendations for other patient subsets, and alternative medications to consider (which can be viewed in Table 251-4). (Harrison's Online, Chapter 251, Part Ten: Disorders of the Respiratory System, Section 2: Diseases of the Respiratory System)

Q 117.17: Which of the following is the most prevalent fatal cancer in the United States? A Cervical B Colon C Esophageal D Liver E Lung

The Correct Answer is: E Lung cancer continues to lead as the major cause of cancer deaths in both men and women in the United States, with cigarette smoking causing greater than 90% of cases. Despite educational campaigns highlighting the risks of smoking, lung cancer continues to kill more individuals that colorectal, breast, and prostate cancers combined. (McPhee SJ, Papadakis MA. Current Medical Diagnosis & Treatment 2011, Chapter 39, Cancer)

Q 116.15: An elderly patient is brought in to the emergency department (ED) complaining of incontinence of liquid "like tea water" stool. He is complaining of rectal pressure and lower abdominal pain. The pain is cramping in quality and the patient's abdomen is "bloated." Digital rectal exam reveals hard stool in the rectum. Which of the following should be selected as the initial treatment for this patient? A passing a nasogastric tube B milk of magnesia C opiate analgesics for pain D oral sodium phosphate E manual disimpaction

The Correct Answer is: E Mechanical bowel obstruction in the rectum does not usually respond to oral laxatives. A nasogastric tube would not be used for an obstruction in the distal colon/rectum. One would avoid opiates in fecal impactions and other constipation problems because they tend to be more constipating. This patient needs to be disimpacted. Oral agents are unlikely to be effective against the fecal impaction and may cause complications. (McQuaid, 2008, p. 481) McQuaid KR. Gastrointestinal disorders. In: McPhee SJ , Papadakis MA, eds. Current Medical Diagnosis and Treatment. 47th ed. New York, NY: McGraw-Hill; 2008.

Q 80.4: A 34-year-old female with a definitive diagnosis of multiple sclerosis presents with an acute symptomatic exacerbation for two days, causing difficulty ambulating and markedly painful paresthesias, which is affecting her ability to sleep. She is currently on Interferon-β-1A, although she admits to missing some recent doses. Which of the following medications is most appropriate for treating her acute exacerbation? A Glatiramer acetate B Interferon-β-1B C Mitoxantrone D Natalizumab E Prednisone

The Correct Answer is: E Multiple sclerosis (MS) therapy is often discussed by the goal of the treatment. Medications, such as the glucocorticoids, are utilized for acute exacerbations or initial episodes of MS, to decrease exacerbation severity. Additionally, they are used for associated conditions, such as optic neuritis. Steroids have not been shown to decrease MS progression or impact the relapse rate. Other medications have shown to be beneficial regarding altering disease progression and/or the relapse rate. These medications include Interferon-β agents, glatiramer acetate, and Natalizumab. Mitoxantrone, an antineoplastic agent, is also indicated as an MS disease-altering agent, but due to a high risk of cardiotoxicity with prolonged use, it is a second-line agent. (McPhee SJ, Papadakis MA. Current Medical Diagnosis & Treatment 2011, Chapter 24, Nervous System Disorders)

Q 72.4: A 24-year-old male has an eight-month history of loose thought associations, social withdrawal, auditory hallucinations, and deterioration in his personal appearance and hygiene. Upon examination, he is noted to have a flat affect, perceptual distortions, and behaves like he is detached from his own actions. An atypical antipsychotic medication is chosen for this patient. Which of the following medications is classified as an atypical antipsychotic? A Chlorpromazine (Thorazine) B Haloperidol (Haldol) C Loxapine (Loxitane) D Molindone (Moban) E Olanzapine (Zyprexa)

The Correct Answer is: E Olanzapine is the only atypical antipsychotic listed in the given answers. The remainder of the medications listed are typical antipsychotic medications. (McPhee SJ, Papadakis MA. Current Medical Diagnosis & Treatment, 2010, p. 954)

Q 101.6: A 55-year-old woman with a history of emphysema, who is undergoing chemotherapy for lung cancer, is sent to see you by her oncologist regarding a sudden increase in dyspnea, with exertion and fatigue. On physical exam, a drop in systolic blood pressure of 20 mm Hg is noted upon inspiration. What is this physical exam finding known as? A Pulsus parvus B Pulsus alternans C Bisferiens pulses D Pulsus bigeminus E Pulsus paradoxus

The Correct Answer is: E Pulsus paradoxus is defined as a decrease in systolic arterial pressure of greater than 10 mmHg. It is an accentuation of the normal decrease in systolic arterial pressure of less then 10mm Hg that normally accompanies inspiration. It is frequently noted in patients with pericardial tamponade. Pulsus parvus means a small weak pulse. Pulsus alternans is noted in patients who despite a regular rhythm, demonstrate a regular alteration of the pressure pulse amplitude. This is frequently found in patients with severe left ventricular systolic dysfunction. A bisferiens pulse is a pulse with two systolic peaks, commonly seen in patients with aortic regurgitation or hypertrophic cardiomyopathy. (Fauci et al., 2001, p. 1256)

Q 88.3: A 62-year-old male diabetic patient with a past medical history of percutaneous transluminal coronary angioplasty (PTCA) with drug-eluting stent presents to review his blood work. He currently takes rosuvastatin 10 mg daily, metformin 1,000 mg BID, an 81 mg aspirin, and ramapril 5 mg, along with metoprolol ER 50 mg daily. His total cholesterol is 212 mg/dL, LDL-C is 126 mg/dL, HDL-C is 46 mg/dL, and triglycerides at 145 mg/dL. Of the following, what is the best choice for him to reach ATP-III revised goals? A Add a fenofibrate 145 mg daily. B Add garlic to his diet. C Add an omega-3 ethyl acid ester daily. D Increase his rosuvastatin to 20 mg daily. E Increase his rosuvastatin to 20 mg daily and add ezetimibe 10 mg daily.

The Correct Answer is: E Statins lower LDL-C by 20% to 55%, depending on the dose and statin used. Regardless, analysis of dose-response relationships for all statins demonstrates that the efficacy of LDL-C lowering is log-linear; LDL-C is reduced by 6% (from baseline) with each doubling of the dose. In doubling his rosuvastatin, it would be reasonable to expect his LDL-C to drop from 126 to 119 mg/dl, and this would not be a goal for the patient. While there is no outcome data, laboratory data would indicate that if ezetimibe were then added, LDL-C would drop approximately another 25% to approximately 89 mg/dl, closer to his ATP-III goal of 70 mg/dl. Intensifying his life style changes might then allow him to reach goal. (Brunton, et al., 2006, Chapter 35)

Q 110.15: You are asked to counsel a patient concerning prognosis of treatment for their non-Hodgkin lymphoma. The patient is 70 years old, has a prior history of smoking and a myocardial infarction 15 years ago, is overweight and has a sedentary lifestyle, and his disease is stage IV. What is the most likely prognosis? A 100% five year survival B 80% five year survival C 60% five year survival D 40% five year survival E Less than 20% five year survival

The Correct Answer is: E The International Prognostic Index is widely used to categorize patients into prognostic groups. Patients with greater than two risk factors have the lowest response rates to treatment. Worse prognosis is for patients over 60 years of age with elevated serum LDH, stage III or stage IV disease, and/or poor performance status. This patient has all but the elevated serum LDH. (Non-Hodgkin Lymphomas, Quick Answers to Medical Diagnosis and Therapy)

Q 120.4: What is the hallmark finding on an EKG that is consistent with Wolff-Parkinson-White syndrome? A Prolonged PR interval B Long QT interval C Widened QRS complex D Sinus arrhythmia E Delta wave formation

The Correct Answer is: E The delta wave appears as an up sloping curvature that begins the QRS complex (as seen on the ECG shown). (Calkins H. Hurst's the Heart, Chapter 38, Supraventricular Tachycardia: AV Nodal Reentry and Wolff-Parkinson-White Syndrome)

Q 111.5: A 50-year-old male states that his eye is bothering him since yesterday. He complains of pain and redness. He states that he mowed his lawn yesterday and that it was windy outside. He attempted to irrigate the eye but still has significant irritation. He notes that it hurts to blink his eyes. What is the correct sequence of steps to treat this condition? A Anesthetic drops, irrigate the eye, and perform tonometry B Prescribe antibiotic cream and pain medication C Fluorescein stain, irrigate the eye, and prescribe antibiotic cream D Fluorescein stain and lid eversion E Anesthetic drops, fluorescein stain, and lid eversion

The Correct Answer is: E The history suggests a retained foreign body to the upper eyelid. A fluorescein stain will reveal significant superficial vertical scratches on the cornea. An upper eyelid eversion must be done, to inspect for and remove the foreign body. If the practitioner is successful in removing the foreign body, relief of the irritation will be immediate. (Tintinalli et al., Chapter 236)

Q 88.9: A 42-year-old woman has a developed a solid and quite firm thyroid mass that is approximately 1 cm diameter by palpation. She does not have any hoarseness, difficulty breathing or swallowing, or symptoms of thyroid disease. She has never been exposed to radiation and has no history of cancer, nor does she have a family history of thyroid or other cancers. A thyroid panel is within normal limits. What is the next step in the care of this patient? A fine-needle aspiration biopsy of the nodule B observation to see if the nodule increases in size C radionuclide thyroid scan D resection of the nodule E ultrasonography of the neck

The Correct Answer is: E The next step in the care of this woman is neck ultrasonography to determine its size and characteristics; this technique is preferred to CT or MRI scanning because it is accurate, noninvasive, far less costly. Should the ultrasound results be suspicious for malignancy, a fine-needle aspiration biopsy (A) should follow. A thyroid mass 1 cm in diameter or larger must be evaluated rather than observed (B). Radionuclide scanning (C) would be indicated if the patient's TSH were low but is otherwise limited in determining whether a thyroid mass is malignant. Resection (D) would be inappropriate until further testing is done. Fitzgerald PA, Endocrine Disorders, in Current Medical Diagnosis and Treatment, 52 nd ed. 2013.

Q 91.2: A 45-year-old woman with recent diagnosis of rheumatoid arthritis has begun treatment with celecoxib. She has been on this medication for 3 months and notes that her pain continues. Early signs of joint involvement are present in the patient's hands. Which of the following medications is the most appropriate to add to her treatment? A aspirin B rituximab C etanercept D leflunomide E methotrexate

The Correct Answer is: E The treatment of rheumatoid arthritis (RA) is aimed at reduction of pain, preservation of function, and prevention of deformity. Although non-steroidal anti-inflammatory drugs (NSAIDs) provide symptomatic relief, they do not alter progression or prevent erosion of the joint. Consequently, in addition to NSAID therapy, disease-modifying anti-rheumatological drugs (DMARDs) should also be initiated as soon as the diagnosis is confirmed. The most common initial DMARD used as treatment of choice in RA is methotrexate. Aspirin should not be added because of the increased risk of gastrointestinal side effects as well as having no effect on altering RA disease progression. Rituximab is a biological DMARD and is indicated to be added in patients with RA refractive to treatment with combination therapy of methotrexate and a tumor necrosis factor inhibitor (TNF). Etanercept is a TNF inhibitor. This class of medication is often added in patients with RA who are not responding to methotrexate therapy alone. Leflunomide is a pyrimidine synthesis inhibitor that is approved for the treatment of RA; however, it is contraindicated for use in premenopausal women secondary to its carcinogenic and teratogenic potential. (Hellmann and Imboden, 2008, pp. 722-725) Hellmann DB , Imboden JB. Arthritis and musculoskeletal disorders. In: Tierney LM , McPhee SJ , Papadakis MA, eds. 2008 Current Medical Diagnosis and Treatment. New York, NY: McGraw-Hill; 2008:703-756.

Q 119.4: Which medication is the treatment of choice for symptomatic patients with hypertrophic cardiomyopathy? A Calcium channel blockers B Nitrates C Thiazide diuretics D Alpha antagonists E Beta-blockers

The Correct Answer is: E The use of beta-blockers in symptomatic hypertrophic cardiomyopathy is useful for gaining rate control. This will allow for the optimal amount of filling in order to maintain enough of an ejection fraction. (Bashore et al., Current Medical Diagnosis and Treatment, Chapter 10)

Q 66.10: A 49-year-old man presents to the office complaining of general malaise with muscle aches, anorexia, fever, and severe pain over his anterior neck radiating to his ears. He states that he was ill about 2 weeks ago with a sore throat, but it resolved within a few days. On palpation, the thyroid gland is enlarged and tender. His laboratory workup shows a high T4 level and increased erythrocyte sedimentation rate (ESR). What is the most appropriate therapy for this patient's disease? A levothyroxine sodium B PTU therapy C radioiodine ablation D surgery E supportive therapy only

The Correct Answer is: E This is subacute, painful thyroiditis. This is a self-limiting disorder that at most requires symptomatic therapy. In mild cases, analgesics (ASA) are sufficient for pain relief and to decrease the inflammation. Prednisone may bring more relief if needed. Transient hypothyroidism should be treated as well. (Ladenson and Kim, 2008, p. 1708) Ladenson P , Kim M. Thyroid. In: Goldman L , Ausiello D, eds. Cecil Textbook of Medicine. 23rd ed. Philadelphia, PA: WB Saunders; 2008.

Q 110.14: A 68-year-old male presents to the clinic stating that his wife has noticed a resting tremor in his right hand, and that his gait is more shuffling over the past six months. Which of these findings on physical exam is most likely for the suspected diagnosis? A Aphasia B Atonia C Chorea D Hyperreflexia E Masked facies

The Correct Answer is: E This patient exhibits classic findings of Parkinson's disease. Parkinson's is a nervous system disorder due to decreased dopamine, resulting from a degeneration of the dopaminergic nigrostriatal system. Symptoms may include a combination of tremor, rigidity, bradykinesia, progressive postural instability, slowing of automatic movements, gait changes, decreased facial expression, speech changes, and cognition deficits. Muscle strength and reflexes are typically preserved. Chorea, an irregular, rapid, and involuntary movement, is typically seen with Huntington's and a variety of other disorders. Parkinson's must be differentiated from other nervous system disorders. (McPhee SJ, Papadakis MA. Current Medical Diagnosis & Treatment 2011, Chapter 24, Nervous System Disorders)

Q 117.3: A 52-year-old male with chronic alcoholism is brought to the emergency department by his family, due to his acting differently for several days. A physical exam reveals nystagmus, eye muscle weakness, global confusion, retrograde amnesia, and ataxia. Which of the following is the most likely etiology of the diagnosis? A Cerebrovascular accident B Hypoxemia C Uremia D Vitamin B12 deficiency E Vitamin B1 (thiamine) deficiency

The Correct Answer is: E This patient exhibits classic symptoms associated with Wernicke encephalopathy. Wernicke encephalopathy is due to vitamin B1 (thiamine) deficiency. In the United States, this condition occurs most frequently in chronic alcoholics, but it may be seen in any condition affecting thiamine levels. Each of the other etiologies listed may also cause neurologic findings, and should be considered in the differential diagnosis and evaluation of this patient. (Simon RP, Greenberg DA, Aminoff MJ. Clinical Neurology, 7e, Chapter 1. Disorders of Cognitive Function)

Q 95.2: A patient presents with clinical features of thrombotic thrombocytopenic purpura. They also have an associated diarrhea positive hemolytic uremic syndrome. Which is the best treatment option for this patient? A Aspirin and dipyridamole B Glucocorticoids C Plasma exchange D Splenectomy and prostacyclin E Supportive therapy and dialysis

The Correct Answer is: E This patient has a diarrhea-associated hemolytic uremic syndrome with clinical features of TTP. D+HUS is associated with infectious etiologies, usually through food borne illnesses. Treatment consists of supportive therapy, treating the underlying infection, and dialysis if renal function warrants. All other therapies listed are used in the treatment of TTP or diarrhea negative HUS. (Lichtman et al., Williams Hematology 8e, Chapter 133, Antibody-Mediated Thrombotic Disorders: Thrombotic Thrombocytopenic Purpura and Heparin-Induced Thrombocytopenia)

Q 82.4: A 50-year-old male with an elevated WBC of 143,000, hematocrit 47% (45 to 52%), platelet count of 247,000 (150,000 to 4000,000), and BCR/ABL found in peripheral blood. He complains of fatigue, night sweats, and abdominal fullness. The best initial course of therapy would include which of the following? A Bone marrow transplant B Cytarabine C Dasatinib D Hydroxyurea E Imatinib mesylate

The Correct Answer is: E This patient has classic signs of chronic myelogenous leukemia. The first line of therapy for patients in the chronic phase of this disease is Imatinib mesylate, starting at 400 mg/day. For patients that demonstrate to Imatinib alone, Dasatinib may be an adjunct therapy. Hydroxyurea and Cytarabine were used prior to formulation of Imatinib, and may play a role for resistant CML. Bone marrow transplant would be reserved for patients that are resistant to current therapy or are in relapse. (Lichtman et al., Williams Hematology 8e, Chapter 90, Chronic Myelogenous Leukemia and Related Disorders)

Q 80.12: A patient being treated for Burkitt's lymphoma is found to have hyperkalemia, hyperphosphatemia, hypocalcemia, a metabolic acidosis, hyperuricemia, and appears to be going into renal failure after the first dose of chemotherapy. To prevent or minimize this finding, the best therapy is to start which of the following? A Allopurinol and rasburicase B Colchicine and monitored hydration of at least 3 L/day C Continuous venovenous hemofiltration and colchicine D Continuous venovenous hemofiltration and hydration E Monitored hydration of at least 3 L/day and allopurinol

The Correct Answer is: E This patient has developed a tumor lysis syndrome. The usual prophylactic therapy is carefully monitored hydration of at least 3 L of saline per day, and either allopurinol or rasburicase to decrease serum uric acid concentration and hyperuricosuria. Allopurinol and rasburicase alone are not enough to prevent TLS. Colchicine is a gout medication and not appropriate in this setting. Continuous venovenous hemofiltration is useful in allowing concomitant full-dose chemotherapy and preventing tumor lysis and renal failure, but is not the preventative treatment prior to starting chemotherapy. (Lichtman et al., Williams Hematology 8e, Chapter 104, Burkitt Lymphoma)

Q 109.13: You are evaluating a generally healthy 75-year-old male patient with a history of hypertension and persistent elevated blood pressure, despite atenolol 25 mg QD. He also complains of some urinary frequency, urgency, and nocturia over the past year and is bothered by these symptoms. His blood pressure today is 146/90 P = 56. His urinalysis is negative. His prostatic-specific antigen (PSA) from his last visit 4 months ago was 1. His rectal examination reveals a smooth, enlarged prostate without masses. What medication, in addition to atenolol, would be the best choice to treat this patient at this time? A carvedilol B lisinopril C amlodipine D hydrochlorothiazide E terazosin

The Correct Answer is: E This patient has uncontrolled hypertension as well as symptomatic benign prostatic hypertrophy (BPH). Although the addition of lisinopril and amlodipine are great options for blood pressure control they will not improve his symptoms of BPH. The addition of carvedilol, a beta blocker, to atenolol (also a beta blocker) is potentially dangerous, especially considering the bradycardic pulse. The addition of HCTZ is an option for further control of his blood pressure, but has the potential of increasing urinary frequency. Due to effects on blood pressure and BPH, the best choice for this patient would be a trial of terazosin, an alpha blocker. (McPhee and Papadakis, 2011, Chapter 11)

Q 112.6: A 23-year-old male presents with uncontrolled mucocutaneous bleeding following a wisdom tooth extraction. Traditional methods of controlling the bleeding have not significantly reduced the bleeding. On history, he reports epistaxis that takes longer than usual to stop bleeding. He also states that he seems to bruise more frequently than his friends. What is the most likely diagnosis? A Acute myelogenous leukemia B Aplastic anemia C Idiopathic thrombocytopenic purpura D Thalassemia E Von Willebrand's disease

The Correct Answer is: E This patient has von Willebrand's disease. Common clinical symptoms include mucocutaneous bleeding, epistaxis, bruising, and hematomas (and menorrhagia in women). Idiopathic thrombocytopenic purpura is the only other diagnosis that is a coagulation disorder, and typical presentation includes purpura and petechiae to the lower extremities. (Lichtman et al., Williams Hematology 8e, Chapter 127, von Willebrand Disease)

Q 67.4: A 66-year-old male with a history of hypertension, diabetes mellitus, and hypercholesterolemia presents by emergency medical services (EMS) to the emergency department complaining of severe chest pain with radiation into his back. The patient states that he was feeling well in the morning, but while performing some light activity he felt a "ripping" sensation in his back, which he initially thought was a pulled muscle. The pain continued and the patient started to have chest pain, shortness of breath, and lightheadedness. On initial examination the patient is still in pain, pale, diaphoretic, and has a blood pressure of 85/40. His chest is clear to auscultation, and he has a 3/6 diastolic murmur best appreciated at the base of the heart. Given this clinical scenario, what is the best test to definitively diagnose this medical problem? A Chest x-ray B Transthoracic echocardiography C Transesophageal echocardiography D Cardiac catheterization E Computed tomography

The Correct Answer is: E This patient is exhibiting a history and physical examination that is consistent with a thoracic aneurysm. The patient's history of hypertension, along with the "ripping" sensation in his back and hypotension give a clinical presentation that is suggestive of a thoracic aneurysm dissection. Given this clinical situation, the best test to evaluate for a potential dissection is by computed tomography (E). This test is sensitive enough to determine if there is a luminal irregularity. While echocardiography (B and C) may be able to show evidence of an aneurysm, it is not specific enough to show all the areas of an aneurysm. In this situation a chest x-ray (A) would not give enough specificity to appropriately diagnose an aneurysm, as well as a cardiac catheterization (D). Source: http://www.accessmedicine.com/content.aspx?aID=3651494

Q 82.5: A 65-year-old woman presents to the office with decreased hearing, and pain over her sternum, pelvis, and her right tibial tubercle. On x-ray, the involved bones are noted to be expanded and denser than normal. Her serum calcium and phosphorus levels are normal, but serum alkaline phosphatase level is markedly elevated. Which of the following would be the appropriate initial treatment for this patient? A ibuprofen 600 mg po every 6 hours B indomethacin 25 mg po tid C meclizine 25 mg po tid D methotrexate 7.5 mg po qd E tiludronate 400 mg po qd

The Correct Answer is: E This patient's signs and symptoms are consistent with Paget disease of bone. Biphosphates have become the treatment of choice for this disease. Tiludronate, taken orally for 3 months, is very effective in treatment of this disease. (Fitzgerald, 2009, pp. 1018-1020) Fitzgerald PA. Endocrine diseases. In: McPhee SJ , Papadakis MA, eds. Current Medical Diagnosis and Treatment. 48th ed. New York, NY: McGraw-Hill; 2009.

Q 119.8: Which of the following is the greatest risk associated with a patient suffering a transient ischemic attack (TIA)? A a substantial decreased risk of returning to baseline mentation. B a substantial increased risk of subsequent Alzheimer's disease. C a substantial increased risk of pulmonary embolism. D a substantial increased risk of a subarachnoid hemorrhage. E a substantial increased risk of a subsequent stroke.

The Correct Answer is: E Transient ischemic attacks (TIAs) are a very strong predictor of subsequent and escalating neurovascular events. A TIA that resolves fully is something that can be taken advantage of—this is an indicator that the clinician needs to treat the patient aggressively to prevent a subsequent event. (McPhee and Papadakis, 2011, Chapter 24)

Q 70.6: An adult male, not previously vaccinated for rabies, presents to the emergency department after being bitten by an aggressive stray dog. The dog was captured, and declared "probably rabid" by a local veterinarian. Which of the following treatment options should you select for this patient? A administer human rabies immune globulin only B administer equine rabies antiserum only C administer human rabies immune globulin and equine rabies antiserum D administer human rabies immune globulin and equine rabies antiserum and human diploid cell rabies vaccine E administer human rabies immune globulin and human diploid cell rabies vaccine

The Correct Answer is: E Transmission of rabies to this patient must be seriously considered, and postexposure immunization should begin immediately by the administration of human rabies immune globulin (HRIG; 20 IU/kg). About half the HRIG should be infiltrated around the bite wound, and the remainder injected intramuscularly. Human diploid cell rabies vaccine (HDCV) should also be given (1 mL IM in the deltoid), and again on days 3, 7, 14, and 28. HDCV should be delivered in a different syringe and administered at a different site than HRIG. (Shandera and Corrales-Medina, 2009, p. 1225) Shandera WX , Corrales-Medina VF. Viral & rickettsial infections. In: McPhee SJ , Papadakis MA, eds. Current Medical Diagnosis and Treatment. 48th ed. New York, NY: McGraw-Hill; 2009.

Q 113.3: A central component of hemostasis, it also functions as a carrier factor and an adhesive link between platelets and injured blood vessel walls. Dysfunction of this factor is the most common inherited bleeding disorder in humans. Which factor has these characteristics? A Factor VII B Factor VIII C G6PD deficiency D Tumor necrosis factor E Von Willebrand's factor

The Correct Answer is: E Von Willebrand's factor is a central component in hemostasis, and is a carrier for factor VIII as well as an adhesive molecule. It is the most common bleeding disorder in the general population. G6PD deficiency causes anemia. Tissue necrosis factor plays a role in the regulation of immune cells. Factor VII and VIII deficiencies cause coagulation disorders, but are not the most common. (Lichtman et al., Williams Hematology 8e, Chapter 127, von Willebrand Disease)

Q 66.8: A 66-year-old male patient complains of pain and swelling in his left foot intermittently over the past year. He denies any current symptoms. On examination you note the following findings. His left foot is unremarkable. Laboratory finding include an elevated uric acid. What is the most likely diagnosis? A rheumatoid arthritis B gout C osteoarthritis D septic arthritis E systemic lupus

The correct answer is (B). The patient has gouty tophi. His uric acid level is elevated, which further supports a diagnosis of gout. It is suspected that the intermittent left-foot swelling is related to acute flares of gout, which usually affects the first MCP joint. Choices (A), (C), (D), and (E) are not associated with elevated uric acid level or tophi. The patient may have osteoarthritis, but this does not explain the high uric acid level and the tophi on exam.

Q 89.1: A 24-year-old female HIV-positive patient, who is not currently on medication, presents to the emergency department with acute dyspnea, tachycardia, fever, nonproductive cough, and a room air oxygen saturation of 92%. She admits feeling poorly for the past five days. A physical exam reveals bilateral basilar crackles. An x-ray reveals the image shown. What is the most likely causative organism of this clinical picture? A Chlamydia psittaci B Histoplasmosis C Klebsiella pneumoniae D Pneumocystis jiroveci E Streptoccocus pneumoniae

The correct answer is (D). Pneumocystis is an opportunistic fungal infection of immunocompromised patients, most frequently seen in patients with untreated HIV/AIDS. Patients often present with fever, dyspnea, a nonproductive cough, decreased arterial oxygen pressure, and tachycardia. Lung auscultation may reveal adventitious sounds, but may also be without abnormality. Chest x-ray results classically include bilateral diffuse infiltrates with perihilar involvement. Although the other etiologies are associated with pneumonia, and may be seen in immunocompromised patients, pneumocystis pneumonia occurs in up to 80% of untreated HIV patients and is a leading cause of death.

Q 67.1: Which of the following lab test is considered the "gold standard" for detecting/diagnosing an inherited thrombophilia? A anticardiolipen deficiency B factor V Leiden deficiency C hyperhomocysteinemia D protein C and S deficiency E There is no single lab test.

The correct answer is (E). There is no single "gold standard" lab test for diagnosis of acquired/hereditary thrombophilia.

Q 72.8: A 40-year-old patient presents to your walk-in clinic with symptoms of hyperhydrosis, oily skin, daytime sleepiness, and snoring. Upon exam, you note large fleshy heel pads and hands with sweaty palms. The patient also has coarse facial features. When asked, the patient isn't aware of any major changes in her face or body. She has not seen another health care provider in many years and has not kept up with any health care maintenance schedule. The patient lives with her ill mother and is not currently employed. Which of the following screening tests would best aid in the diagnosis of this patient? A CT of the chest and abdomen B Thyroid scan C 24-hour urine for catecholamines D Serum calcitonin level E Serum IGF-I level

he correct choice is E, serum IGF-I level. Age and gender matched levels of IGF-I are elevated in patients with acromegaly. IGF-I is the mediator of most of the effects of GH on the body, and lead to the proliferation of bone, cartilage, and soft tissue. Although GH levels may be elevated in patients with acromegaly, they are secreted in a pulsatile fashion and are not consistently elevated. Serum GH levels are not the best screening test for acromegaly. Choices A, a CT of the chest and abdomen, and B, a thyroid scan, are expensive imaging studies that are not usually used as screening tests. They also have no role in the routine workup of patients with suspected acromegaly. Choice C, 24-hour urine for catecholamines, is a test that can be used in the work up of patients with suspected pheochromocytoma. Choice D, serum calcitonin levels, are associated with medullary thyroid cancer and other thyroid disease. (Melmed et al., 2008, Chapter 333) (Greenspan et al., 2007, Chapter 5)

Q 97.4: A 50-year-old woman with a history of hypertension complains of chest tightness and dyspnea while walking up one flight of stairs. She recently experienced an episode of near-syncope. She denies a history or rheumatic fever. On auscultation, a crescendo-decrescendo systolic ejection murmur is heard at the upper right sternal border radiating to the carotids bilaterally. Given the patient's physical exam findings, which of the following is the most likely diagnosis? A Severe aortic stenosis secondary to congenital bicuspid aortic valve B Aortic regurgitation/insufficiency C Mitral stenosis D Aortic regurgitation/insufficiency E Tricuspid regurgitation/insufficiency

A Choice A is correct, as the murmur of aortic stenosis is usually described as a crescendo-decrescendo or systolic ejection murmur heard best at the right upper sternal border. In addition, the murmur of aortic stenosis is frequently transmitted to the carotid arteries. In a patient of this age, with symptoms suggestive of severe aortic stenosis and with these physical exam findings, a congenital bicuspid aortic valve is the best choice, especially as the patient denies a history or rheumatic fever, which is also a cause of developing aortic valve stenosis in individuals under the age of 65. Patients with a congenital bicuspid aortic valve typically develop symptoms once the valve leaflets have become calcified and thickened, secondary to the undue stress over many years on a structurally abnormal aortic valve. Choice B is incorrect, as the murmur of aortic regurgitation is usually described as a high-frequency decrescendo early diastolic murmur heard best at the left upper sternal border or at the right upper sternal border. Choice C is incorrect, as the murmur of mitral stenosis is described as a low-frequency rumbling diastolic murmur that is decrescendo in early diastole, but may become crescendo up to the first heart sound with moderately severe mitral stenosis and sinus rhythm. Choice E is incorrect, as the murmur of tricuspid regurgitation is described as a holosystolic descrescendo murmur. (McPhee et al., 2011, Chapter 10)

Q 76.8: A 58-year-old female patient has a history of cirrhosis. She undergoes a diagnostic endoscopy, which shows large varices. Which of the following medications is the most appropriate to reduce the risk of a first variceal hemorrhage? A Beta blocker B ACE inhibitor C Calcium channel blocker D Angiotensin receptor blocker E Alpha blocker

A The correct answer is a beta blocker. This is the recommended medication to reduce the risk of first variceal hemorrhage in patients with large or small varices, who either have variceal red wale marks or advanced cirrhosis. (McPhee SJ, Papadakis MA. Current Medical Diagnosis & Treatment, 2010, p. 541)

Q 82.9: A 55-year-old patient presents with symptoms of polyuria and is worried that he may have diabetes mellitus. His past medical history includes hypertension and lung cancer. His family history is negative for diabetes mellitus. He works as an accountant and has no history of head trauma. He has an average diet and fluid intake. His fasting plasma glucose is 110 mg/dL and his hemoglobin A1c is 5%. What is the most likely cause of his polyuria? A Cancer related diabetes insipidus B Type 2 diabetes mellitus C Nephrogenic diabetes insipidus D Familial hypothalamic diabetes insipidus E Primary polydipsia

A The correct choice is A, cancer related diabetes insipidus. Solid tumors, such as cranipharyngiomas and metastatic lung and breast cancer, are known to cause diabetes insipidus. This patient has a history of lung cancer. Choice B, diabetes mellitus, is not present in this patient. His pasting glucose and hemoglobin A1c are within the reference range. Choices C, nephrogenic diabetes insipidus, and choice D, familial hypothalamic diabetes indipidus, are causes of polyuria. Both are noted in young patients. Choice E, primary polydipsia, is associated with excessive drinking of liquids, which leads to excessive urination. There is no history of this occurring in this patient. (Robinson et al., 2007, Chapter 6)

Q 89.3: Which of the following types of thyroid cancer involve the parafollicular cells (or C cells) of the thyroid? A Medullary B Papillary C Follicular D Lymphoma E Anaplastic

A The correct choice is A, medullary thyroid carcinoma. Unlike papillary (choice B) and follicular (choice C) thyroid cancers, which arise from thyroid epithelial cells, medullary thyroid cancer arises from the parafollicular cells of the thyroid. Anaplastic thyroid cancer, choice E, arises from undifferentiated cells. Thyroid lymphoma, choice D, is most commonly of B cell follicle center cell origin. (Lee et al., 2008, Chapter 41)

Q 77.8: A 32-year-old female with history of non-insulin-dependent diabetes mellitus (NIDDM) returns to the clinic for a routine visit. She reports that she her blood sugars have been controlled. However, her vitals today show a blood pressure (BP) of 136/78 P = 72. You note the past three office visits that her blood pressures were in the mid 130s to 140 systolic. She is not currently taking anything for her blood pressure but admits that she has not been exercising as much as usual. What do you advise the patient? A Advise lifestyle modification and continue to monitor BP next visit. B Initiate antihypertensive therapy to target BP < 130/80. C Initiate DASH diet and recheck blood pressure in 6 months. D Monitor blood pressure next visit, and if > 140/90, initiate antihypertensive therapy. E Her blood pressure indicates prehypertension, there is no need for treatment other than exercise at this time.

B The correct answer is (B). According to the JNC-7 guidelines, a patient with diabetes mellitus (DM) or chronic kidney disease (CKD) with this patient's blood pressures should be treated to achieve a goal BP of < 130/80. Although she would be classified as prehypertensive, due to her compelling indication of DM initiation of antihypertensive treatment should begin because she is at higher risk for cardiovascular disease. Lifestyle modifications such as increased exercise, low salt diet, and weight loss are recommended, but in addition to drug therapy at this time due to her increased risk of cardiovascular disease. (McPhee and Papadakis, 2011, Chapter 11)

Q 109.4: Your patient is a 70-year-old female who has a history of non-insulin-dependent diabetes mellitus (NIDDM), hypertension, chronic kidney disease (CKD), and gastroesophageal reflux disease (GERD). You receive a call that her potassium is elevated at 5.8 mEq/L. Last week you treated her for potassium of 6 mEq/L. Her current medications are listed in the following choices. Which of the following medications should you consider discontinuing due to persistent hyperkalemia? A acetaminophen B lisinopril C amlodipine D glyburide E omeprazole

B The correct answer is (B). A potential complication of CKD, usually in the later stages, is hyperkalemia. Hyperkalemia can also be induced by ACE inhibitors such as lisinopril or ARBs and may need to be discontinued. Another antihypertensive that does not cause hyperkalemia may be substituted. The remaining choices are unlikely to cause hyperkalemia. (McPhee and Papadakis, 2011, Chapter 22)

Q 99.3: It is a busy day in the internal medicine office. A 49-year-old noncompliant male with history of non-insulin-dependent diabetes mellitus (NIDDM) and hypertension returns for his 3-month follow-up, now 1 year later. His blood pressure is 156/92, P = 88, BS 250 mg/dl fasting, urine +microalbumin, +glucose, -ketones. He is not taking any medications at this time. You request that he resume his metformin from his last visit, but want to give him something else based on your current findings. What single medication would be best choice for this patient to treat both his blood pressure and microalbuminuria? A atenolol B lisinopril C amlodipine D hydrochlorothiazide (HCTZ) E terazosin

B The correct answer is (B). ACE inhibitors, such as lisinopril, or ARBs are both ideal choices for treating patients with diabetes that have hypertension and microalbuminuria. Although beta blockers (atenolol), calcium channel blockers (amlodipine), and diuretics (HCTZ) are recommended for use in diabetes to control blood pressure, they are not the first choice for patients with chronic kidney disease and will not reduce microalbuminuria. Terazosin (alpha blocker) would not be recommended first-line for a diabetic with hypertension and has no effect on microalbuminuria. (McPhee and Papadakis, 2011, Chapter 11)

Q 94.1: A 63 -year-old male returns to the office for follow-up complaining of increasing abdominal girth and weight gain of 10 pounds in the past month, despite his usual caloric intake. His vital signs are normal. He continues to drink at least a six-pack of beer daily despite your previous recommendations. On examination you note ascites, palmer erythema, and gynecomastia. The patient finds his increasing abdominal girth to be uncomfortable and is requesting treatment. He admits that his diet is low in salt. What treatment is indicated at this time? A increase in salt intake B spironolactone C hydrochlorothiazide (HCTZ) D large volume paracentesis E transjugular intrahepatic portosystemic shunt (TIPS)

B The correct answer is (B). Spironolactone is the treatment of choice initially and may be combined with furosemide. Salt intake restriction, not increase, is indicated initially. HCTZ is not indicated and will likely be ineffective. Paracentesis and TIPS are treatment options for refractory ascites and due to increased risks are not initial treatments. (McPhee and Papadakis, 2011, Chapter 16)

Q 104.3: A 78-year-old Caucasian female has a 3-year history of stiffness and achiness of bilateral shoulders and hips. She has been tested for rheumatoid arthritis in the past and has been found negative. Multiple radiographs of her hips and shoulders are unremarkable. She admits that she was placed on prednisone for an allergic reaction and noted a temporary resolution of her symptoms. For the past two weeks she complains of increasing symptoms now involving her neck and pain in her jaw with chewing. Today she noticed that her scalp is sore when she brushed her hair on the right side. What test is the gold standard for diagnosis of this patient's current symptoms? A ESR 60 mm/h B temporal artery biopsy C Elevated CRP D ultrasound of temporal artery E polymyalgia rheumatica (PMR) of temporal artery

B The correct answer is (B). This patient has long standing symptoms of polymyalgia rheumatica (PMR) with current symptoms suggestive of giant cell (temporal) arteritis. Temporal artery biopsy is considered the gold standard for diagnosis of giant cell (temporal) arteritis. Patients with temporal arteritis may have an elevated erythrocyte sedimentation rate (ESR) or CRP, but this is not required for diagnosis. A color ultrasound of the temporal artery will sometimes show edema or stenosis of the affected artery but is not very sensitive for giant cell arteritis. MRA is used for diagnosis of larger arteries with vasculitis and not routinely used in the diagnosis of temporal arteritis. (Imboden et al., 2007, Chapter 31)

Q 66.1: Your patient is asked to see you in followup for his fasting labs. His total cholesterol = 230 mg/dl, triglycerides = 1200 mg/dl, unable to calculate LDL. Your patient should be advised that due to these findings he is at risk for which of the following? A diabetes mellitus B pancreatitis C gout D diabetes insipidus E hypertension

B The correct answer is (B). Very elevated triglycerides, especially over 1000 mg/dl, increase the patient's risk of acute pancreatitis. This patient requires prompt treatment. Hypertriglyceridemia may be a component of metabolic syndrome but does not directly cause diabetes mellitus or hypertension. Hypertriglyceridemia does not increase the risk of gout and diabetes insipidus.

Q 108.10: A 64-year-old female presents to the office with a complaint of retrosternal chest pain, painful swallowing, and difficulty swallowing. The symptoms occur about three hours after she takes a daily medication. Which of the following medications is most likely to be the cause of her symptoms? A Lisinopril (Zestril) B Alendronate (Fosamax) C Levothyroxine (Synthroid) D Drospirenone, ethinyl estradiol (Yaz) E Rosuvastatin (Crestor)

B The correct answer is alendronate (fosamax), which is known to cause esophagitis. The other choices are very unlikely to cause pill-induced esophagitis. Lisinopril is an ACE inhibitor that is used to treat hypertension. Levothyroxine is a syntethic T4 and is used to treat hypothyroidism. Drospirenone, ehtinyl estradiol is an oral contraceptive. Rosuvastatin is a statin and is used to treat hypercholesterolemia. (McPhee SJ, Papadakis MA. Current Medical Diagnosis & Treatment, 2010, p. 536)

Q 61.10: You are evaluating a 59-year-old male with history of heavy alcohol use for the past 5 years who appears acutely ill. Which of the following findings is most likely to be an indicator of increased mortality in this patient with severe alcoholic hepatitis? A fatty liver B INR of 7 C AST > ALT by a factor of 2:1, but less than 300 units/L D serum bilirubin 1.8mg/dl E folic acid deficiency

B The correct answer is choice (B). A significantly elevated PT/INR as well as bilirubin > 10 mg/dl are indicators of severe alcoholic hepatitis and increased mortality requiring hospitalization. Choices (A), (C), and (E) are all common findings in alcoholic hepatitis. Mild elevation of serum bilirubin, choice (D), is also common.

Q 118.12: A 40-year-old female presents to the clinic with symptoms of a 10 pound unintentional weight loss, diarrhea, and palpitations. She has a positive family history for thyroid disease and diabetes mellitus. Her screening TSH level is 0.15mIU/L, and her fasting plasma glucose is 105mg/dL. What is the most common cause of this patient's symptoms? A Pituitary tumor B Graves' disease C Thyroid carcinoma D Hashimoto's thyroiditis E Euthyroid sick syndrome

B The correct choice is B, Graves' disease. Graves' disease is an autoimmune disorder, resulting in an increase in synthesis and release of thyroid hormone. It is the most common cause of hyperthyroidism. Choice A, pituitary tumor, and choice C, thyroid carcinoma, are rare causes of hyperthyroidism. Choice D, Hashimoto's thyroiditis, is a common cause of hypothyroidism. Choice E, euthyroid sick syndrome, is a condition of normal thyroid function that occurs after major surgery or an acute illness. This patient is not acutely ill and does have symptoms of true hyperthyroidism. (Fitzgerald et al., 2011, Chapter 26)

Q 83.5: Which of the following is the most common cause of short stature worldwide? A Acromegaly B Malnutrition C Prader-Willi syndrome D Congenital growth hormone deficiency E IGF-I receptor deficiency

B The correct choice is B, malnutrition. When not associated with chronic diseases, this is the most common cause of short stature worldwide. Children with malnutrition commonly present with failure of weight gain before growth rate decreases. A dietary history is key to the diagnosis, as well as a history of any parasites in the local area. Choice A, acromegaly, is a disorder of growth hormone excess. Choice C, Prader-Willi syndrome, choice D, congenital growth hormone deficiency, and choice E, IGF-I receptor deficiency, have been found to cause short stature, but are not seen as commonly as malnutrition. (Styne et al., 2007, Chapter 7)

Q 77.1: An 84-year-old female is taking prednisone for the past year, secondary to a diagnosis of biopsy-confirmed giant cell arteritis. Her dose was tapered to 20 mg/day, but her symptoms returned several months ago and her dose was increased. In addition she is taking hydrochlorothiazide (HCTZ) for hypertension, but denies any other medications or medical problems. What treatment should you recommend to this patient at this time? A glucophage B simvastatin C calcium and vitamin D D fosamax E naprosyn

C The correct answer is (C). This patient requires chronic prednisone treatment. Common complications of chronic glucocorticoid treatment include osteoporosis and diabetes. At this point with this patient there is no information stating that she has diabetes or osteoporosis. Before initiation of treatment with glucophage, the patient should have a diagnosis of diabetes mellitus. Before initiation of fosamax the patient should have a bone density scan to look for osteopenia/osteoporosis. In the meantime, calcium supplementation with vitamin D is warranted for prevention of osteoporosis. Simvastatin is not indicated, and naprosyn may increase risk of gastrointestinal bleeding with chronic prednisone. (Imboden et al., 2007, Chapter 31)

Q 116.18: You are evaluating a 55-year-old female with a history of hypertension. Her blood pressures have been in the 120s to 130s systolic and 60s to 70s diastolic until recently. She notices some days her blood pressure is normal and other days her systolic readings are in the 150s to 160s and wonders if a medication she could be taking might be contributing to the elevation. Her blood pressure today is 142/66. Which of the following medications is most likely to result in her blood pressure being elevated? A loratadine B simvastatin C pseudoephedrine D acetaminophen E lisinopril

C The correct answer is (B). Decongestants, such as pseudoephedrine, are known to increase blood pressure. Discontinuing pseudoephedrine and rechecking the blood pressure off of this medication may provide further information on the need for additional antihypertensive drug therapy. Loratadine, simvastatin, and acetaminophen are not known to cause secondary hypertension. Lisinopril is an ACE inhibitor used to treat blood pressure. (McPhee and Papadakis, 2011, Chapter 11)

Q 102.5: Your patient has a history of primary hyperparathyroidism. Recently she has been hospitalized due to obstructing kidney stones. She has had several fractures including her hip, sacrum, and forearm in the past year, all on separate occasions. She is constantly complaining of a lack of energy. What is the recommended treatment for her symptomatic hyperparathyroidism? A surgical removal of the pituitary B high-dose calcium supplementation C parathyroidectomy D thyroidectomy E thiazide diuretics

C The correct answer is (C). Primary hyperparathyroidism is most commonly secondary to a single parathyroid adenoma. The recommended treatment for symptomatic primary hyperparathyroidism is parathyroidectomy. High-dose calcium supplementation and thiazide diuretics, choices (B) and (E), can worsen the hypercalcemia associated with hyperparathyroidism. Neither surgical removal of the pituitary, choice (A), nor or a thyroidectomy, choice (D), is an indicated treatment for this condition. (McPhee and Papadakis, 2011, Chapter 26)

Q 115.2: A patient presents to for a pre-op physical secondary to a right rotator cuff injury. He is a 77-year-old male with a history of chronic obstructive pulmonary disease (COPD), non-insulin-dependent diabetes mellitus (NIDDM), hypertension, and hyperlipidemia. His medications include albuterol MDI, glipizide, candesartan, and simvastatin. A routine EKG is taken. What laboratory abnormality would you expect to find in this patient based on the portion of the EKG revealed below? Source: (Knoop et al., 2010, Chapter 23) A hypernatremia B hypercalcemia C hyperkalemia D hypocalcemia E hypokalemia

C The correct answer is (C). The classic EKG findings in the figure, displaying peaked T waves and widening of the ST segment, are consistent with hyperkalemia. The patient is taking candesartan, an ARB that is a frequent cause of hyperkalemia alone or when combined with other medications that may cause hyperkalemia, such as spironolactone. Flattened T waves, U waves, and ST depression is characteristic of hypokalemia. Prolongation of the QT interval is characteristic of hypocalcemia while shortening of the QT interval is characteristic of hypercalcemia. The patient is not on any medications that would expect to prolong or shorten the QT interval or cause hypocalcemia or hypercalcemia. Changes associated with hyponatremia are not found on an EKG. (Fauci, et al., 2008, Chapter 221)

Q 92.6: A 64-year-old caucasian male presents to the office with a complaint of worsening difficulty swallowing over the last two months. He also has a long history of gastroesophageal reflux disease. He is now becoming concerned because he has lost 15 pounds in the last two weeks. What is the most likely diagnosis? A Leiomyosarcoma B Esophageal stricture C Adenocarcinoma D Achalasia E Squamous cell carcinoma

C The correct answer is adenocarcinoma. Adenocarcinoma is more common in caucasians, and the majority develop as a complication of Barret metaplase, due to chronic gastroesophageal reflux. Leiomyosarcoma is a rare smooth muscle malignancy. Esophageal stricture would present with the gradual development of solid food dysphagia over many months to years. Achalasia is a motility disorder, in which there is loss of peristalsis in the distal two-thirds of the esophagus and impaired relaxation of the lower esophageal sphincter. Squamous cell carcinoma is more common in African Americans, and is associated with chronic alcohol and tobacco use. (McPhee SJ, Papadakis MA. Current Medical Diagnosis & Treatment, 2010, p. 1468)

Q 83.7: A 52-year-old male presents to the office with a complaint of intermittent dysphagia. His symptoms occur mostly when he eats steak, and have been ongoing for about six months. The symptoms have not worsened and he denies weight loss. You are concerned that he may have an esophageal web or ring. The diagnostic test that best visualizes an esophageal web or ring is which of the following? A Endoscopy B Esophageal manometry C Barium esophagram D Esophageal pH monitoring E CT scan of the abdomen

C The correct answer is barium esophagram, which provides the best visualization of an esopahgeal web or ring. Endoscopy and CT scan of the abdomen are less sensitive than barium esophagography. Esophageal manometry is used to measure and record esophageal pressures. pH monitoring is used to assess for reflux into the esophagus. (McPhee SJ, Papadakis MA. Current Medical Diagnosis & Treatment, 2010, p. 538)

Q 99.6: A patient describes an insidious onset of vague gastrointestinal symptoms including nausea, vomiting, and abdominal pain. She has been losing weight over the last few weeks as well. Past medical history reveals a new diagnosis of anemia. The patient is not currently taking any medications. Physical exam findings include hypotension and dark freckles with dark, bluish black pigmentation of the mucosal membranes. Serum sodium is decreased, while the potassium and plasma ACTH levels are elevated. What is the most likely diagnosis for this patient? A Acute abdomen B Cushing's syndrome C Primary adrenal insufficiency D Disseminated intravascular coagulation E Diabetic ketoacidosis

C The correct choice is C, primary adrenal insufficiency. This disorder presents with symptoms resulting from progressive destruction of the adrenal glands and resultant decrease in aldosterone and cortisol secretion. The presentation can range from subtle to fulminant adrenal crisis. Common clinical findings include hyperpigmentation, weakness, anorexia, nausea, vomiting, diarrhea, and hypotension. Some patients with adrenal insufficiency may initially be treated for an acute abdomen, choice A. In this patient, ACTH levels are known to be elevated, pointing the cause to the pituitary-adrenal axis. Choice B, Cushing's syndrome, presents with signs and symptoms related to an excess in plasma cortisol, such as truncal obesity, hyperglycemia, hypertension, skin changes, and weight gain. Choice D, disseminated intravascular coagulation, is a coagulation disorder presenting with bleeding and thrombosis. Choice E, diabetic ketoacidosis, is an acute complication seen in patients with type 1 diabetes mellitus. (Williams, Harrison's Online, Chapter 336)

Q 75.3: What is the definitive treatment for the majority of patients presenting with mild symptoms of hyperthyroidism secondary to subacute thyroiditis? A Subtotal thyroidectomy B Oral methimazole C Symptomatic treatment D Radioactive iodine E Antibiotics

C The correct choice is C, symptomatic treatment. Most patients with subacute thyroiditis and symptoms of hyperthyroidism require only symptomatic treatment, with non-steroidal anti-inflammatory medications and/or beta blockers, for any cardiac symptoms including palpitations and tachycardia. Occasionally, patients may require a course of prednisone for this acute inflammatory condition. Most patients will recover spontaneously within a few months. Choices A, B, D, and E are not necessary in this condition. Most cases of subacute thyroiditis are associated with viral infections, and resolve without additional thyroid medications. (Cooper et al., 2007, Chapter 8)

Q 114.9: A 37-year-old woman presents with symptoms of weight gain, weakness, and poor concentration. Her past medical history includes a vertebral fracture two years ago. Vital signs include the following: BP = 145/102, pulse = 80, and respirations = 16. A physical exam reveals truncal obesity, hirsuitism, and thin skin. Laboratory results include an elevated 11 p.m. salivary cortisol and lack of suppression with an overnight dexamethasone suppression test. Gadolinium enhanced MRI of the pituitary reveals a microadenoma. What is the treatment of choice for this patient? A Adrenal radiation therapy B Bilateral adrenalectomy C Transsphenoidal resection D Chemotherapy regimen E Glucocorticoid replacement

C The correct choice is C, transsphenoidal resection of the pituitary microadenoma. This is the treatment of choice for this patient. This patient has developed Cushing's syndrome secondary to an ACTH secreting pituitary adenoma (Cushing's disease). This more commonly occurs in women who are between 20 and 40 years old. It is the most common cause of ACTH dependent Cushing's syndrome. The remission rate for patients with pituitary microadenomas is approximately 80%. Since the pathology lies in the pituitary gland, choice A, adrenal radiation, and choice B, bilateral adrenalectomy, would not address the cause. Cushing's syndrome is not treated with chemotherapy, choice D. Choice E, glucocorticoid replacement, would not be advised as this patient already has an elevated level of plasma cortisol. (Melmed et al., 2008, Chapter 333)

Q 83.10: A patient presents with symptoms of polyuria for several months. There is no history of diabetes mellitus or intrinsic kidney disease. Which of the following tests can best determine whether the polyuria is resulting from primary polydipsia or some form of diabetes insipidus? A 24-hour urine volume test B Routine urinalysis C Water deprivation test D Urine culture and sensitivity E Vasopression suppression test

C The correct choice is C, water deprivation test. This test helps the practitioner determine if the patient can concentrate his/her urine with or without becoming hyperosmolar. The patient is restricted from liquids and food during the test, which must be done in a controlled environment. A patient with primary polydipsia will be able to concentrate his/her urine without becoming hyperosmolar. A patient with diabetes insipidus will become hyperosmolar without concentrating the urine. Choice A, 24-hour urine volume test, can provide quantification of the polyuria, but will not determine its cause. Choice B, routine urinalysis, may provide evidence of dilute urine, but will not provide evidence of its cause. Choice D, urine culture and sensitivity, is used when trying to determine the cause of a urinary tract infection. Choice E, vasopression suppression test, is not a test that is performed. It could be dangerous to try to suppress a hormone that is already thought to be deficient. (Robinson et al., 2007, Chapter 6)

Q 115.11: A patient with long-standing, untreated acromegaly is seen in your office with symptoms of severe headaches. After completing a thorough history and physical exam, you order a set of x-rays including a skull series. Which of the following findings would you expect in this patient? A Punched out lesions B Basilar skull fracture C Metastatic bone lesions D Enlarged sella tursica E Thinning of the skull

D *The correct choice is D, enlarged sella tursica. This finding is seen in 90% of patients with acromegaly. Other findings on skull radiographs include thickened calvarium (upper portion of the skull), enlarged mandible, and sinuses.* Bony growth is a hallmark of the disease. The pituitary adenoma, which typically causes the disease, can be found in the sella tursica. This disorder doesn't typically metastasize, and is not associated with metastatic bone cancer, as noted in choice C. Choice A, punched out lesions, are commonly associated with Paget's disease. There is no history of head trauma, as would be the case in choice B, basilar skull fracture. As noted earlier, the skull may be thickened. Therefore, choice E, thinning of the skull, would not fit this patient's presentation. (Aron et al., 2007, Chapter 7)

Q 105.2: Your patient returns to your office for a follow up for non-insulin-dependent diabetes mellitus (NIDDM). Her HgA1c in the office is 6.4%. She is concerned about developing kidney disease from her diabetes and requests that you test her for this. What initial screening test should you order that would provide clues to potential diabetic nephropathy allowing for treatment to slow the disease progression? A 24-hour urine for protein B serum BUN/CR C urine microscopic D urine microalbumin E serum protein

D The correct answer is (D). An easy office dipstick or laboratory test for urine microalbumin should be done initially and periodically on diabetic patients who are at risk for diabetic nephropathy. Treatment should be initiated if microalbuminuria is found to slow disease progression. A urine microscopic for renal casts may be helpful if the patient has symptoms of kidney disease, but is not an initial screening test. Serum BUN/CR and GFR are useful tests for patients with known diabetic nephropathy to indicate the stage of chronic renal failure but is not elevated early in the disease progression, before urine microalbumin. A 24-hour protein is not indicated in this case as an initial screening test. (McPhee and Papadakis, 2011, Chapter 22)

Q 118.13: A 55 -year-old male returns for a routine follow up. He has a history of hypothyroidism and is taking levothyroxine 100 mcg QD for the past 2 years. He has not had a TSH since his dose was changed 2 years ago. He states that he has felt very nervous and tired in the past 3 months. He has lost 6 pounds but has not changed his diet and states he is always hungry. His TSH is 0.5 mU/L. His FT 4 is elevated. What treatment would you recommend at this time? A increase levothyroxine to 125 mcg QD B change to methimazole C change to propylthiouracil D decrease levothyroxine to 50 mcg E Radioa

D The correct answer is (D). The patient has symptoms of hyperthyroidism but has a history of hypothyroidism. He is currently taking thyroid replacement. The most likely cause of his low TSH is too much thyroid replacement. A reasonable treatment is to decrease the dose of his levothyroxine. Increasing the dose would likely cause worsening symptoms of hyperthyroidism. Although the patient has symptoms of hyperthyroidism and a suppressed TSH treatment for hyperthyroidism, choices (B), (C), and (E) are not recommended and can be dangerous. (McPhee and Papadakis, 2011, Chapter 26)

Q 118.5: A 63-year-old female complains of a 5-day history of a persistent left-sided headache, which she has not experienced before. She also notes a tender swollen area around her left temple, which appeared around the same time. On examination you note tenderness and prominence of the left temporal artery. You order an ESR, which is 75 mm/h. What is your best course of action at this time? A Repeat the ESR in 72 hours. B Begin prednisone 20 mg/d and increase if symptoms persist. C Refer to a rheumatologist for appointment next month, with a trial of nonsteroidal anti-inflammatory drugs (NSAIDs). D Begin prednisone 60 mg/d immediately. E Refer for a temporal artery biopsy next week, with a trial of hydrocodone for analgesia.

D The correct answer is (D). The patient's history and physical examination findings point to giant cell arteritis (temporal arteritis) as the most likely cause, prompting immediate treatment with high-dose prednisone to prevent visual loss. The patient meets the criteria for clinical diagnosis of giant cell arteritis without a temporal artery biopsy, but it is recommended for definitive diagnosis due to the complications associated with long-term corticosteroid treatment. Treatment with prednisone should not be withheld while waiting for a temporal artery biopsy. NSAIDs and hydrocodone do not prevent the complications of temporal arteritis. (Imboden et al., 2007, Chapter 31)

Q 113.9: You are examining n 42-year-old male with the following physical examination findings (see picture below). What symptoms would most likely correspond to his diagnosis? A anxiety B palpitations C excessive snoring D polyuria and polydipsia E weight gain

D The correct answer is (D). The patient's physical examination findings suggest acanthosis nigricans, a condition associated with diabetes mellitus. This patient would most likely present with polydipsia and polyuria at diagnosis. He may have weight loss and fatigue. Anxiety and palpitaions are not as likely to present symptoms of diabetes. (McPhee and Papadakis, 2011, Chapter 27)

Q 79.7: A 39-year-old male complains of intermittent episodes of hematuria over the past 6 months, which resolved spontaneously without treatment. He also has noted some dull bilateral flank discomfort recently that he attributed to a strain after moving. He denies any dysuria or penile discharge. He further denies any history of kidney stones. His mother died in her 60s of kidney disease that required dialysis. On examination his BP is 148/92, P = 68, T = 98.2˚F. His examination is unremarkable except for bilateral palpable enlarged kidneys. Urine dip is positive for +1protein. What diagnostic test would be most helpful to confirm your suspected diagnosis? A complete blood count (CBC) with diff B acute abdominal series C 24-hour urine protein D renal ultrasound E kidney-ureter-bladder (KUB)

D The correct answer is (D). This patient's history and physical examination findings are suggestive of polycystic kidney disease. His family history also suggests the potential for polycystic kidney disease, which is a common hereditary disease that may lead to end-stage renal disease (ESRD). A renal ultrasound is the preferred test to confirm the diagnosis revealing multiple renal cysts. Radiographs and labs, choices (A) and (C), can help in your differential diagnosis but are not diagnostic of polycystic kidney disease. (Fauci et al., 2008, Chapter 278)

Q 108.4: A 52-year-old male undergoes an upper endoscopy for evaluation of GERD, and is diagnosed with Barrett's esophagus. There is no evidence of dysplasia at this time. In addition to placing him on a PPI, the most appropriate management includes which of the following? A Balloon dilation of the lower esophageal sphincter B Barium swallow every two years C Nissen fundoplication D Endoscopy every three years E Esophagectomy

D The correct answer is endoscopy every three years. Balloon dilation of the lower esophageal sphincter is performed for achalasia. A barium swallow will show motility disorders, but cannot be used to detect dysplasia. Nissen fundoplication is a surgical treatment used in gastroesophageal reflux disease. Esophagectomy is used for patients with high grade dysplasia or adenocarcinoma of the esophagus. (McPhee SJ, Papadakis MA. Current Medical Diagnosis & Treatment, 2010, p. 532)

Q 112.2: Which of the following is the most common cause of primary adrenal insufficiency in the United States? A Tuberculosis B Adrenal hemorrhage C Lymphoma D Autoimmune destruction E Metastatic carcinoma

D The correct choice is D, autoimmune destruction. This is responsible for 80% of cases of primary adrenal insufficiency in the United States. All of the other choices can cause adrenal insufficiency, but they are less common. Tuberculosis, choice A, is a common cause of adrenal insufficiency in other areas of the world, where the infection is more common. Bilateral adrenal hemorrhage, choice B, can occur as a complication of sepsis, heparin use, anti-phospholipid syndrome, and after major trauma or surgery. Lymphoma, choice C, and metastatic carcinoma, choice E, are rare causes of adrenal insufficiency. (Fitzgerald et al., 2011, Chapter 26)

Q 83.2: To which of the following areas does follicular thyroid cancer most commonly first spread? A Intraglandular metastasis B Local spread into regional vocal cords C Distant lymph nodes D Bone and lung via bloodstream E Local extension into the muscle and trachea

D The correct choice is D, bone and lung via bloodstream. Follicular thyroid cancer can spread to regional lymph nodes and distant sites via the blood stream. Choice A, intraglandular metastasis, is seen more commonly in papillary thyroid carcinoma, and choice B, local spread into regional vocal cords, occurs in anaplastic thyroid carcinoma. Choice C, distant lymph nodes, and choice E, local extension into the muscle and trachea, are more commonly seen in patients with medullary thyroid cancer. (Bauer et al., 2010, Chapter 20) (Lee et al., 2008, Chapter 41)

Q 116.12: What is the most sensitive test available for the screening and detection of early thyroid dysfunction? A Radioactive iodine uptake B Serum T3 resin uptake C Serum total T4 level D Serum TSH level E Thyroid scan

D The correct choice is D, serum TSH level. Very small changes in serum TSH level can provide clues that there are changes in the functioning of the hypothalamic-anterior pituitary-thyroid axis. The test is easier and less expensive than any thyroid imaging tests. All of the other choices can be used in the work up of patients for thyroid dysfunction, but they are less sensitive, and many are more expensive and more invasive. (Bauer et al., 2010, Chapter 20)

Q 121.14: A 62-year-old African-American male is seen for his yearly physical exam. He has no complaints. He denies any current medications or medical problems, but the occupational medicine nurse has taken his blood pressure several times in the past year and told him it was high. He denies any tobacco or alcohol use. His blood pressure is 156/92 today. What is the most likely cause of his elevated blood pressure? A sleep apnea B primary aldosteronism C pheochromocytoma D renal artery stenosis E essential hypertension

E The correct answer is (E). The most common cause of hypertension (HTN) is essential, also known as primary hypertension, making up about 95% of patients with hypertension. Secondary causes of hypertension are less common and include sleep apnea, primary aldosteronism, pheochromocytoma, and renal artery stenosis. There is no reason to suspect these other causes in this patient, who is otherwise healthy with stage 1 hypertension based on the stated history. Sutters Michael, "Chapter 11. Systemic Hypertension" (Chapter). McPhee SJ, Papadakis MA: CURRENT Medical Diagnosis & Treatment 2011: http://www.accessmedicine.com/content.aspx?aID=3177080.

Q 110.3: You are called for a consult of a 30-year-old female who appears well, but is noted to have hepatomegaly, spider nevi, and elevated serum amino transaminases greater than 1000 units/L. Because of your suspected diagnosis, you decide to give her a trial of corticosteroids resulting in improvement of her serum amino transaminases. What is the most likely diagnosis? A hepatitis A B hepatitis B C hepatitis C D hepatocellular carcinoma E autoimmune hepatitis

E The correct answer is (E). The symptoms indicate chronic hepatitis. Autoimmune hepatitis is generally the only hepatitis of the choices that responds to corticosteroids. Patients with autoimmune hepatitis may develop cirrhosis and may be at higher risk for hepatocellular carcinoma, but the history is more consistent with autoimmune hepatitis. A liver biopsy is indicated. The patient likely has a positive antinuclear antibody (ANA). (McPhee and Papadakis, 2011, Chapter 16)

Q 80.7: An 84-year-old female complains of intermittent dyspnea and dizziness over the past 4 months. She is found to have an irregular rhythm on exam. Her EKG findings are below. Based on the EKG findings, which single laboratory test may lead to finding a secondary cause? Source: (Tintinalli, et al., 2010, Chapter 22) A B12 level B folic acid C lipid profile D erythrocyte sedimentation rate (ESR) E thyroid-stimulating hormone (TSH)

E The correct answer is (E). This patient has atrial fibrillation, which can occur in patients with hyperthyroidism, Therefore, it is important to check a TSH in patients who present with atrial fibrillation, especially in the elderly. Choices (A), (B), (C), and (D) are not secondary causes of atrial fibrillation. (McPhee and Papadakis, 2011, Chapter 26)

Q 104.7: Your patient is a 47-year-old female who complains of leg cramps and fatigue over the past few weeks. Her examination is completely normal. She is taking an unknown medication for hypertension, which she did not bring with her. Labs include a normal complete blood count (CBC) and a BMP that reveals a potassium level of 3.2 m&thinsp;Eq/L, otherwise normal. Which of the following is the most likely cause of her laboratory abnormalities? A quinapril B labetalol C verapamil D valsartan E hydrochlorothiazide (HCTZ)

E The correct answer is (E). This patient's symptoms are likely due to hypokalemia, which is a potential side effect of thiazide diuretics such as hydrochlorothiazide. Hyponatremia may also be another possible side effect. Choice (A), an ACE inhibitor, and choice (D), an ARB, can both cause hyperkalemia. Choice (B), a beta blocker, and choice (C), a calcium channel blocker, do not cause hypokalemia but may be associated with bradycardia. (McPhee and Papadakis, 2011, Chapter 11)

Q 86.1: What is the radionuclide imaging pattern noted during a thyroid scan in patients with subacute thyroiditis? A Single area of increased uptake B Diffusely high uptake C Multiple areas of increased uptake D Single area of low uptake E Diffusely low uptake

E The correct choice is E, diffusely low uptake. Acute inflammation is occurring during subacute thyroiditis, causing leakage of stored thyroid hormone into the circulation. The thyroid is not metabolically active, and therefore there is low radioactive iodine uptake on scan. This is in distinction to the diffuse increased uptake seen on scan in Graves' disease, choice B. Choice A, a single area of increased uptake, is commonly seen with a "hot" or toxic nodule. Choice C, multiple areas of increased uptake, are found in patients with multinodular goiters. This patient had no nodules noted on exam. Choice D, single area of low uptake, is seen as cold nodules and may be related to a benign nodule or thyroid cancer. (Cooper et al., 2007, Chapter 8)

Q 77.4: During a hospitalization for acute exacerbation of COPD, troponin levels are drawn on a 62-year-old man with a history of hypertension, hyperlipidemia, and chronic tobacco use, and found to be elevated above the 99 th percentile of normal. Which of the following choices would qualify this patient for the most recent ACC/AHA consensus guideline's definition of myocardial infarction? A Ischemic symptoms B New right bundle branch-block on EKG C J wave on EKG D Pulmonary vascular congestion on CXR E Elevated WBC count

The Correct Answer is: A Choice A is the most appropriate choice, as troponin elevation may occur in the setting of patients who do not suffer from acute coronary syndrome. Therefore, the 2007 consensus guidelines recommended that the definition of myocardial infarction be applied to those patients who not only had troponin elevation above the 99 th percentile, but also met one of the following criteria: "ischemic symptoms, new left bundle branch block (not right bundle branch-block as in choice B), new ST and T-wave changes, new Q waves, or imaging evidence of a new loss of viable myocardium or new regional wall-motion abnormality." Choice C, J wave, is characteristic of patients with hypothermia. Choice D, pulmonary vascular congestion, is frequently noted on CXR of patients with congestive heart failure. Choice E, an elevated WBC count, is indicative of an infectious process. (Tintinalli et al., 2011, Chapter 52)

Q 113.10: A patient is being evaluated for fatigue, weight loss, a two-week cough, and erythema nodosum. Today, the patient noted eye symptoms and your exam reveals iritis. A chest xray reveals hilar and paratracheal lymphadenopathy. You suspect sarcoidosis. Which of the following is the most appropriate next step of evaluation to determine the diagnosis? A Bronchoscopy with tissue biopsies B CT chest with contrast C MRI chest D PET Scan E Serum ACE level

The Correct Answer is: A A diagnosis of sarcoidosis must incorporate clinical findings and radiologic findings, ruling out other conditions and obtaining definitive information. A biopsy should be performed, and may be performed utilizing tissue from any affected organ, including skin or from a transbronchial biopsy, which has a high conclusive yield. Histologically, sarcoidosis is associated with noncaseating granulomas; however, other granulomatous disease must be ruled out. The additional testing listed may also be utilized for further evaluation, but is considered adjunctive (see Figure 322-8 for more information). (Fauci et al., Harrison's Principles of Internal Medicine, 17e, Chapter 322, Sarcoidosis) Proposed approach to management of patient with possible sarcoidosis. Presence of one or more of these features supports the diagnosis of sarcoidosis: uveitis, optic neuritis, hypercalcemia, hypercalciuria, seventh cranial nerve paralysis, diabetes insipidus.

Q 108.1: A known patient who carries the diagnosis of hemophilia A wants to know what medications they can use to relieve the symptoms of the flu. Which of the following is the most appropriate to recommend? A Acetaminophen B Acetylsalicylic Acid C Diclofenac D Ibuprofen E Naproxen

The Correct Answer is: A Acetaminophen is the only product on the list that won't exacerbate or interfere with platelet aggregation. All other products are to be avoided, as they interfere with platelet aggregation and exacerbate bleeding. (Lichtman et al., Williams Hematology 8e, Chapter 124, Hemophilia A and Hemophilia B)

Q 120.16: A 43-year-old female patient presents with back pain and hematuria. The patient reports having this problem earlier this year and recalls her previous clinician telling her, "they're just cysts." Denying any history of urinary tract infections, the patient reports her mother was on dialysis before passing away. The patient is afebrile and her physical examination is positive for diffuse back tenderness and bilateral flank masses with palpation. Urine dipstick is positive for 3+ blood and is negative for leukocytes and nitrites. What is this patient's most likely diagnosis? A adult polycystic kidney disease B renal cyst C horseshoe kidney D renal cell carcinoma

The Correct Answer is: A Adult polycystic kidney disease is a hereditary condition that almost always has a bilateral presentation (95% of the cases). It does not appear until after the age of 40, and dialysis or kidney transplantation is necessary for survival. Renal cysts and renal cell carcinoma generally present unilaterally. A horseshoe kidney (fusion of the renal tissue) may be palpated bilaterally; otherwise, the patient is asymptomatic. (McAninch, 2008, pp. 507-512) McAninch JW. Disorders of the kidneys. In: Tanagho EA , McAninch JW, eds. Smith's General Urology. 17th ed. New York, NY: McGraw-Hill; 2008:506-520. McAninch JW. Disorders of the penis and male urethra. In: Tanagho EA , McAninch JW, eds. Smith's General Urology. 17th ed. New York, NY: McGraw-Hill; 2008:625-637.

Q 102.10: A 54-year-old male with a known history of hypercholesterolemia presents to your office complaining of vision changes in his right eye. He states that this morning, while working on his computer, he noticed darkening of his vision, which he described as a "shade being pulled down over my eye." This was followed by approximately three minutes of vision loss in the right eye, which resolved. He denies any other symptoms and does not have a history of eye problems. Which of the following is the most likely diagnosis? A Amaurosis fugax B Intracranial tumor C Retinal detachment D Retinal vascular spasm E Uveitis

The Correct Answer is: A Amaurosis fugax, often characterized as transient blindness or vision impairment lasting 1 to 5 minutes, can result when blood flow to the retina is disrupted. Emboli, often due to atherosclerosis in carotid arteries or cardiac sources, impact the retinal arteries, causing the visual changes. As the body resolves the emboli and restores blood flow to the retina, symptoms improve. Retinal detachment is also associated with vision changes and may be described as having a curtain effect. This typically starts in the superior temporal area of vision and progresses, with central vision remaining intact, unless the macula is involved. Retinal vascular spasm and vasculitis are more rare causes of transient vision loss. Intracranial tumor, depending on location, may impact vision, with transient symptoms less likely. Acute uveitis typically presents with unilateral pain, redness, vision changes, and photophobia. (McPhee SJ, Papadakis MA. Current Medical Diagnosis & Treatment 2011, Chapter 7, Disorders of the Eyes & Lids)

Q 86.8: A 38-year-old woman presents with a history of frequent headaches that begin behind her right eye and are associated with a visual aura, photophobia, and phonophobia. She feels her headaches are worse due to job-related stress and insomnia. She is having six to eight instances of headache a month. Her medical history is remarkable for exercise-induced asthma. Which of the following agents is the best prophylactic agent for this patient? A amitriptyline (Elavil) B celecoxib (Celebrex) C propranolol (Inderal) D sumatriptan (Imitrex) E butalbital/caffeine (Midrin)

The Correct Answer is: A Amitriptyline would be an appropriate prophylactic agent that may also treat her insomnia. Celecoxib and sumatriptan are common abortive agents that also have a limited role as prophylactic agents. They are inappropriate choices for this patient due to the frequency of her headaches and ill-defined pattern. Propranolol is contraindicated because of her history of exercise-induced bronchospasm. Butalbital/caffeine is not indicated for use as a prophylactic agent. (Wells et al., 2009, p. 609) Wells BG , DiPiro JT , Schwinghammer TL , et al. Pharmacotherapy Handbook. 7th ed. New York, NY: McGraw-Hill; 2009.

Q 101.7: Two days following an uneventful 4-vessel CABG, a 57-year-old man develops a sudden onset of lightheadedness and palpitations. His vital signs are stable, and physical examination demonstrates no abnormalities. Given the results of his EKG, as shown (Figure 2), which of the following is the most appropriate next step in management? A Direct-current cardioversion B Nitroglycerin patch C Digoxin 0.125 mg PO daily D Neurology consult E Meclizine 25 mg PO Q6H

The Correct Answer is: A Among the choices offered here, choice A is the most appropriate next step in management of a patient with new onset atrial flutter, as determined by EKG; it most effectively converts most patients to normal sinus rhythm. Choice B is inappropriate, as the patient is not demonstrating angina pectoris, and the EKG does not demonstrate evidence of ischemia or infarction. Choice C is inappropriate, as it is the least effective agent for slowing the ventricular response when compared to beta blockade or calcium channel blockers, all of which act by blocking the AV node (digixon may occasionally convert atrial flutter to atrial fibrillation). Choice D is inappropriate, as the patient's symptoms of lightheadedness do not stem from neurologic changes. Choice E is inappropriate, as the patient's symptoms do not stem from vertigo. (McPhee et al., 2011, Chapter 10)

Q 119.7: A patient describes a desire for close relationships and to be more successful at work. However, she views herself as being undesirable and inferior. Because of these feelings she avoids social activities and extra occupational projects out of fear of criticism, rejection, and embarrassment. Which diagnosis would best fit this description? A avoidant personality disorder B borderline personality disorder C histrionic personality disorder D schizoid personality disorder

The Correct Answer is: A An individual with avoidant personality disorder differs from schizoid in that they desire interaction and closeness but are unable to overcome their deep seated self-beliefs and fears. They tend to be less impulsive and more stable than borderline personality disorder patients and have less of a need to be the center of attention than those with histrionic personality disorders. (Sadock and Sadock, 2008, p. 385) Sadock BJ , Sadock VA. Concise Textbook of Clinical Psychiatry, 3rd ed. Philadelphia, PA: Lippincott, Williams & Wilkins; 2008.

Q 117.8: A 45-year-old male presents with a complaint of itching and burning of his upper eyelid. This has been present for a week. Some dried discharge is seen clinging to his eyelashes. Which of the following is the most likely diagnosis? A Blepharitis B Chalazion C Dacryocyctitis D Hordeolum E Conjunctivitis

The Correct Answer is: A Anterior Blepharitis is common and can be bilateral. Causes include staphylococcus and seborrhea. Posterior blepharitis is caused by meibomian gland dysfunctions. (Riordan et al., 2008, Chapter 4)

Q 55.6: A 26-year-old woman has decreased appetite, weight gain, cold intolerance, hoarse voice, constipation, and arthralgias. What is the most likely etiology of her condition? A autoimmune thyroiditis B congenital hypothyroidism C dietary iodine deficiency D surgical resection of the thyroid gland

The Correct Answer is: A Auto immune thyroiditis is the most common cause of hypothyroidism in the United States. Dietary iodine deficiency is the most common cause in underdeveloped regions of the world.

Q 108.8: A 26-year-old female is returning for a follow-up visit for gastroesophageal reflux disease. She has been seen in the office frequently over the past three months for symptoms associated with reflux, but she has also exhibited signs and symptoms that are consistent with a personality disorder. In particular, she is withdrawn, shy, introverted, and avoids close relationships. These signs are most consistent with which personality disorder? A Avoidant B Dependent C Histrionic D Schizoid E Schizotypal

The Correct Answer is: A Avoidant personality disorder presents clinically as someone who fears rejection, overreacts to rejection and failure, and has poor social endeavors and low self-esteem. Signs of a dependant personality disorder include someone who lacks confidence and self-esteem, has difficulty making decisions, and is passive and overaccepting. Clinical findings of histrionic personality disorder include being dependent, immature, seductive, egocentric, vain, and emotionally labile. Schizotypal disorder is characterized by being superstitious, socially isolated, and suspicious, and having limited interpersonal ability, odd speech, and eccentric behaviors. (McPhee SJ, Papadakis MA. Current Medical Diagnosis & Treatment, 2010, p. 951)

Q 71.3: A 47-year-old female patient is diagnosed with a duodenal ulcer. She was determined to have H. pylori infection. Which of the following medications used to treat peptic ulcer disease is specifically used to eradicate H. pylori? A Omeprazole B Amoxicillin C Ranitidine D Lansoprazole E Misoprostol

The Correct Answer is: B Amoxicillin is an antibiotic that is specifically used to eradicate the H. pylori bacteria. Omeprazole and lansoprazole are proton pump inhibitors, and are used in peptic ulcer disease to suppress acid production in the stomach. Ranitidine is an H2 receptor antagonist, and its role is also to reduce acid production in the stomach. Misoprostol is a prostaglandin analog that stimulates gastric and duodenal mucus and bicarbonate secretion. (McPhee SJ, Papadakis MA. Current Medical Diagnosis & Treatment, 2010, p. 550)

Q 113.2: Which of the following is the third component of the atopic triad, besides allergic rhinitis and asthma? A Psoriasis B Dermatitis C Arthritis D Urticaria E Pruritus

The Correct Answer is: B Atopic dermatits, or eczema, is the third chronic finding, along with asthma and allergic rhinitis, in patients who are atopic. Urticaria are common in acute and chronic allergies. (Lalwani A.K., 2008, Chapter 13)

Q 62.11: A 29-year-old female complains of a two-month history of easy bruising. She describes that the bruising is located primarily on her shins, but has noted them on other areas as well. She also describes red freckles on her lower extremities. On exam, you note non-blanching and non-palpable purpura to both legs and petechiae. She denies recent illnesses and states that she has essentially been feeling fine. The PE is normal other than the skin findings. A platelet count of 118,000 is noted. What is the most likely diagnosis? A Antiphospholipid syndrome B Idiopathic thrombocytopenic purpura C Systemic lupus eruthematosus D Thrombotic thrombocytopenic purpura E Von Willebrand's disease

The Correct Answer is: B Diagnosis is based on clinical manifestations including history, physical exam, blood count, and blood film. Typically, these patients have felt fine until they notice development of purpura and petechiae. No additional physical findings are typically found. Blood counts note a thrombocytopenia, with other tests usually in the normal range.

Q 109.16: A 56-year-old chronic alcoholic presents with signs of anemia. Laboratories reveal a moderate anemia with a hematocrit of 30% (45 to 62%) and a dimorphic population of red cells, one normal and one hypochromic. The MCV (mean corpuscular volume) is normal, the serum iron level is elevated, and the transferrin saturation is high. The diagnosis was made using which diagnostic study? A 24-hour urine for porphyrin B Bone marrow biopsy C Echocardiogram D Ferritin level E Liver biopsy

The Correct Answer is: B Diagnosis of sideroblastic anemia is made by examination of the bone marrow, using Prussian blue staining and noting the presence of ringed sideroblasts, which are cells with iron deposits encircling the red cell nucleus. None of the other studies are useful in making this diagnosis. (McPhee SJ, Papadakis MA, Tierney LM. Current Medical Diagnosis and Treatment, 2010, Chapter 13, Blood Disorders)

Q 97.6: A 62-year-old man presents to the emergency department with aphasia and right lower extremity weakness that started about 4 hours ago. He now has progressing right upper extremity weakness, worsening right lower extremity weakness, and decreased sensation throughout his right side. This cerebral ischemia is best characterized as A transient ischemic attack B stroke in evolution C completed stroke D subarachnoid hemorrhage E global cerebral ischemia

The Correct Answer is: B During a stroke in evolution, symptoms will worsen or new symptoms will appear. A completed stroke is one in which neurologic symptoms have stabilized, whereas a transient ischemic attack produces deficits that resolve over time. This patient's symptoms do not match those of an acute subarachnoid hemorrhage. Global cerebral ischemia as seen in sudden cardiac arrest would involve loss of consciousness. (Aminoff et al., 2005, pp. 286-297) Aminoff MJ , Greenberg DA , Simon RP. Clinical Neurology. 6th ed. New York, NY: McGraw-Hill; 2005.

The Correct Answer is: C This genetic mutation is found in patients with MEN1 syndrome, formerly known as Wermer Syndrome. About two-thirds of patients with this syndrome present with hyperparathyroidism. Gastrinomas (A) occur in about one-third and pituitary adenomas (E) in about half. Hirschsprung disease (B) and medullary thyroid carcinomas (D) are found in MEN2a syndrome. Fitzgerald PA, Endocrine Disorders, in Current Medical Diagnosis and Treatment, 52 nd ed. 2013.

The Correct Answer is: B Entamoeba histolytica has two stages in its life cycle. In the active stage in the human intestine, it causes symptoms of dysentery, abdominal pain, stool mucus, and tenesmus. In the dormant stage, the cystic form is excreted in the stool and in developing nations frequently contaminates the supply of drinking water. When the amoeba is in the dormant stage, the cystic form can be excreted in the stool and, in the case of food handlers with poor personal hygiene, be transmitted to others. In addition, because of the cystic stage, individuals engaging in anal intercourse can transmit the infection unknowingly. Diagnosis is made by microscopic evaluation of a stool wet prep and confirmed by serology. Treatment includes agents such as metronidazole or tinidazole. (Schuster and Glaser, 2007, pp. 2404-2405) Schuster FL , Glaser CA. Amebiasis. In: Goldman L , Ausiello D, eds. Cecil Medicine. 23rd ed. Philadelphia, PA: Saunders Elsevier; 2007.

Q 119.9: You suspect that a 44-year-old female with a history of fatigue, anxiety, and weight loss has hyperthyroidism. What physical examination findings would support hyperthyroidism as your diagnosis? A dry skin B exophthalmos C decreased tendon reflexes D thinning hair E bradycardia

The Correct Answer is: B Exophthalmos is a physical examination finding that is found in some patients with hyperthyroidism due to Grave's disease. Other potential physical examination findings may be a tremor, moist skin, and tachycardia. The other choices may be found in patients with hypothyroidism. (McPhee and Papadakis, 2011, Chapter 26)

Q 69.5: Which of the following treatments is first-line therapy for sputum culture-positive Legionnaire's pneumonia in an immunocompetent patient? A ampicillin/sulbactam B erythromycin C ceftriaxone D vancomycin E clindamycin

The Correct Answer is: B First-line therapy for legionella pneumonia (mild to moderate) in the immunocompetent host is erythromycin 500 mg to 1 g IV qid or 500 mg po qid for 14 to 21 days. Another option for first-line therapy is doxycycline 200 mg IV or po once daily for 14 to 21 days. Alternatives include levofloxacin 500 mg IV or po q day for 7 to 10 days or azithromycin 500 mg IV or po q day for 3 days. Severe infection or treatment in the immunocompromised patient is levofloxacin or azithromycin. (Edelstein, 2007, pp. 2264-2265) Edelstein PH. Legionella infection

Q 55.1: A 41-year-old male presents due to concerns regarding headaches. He states that for the past month and a half he has had daily headaches, described as aching, which he notes are worse first thing in the morning and do not seem to change with activity or position. He has noted that when he drives, his left hand seems weaker on the steering wheel, and he drops things more often. He denies any other neurologic symptoms or recent illnesses. A physical exam reveals weakness of the left hand, forearm, and upper arm, with mildly decreased reflexes. There are no other significant findings. Which of the following is the most likely diagnosis? A Amyotrophic lateral sclerosis B Intracerebral neoplasm C Lyme encephalitis D Simple partial seizure E Transient ischemic attack

The Correct Answer is: B Headaches and neurological changes may be seen with many conditions. However, *intracerebral neoplasms are often associated with persistent headaches and described as worse in the morning, and may involve neurologic disturbances of many forms, based on the location of the lesion. Coordination deficits, sensory deficits, ataxia, and limb involvement are often seen with brainstem lesions.* Amyotrophic lateral sclerosis, a progressive, degenerative nerve disorder with associated weakness, may have similar symptoms, but is not classically associated with headaches. With Lyme encephalitis, signs and symptoms such as fever, vomiting, meningeal signs, and photophobia would be expected. Simple partial seizures are not associated with headaches, although focal neurologic findings are possible. Transient ischemic attacks may have associated headache and neurologic symptoms, but should have improving symptoms and resolution within 24 hours, and are not likely to recur consistently.

Q 109.19: A 28-year-old woman presents with nervousness and palpitations associated with heat intolerance. On examination, there is no evidence of thyromegaly, but there is a palpable nodule that is "hot" on a thyroid scan. The TSH was low and T3 and T4 were both elevated. Which of the following is the recommended treatment for this patient? A Propylthiouracil (PTU) B Thyroid lobectomy C Total thyroidectomy D Radioiodine ablation

The Correct Answer is: B In Graves' disease, the thyroid is diffusely enlarged in contrast to a toxic adenoma in which the thyroid is normal sized but with a palpable nodule. Surgery is the treatment of choice for a toxic adenoma. Surgical treatment of a toxic adenoma is a thyroid lobectomy and isthmusectomy. A subtotal or total thyroidectomy is indicated for toxic multinodular goiters or Plummer disease. Thionamides and radioiodine ablation are not effective therapies for toxic adenomas. (Coe, 2006, pp. 404-406) Coe NPW. Surgical endocrinology: thyroid gland. In: Lawrence PF, ed. Essentials of General Surgery. 4th ed. Philadelphia, PA: Lippincott Williams & Wilkins; 2006.

Q 118.15: What is the most common ECG abnormality in patients with a pulmonary embolism (PE)? A Atrial fibrillation B Sinus tachycardia C Ventricular ectopy D Sinus bradycardia E High grade AV block

The Correct Answer is: B In most cases, sinus tachycardia is the only abnormality in patients with a PE. You may also find some ECGs that will have non-specific ST-T wave changes. Sinus bradycardia and AV blocks are not common findings that are associated with PE. (Chesnutt MS, Prendergast TJ. Current Medical Diagnosis and Treatment, 2011, Chapter 9, Pulmonary Disorders)

Q 100.2: A 26-year-old male has a history of schizophrenia that has been characterized by marked incoherence and a silly affect. Which classification of schizophrenia does this most closely resemble? A Catatonic B Disorganized C Paranoid D Residual E Undifferentiated

The Correct Answer is: B The signs and symptoms that this patient has exhibited is most consistent with disorganized schizophrenia. Catatonic schizophrenia is characterized by severe psychomotor disturbances of either rigidity with mutism or excitement. Paranoid schizophrenia is characterized by persecutory or grandiose delusions, often accompanied by hallucinations. Residual schizophrenia is characterized by an episode that warrants a diagnosis of schizophrenia, but there are no current psychotic symptoms present; however, they continue to exhibit milder signs, such as social withdrawal, flat affect, and eccentric behaviors. In undifferentiated schizophrenia, the symptoms are not specific enough to allow for categorization in any of the other subtypes. (McPhee SJ, Papadakis MA. Current Medical Diagnosis & Treatment, 2010, p. 952)

Q 117.15: What is the most common cause of treatment failure in tuberculosis? A Drug resistance B Noncompliance to therapy C Inappropriate selection of medication D Sepsis E Death

The Correct Answer is: B The usual reason for failure is simply due to the patient not continuning their treatment plan, regardless of the severity of the disease. Drug resistence, while present in some cases does not preclude the patient from treatment failure. Ongoing sepsis is not a reason to have treatment failure. (Chesnutt MS, Prendergast TJ. Current Medical Diagnosis and Treatment, 2011, Chapter 9, Pulmonary Disorders)

Q 76.9: A 30 year old female complains of a dome shaped slightly erythematous nodule on her right thigh. Upon clinical examination it exhibits the "dimple sign". The patient states the lesion is not changing and is asymptomatic. What is the most likely diagnosis? A Basal cell carcinoma B Dermatofibroma C Closed comedone D Hypertrophic scar

The Correct Answer is: B This is the classic presentation of a Dermatofibroma. The dimple sign is when a depression forms after the lesion is laterally compressed between the fingers (Wolff & Johnson, p 224-226). (Fig 9-54, Wolff K, Johnson RA: Fitzpatrick's Color Atlas & Synopsis of Clinical Dermatology, 6 th Ed: http://www/accessmedicine.com)

Q 63.2: A 25-year-old female presents for a skin exam. She has no family history of skin cancer, and has no moles that itch, bleed, or ulcerate. She is concerned about a mole on her arm that is surrounded by a hypopigmented area. She states that the mole appears to be decreasing in size. What would appropriate management of the lesion include? A excision with 2mm margin B reassurance C hydrocortisone 2.5 % ointment bid x 2 weeks D ketoconazole cream bid x 2 weeks

The Correct Answer is: B This lesion is consistent with a halo nevus. The depigmented macule that surrounds the nevus is similar to vitiligo and may consume the nevus; therefore, reassurance that this will resolve is appropriate for this patient. There is no need for excision with margins as this is not indicative of malignant changes. Hydrocortisone 2.5% ointment is a low potency topical steroid. This treatment is not indicated for a halo nevus. Ketoconazole is an antifungal used to treat fungal infections. There are no indications of fungal infection in this halo nevus.

Q 116.14: A 42-year-old woman has experienced recent weight gain, heavy periods, fatigue, cold intolerance, and constipation. She has a rough voice, and her rate of speech is slow. Physical exam is significant for an enlarged thyroid, slow reflexes, and the presence of brittle and coarse hair. She denies any history of bipolar disease or treatment with lithium. Laboratory tests show an elevated TSH and low free T 4 . What is the most appropriate treatment for this patient? A propylthiouracil (PTU) B levothyroxine C surgical resection D radioiodide ablation

The Correct Answer is: B This patient's signs and symptoms are consistent with hypothyroidism. Treatment of choice is levothyroxine, which is partially converted in the body to T 3 . Significant increases are seen within 1 to 2 weeks, with maximum levels reached in 3 to 4 weeks. (Friedman and Herman-Bonert, 2009, pp. 652-653) Friedman TC , Herman-Bonert VS. Thyroid gland. In: McPhee SJ , Papadakis MA, eds. Current Medical Diagnosis and Treatment. 48th ed. New York, NY: McGraw-Hill; 2009.

Q 60.8: A 41-year-old woman presents with complaints of weight gain, infrequent menses, and mood changes. You observe her to have moon facies, centripetal fat distribution, and purple striae on her abdomen (see Figure 4-3). Her blood pressure is 152/98 mm Hg. What is the first step in confirming this diagnosis? (Reproduced, with permission, from Fauci AS, Braunwald E, Kasper DL, et al. Harrison's Principles of Internal Medicine, 17th edition. New York: McGraw-Hill, 2008: 2255.) A random cortisol level B overnight dexamethasone suppression test C thyroid studies D MRI

The Correct Answer is: B This patient's signs and symptoms indicate possible Cushing syndrome. Overnight dexamethasone testing is the most widely used test, with normal results excluding Cushing syndrome. Cortisol levels are not useful because of diurnal variations.

Q 68.7: Which of the following pulmonary function test results demonstrates emphysema, a form of obstructive pulmonary dysfunction? A Decreased total lung capacity, decreased residual volume B Increased total lung capacity, increased residual volume C Decreased total lung capacity, decreased FEV1 D Increased total lung capacity, decreased residual volume E Increased total lung capacity, increased FEV1

The Correct Answer is: B Total lung capacity represents the vital capacity, defined as the amount of gas exhaled after a maximal inhalation, plus the residual volume within the lung after maximum exhalation. With emphysema, the lung parenchymal damage and decreased elasticity results in all flow rates being reduced, including FEV1, FVC, and FEV1/FVC levels. Expiratory time is increased and gas trapping occurs, thus increasing the total lung capacity. (McPhee SJ, Hammer GD. Pathophysiology of Disease, 6e, Chapter 9, Pulmonary Disease)

Q 67.9: What is the most appropriate management for the lesion shown, which is noticed on a 50-year-old female? A punch biopsy B imiquimod cream 3x week C excision with 1 cm margin D cryotherapy

The Correct Answer is: C *The lesion is asymmetric with irregular margins. The optimal treatment of this lesion would be excision with 1 cm margins. A punch biopsy would only be performed if excision cannot be performed.* Cryotherapy would destroy the lesion and prohibit a diagnosis and staging. (Wolff et al., 2009, Page 332)

Q 120.14: A 54-year-old man with a history of chronic alcohol abuse presents to the emergency department with complaints of a subjective fever and severe epigastric pain radiating to the back. The pain has been present for the past 8 hours and is associated with nausea and vomiting, which has not relieved the pain. Laboratory data reveal a WBC of 14,000/mm 3 and a serum amylase of 500 U/L (reference range 0-286 U/L). Plain films of the abdomen were unremarkable. Which of the following is the most likely diagnosis? A Perforated duodenal ulcer B Acute cholecystitis C Acute pancreatitis D Mesenteric ischemia E Choledocholithiasis

The Correct Answer is: C Acute pancreatitis typically presents with severe, steady midepigastric abdominal pain that radiates through to the back; pain is associated with fever, nausea, and vomiting. The most common causes of acute pancreatitis are gallstones and alcohol. Laboratory studies will show elevated WBC and serum amylase levels. Amylase elevations are nonspecific and can be elevated with perforated ulcers and mesenteric ischemia. A perforated ulcer will show evidence of free air on plain film; mesenteric ischemia will not present with fever or an elevated WBC unless there is the presence of infarcted bowel at which point the patient would appear septic. Acute cholecystitis may be associated with elevations in amylase but they are typically only a modest increase. (Sharp, 2006, pp. 355-357) Sharp KW , Goldin SB , Lomis KD. Pancreas. In: Lawrence PF, ed. Essentials of General Surgery. 4th ed. Philadelphia, PA: Lippincott Williams & Wilkins; 2006.

Q 81.10: A physician assistant student suffers a needlestick injury while caring for an HIV-positive patient whose viral load is currently undetectable. Of the following, which is the most appropriate management for the student? A no drug treatment unless HIV testing performed immediately and at 6 weeks, 3 months, and 6 months results become positive B administration of zidovudine and lamivudine until results of baseline testing are received C administration of zidovudine and lamivudine for 4 weeks D administration of zidovudine, lamivudine, and indinavir for 4 weeks

The Correct Answer is: C After a needle-stick injury, a health-care worker should have baseline testing with follow-up testing at 6 weeks, 3 months, and 6 months. Risk of seroconversion is approximately 1:300. Administration of antiviral therapy decreases this risk by 79%, so the worker should be offered treatment with zidovudine and lamivudine as soon as possible after the injury, for a total of 4 weeks. However, workers with a high-risk exposure (source patient with advanced disease, a viral load >50,000, or with resistant organisms) should have a protease inhibitor added to the prophylactic regimen. (Katz and Zolopa, 2009, p. 1192) Katz MH , Zolopa AR. HIV infection. In: McPhee SJ , Papadakis MA, eds. Current Medical Diagnosis and Treatment. 48th ed. New York, NY: McGraw-Hill; 2009.

Q 63.5: A 68-year-old female presents to the emergency department with signs and symptoms of an acute ischemic stroke. The initial CT scan is normal. Her blood pressure is 164/105. What is the most appropriate treatment for the blood pressure of this patient? A Atenolol PO B Clonidine PO C Close monitoring D Labetolol IV E Nicardipine IV

The Correct Answer is: C Aggressively lowering blood pressure may decrease blood flow to the ischemic tissue, thus decreasing the chances of recovery or increasing the risk of further infarction. In the setting of an acute ischemic stroke, blood pressure elevation should be monitored closely, with some elevation expected. This elevation is expected to decline without medication in the first few hours to days, but if elevation continues to a systolic blood pressure greater than 220mmHg, or mean arterial pressure greater than 120mmHg, medication is advised. Medications may include intravenous labetolol or nicardipine, with close monitoring of the patient. After the acute phase following a stroke, appropriate oral medications may be considered for outpatient hypertension management.

Q 119.15: You are asked to examine an 88-year-old female resident of a nursing home, who presents with a red eye. Her notes from the nursing home say that the patient has had this problem for six months, but now seems to be getting worse despite using daily artificial tears and occasional topical antibiotic drops. On physical exam you notice markedly injected conjunctiva to the right eye, with no discharge. The lower lid appears to be curled in toward the bulbar conjunctiva, with the eyelashes pointing inward. What is the name of this condition? A Conjunctivitis B Dacryoadenitis C Entropion D Ectropion E Endophthalmos

The Correct Answer is: C Aging causes a relaxation in the lower lid retractors, resulting in an entropion. This causes chronic irritation to the bulbar conjunctiva and corneal abrasions. Treatments include taping the lower lid to the cheek, botulinum toxin injection, or surgery. (Riordan et al., 2008, Chapter 3)

Q 66.4: Which of the following conditions is the most common cause of massive lower gastrointestinal bleeding? A Hemorrhoids B Colon cancer C Diverticular disease D Upper gastrointestinal hemorrhage E Meckel diverticulum

The Correct Answer is: C Although hemorrhoids are a common cause of lower gastrointestinal hemorrhage, they do not cause massive hemorrhage. Massive lower gastrointestinal hemorrhage is most commonly due to diverticular disease. Upper gastrointestinal hemorrhage may present as massive lower hemorrhage due to the cathartic effect of blood. Colon cancer usually presents with occult bleeding. Meckel diverticulum may also present significant lower gastrointestinal hemorrhage, but it affects only approximately 2% of the population and is therefore not as common.

Q 115.3: A patient presented with intermittent rectal bleeding associated with decreased caliber in the size of his stool. On examination, there were no palpable abdominal masses, but the fecal occult testing was positive. A barium enema was obtained with the results pictured below. What is the most likely diagnosis for this patient? (Reproduced, with permission, from Fauci AS, Braunwald E, Kasper DL, et al. Harrison's Principles of Internal Medicine, 17th ed. New York: McGraw-Hill, 2008:577.) A Diverticular disease B Crohn disease C Colorectal carcinoma D Intussusception

The Correct Answer is: C Barium enema finding of carcinoma of the sigmoid colon causing high-grade obstruction shows the classic "apple core" lesion. Crohn's disease is typically associated with the string sign, which is an area of stricture or stenosis that shows up as a narrow line of contrast, giving the appearance of a string associated with the stricture. Diverticular disease is associated with outpouchings from the colon that will be filled with barium. (Chang, 2006, p. 1111; Mellinger, 2006, p. 291; Dayton, 2006, p. 313) Chang AE , Morris AM. Colorectal cancer. In: Mulholland MW , Lillemoe KD , Doherty GM , Maier RV , Upchurch GR, eds. Greenfield's Surgery: Scientific Principles and Practice. 4th ed. Philadelphia, PA: Lippincott Williams & Wilkins; 2006. Mellinger JD , Macfadyen BV , Mercer DW , et al. Small intestine and appendix. In: Lawrence PF, ed. Essentials of General Surgery. 4th ed. Philadelphia, PA: Lippincott Williams & Wilkins; 2006. Dayton MT , Tradel JL. Colon, rectum, and anus. In: Lawrence PF, ed. Essentials of General Surgery. 4th ed. Philadelphia, PA: Lippincott Williams & Wilkins; 2006.

Q 120.20: You are evaluating a patient who is complaining of facial drooping , and inability to close his eye. During the cranial nerve exam you notice he is unable to wrinkle his forehead. Based on this information what is the most likely diagnosis? A Cerebrovascular accident B Transient ischemic attack C Bell's palsy D Horner's syndrome E Isolated oculomotor palsy

The Correct Answer is: C Bell's palsy affects cranial nerve VII, the facial paralysis conforms to the all branches of the peripheral nerve including the side of the face, eyelid and forehead muscles. An acute cerebrovascular accident would present only with a facial droop, the ability to close the eye and wrinkle the forehead would be preserved and there would likely be other focal weakness on physical exam. Horner's syndrome is miosis, ptosis and facial flushing and anhydrosis caused by abnormalities of the supercervical ganglion along the internal carotid artery. (Riordan et al., 2008, Chapters 65, 68)

Q 75.5: Which one of the following is a characteristic finding on computed tomography (CT) of the abdomen in a patient with acute diverticulitis? A toxic megacolon B air-fluid levels C soft tissue inflammation of the pericolic fat D thinning of the colon wall E paucity of bowel gas in the colon

The Correct Answer is: C CT findings consistent with diverticulitis include soft tissue thickening of the pericolic fat (98%), diverticula, and thickening of the bowel wall. In immunosuppressed patients, findings may include intraperitoneal and extraperitoneal gases without fluid or abscess formation. (Travis, 2009, p. 249) Travis AC , Blumberg RS. Diverticular disease of the colon. In: Greenberger NJ, eds. Current Diagnosis & Treatment: Gastroenterology, Hepatology, & Endoscopy. New York, NY: McGraw-Hill; 2009.

Q 73.7: A 37-year-old male presents to your office with a history of vision loss in his right eye. He denies any pain, and states that the vision loss occurred suddenly. He noted there was a wavy, "curtain-like" visual disturbance preceding the vision loss. Upon physical exam you notice a cherry red spot over the macula and retinal pallor. What is the most likely diagnosis? A Macular degeneration B Retinal detachment C Central retinal artery occlusion D Cerebrovascular accident E Central retinal vein occlusion

The Correct Answer is: C Central retinal artery occlusion is characterized by a sudden, painless vision loss. A cherry red spot is characteristic on the macula, along with pallor to the retina. (Tintinalli et al., 2011, Chapter 236)

Q 57.9: Which disorder is characterized by episodes of hypomania and depression for greater than 2 years? A dysthymia B major depressive disorder C cyclothymia D bipolar E mood disorder

The Correct Answer is: C Cyclothymia is characterized by symptoms of depression and hypomania for at least 2 years. Symptoms are milder than a regular depressive or manic episode. Occasionally, patients will have regular depressive or manic symptoms at which time they need to be reclassified as bipolar.

Q 79.4: A 55-year-old woman with a history of emphysema, who is undergoing chemotherapy for lung cancer, is sent to see you by her oncologist regarding a sudden increase in dyspnea, with exertion and fatigue. On physical exam, pulsus paradoxus and muffled heart sounds are noted. Which of the following diagnostic studies would be most effective in establishing a definitive diagnosis given this patient's physical exam findings? A Chest x-ray B CBC with differential C Transthoracic echocardiography D Pulmonary function tests E Tilt-table test

The Correct Answer is: C In patients with a history of malignancy, sudden worsening of dyspnea and physical exam findings of pulsus paradoxus and muffled heart sounds (two of the three components of Beck's triad), a clinical suspicion of cardiac tamponade should be part of a clinician's differential diagnosis. Choice C, transthoracic echocardiography, is most effective and will establish a definitive diagnosis of cardiac tamponade, as it allows direct visualization of the location and amount of pericardial fluid. A CXR may demonstrate a classic "water bottle" shape of the heart, but may also be normal. A CBC with differential would not give a definitive diagnosis of cardiac tamponade. Pulmonary function tests will likely be abnormal in the setting of a patient with emphysema and lung cancer, and thus would also not offer a definitive diagnosis if cardiac tamponade is suspected. Tilt-table testing is useful in patients with syncope, but not in this patient population. (Fauci et al., 2001, p. 1366)

Q 83.3: A 72-year-old male presents with bony pain, Bence Jones protein in his urine, an elevated creatinine level of 2.0, hypercalcemia, and lytic lesions to the long bones of his legs. The best initial treatment to correct this patient's renal insufficiency is which of the following? A Bisphosphonate and plasma phoresis B Dialysis and calcitonin C Hydration and calcitonin D Hydration and rapid single infusion of bisphosphonate E Plasmaphoresis and hydration

The Correct Answer is: C Management of renal impairment in patients with multiple myeloma is primarily supportive care. Hydration and use of calcitonin is the mainstay. If the hypercalcemia needs rapid correction, a slow infusion of bisphosphonate may be used. Dialysis would be useful if the patient has renal failure but has difficulty removing light chains from the blood, which are causing the problem. Plasmaphoresis doesn't play a role in this setting. (Lichtman et al., Williams Hematology 8e, Chapter 109, Myeloma)

Q 118.8: What is the most common pathogen that causes atypical pneumonia? A Chlamydia pneumoniae B Legionella pneumophila C Mycoplasma pneumoniae D Streptococcus pneumoniae E Hemophilus influenzae

The Correct Answer is: C Mycoplasma is the third most common pathogen of all pneumonias, and the most common atypical. Streptococcus pneumoniae is considered a typical bacterium, while Legionella and Chlamydia are not as common when compared to Mycoplasma. (Lee et al., Current Diagnosis and Treatment in Pulmonary Medicine, Section X, Pulmonary Lung Disease, Chapter 37, Viral and Atypical Pneumonia)

Q 121.3: An 18-year-old woman is transferred to your emergency department from a local college infirmary. She presented yesterday with a complaint of headache but became confused and is now febrile. You notice a petechial rash on physical examination and her cerebrospinal fluid comes back with increased WBCs, increased protein, and decreased glucose. What is the most likely organism responsible for her meningitis? A Haemophilus influenzae B cytomegalovirus C Neisseria meningitidis D Mycobacterium tuberculosis E coxsackievirus B

The Correct Answer is: C Neisseria meningitidis and Streptococcus pneumoniae are the most common etiologic agents for bacterial meningitis in this patient's age group. So much so that many colleges and universities require a vaccine for students who live in dormitories. Her fever and the cerebrospinal fluid values are consistent with a bacterial and not a viral infectious source for the meningeal irritation. (Aminoff et al., 2005, pp. 20-30) Aminoff MJ , Greenberg DA , Simon RP. Clinical Neurology. 6th ed. New York, NY: McGraw-Hill; 2005.

Q 82.8: Which of the following is one standard of care to treat a first-time deep vein thrombosis (DVT) without pulmonary embolism (PE)? A Begin the patient on warfarin 5 mg PO once daily and check an INR in five days and adjust to therapeutic levels continuing warfarin for six months. B Begin the patient on enoxaparin 1 mg/kg SQ QD while also starting warfarin 5 mg PO once daily and check INR in five days and adjust to therapeutic levels continuing warfarin for six months and enoxaparin until therapeutic on the warfarin. C Begin the patient on enoxaparin 2 mg/kg SQ QD while also starting warfarin 5 mg PO once daily and check INR in five days and adjust to therapeutic levels continuing warfarin for six months and enoxaparin until therapeutic on the warfarin. D Begin the patient on heparin sodium IV and monitor and adjust levels to achieve therapeutic levels Q six hours and transition to warfarin as soon as possible. E Initiate a hypercoaguability work up and treat accordingly.

The Correct Answer is: C Patients with deep vein thrombosis (DVT) without pulmonary embolism (PE) do not necessarily require hospitalization. Outpatient therapy would include treatment with a low molecular weight heparinoid (LMWH), such as enoxaparin, subcutaneously while the patient becomes therapeutic on oral warfarin. The LMWH can be discontinued once therapeutic on warfarin, with continued monitoring of INRs for six months before considering discontinuation. (McPhee and Papadakis, 2011, Chapter 9)

Q 112.4: A 38-year-old man presents to the emergency department experiencing a severe headache and heart palpitations. He appears to be anxious and perspiring heavily. On exam, he is found to be tachycardic and his blood pressure is 158/102 mm Hg. His urine catecholamines are increased. If imaging were performed, what is the most likely location where a lesion would be found? A pituitary gland B liver C adrenal gland D testicle E kidney

The Correct Answer is: C Pheochromocytomas produce, store, and secrete catecholamines. They are usually derived from the adrenal medulla, although they may be found in other locations. (Fitzgerald, 2009, pp. 1031-1034) Fitzgerald PA. Endocrine diseases. In: McPhee SJ , Papadakis MA, eds. Current Medical Diagnosis and Treatment. 48th ed. New York, NY: McGraw-Hill; 2009.

Q 92.5: Which of the following medications for type 2 diabetes mellitus is thought to best preserve beta cell function in the pancreas? A acarbose B glimepiride C pioglitazone D sitagliptin E insulin glargine

The Correct Answer is: C Pioglitazone is a thiazolidinedione (TZD). TZDs are thought to help preserve beta-cell function. (Fauci et al., 2008, Chapter 338)

Q 72.1: What is the most common hematologic finding in a patient with pulmonary hypertension? A Anemia B Thrombocytopenia C Polycythemia D Leukocytosis E Elevated mean corpuscular volume (MCV)

The Correct Answer is: C Polycythemia is the most common finding. Hematocrits that are >60% usually require phlebotomy, to reduce the numbers and prevent a hypercoagulable state. (Chesnutt MS, Prendergast TJ. Current Medical Diagnosis and Treatment, 2011, Chapter 9, Pulmonary Disorders)

Q 79.10: A 65-year-old male presents to you with a growth on the inner aspect of his left eye. He states that it has been getting slightly larger. On physical exam, you note a fleshy triangle shaped protrusion on the inner bulbar conjunctiva, abutting and slightly crossing the limbic border. Which of the following is the correct diagnosis? A Charazion B Hordeolum C Pterygium D Pinguecula E Cataract

The Correct Answer is: C Pterygium is a complication of exposure to ultraviolet light and wind. It consists of hyaline and elastin tissue. If it encroaches on the cornea, surgical removal is indicated. (Riordan et al., 2008, Chapter 5)

Q 80.2: Which of the following is the most prevalent cause of chronic respiratory acidosis? A Anemia B Cerebrovascular accident C Chronic obstructive pulmonary disease D High altitude E Pneumonia

The Correct Answer is: C Respiratory acidosis is associated with elevated PaCO 2 levels, due to the inability of elimination to keep pace with production. This may be due to a decreased rate of ventilation due to control alteration, decreased ventilatory muscle strength, underlying lung disease, or a systemic insult such as infection or medication. Chronic respiratory acidosis is generally due to underlying lung disease, with the most prevalent cause being chronic obstructive pulmonary disease (COPD). High altitude, pneumonia, and severe anemia are associated with respiratory alkalosis. Cerebrovascular accidents may be associated with either respiratory acidosis or alkalosis, depending on the location, extent, and impact. (Fauci et al., Harrison's Principles of Internal Medicine, 17e, Chapter 48, Acidosis and Alkalosis)

Q 75.1: A patient presents with mild dyspnea, increased cough, and rhinorrhea. On physical exam, you auscultate low-pitched, sonorous, and adventitious sounds over the bilateral upper lung fields, which are suggestive of secretions. Which of the following terms is defined by these findings? A Crackles B Rales C Rhonchi D Vesicular breath sounds E Wheezes

The Correct Answer is: C Rhonchi are defined as low-pitched, often harsh breath sounds, with increased secretions and inflammation. Rhonchi due to secretions may improve with coughing. Crackles, also known as rales, are due to an increase of fluid shifting from the intravascular space into the alveoli, and are often described as brief, nonmusical sounds with popping. Wheezes, which are high-pitched, musical sounds, are due to the narrowing of the airway related to mucosal edema, secretions, and bronchospasm. Vesicular breath sounds are normal lung sounds found over the periphery. (McPhee SJ, Papadakis MA. Current Medical Diagnosis & Treatment 2011, Chapter 9, Pulmonary Disorders)

Q 73.3: A 25 year old female presents with multiple irregular brown macules on her upper back. They are asymptomatic. She has worked as a lifeguard for the past 7 years. She reports a couple of blistering sunburns and admits to maintaining a "healthy tan". What is the most likely diagnosis? A Pityriasis alba B Basal cell nevus syndrome C Solar lentigines D Metastatic melanoma

The Correct Answer is: C Solar lentigines are a result of skin damage from the sun. Unlike ephiledes, they do not fade once exposure to the sun has stopped. There is no risk of malignancy associated with solar lentigines. (Wolff & Johnson, p264-265) (Fig 14-20, Wolff K, Johnson RA: Fitzpatrick's Color Atlas & Synopsis of Clinical Dermatology, 6 th ed: http://www/accessmedicine.com )

Q 106.15: Large numbers of epithelial cells on urine sediment indicate A UTI B acute tubular necrosis C sample contamination D vaginitis in women E prostatitis in men

The Correct Answer is: C Squamous epithelial cells line the distal portion of the urethra in men and the entire urethra in women. They appear in the urine due to inadequate cleaning of the external urinary meatus prior to obtaining the sample and indicate that the sample is contaminated. In women, the source is usually vaginal/perineal. Uncircumcised men commonly have squamous epithelial cells in the urine sample. (McBride, 1998, p. 103) McBride L. Textbook of Urinalysis and Body Fluids. Philadelphia, PA: Lippincott Williams & Wilkins; 1998.

Q 56.10: Which of the following is a major contraindication for surgical resection of a lung carcinoma? A Chest wall invasion B Non-malignant pleural effusion C Superior vena cava syndrome D Unilateral endobronchial tumor E Vagus nerve involvement

The Correct Answer is: C Surgical resection of lung carcinoma is contraindicated in cases of superior vena cava syndrome, extrathoracic metastases, heart, pericardial or great vessel involvement, recurrent laryngeal or phrenic nerve involvement, esophageal or carina involvement, malignant effusion, or contralateral mediastinal lymph nodes. Other contraindications are patient and staging dependent.

Q 104.6: An adult with a high risk for bacterial endocarditis is scheduled for a dental extraction. The patient has a history of penicillin allergy. Which of the following is an appropriate oral prophylactic drug to give this patient? A amoxicillin B vancomycin C clindamycin D doxycycline E gentamicin

The Correct Answer is: C The American Heart Association recommends that patients who are at moderate to high risk for bacterial endocarditis receive antibiotic prophylaxis prior to undergoing oral/dental, respiratory tract, or esophageal procedures. Amoxicillin 2.0 g orally 1 hour before the procedure is the standard regimen. Patients who have a history of amoxicillin/penicillin allergy may be given clindamycin, cephalexin, azithromycin, or clarithromycin. For adults, clindamycin is given at a dose of 600 mg po 1 hour before the procedure. (Schwartz and Chambers, 2009, pp. 1272-1273) Schwartz BS , Chambers HF. Bacterial and chlamydial infections. In: McPhee SJ , Papadakis MA, eds. Current Medical Diagnosis and Treatment. 48th ed. New York, NY: McGraw-Hill; 2009.

Q 116.6: A 71 year old female presents with complaint of a severe headache for 2 days. The patient denies a history of headaches in the past. She complains of a 2 week periods of morning shoulder and pelvic stiffness. There also a history of jaw pain when she chews her food. The past medical history is remarkable for well controlled hypertension, hyperlipidemia. On physical exam there is a markedly tender scalp and left temporal artery. The neurologic exam is normal. Besides a temporal artery biopsy what other diagnostic test is most indicated? A CBC B Computed tomography of the head C Erythrocyte Sedimentation Rate D Electrolytes and renal function E International normalized ratio

The Correct Answer is: C The clinical scenario suggests giant cell arteritis (GCA) or temporal arteritis. An erythrocyte sedimentation rate( ESR) or C-reactive protein( CRP) can aid in the diagnosis of GCA when evaluating an inflammatory vasculitis such as GCA. Treatment is based on history and physical and not the result of the ESR. Temporal artery biopsy is the gold standard for diagnosis. (McPhee et al., 2011, Chapter 20)

Q 117.10: A 78-year-old woman presents to the office complaining of a constant left-sided headache for 2 months. She has tried various over-the-counter (OTC) medications without relief. The patient admits to vision loss of her left eye last night for 10 minutes. The patient states that her vision then returned to normal. She denies pain in her eye. On review of systems, she relates several months of muscle aches and weight loss. On physical examination, she is found to have a tender, nonpulsatile superficial temporal artery. Her sedimentation rate is elevated at 90 mm/h. What is the next, most appropriate step in managing this patient? A stat MRI/MRA of the brain and cranial vessels B aspirin therapy C high-dose prednisone D lumbar puncture E sumatriptan (Imitrex) injection

The Correct Answer is: C The diagnosis is temporal arteritis. This is an arteritis of the temporal branch of the external carotid artery characterized by unilateral or bilateral headaches that may be localized to a tender temporal artery. The temporal artery may be thickened and tender and may be thrombosed and nonpulsatile late in the disease. Many patients present with malaise and have anemia and a low-grade fever. Fifty percent of patients report generalized muscle aches consistent with polymyalgia rheumatica. The most severe complication of temporal arteritis is blindness resulting from thrombosis of the ophthalmic artery. In some cases, this may be preceded by previous episodes of amaurosis fugax before the blindness becomes irreversible. Once blindness occurs in one eye, it may be prevented in the other by initiating treatment. The diagnosis is based on recognizing the clinical picture and obtaining a temporal artery biopsy. Treatment should not be delayed pending the biopsy. Early treatment with prednisone may prevent irreversible blindness. The efficacy of treatment can be measured with serial sedimentation rates. MRI and MRA have no value in establishing the diagnosis of temporal arteritis. Antiplatelet therapy would not be inappropriate but is inadequate for this diagnosis. The potentially unilateral headache should not be confused with a migraine for which Imitrex therapy would be appropriate. In addition, lumbar puncture has no role in establishing this diagnosis. (Langford and Fauci, 2008, p. 2127) Langford CA , Fauci AS. The Vasculitis syndromes. In: Fauci AS , Braunwald E , Kasper DL, et al., eds. Harrison's Textbook of Medicine. 17th ed. New York, NY: McGraw-Hill; 2008.

Q 105.6: A 26-year-old female presents with a whitish coating on her tongue and lips. When you attempt to rub the plaque with a tongue depressor, a small amount of bleeding is noted from the oral mucosa. Her past medical history includes asthma. What diagnostic test would you perform to confirm your diagnosis? A Strep screen B Culture and sensitivity C Wet mount D Viral culture E Gram stain

The Correct Answer is: C The history and physical are consistent with oral thrush. A wet mount potassium hydroxide preparation will reveal fungal spores and nonseptated mycelia. A culture and sensitivity is used for suspected bacterial infections. A gram stain is used to identify whether a bacterial pathogen is gram positive or gram negative. (McPhee et al., 2011, Chapter 8)

Q 121.15: A 43-year-old male farmer from the Southwest United States has been working in a very contaminated barn with rodent feces for the last week. He presents to your office with complaints of fever, non-productive cough, malaise, and decreased appetite. His physical exam reveals a temperature of 102 0 F, pulse rate of 98, blood pressure of 98/62, and O 2 saturation of 93%. Lung sounds have diffuse crackles throughout, and the rest of the exam is unremarkable. Based on the history and exam findings, what is the most likely pathogen for this type of illness? A Influenza pneumonia B Varicella pneumonia C Hantavirus pneumonia D Streptococcal pneumoniae E Cytomegalovirus

The Correct Answer is: C The history of the patient being exposed to the rodent feces is a typical presentation of a pneumonia caused by the hantavirus. There is no treatment for this type of pneumonia, only supportive care. (Lee et al., Current Diagnosis and Treatment in Pulmonary Medicine, Section X, Pulmonary Lung Disease, Chapter 37, Viral and Atypical Pneumonia)

Q 119.19: A 55-year-old male presents with severe swelling to his left eye. He denies injury or allergies. He states that he has had a severe sinus infection for the past two weeks. What is the most likely diagnosis for the following physical finding? A Allergic reaction B Eyelid abcess C Orbital cellulitis D Erysipelas E Zoster ophtalmicus

The Correct Answer is: C The image shows a severe orbital cellulitis. These infections often spread from paranasal sinus infections. Multiple pathogens may be involved, such as s.aureus, s. pheumoniae, and anaerobes. An eyelid abscess would be more localized. Allergic reactions that cause angioedema around the eye are usually bilateral, and also pruritic. (Tintinalli et al., 2011, Chapter 236)

Q 118.16: What is the initial treatment for a patient who is having an acute episode of supraventricular tachycardia? A Caffeine B Beta blockers C Valsava maneuver D No treatment E Synchronized cardioversion

The Correct Answer is: C The initial treatment that should be attempted is a simple vagal maneuver to break the reentry tachycardia. If this is unsuccessful, pharmacologic therapy is indicated. (Piktel JS., Cardiac Rhythm Disturbances, Section 3, Chapter 22. From Tintinalli J et al.,Tintinalli's Emergency Medicine)

Q 65.1: A 27-year-old woman presents with 3 days of fever, chills, headache, and a deep dry cough. She has been working at a pet store for the past month and thinks that one of the parakeets that came in 10 days ago may be sick. On examination, she has dullness to percussion of the right lung base and right-sided coarse crackles. The most likely diagnosis is A sarcoidosis B tularemia C psittacosis D brucellosis E listeriosis

The Correct Answer is: C The key piece of history in this question is the new exposure to parakeets. The symptoms and signs, including atypical pneumonia, are consistent with psittacosis but are not pathognomonic. Sarcoidosis is an illness of unknown cause. Listeriosis has been linked to exposures to contaminated food, particularly dairy products and hot dogs. Brucellosis can be caused by exposure to hogs, cattle, or goats. Tularemia is associated with contact with rabbits, other rodents, and biting arthropods.

Q 75.4: Which of the following agents is a significant cause of pill-induced esophagitis? A fluoxetine B omeprazole C ibuprofen D Vitamin D E ciprofloxacin

The Correct Answer is: C The most common causes of pill-induced esophagitis are nonsteroidal medications. Other commonly prescribed medications causing esophageal injury include slow release of potassium chloride, iron sulfate, quinine sulfate, and alendronate sodium. (McQuaid, 2009, pp. 520-521) McQuaid KR. Gastrointestinal disorders. In: McPhee SJ , Papadakis MA, eds. Current Medical Diagnosis and Treatment. 48th ed. New York, NY: McGraw-Hill; 2009.

Q 115.14: A 23-year-old patient with type 1 diabetes mellitus (DM) has been having difficulty sleeping at night. Usually around 3 am the patient will wake up feeling sweaty, nauseated, and tachycardic. He has recorded the following blood glucose levels: What advise is the best for this patient? A stop eating a bedtime snack B increase the evening regular dosage C decrease the evening Lente dosage D exercise before going to bed at night

The Correct Answer is: C The patient has described the Somogyi effect. This effect occurs because the patient is receiving too much intermediate insulin at dinnertime. This occurs when nocturnal hypoglycemia results in counter-regulatory hormones producing hyperglycemia. Either the intermediate insulin dosage can be shifted to a lower dosage at bedtime or the patient can eat a larger snack at bedtime. (Masharami, 2009, pp. 1074-1075) Masharami U. Diabetes mellitus and hypoglycemia. In: McPhee SJ , Papadakis MA, eds. Current Medical Diagnosis and Treatment. 48th ed. New York, NY: McGraw-Hill; 2009.

Q 114.3: A 45-year-old male presents with a history of thick, adherent yellow scaling in his scalp, and red scaling patches with fissuring in post auricular areas bilaterally. Which of the following are other areas of potential involvement? A antecubital and popliteal fossae B palms and soles C eyebrows, eyelashes, and beard area D trunk and neck

The Correct Answer is: C The patient presents with seborrheic dermatitis. Other areas of potential involvement include the eyebrows, eyelashes, and beard area. Antecubital and popliteal fossae are common areas of involvement in atopic dermatitis; not seborrheic dermatitis. The palms and soles are not involved in seborrheic dermatitis. The trunk and neck are not usually affected in seborrheic dermatitis. (Wolff et al., 2009, Page 49)

Q 116.7: A 27-year-old male with enlarged cervical lymph nodes and shortness of breath is seen. Lungs are clear to auscultation but diminished breath sounds centrally are noted. A chest x-ray reveals a large mediastinal mass. A biopsy of the lymph node is performed and Reed-Sternberg cells are noted on exam. What is the most likely diagnosis? A Acute myelogenous leukemia B Chronic lymphocytic leukemia C Hodgkin lymphoma D Infectious mononucleosis E Multiple myeloma

The Correct Answer is: C The presence of Reed-Sternberg cells in the histologic examination of lymph nodes, along with a large mediastinal mass, is consistent with Hodgkin lymphoma. Reed-Sternberg cells are not seen in multiple myeloma, AML, CLL, or infectious mononucleosis. (Lichtman et al., Williams Hematology 8e, Chapter 99, Hodgkin Lymphoma)

Q 74.2: A 46-year-old female complains of heartburn, steatorrhea, and a 20-lb weight loss. She was diagnosed with a solitary ulcer in the duodenal bulb, but it has been refractory to treatment. Imaging reveals the tumor but no hepatic metastasis. What is the recommended treatment in this patient to cure the disease? A Oral proton pump inhibitors B Resection of the entire duodenum C Resection of the tumor D Long term therapy with oral amoxicillin and an H2 antagonist E Systemic chemotherapy

The Correct Answer is: C The recommended treatment to cure localized disease in patients with Zollinger-Ellison syndrome is resection of the gastrinoma, before hepatic metastasis spread has occurred. (McPhee SJ, Papadakis MA. Current Medical Diagnosis & Treatment, 2010, p. 556)

Q 83.4: A 55-year-old man patient presents with tachycardia and heart palpitations. Physical exam shows a multinodular goiter. He does not have obstructive symptoms. He has suppressed TSH and elevated T 3 and T 4 , and a thyroid scan shows multiple functioning nodules. What is the treatment of choice for this patient? A propylthiouracil B beta-blockers C 131I ablation D surgical resection

The Correct Answer is: C The treatment of choice for multinodular goiter is 131 I ablation. In patients with very large thyroid glands with obstructive symptoms, surgical resection may be the best option. (Herman-Bonert, 2007, p. 651) Herman-Bonert VS. Hypothalamic-pituitary axis. In: Andreoli TA , Carpenter CC , Griggs RC, et al., eds. Cecil Essentials of Medicine. 7th ed. Philadelphia, PA: WB Saunders; 2007.

Q 69.8: A 59-year-old female with history of non-insulin-dependent diabetes mellitus (NIDDM), hypertension, and chronic kidney disease (CKD) returns for follow-up of her labs. You note that she her GFR has decreased from 40 to 36 ml/min/1.73m 2 . What stage of chronic kidney disease is she currently? A 1 B 2 C 3 D 4 E 5

The Correct Answer is: C There are five stages of CKD. Stage 3 Chronic kidney disease is referred to as a moderately decreased GRF between 30-59 ml/min/1.73m 2 . All other choices reflect different ranges of GRF above or below stage 3. (McPhee and Papadakis, 2011, Chapter 22)

Q 120.13: What primitive, or immature, defense mechanism is demonstrated by a patient who attributes their own, unacknowledged, feelings onto others while they search for perceived wrongdoings, no matter how small? A acting out B isolation C projection D splitting

The Correct Answer is: C These patients are sensitive to any criticism and are constantly searching for any insult or mistreatments, no matter how small or unintentional they may be. Confrontation is to be avoided as it is only counterproductive and will reinforce their beliefs. This is commonly seen in paranoid personality disorders. (Sadock and Sadock, 2008, p. 30) Sadock BJ , Sadock VA. Concise Textbook of Clinical Psychiatry, 3rd ed. Philadelphia, PA: Lippincott, Williams & Wilkins; 2008.

Q 113.4: A 46-year-old male comes into the emergency department complaining of severe left eye pain after a champagne cork hit his eye while trying to open a bottle on his honeymoon night. Visual acuity to the affected eye is limited to identifying finger movements only. A physical exam reveals the following findings. What is your diagnosis? A Corneal abrasion B Orbital contusion C Hyphema D Hypopyon E Retinal detachment

The Correct Answer is: C This image demonstrates a traumatic hyphema. There is an air-fluid level of blood in the anterior chamber of the eye. Treatment includes elevation of the head and dilation of the pupil. A corneal abrasion would demonstrate fluorescein staining of the cornea. A hypopyon is a collection of wbc's, or pus in the anterior chamber. A retinal detachment would only be visualized in the posterior eye with a direct ophthalmoscope. (Tintinalli et al., 2011, Chapter 236)

Q 118.11: A 30 year old male presents with bright red erythema over her cheeks and nose after spending a week at the beach. She has had no prior rashes. She states that the rash is pruritic and she has had a low grade fever accompanied by lethargy. Lab results show a (+) ANA and anti DS-DNA with an elevated ESR. What is the most likely diagnosis? A Rosacea B Roseola C Systemic lupus erythematosus D Fixed Drug eruption

The Correct Answer is: C This is a classic presentation of systemic lupus erythematosus (SLE). This autoimmune disorder is more common in women who are in their 20s or 30s. Frequently it is precipitated or worsened by sun exposure. (Wolff & Johnson, pg 382) (Fig 14-34A, Wolff K, Johnson RA: Fitzpatrick's Color Atlas & Synopsis of Clinical Dermatology, 6 th Ed: http://www/accessmedicine.com)

Q 81.6: A 28-year-old male presents with a tan-pink, well-demarcated waxy plaque, with raised firm borders located in the pretibial region of the left lower extremity. What is the next appropriate step to confirm the diagnosis? A liver function test B lipid panel C glucose tolerance test D complete blood cell count

The Correct Answer is: C This lesion is consistent with necrobiosis lipoidica, and is associated with diabetes mellitus. The patient should undergo glucose tolerance testing to be evaluated for diabetes. (Wolff et al., 2009, Page 428)

Q 86.10: A 70-year-old male is being followed for his chronic lymphocytic leukemia. On prior examination, he was staged at Binet Stage A, with three cervical lymph nodes and one right axillary lymph node palpated. Six months later he is seen for follow-up, where he now has five palpable cervical lymph nodes and two left and one right axillary node. His lymphocyte count has risen from 12,500 to 18,300. What is the best therapy at this time? A Alemtuzumab B Chlorambucil C Continue monitoring D Etoposide E Fludarabine

The Correct Answer is: C This patient continues to remain in Binet Stage A, and therefore would not benefit from beginning any additional therapy. Monitoring should be continued. If they would develop 3/5 lymphoid regions of involvement (cervical, axillary, ileofemoral, splenomegaly, or hepatomegaly) or doubling of the lymphocyte count in less than six months, then adding additional therapy would be indicated. Fludarabine is first-line therapy for progressing CLL. Chlorambucil is the main alkylating agent used in CLL. Alemtuzumab is a monoclonal antibody specific for human CD52, found on most lymphocytes and useful in CLL treatment. Etoposide is used in patients who failed alkylator-based chemotherapy. (Lichtman et al., Williams Hematology 8e, Chapter 94, Chronic Lymphocytic Leukemia and Related Diseases)

Q 95.3: A 31-year-old female is admitted to the hospital with a microangiopathic hemolytic anemia, thrombocytopenia, and elevated reticulocyte count and serum lactate dehydrogenase. Haptoglobin is decreased and a direct Coombs test is negative. There is a marked increase in schistocytes on blood smear. PT and aPTT are normal. What is the best first line of treatment? A Aspirin B Glucocorticoids C Plasma Exchange D Platelet transfusion E Vincristine

The Correct Answer is: C This patient has TTP (thrombotic thrombocytopenic purpura). The mainstay of therapy is plasma exchange. Glucocorticoids are used, but their efficacy has not been demonstrated conclusively. Aspirin is controversial, but has been used in additional therapy. Platelet transfusion is contraindicated. Vincristine is used if plasma exchange does not work. (Lichtman et al., Williams Hematology 8e, Chapter 133, Antibody-Mediated Thrombotic Disorders: Thrombotic Thrombocytopenic Purpura and Heparin-Induced Thrombocytopenia)

Q 109.3: A 48-year-old male presents with an 8-day history of a productive cough, subjective fevers, and malaise. He is otherwise healthy without any active medical problems. He is a social drinker of alcohol, and denies any tobacco or drug use. On physical examination, the patient is alert and oriented. His temperature is 100.4 0 F, pulse rate is 56, respiratory rate is 18, and blood pressure is 133/64. HEENT is within normal limits and a chest exam has diffuse expiratory wheeze with decreased sounds to the right lower lung fields. Blood labs reveal WBC 14.4, Hgb 11.3, Plt 233, ALT 65, AST 102, and PO 4 2.1. A chest x-ray reveals a dense consolidation with bulging fissures. Based on these findings, what is the best medication to treat this patient? A Ceftazidime B Vancomycin C Azithromycin D Penicillin E Gentamicin

The Correct Answer is: C This patient has a history and physical examination that is consistent with Legionella infection, as noted by the chest x-ray findings. The treatment of choice for the Legionella infection is the use of macrolides. In this case, azithromycin would be the best choice for this case. (Sabria M, Yu VL. Harrisons Online, Part 7, Infectious Disease, Section 6, Diseases Caused by Gram-Negative Bacteria, Chapter 141, Legionella Infection)

Q 85.4: A 45-year-old male is being treated with Imatinib Mesylate for his Ph chromosome positive disease. He obtained a complete cytogenetic response at six months. He is now one year out from diagnosis and initiation of therapy. Standard monitoring guidelines for this patient would include which of the following? A Bone marrow biopsy and cytogenetics yearly B Bone marrow biopsy and FISH for t(9;22) every six months C Bone marrow biopsy yearly and quantitative PCR on blood cells every three months D FISH for t(9;22) and quantitative PCR yearly E FISH for t(9;22) on blood cells and quantitative PCR every three months

The Correct Answer is: C This patient has chronic myelogenous leukemia Ph chromosome positive. With a complete cytogenetic response at six months, standard guidelines for monitoring of these patients include a yearly bone marrow biopsy and quantitative PCR on blood cells every three months. All other tests are done, but not in the frequency stated or necessarily following complete cytogenetic response. (Lichtman et al., Williams Hematology 8e, Chapter 90, Chronic Myelogenous Leukemia and Related Disorders)

Q 119.10: Which of the following is a result of untreated or partially treated otitis media, which presents with fever, ear pain, otorrhea, tenderness behind the ear, fluid collection, and destruction of air cells seen on head CT? A Suppurative otitis media B Peritonsillar abcess C Mastoiditis D Meningitis E Ethmoid sinusitis

The Correct Answer is: C Untreated or partially otitis media can result in mastoiditis. Tenderness, redness, and fluctuance over the mastoid bone is characteristic. Peritonsillar abcess symptoms include severe sore throat, drooling, dysphonia, and outpouching of the tonsillar pillar on the affected side and trismus. Ethmoid sinusitis presents with nasal congestion, discharge, and headache. Suppurative otits media is contained in the middle ear, without spreading to adjacent structures. (Knoop et al., 2010, Chapter 5)

Q 111.9: A 45-year-old nonsmoking female is found to have a 2-cm mass in the periphery of the left lung on a routine chest radiograph. She has no significant medical history, known exposures, or contributory family history. A subsequent biopsy confirms malignancy. Which of the following is the most likely underlying cell type? A Squamous cell B Large cell C Adenocarcinoma D Small cell

The Correct Answer is: C While adenocarcinoma, like other lung cancer types, is more likely to appear in smokers; it is the most common lung cancer seen in nonsmokers, particularly women and younger patients. It generally originates in the periphery of the lung. All other answers are associated with significant smoking history and, with the exception of large cell carcinoma, originate centrally. (Minna, 2008, p. 552) Minna JD , Schiller JH. Neoplasms of the lung. In: Fauci AS , Kasper DL , Longo DL , Braunwald E , Hauser SL , Jameson JL , Loscalzo J, eds. Harrison's Principles of Internal Medicine. 17th ed. New York, NY: McGraw-Hill; 2008.

Q 95.8: A 68-year-old female presents with symptoms of kidney failure. Her creatinine is elevated at 1.9, and monoclonal light chains are seen in the urine. She complains of fatigue and bony pain that has been increasing over the last several months. This patient is most likely experiencing symptoms from which disease? A Acute myelogenous leukemia B Chronic myelogenous leukemia C Hodgkin lymphoma D Multiple myeloma E Non-Hodgkin lymphoma

The Correct Answer is: D 30 to 50% of patients presenting with multiple myeloma will have some form of renal impairment. The most frequent cause is the formation of myeloma cast nephropathy. This is due to formation of tubular casts in the distal nephron, formed by the binding of light chains to uromodulin. These tubular casts obstruct the distal nephron and parts of the ascending loop of Henle, and contribute to development of interstitial nephritis. Light chains are not seen in the urine of any of the other diseases listed. (Lichtman et al., Williams Hematology 8e, Chapter 109, Myeloma)

Q 105.17: While interviewing a 29-year-old computer programmer, you find that he denies any close friends or prior sexual relationships and has no interest in developing them. He describes little enjoyment in any activities except role play video games. He denies past emotional difficulties or stressors. His exam reveals a flat affect throughout the visit but is otherwise normal. Which is the most likely diagnosis in this scenario? A antisocial personality disorder B adjustment disorder C seasonal affective disorder D schizoid personality disorder

The Correct Answer is: D A patient with ambivalence toward sexual relationships, no close contacts, and no desire for either, along with anhedonism and flat affect are typical for this disorder. The preference for solitary activities and use of fantasy furthers this picture. The lack of aggressiveness and risk-taking behavior lessons the antisocial diagnosis. The patient denied any precipitating event that would lend the problem to an adjustment disorder and the lack of variance, seasonal or otherwise, lessens the seasonal affective disorder diagnosis. (Eisendrath and Lichtmacher, 2009, pp. 925-926; Sadock and Sadock, 2008, p. 378) Eisendrath SJ , Lichtmacher JE. Psychiatric disorders. In: McPhee SJ , Papdakis MA, eds. Current Medical Diagnosis and Treatment, 48th ed. New York: McGraw-Hill; 2009. Sadock BJ , Sadock VA. Concise Textbook of Clinical Psychiatry, 3rd ed. Philadelphia, PA: Lippincott, Williams & Wilkins; 2008.

Q 62.8: A 35-year-old homeless male presents with a painful red right eye and decreased visual acuity, which occured over the past 48 hours. He doesn't recall any trauma, and prior to this his vision was good. His past history includes alcoholism and liver disease. On physical exam you notice a white opacity in the center of his right cornea. You are unable to do an ophthalmoscopic exam due to the opacity, and a fluorescein staining is strongly positive. What is the likely etiology of the white opacity? A Hypopyon B Hyphema C Corneal Infiltrate D Corneal Ulcer E Corneal Abrasion

The Correct Answer is: D A rapidly progressing central corneal ulcer must be considered first and treated aggressively. An ulcer will show fluorescein staining, due to a break in the corneal epithelium. Pseudomonas, strep pneumonia, herpes, and fungus must be considered as possible causes. An emergent ophthalmology consult can be sight saving.

Q 85.5: Administration of a drug that inhibits acetylcholinesterase would most likely lead to which of the following? A bronchodilation B decreased lacrimation C decreased salivation D increased gastric juice secretion E increased heart rate

The Correct Answer is: D Acetylcholinesterase is the enzyme responsible for metabolizing acetylcholine, which is the major neurotransmitter released from postganglionic neurons of the parasympathetic nervous system. Inhibition of acetylcholinesterase increases the level of acetylcholine at target tissues and can thereby create parasympathomimetic effects. These would include bronchoconstriction, increased lacrimation, increased salivation, bradycardia, and increased gastric juice secretion. (Costanzo, 2006, pp. 51-54) Costanzo LS. Physiology. 3rd ed. Philadelphia, PA: Saunders Elsevier; 2006.

Q 106.4: A 54-year-old male presents to you with a sudden onset of severe left eye pain and blurred vision. He states that he is nauseated and vomited twice. He denies any history of eye problems, other than having to wear glasses for reading. His only recent problem has been a mild upper respiratory infection, for which he is taking an over-the-counter decongestant. On physical exam, the vision in the affected eye is 20/200. His pupil is mid-sized and non-reactive to light, and the conjunctiva is markedly injected. What diagnosis must you consider first? A Retinal detachment B Central retinal artery occlusion C Open angle glaucoma D Angle closure glaucoma E Optic neuritis

The Correct Answer is: D Acute angle closure is characterized by sudden onset of severe eye pain, blurred vision, nausea, vomiting, visual halos, and headache. Physical exam findings can include conjunctival injection, a rock hard ocular globe on palpation, a cloudy cornea, and a mid-position fixed pupil. Normal intraocular pressure is below 21mm Hg. Acute angle closure glaucoma can develop pressures of 60 to 80mm Hg. (Tintinalli et al., 2011, Chapter 236)

Q 111.6: During a hospitalization for pneumonia, troponin levels are drawn on a 62-year old-man with a history of hypertension, hyperlipidemia, and chronic tobacco use, and found to be elevated above the 99 th percentile of normal. If acute myocardial infarction is ruled out, which of the following disease entities could also cause troponin elevation? A Mitral regurgitation B Gout C Parkinson's disease D Sepsis E Herpes zoster

The Correct Answer is: D Choice D, sepsis, is one of a long list of disease entities that can cause troponin elevation, including arrhythmias (both tachycardic and bradycardic), aortic valve disease, hypertrophic cardiomyopathy, invasive cardiac surgeries and procedures, severe pulmonary hypertension, pulmonary embolism, myocardial infiltrative diseases (such as amyloidosis, sarcoidosis, scleroderma, and hemochromatosis), acute respiratory failure, burns, pericarditis, endocarditis, myocarditis, and even occasionally due to extreme athletic activities such as marathon running. Not included on this long list, however, are choices A, B, C, and E. (Tintinalli et al., 2011, Chapter 52)

Q 68.4: A 45-year-old man with a history of NSTEMI, CABG X 3, HTN, and hyperlipidemia presents to your office with complaints of progressive dyspnea over the last three weeks, to the point that he is now dyspneic while walking across the room. In the last few days, he has noticed bilateral lower extremity edema. His EKG is unchanged, demonstrating evidence of his prior infarction but no acute ST or T wave changes. Which of the following is the most appropriate next diagnostic study for this patient? A Transesophageal echocardiogram B Cardiac catheterization C Pulmonary function testing D Transthoracic echocardiogram E Holter monitor

The Correct Answer is: D Choice D, transthoracic echocardiogram, is usually necessary in patients demonstrating symptoms suggestive of congestive heart failure, as the history and physical examination are usually not sufficient to determine the etiology of the patient's heart failure. When compared to choice A, it is a less invasive test. It is an inexpensive, yet sensitive test for the evaluation of not only systolic, but also diastolic function, as well as valvular function, all of which can play a role in congestive heart failure. Choice B would not be the most appropriate next step, although it would be useful in evaluation of coronary artery stenosi if a nuclear stress test is found to be positive for myocardial ischemia. Choice C is a useful test if a pulmonary etiology is suspected; however, in this patient with a prior history of NSTEMI and CABG prior to the age of 45, a cardiac etiology should be ruled out prior to a pulmonary one. Choice E, Holter monitor, is inappropriate in a patient with no complaints of palpitations without arrhythmia on EKG. (Tintinalli et al., 2011, Chapter 57)

Q 117.18: Following a gunshot wound to the lower abdomen, a 29-year-old man is hospitalized and treated with clindamycin for a potential anaerobic infection. After 3 days of clindamycin therapy, while recuperating in the hospital, he develops severe diarrhea, dehydration, and lower abdominal cramping. A stool culture is ordered and later discovered to contain Clostridium difficile. After discontinuing the clindamycin, which of the following would be the most appropriate treatment? A cefaclor B doxycycline C amoxicillin D metronidazole E cephalexin

The Correct Answer is: D Clostridium difficile is a gram-positive, anaerobic, spore-forming bacillus that is responsible for the development of antibiotic-associated diarrhea and colitis. C difficile colitis results from a disturbance of the normal bacterial flora of the colon, colonization with C difficile, and release of toxins that cause mucosal inflammation and damage. Antibiotic therapy is the key factor that alters the colonic flora. Specific therapy aimed at eradicating C difficile is indicated if symptoms are persistent or severe. The drug of choice is metronidazole, 500 mg orally three times daily or 250 mg orally four times daily. Oral metronidazole and vancomycin are equally effective in treating diarrhea caused by C difficile. Despite the isolation of metronidazole-resistant strains of C difficile, metronidazole is the drug of first choice because of its lower cost and the fact that it can promote vancomycin-resistant nosocomial infections. (Martin and Jung, 2008, p. 1863; McQuaid, 2008, pp. 543-544) Martin S , Jung R. Gastrointestinal infections and enterotoxigenic poisonings. In: DiPiro JT , Talbert RL , Yee GC, et al., eds. Pharmacotherapy: A Pathophysiologic Approach. 7th ed. New York: McGraw-Hill; 2008. McQuaid KR. Gastrointestinal disorders. In: Tierney LM Jr , McPhee SJ , Papadakis MA, eds. Current Medical Diagnosis & Treatment. 47th ed. New York: McGraw-Hill; 2008.

Q 121.13: Which of the following will cause conductive hearing loss? A Mumps B Syphilis C Multiple sclerosis D Otitis media E Medications

The Correct Answer is: D Conductive hearing loss is the result of blockage of sound waves from the external canal to the inner ear. Causes include cerumen, middle ear effusion, otitis media, and occiscle disruption. Multiple sclerosis causes VIIIth cranial nerve disruption and neural hearing loss. Mumps and syphilis can cause sensoryneural hearing loss. (McPhee et al., 2011, Chapter 8) (Lalwani A.K., 2008, Chapter 52)

Q 109.7: Which of the following would raise your suspicions the most and likely warrant consideration of testing for an inherited thrombophilia? A a deep femoral vein deep vein thrombosis (DVT) after a flight from Mumbai, India B an iliac vein DVT after a round trip bus trip to Atlantic City and playing slots all day C any DVT after a total knee replacement D an upper extremity DVT after tripping falling down a flight of stairs E any DVT in a woman who recently started oral contraceptives and smokes

The Correct Answer is: D DVTs most commonly arise from the deep femoral veins and iliac arteries, most commonly in patients who smoke and take oral contraceptives, after immobilizing surgeries, and/or after immobilization due to long periods of time seated, including but not limited to airplane flights, bus rides, etc. Upper extremity DVTs are rare, even after trauma, and warrant a hypercoaguability work-up to rule out inherited disease. (Current Diagnosis and Treatment: Surgery, Chapter 35)

Q 120.12: Which of the following drugs is indicated for the treatment of anemia associated with chronic renal failure? A deferoxamine B warfarin C protamine sulfate D erythropoietin E argatroban

The Correct Answer is: D Erythropoietin (EPO) is a naturally occurring hormone synthesized and secreted by the kidneys. Synthetic forms of EPO include Epogen and Procrit. EPO works at the red bone marrow to stimulate erythropoiesis. In patients with chronic renal failure, EPO production is usually impaired, and this EPO deficiency leads to anemia. Deferoxamine is an iron-chelating compound that can be given systemically in situations of iron overdose. Warfarin and argatroban are both anticoagulants and do not typically affect red cell count. Protamine sulfate is a heparin-chelating compound that can be given in cases of heparin overdose. (Watnick and Morrison, 2008, p. 797) Watnick S , Morrison G. Kidney disease. In: Tierney LM Jr , McPhee SJ , Papadakis MA, eds. Current Medical Diagnosis & Treatment. 47th ed. New York: McGraw-Hill; 2008.

Q 80.9: A 62-year-old female presents to your office with a complaint of hematuria. She had a cardiac valve replacement three months ago and was placed on warfarin. You perform an in-office international normalized ratio (INR) and discover it is 6.8. Which of the following agents would provide the appropriate reversal of the effects of the warfarin? A folic acid B Fresh frozen plasma C vitamin B D vitamin K (phytonadione) E protamine sulfate

The Correct Answer is: D Excessive anticoagulant effect and bleeding from warfarin can be reversed by stopping the drug and administering oral or parenteral vitamin K1 (phytonadione). The speed and extent of reversal must be balanced against the risk of recurrent thromboembolism in patients who require therapeutic anticoagulation. For example, an over-anticoagulated patient with a prosthetic mitral valve may develop fatal thrombosis if supratherapeutic anticoagulation is rapidly and fully reversed. (Tintinalli, et al., 2011, Chapter 234)

Q 109.14: A 46-year-old male presents to your office for consultation of supraventricular tachycardia (SVT). His history shows that he has had multiple episodes of SVT, and on three occasions has been treated in the emergency department with adenosine, which has converted his rhythm back to sinus. The patient is otherwise healthy, has no medical problems, and is on no medications. His vital signs are stable and his physical examination is within normal limits. Based on this history, which medication is the best choice for treating this patient long-term? A Magnesium sulfate B Spironolactone C Digoxin D Flecanide E Diltiazem

The Correct Answer is: D Flecanide doses up to 200 mg BID can be used to help control rate and prevent recurrences of reentry tachycardias. In this case, the use of digoxin would not be ideal, and the other medications would not have a direct effect on the nodal aberration that causes the tachycardia. (Calkins H. Hurst's the Heart, Chapter 38, Supraventricular Tachycardia: AV Nodal Reentry and Wolff-Parkinson-White Syndrome)

Q 119.1: In a patient whom you suspect has giant cell arteritis, which of the following medication and dosage combinations is indicated? A Naproxen 500 mg twice daily for 1 month. B Prednisone 20mg daily for 1 month C Methylprednisolone dose-pack D Prednisone 60 mg daily for 1 month E Infliximab 5mg/kg daily for 1 month

The Correct Answer is: D Giant cell arteritis (GCA) or temporal arteritis is a large vessel vasculitis that can cause ischemic optic neuropathy and blindness. The first line treatment of high dose prednisone 60 mg/day for one month is sight saving. The lower dose of prednisone 20mg per day is effective in treating polymyalgia rheumatic (PMR). Methylprednisolone dose-pack is a very low dose of steroid and would be ineffective in treating GCA. Infliximab is a tumor necrosis factor inhibitor used for treating ankylosing spondylitis, rheumatoid arthritis and crohn's disease. (McPhee et al., 2011, Chapter 20)

Q 71.8: A 30-year-old female patient comes into your family practice office to discuss the fact that her mother just tested positive for the BRCA 1 and BRCA 2 genes. She has also been tested and is positive. What is her risk for developing breast cancer by the age of 70? A Her risk is identical to the population. B Her risk is approximately 25%. C Her risk is approximately 55%. D Her risk is approximately 85%. E Her risk is approaches 100%.

The Correct Answer is: D Hereditary breast and ovarian cancers put both male and female patients at increased risk for multiple cancers, including breast, ovarian, prostate, and pancreatic. In this case, the risk of breast cancer by the age of 70 is approximately 85%. When in doubt, consult a genetic counselor. (Kantarjian et al., 2006, Chapter 21)

Q 114.1: A 62-year-old man with a history of hypertension, diabetes mellitus type 2, hyperlipidemia, and chronic tobacco use presents to the office with complaints of a retrosternal chest pressure associated with diaphoresis, nausea, and dyspnea, radiating down his left arm for the last 45 minutes after mowing his lawn. The patient's vital signs are stable, and on physical examination a new systolic murmur is appreciated. His EKG demonstrates evidence of acute anterolateral myocardial infarction on EKG, with ST segment elevation across the precordial leads, indicative of left anterior descending coronary artery stenosis. Which of the following is the most appropriate next step in management of this patient? A Dobutamine stress echocardiogram B Transthoracic echocardiogram C Nuclear stress test D Cardiac catheterization E Exercise treadmill stress test

The Correct Answer is: D In patients suffering from acute ST elevation myocardial infarction (STEMI), cardiac catheterization with percutaneous coronary intervention within 90 minutes substantially decreases morbidity and mortality outcomes; thus, choice D is the most appropriate next step in management of this patient. A transthoracic echocardiogram, choice B, will likely be performed during this patient's hospitalization, as it is can assist in assessment of complications of MI. However, the primary goal for acute STEMI is reperfusion in a timely manner, and thus TTE would not be the next step in the management of this patient. Choices A, C, and E are all forms of stress testing, which are useful testing modalities for patients with chest pain who are not actively infarcting. (McPhee et al., 2011, Chapter 10)

Q 110.9: An 18-year-old woman comes in for evaluation of "losing weight without meaning to." She also feels weak and in "always in the bathroom." Her appetite is normal but she "can't get enough to drink." Examination shows that she has lost 17# since her last visit a year ago. She is 66" tall and now weighs 120#. She is mildly orthostatic, but no other abnormalities are noted. A random blood sugar done in the office is 260mg/dl. Which results are most likely on measurement of her lipoproteins at this time? A decreased high density lipoprotein level B extremely elevated triglycerides C markedly increased total cholesterol D mildly elevated low density lipoproteins E normal profile

The Correct Answer is: D In persons with diabetes mellitus, type 1, low density lipoproteins, trigycerides (B), and total cholesterol (C) are likely to be slightly elevatated. High density lipoproteins (A) remain about the same as the patient's baseline. Once the glucose level is controlled, the lipid levels (E) on the profile typically return to normal. Masharani U, Diabetes Mellitus & Hypoglycemia, in Current Medical Diagnosis and Treatment, 52 nd ed. 2013.

Q 110.8: A surgical physician assistant suffers a deep puncture wound during surgery on an HIV-positive patient. The patient, who is on a multidrug regimen, has a viral load of 120,000 copies. Which of the following drugs is contraindicated for the physician assistant because of its potential for hepatotoxicity in the setting of HIV prophylaxis? A abacavir B indinavir C lamivudine D nevirapine E zidovudine

The Correct Answer is: D Nevirapine should be avoided for HIV prophylaxis as reports have linked it to hepatotoxicity in the prophylactic setting. Abacavir may cause rash and fever, indinavir kidney stones, lamivudine rash and peripheral neuropathy, and zidovudine anemia, neutropenia, nausea, malaise, headache, insomnia, and myopathy. (Katz and Zolopa, 2009, p. 1192) Katz MH , Zolopa AR. HIV infection. In: McPhee SJ , Papadakis MA, eds. Current Medical Diagnosis and Treatment. 48th ed. New York, NY: McGraw-Hill; 2009.

Q 104.4: Which of the following symptoms is most concerning for gastric cancer? A Dyspepsia exacerbated by meals B Alternating bowel habits from constipation to diarrhea C Dyspepsia that is exacerbated by recumbency D New-onset of dyspepsia in a 58 year-old patient E Steatorrhea

The Correct Answer is: D New-onset dyspepsia in a patient more than 55 years of age is the most concerning symptom of the ones listed for gastric cancer. It is quite common for GERD symptoms to be exacerbated by meals and recumbency. Alternating bowel habits, from constipation to diarrhea, can be associated with colon cancer. Steatorrhea can occur in many disease processes involving the small intestine, but is not typical in gastric cancer. (McPhee SJ, Papadakis MA. Current Medical Diagnosis & Treatment, 2010, p. 1472)

Q 58.2: The triad of "dermatitis, diarrhea, and dementia" (pellagra) results from a severe deficiency of which of the following vitamins? A thiamine B vitamin K C riboflavin D niacin E pyridoxine

The Correct Answer is: D Niacin deficiency is known as pellagra. It is rare in the United States and is most often a complication of alcoholism or malabsorption syndrome. Clinical signs of pellagra are known as the 3 Ds—dermatitis, diarrhea, and dementia.

Q 70.8: What is the treatment for a patient who has recurrent ventricular tachycardia with no reversible cause, and has failed oral medication therapy? A Biventricular pacemaker insertion B Ablation therapy C Heart transplantation D Implantable defibrillator E Single chamber pacemaker

The Correct Answer is: D Patients with recurrent symptoms benefit from the implantation of a defibrillator, which will reduce sudden death. Ablation therapy is usually not indicated. In rare cases of patients who do not have any other underlying disease, cardiac transplantation is an option. Pacemakers are options if the underlying rhythm is in need of pacing. (Bashore et al., Current Medical Diagnosis and Treatment, Chapter 10)

Q 118.7: A 42-year-old woman with a history of migraine cephalgia and Raynaud's phenomenon comes to the emergency department with complaints of severe chest discomfort that occurs at rest every morning (at approximately 10 AM). An EKG performed during an episode of chest discomfort demonstrates transient ST segment elevation, which is relieved with sublingual nitroglycerin. There is no troponin elevation. Cardiac catheterization is performed, and reveals coronary artery spasm, which corresponds with ST segment elevation, and no significant coronary artery stenosis. Which of the following choices is the most likely diagnosis? A Pericarditis B Acute myocardial infarction C Costochondritis D Prinzmetal angina E Myocarditis

The Correct Answer is: D Prinzmetal angina, or variant angina pectoris, is defined as coronary artery spasm associated with ST-segment elevation, and usually occurs at rest and at the same time of the day. Patients with a history of migraine cephalgia and Raynaud's phenomenon demonstrate Prinzmetal angina more frequently than the rest of the patient population. This can occur in patients with normal coronary arteries and with coronary artery stenosis. Choice A, pericarditis, would present with chest discomfort that is worse while supine and improves with sitting up, as well as a pericardial friction rub. Choice B, acute myocardial infarction, would present with troponin elevation, and is unlikely in the setting of a patient with normal coronary arteries on cardiac catheterization. Choices C and E would not be relieved with sublingual nitroglycerin or demonstrate transient ST-segment elevation. (LeBlond et al., 2009, Chapter 8)

Q 106.20: Which of the following laboratory findings will be observed in a patient with noninflammatory nonbacterial prostatitis? A positive bacterial culture from postmassage urine (chronic) B positive bacterial culture with expressed prostatic secretions (chronic) C negative bacterial culture with elevated leukocytes D negative bacterial culture with normal leukocytes

The Correct Answer is: D Prostatitis includes a continuum of prostate characteristics ranging from acute episodes to prostatodynia, a noninflammatory disorder. Patients with acute bacterial prostatitis will have an exquisitely tender prostate gland, and prostatic massage is contraindicated in these patients. Chronic prostatitis patients may have no evidence of an acute infection but will have increased leukocytes. Nonbacterial prostatitis is broken into two subcategories with the noninflammatory classification having neither bacteria nor leukocytes. (Nguyen, 2008, pp. 208-210; Stoller et al., 2009, pp. 830-832) Nguyen HT. Bacterial infections of the genitourinary tract. In: Tanagho EA , McAninch JW, eds. Smith's General Urology. 17th ed. New York, NY: McGraw-Hill; 2008:193-218. Stoller ML , Kane CJ , Meng MV. Urologic disorders. In: McPhee SJ , Papadakis MA, eds. Current Medical Diagnosis and Treatment. 48th ed. New York, NY: McGraw-Hill; 2009:827-847.

Q 82.3: You are treating a healthy 50-year-old man with no cardiac risk factors. This patient has no other medical conditions and takes no medications. His fasting lipid panel from last week demonstrates the following: What is the LDL goal for this patient? A less than 100 mg/dL B less than 120 mg/dL C less than 130 mg/dL D less than 160 mg/dL E less than 190 mg/dL

The Correct Answer is: D Recommendations of the National Cholesterol Education Program (NCEP) Adult Treatment Panel III Report states the LDL goal for patients with 0 to 1 risk factor to be less than 160 mg/dL. (Baron, 2009, p. 1101) Baron RB. Lipid disorders. In: McPhee SJ , Papadakis MA, eds. Current Medical Diagnosis and Treatment. 48th ed. New York, NY: McGraw-Hill; 2009.

Q 119.11: A 73-year-old male presents to clinic with a history of blood in his urine for the past month. He has mild irritation with voiding but denies any other symptoms except an unintentional weight loss of 20 pounds in the past 6 months. His past medical history includes hyperlipidemia, seasonal allergies, and fibromyalgia. He denies alcohol use and has a 22 pack-year history of tobacco use. Which risk factor supports your diagnosis? A Fibromyalgia B Hyperlipidemia C Seasonal allergies D Cigarette smoking E Age

The Correct Answer is: D Risk factors for bladder cancer include cigarette smoking (D) and exposure to industrial dyes and solvents. Age (E) is not a risk factor but the mean age of diagnosis is 73 years old and is more common in men than women (3.1:1). Fibromyalgia (A), hyperlipidemia (B), and seasonal allergies (C) have not shown any correlation to predisposing a person to bladder carcinoma. Papadakis MA, McPhee SJ, "Bladder Cancer." Quick Medical Diagnosis & Treatment: http://www.accessmedicine.com/quickam.aspx

Q 64.9: An otherwise healthy 20-year-old man has been given the accurate diagnosis of migraine. His frequency of attack is about one per month. He has never experienced adequate relief with any over-the-counter analgesics. Which of the following would be appropriate to try next? A verapamil 120 mg once daily B amitriptyline 25 mg once nightly C codeine/acetaminophen 15 mg prn D rizatriptan 10 mg at onset, may repeat once in 2 hours E oxygen at 7 L/min inhaled via nonrebreather mask

The Correct Answer is: D Rizatriptan, like the other 5-HT receptor agonists, is an extremely effective medication for the acute treatment of migraine. This patient has tried over-the-counter analgesics, which can work for mild forms of migraine, and so the use of a migraine-specific abortive agent is appropriate. Narcotic analgesics should be avoided not only because of the possibility of dependence but more importantly because they are not as effective as other analgesics for targeting the neurochemical causes of migraine. With a frequency of about 1 attack per month, a preventive agent such as verapamil or amitriptyline is not needed. Oxygen can be useful as an acute treatment for cluster headache but has not been shown to be useful for migraine.

Q 91.10: A 42-year-old man who is HIV positive develops fever of 38.8°C, mild nonproductive cough, and shortness of breath. He takes no medications other than a multivitamin tablet, does not smoke cigarettes, or use alcohol or illicit drugs. Of the following findings on diagnostic studies, which is most consistent with a diagnosis of Pneumocystic jiroveci pneumonia in this man? A apical infiltrates on chest radiography B bronchiolar consolidation on computed tomographic (CT) scan C CD4 count of 300 cells/mL D PO2 of 54 mm Hg E serum lactate dehydrogenase (LDH) level of 54 units/L

The Correct Answer is: D Severe hypoxemia is a common finding in Pneumocystis pneumonia even when symptoms are not severe. The characteristic chest radiograph findings are diffuse or perihilar infiltrates. Apical infiltrates are more likely to be seen in patients who have been receiving aerosolized pentamidine prophylaxis. High-resolution chest CT scanning would most likely demonstrate interstitial lung disease. This pneumonia is rare unless the CD4 count is less than 250. An elevated LDH is found in about 95% of patients, but this is not specific. (Katz and Zolopa, 2009, pp. 1183, 1361) Katz MH , Zolopa AR. HIV infection. In: McPhee SJ , Papadakis MA, eds. Current Medical Diagnosis and Treatment. 48th ed. New York, NY: McGraw-Hill; 2009.

Q 79.5: Which of the following is a tumor of bronchial origin that is known to grow rapidly and have diffuse metastases at the time of diagnosis? A Adenocarcinoma B Carcinoid C Large cell D Small cell E Squamous cell

The Correct Answer is: D Small cell lung cancer is a fast-growing, rapidly spreading form of lung cancer. Although the cells are small, they grow very quickly, metastasize to many parts of the body, and form large tumors. At the time of diagnosis, tumor spread is presumed. The growth and spread is considered much faster than that of non-small cell lung cancers. Staging is also different, utilizing a two-stage system based on the extent of spread. (McPhee SJ, Papadakis MA. Current Medical Diagnosis & Treatment 2011, Chapter 39, Cancer)

Q 97.3: A 56-year-old male is diagnosed with a duodenal ulcer through endoscopy. He is placed on an appropriate regimen of medications to heal the ulcer. Which of the following lifestyle choices is known to retard the rate of ulcer healing and should be discouraged? A Moderate alcohol intake B Ingestion of spicy foods C Eating at regular intervals D Smoking cigarettes E Ingestion of fruit juices

The Correct Answer is: D Smoking cigarettes does retard the rate of ulcer healing and increases the frequency of recurrence. Patients should be encouraged to eat balanced meals at regular intervals. There is no justification for bland or restrictive diets. Moderate alcohol intake is not harmful. (McPhee SJ, Papadakis MA. Current Medical Diagnosis & Treatment, 2010, p. 550)

Q 59.3: You are caring for a 29-year-old G1P0 at who is pregnant with twins. She has received routine obstetrical care and her pregnancy has been uneventful to date. What is the average gestation age for twins at delivery? A 33 to 34 B 39 to 40 C 35 to 36 D 36 to 37 E 37 to 38

The Correct Answer is: D The average length of gestation for a single fetus is 40 weeks; the average age of gestation decreases with increasing number of fetuses.

Q 82.2: A 30-year-old female presents with a five day history of a sore throat. She denies cough or nasal congestion. She also denies vomiting or diarrhea. On physical exam, her temperature is 101˚F, the pharynx is red with tonsillar exudates, and she has tender anterior cervical lymphadenopathy. What is your next step? A Perform culture and sensitivity B Perform rapid strep C Treat symptomatically with antipyretics D Begin oral penicillin E Begin oral ciprofloxacin

The Correct Answer is: D The centor criteria include fever, tonsillar exudates, tender anterior lymphadenopathy and lack of cough. These signs and symptoms highly suggest group A beta hemolytic strep. Treatment with penicillin would be the most appropriate step, especially if cost is a concern to the patient. Penicillin v potassium, 250 mg three times per day, or 500 mg twice daily for 10 days, is highly effective. Some studies show a five-day regime to be as effective. (McPhee et al., 2011, Chapter 8)

Q 111.3: A 28-year-old male presents with hypotension, marked tachypnea, and severe dyspnea following a fistfight. A physical exam reveals ecchymoses over the lateral left rib cage, hyperresonance on the left, and tracheal shift to the right. Which of the following is the most appropriate next step for management of this patient? A Anesthesia consultation B Chest CT scan C Chest x-ray D Needle thoracostomy to the second intercostal space, midclavicular line E Tube thoracostomy to the fifth intercostal space, midclavicular line

The Correct Answer is: D The diagnosis for this patient is a traumatic tension pneumothorax, a true medical emergency. Diagnosis can be made based on physical exam findings. Although definitive treatment with a properly placed tube thoracostomy is preferred, choice E has the position incorrect. Needle decompression in the second intercostal space, midclavicular line is the correct choice, with this remaining in place until a chest tube is properly in place. (Fauci et al., Harrison's Principles of Internal Medicine, 17e, Chapter 257, Disorders of the Pleura and Mediastinum)

Q 58.7: A 42-year-old male with a past medical history of renal failure and diabetes type II presents with facial swelling and pain. He states that it has been getting worse since it started five days ago. He also states that the side of his cheek became acutely swollen and painful five days ago when he was eating. His physical exam reveals a markedly swollen left submandibular space, with a firm and tender 1.5 cm nodule, palpable near the mandible, on the left side. When pressed, pus is seen coming out of the submandibular salivary duct. What is the most likely diagnosis? A Sialolithiasis B Parotitis C Ludwig's angina D Sialadenitis E Dental abcess

The Correct Answer is: D The history and physical exam is consistent with suppurative sialadenitis. The preceding episode of pain and swelling while eating indicates that the patient may have a salivary duct stone, which predisposed the patient to the salivary gland infection. Ludwig's angina is a bilateral submandibular and sublingual abcess, most often caused by infected mandibular molars. Parotitis is seen in mumps, a bilateral inflammation of the parotid glands caused by paramixovirus. Sialolithiasis are salivary duct stones without infection, are unilateral, and can affect the sublingual, submandibular, and parotid glands.

Q 106.5: Of the following diseases, which would you consider screening for first in a 55-year-old diabetic with no other comorbidities? A abdominal aortic aneurysm with a sonogram B carotid arterial disease with a sonogram C coronary artery disease with an angiogram D peripheral arterial disease with an ABI

The Correct Answer is: D The majority of patients with peripheral artery disease (PAD) will be asymptomatic. In fact, clinicians will miss 90% of patients with PAD if they wait for classic symptoms to appear. According to the American Heart Association, diabetics over the age of 50 are at "extremely high risk" of PAD and should be considered for screening. Generally speaking, screening for PAD should be performed in patients 50 years of age and older who smoke and/or have diabetes, all patients 70 years of age and older, as well as any patient with symptoms of PAD. (Fauci et al., 2008, Chapter 243)

Q 115.19: A 31-year-old male presents due to excessive daytime somnolence affecting his ability to work. He states that he often sleeps at inappropriate times and alters his schedule to avoid being caught. He describes an episode yesterday where he felt almost paralyzed upon awakening from sleep. He denies any other neurologic symptoms, recent illness, or previous sleep testing. He denies depression symptoms. Which of the following is the most appropriate diagnostic test to confirm the suspected diagnosis? A Ambulatory pulse oximetry monitoring B Brain MRI C Electroencephalogram D Multiple sleep latency test E Thyroid stimulating hormone

The Correct Answer is: D The most likely diagnosis is narcolepsy, based upon the patient's symptoms. Evaluation should include a multiple sleep latency test. Consideration should also be given to a sleep study with polysomnography, to evaluate for underlying sleep apnea. An in-depth sleep history should be obtained to rule out chronic sleep deprivation. Additional differentials should be considered (and are suggested by the other answer choices), but will be based upon the patient's presentation, symptoms, past medical history, and likelihood of additional diagnoses. (Fauci et al., Harrison's Principles of Internal Medicine, 17e, Chapter 28, Sleep Disorders)

Q 121.12: You are conducting a physical exam on a female, who was referred to you from an optometrist. She sought a visual screening due to progressive loss of visual acuity. She has not been seen by a physician in 10 years due to lack of insurance. She admits to a 15 lb weight gain in the past three years, and also complains of parasthesias in her feet. During an ophthalmoscopic exam you notice deep retinal microvascular hemorrhages, and cotton wool spots. What is the most likely cause of her visual disorder? A Macular degeneration B Retinal detachment C Central retinal artery occlusion D Diabetic retinopathy E Central retinal vein occlusion

The Correct Answer is: D The patient's symptoms suggest a likelihood of diabetes. Retinal findings can include microaneurysms, deep hemorrhages, a flame-shaped hemorrhage, exudates, and cotton wool spots. (Riordan et al., 2008, Chapter 6)

Q 54.3: The best initial diagnostic study for a suspected perforated peptic ulcer is which of the following? A abdominal ultrasound B upper GI barium swallow C esophagogastroduodenoscopy (EGD) D upright/decubitus abdominal plain film E colonoscopy

The Correct Answer is: D The presence of free intraperitoneal air on an upright or decubitus film in the majority of patients with peptic ulcer perforation. This finding along with a classic history of sudden onset of severe abdominal pain and a rigid, quiet abdomen should establish the diagnosis in most cases without the need for further studies. Barium studies are contraindicated in patients with a possible perforation.

Q 105.9: What is the radiographic pattern of a pneumonia that is caused by Pneumocystosis jiroveci in an HIV patient? A Lobar consolidation B Consolidation with effusion C Cavitary lesion with infiltrate D Diffuse interstitial infiltration E Lingular infiltrate

The Correct Answer is: D The presentation of diffuse interstitial infiltrates is what differentiates the patient from a typical pneumonia patient, including the presentation of the history, followed by the chest x-ray results. (Shelburne SA, Hamill RJ. Medical Diagnosis and Treatment, 2011, Chapter 36, Mycotic Infections)

Q 120.17: A new patient to the practice reports that his paternal grandfather had a tumor of the pituitary gland, his father had hyperparathyroidism, one uncle had a pancreatic tumor and another a thyroid cancer. His first cousin has "some kind of facial tumors" and another relative had treatment for "producing too much stomach acid." Given this family history, he should be screened for which of the following? A Carney complex B Cowden disease C McCune-Albright syndrome D MEN1 E Sipple syndrome

The Correct Answer is: D This family history strongly suggests Multiple Endocrine Neoplasia 1, which are inherited in an autosomal dominant manner and involve mutations on the long arm of chromosome 11 (11a13). Carney complex (A) is another syndrome of multiple endocrine neoplasia (MEN) with tumors of the adrenal cortex, pituitary gland, thyroid, and gonads and with hyperpigmentation and cardiac myxomas. Cowden disease (B) is also a syndrome of MEN including thyroid abnormalities, breast cancer and hamartomas. Another MEN, McCune-Albright syndrome (C) is associated with precocious puberty, Cushing syndrome, hyperthyroidism, and acromegaly. Sipple syndrome (E), also known as MEN2a. may develop medullary thyroid carcinoma, pheochromocytomas, or Hirschsprung disease. Fitzgerald PA, Endocrine Disorders, in Current Medical Diagnosis and Treatment, 52 nd ed. 2013.

Q 88.4: A two-month-old bottle fed infant female presents with a sharply demarcated scaling red rash on the face and in the diaper area. The mother reports that her child has been irritable and has had diarrhea. The primary care provider has treated with hydrocortisone 2.5% ointment bid for two weeks with no improvement. What should be the next step in confirming the diagnosis? A punch biopsy at border of genital rash B bacterial culture and viral culture C KOH and bacterial culture D cbc with diff and Zn level

The Correct Answer is: D This infant is displaying classic signs of zinc deficiency. This disorder can occur in infants who are bottle fed. Treatment with topical steroids will not improve the condition until the zinc level is corrected. A punch biopsy would not help in diagnosing the zinc deficiency that this patient is exhibiting. Bacterial and viral cultures will not be helpful in establishing the diagnosis. A KOH is used to diagnose fungal infections. The KOH and bacterial culture will not be helpful in establishing the diagnosis of zinc deficiency. (Wolff et al., 2009, Pages 442-443)

Q 110.4: A 22-year-old complains of a spreading rash. He states it began about a week ago, with one large spot on his abdomen. Very shortly thereafter, the rash rapidly spread on his torso. The lesions are small oval red plaques with a collarette of scale. He states they are only mildly pruritic. What is the most likely diagnosis? A tinea corporis B guttate psoriasis C atopic dermatitis D pityriasis rosea

The Correct Answer is: D This is the classic presentation of pityriasis rosea. The large initial lesion is the herald patch. The ensuing lesions are small, oval, and have a collarette of scale. The lesions are distributed along Blaschkoe's lines, which is the "Christmas tree" distribution. Presentation of tinea corporis is pruritic, annular scaling patches that enlarge with central clearing. Guttate psoriasis presents as salmon-pink papules or small plaques with overlying fine silvery scales. Atopic dermatitis is usually manifested prior to the age of six. The classic distribution is the flexural surfaces of the extremities. (Wolff et al., 2009, Page 122)

Q 106.16: A 31-year-old female presents with headache, fever, and petechiae to her lower extremities. A microangiopathic hemolytic anemia is noted. She denies diarrhea, recent infectious episodes, cough, shortness of breath, or urinary symptoms. What other finding would you expect to find in this woman to confirm the diagnosis? A Blasts on blood smear B Edema C Hepatomegaly D Thrombocytopenia E Sickle Cell

The Correct Answer is: D This patient has classic signs of thrombotic thrombocytopenic purpura. Microangiopathic hemolytic anemia and thrombocytopenia are seen in all cases of TTP in the absence of another plausible explanation. Edema is not seen in TTP. Sickle cell is found in sickle cell anemia. Hepatomegaly is seen in less than 20% of cases of TTP and does not confirm the diagnosis. Blasts are seen in leukemia and do not confirm the diagnosis. (Thrombotic Microangiopathy, Quick Answers to Medical Diagnosis and Therapy)

Q 64.1: A 45-year-old male presents to your office complaining of severe unilateral eye pain with some photophobia for one day. He denies any history of trauma. On examination and with staining, you notice a dendritic lesion to the cornea, and an otherwise normal examination. Which of the following medications would be contraindicated in this patient? A atropine ophthalmic drops B azelastine ophthalmic drops C levofloxacin ophthalmic drops D prednisolone ophthalmic drops

The Correct Answer is: D This patient has herpes simplex keratitis is an important cause of ocular morbidity. The ability of the virus to colonize the trigeminal ganglion leads to recurrences precipitated by fever, excessive exposure to sunlight, or immunodeficiency. The dendritic (branching) ulcer is the most characteristic manifestation. More extensive ("geographic") ulcers also occur, particularly if topical corticosteroids have been used. Ophthalmic corticosteroids in cases of suspected herpes simplex keratitis are contraindicated.

Q 110.18: A 65-year-old woman presents with severe mid-back pain of 2 weeks duration. She has no history of trauma. Radiographic evaluation reveals compression fractures of T11 and T12. A complete blood count, erythrocyte sedimentation rate, serum protein, serum calcium, phosphate, and parathyroid hormone levels are all within normal ranges. In addition to ordering a dual-energy x-ray absorptiometry (DEXA) scan, which of the following laboratory evaluations is most helpful in evaluating this patient for secondary causes of this presentation? A bone biopsy B rheumatoid factor C serum magnesium D 25-hydroxyvitamin D E antinuclear antibodies test

The Correct Answer is: D This patient has typical findings associated with osteoporosis. Most patients with osteoporosis are asymptomatic until fractures present. Fractures occur spontaneously and are associated with back pain of varied degrees. Serum calcium, phosphate, and parathyroid hormone levels are often normal. Since vitamin D deficiency state is common in osteoporosis, a 25-hydroxyvitamin D should be ordered. Bone biopsy is not indicated with this patient; this would be reserved for evaluating for osteomalacia. Rheumatoid factor and antinuclear antibodies would not be of importance with this patient presentation. Serum magnesium is associated more with evaluating for parathyroid or thyroid disorder and has no value with osteoporosis. (Fitzgerald, 2008, pp. 994-995) Fitzgerald PA. Endocrine disorders. In: Tierney LM , McPhee SJ , Papadakis MA, eds. 2008 Current Medical Diagnosis and Treatment. New York, NY: McGraw-Hill; 2008:949-1031.

Q 70.1: A 70-year-old man with a history of pulmonary hypertension and obstructive sleep apnea presents with complaints of increasing dyspnea while walking his dog. He has also recently noted increased lower extremity edema. On physical examination, jugular venous distension is noted. Auscultation of the chest demonstrates a high-pitched blowing diastolic murmur. With inspiration, the murmur increases in intensity and is heard over the second and third left intercostal spaces. An S 3 is appreciated. Palpation of the precordium reveals a hyperdynamic right ventricle, and both a systolic and diastolic thrill. The abdominal exam reveals hepatomegaly and splenomegaly. Based on this patient's history and physical exam findings, which of the following is the most likely finding on echocardiogram? A Tricuspid regurgitation B Aortic stenosis C Atrial septal defect D Pulmonic regurgitation/insufficiency E Mitral stenosis

The Correct Answer is: D This patient is demonstrating signs and symptoms of right heart failure, and with a history of pulmonary hypertension and a high-pitched diastolic blowing murmur (Graham Steell murmur), Choice D is the most likely of the choices offered. A blowing holosystolic murmur at the left lower sternal border is characteristic for tricuspid regurgitation, Choice A. Choice B, aortic stenosis, presents with a systolic ejection murmur. An atrial septal defect, Choice C, if large, could present with similar symptoms of exertional dyspnea secondary to a large shunt, but auscultation would reveal a moderately loud systolic ejection murmur that is heard best in the second and third interspaces. This is secondary to increased pulmonary arterial flow. Choice E, mitral stenosis, presents with a diastolic murmur heard best in the left lateral decubitus position, with the bell of the stethoscope at the apex. (McPhee et al., 2011, Chapter 10)

Q 65.5: A 49-year-old female with a known history of hypertension presents to the emergency department with a generalized headache that is throbbing. She states she had run out of her normal blood pressure medication about a week ago and since then she has noticed that her headache came about and has been getting worse. She denies any nausea, vomiting, visual changes, chest pain, or other symptoms. On exam the patient has a BP 227/120, P 78, R 18. Her HEENT exam is essentially normal, lungs are clear to auscultation, and heart is a regular rhythm without murmur or gallop. Given this clinical situation, which medication would be the most appropriate to address the patients condition? A IV nitroprusside B Oral furosemide C Spironolactone D IV labetalol E Oral hydralazine

The Correct Answer is: D This patient's clinical situation is one of a hypertensive emergency. In this situation the goal is to bring down the systolic pressure to prevent end organ damage. Given the possible choices, the best choice would be intravenous labetalol (D) due to its effective quick onset, and its ability to be tolerated with most patients. While oral furosemide (B) and hydralazine (E) can both be effective in managing hypertension, the IV dosing of labetalol would be the better choice. Nitroprusside (A) is no longer a treatment option. Spironolactone (C) would not have strong enough effects to appropriately lower the blood pressure in an efficient manner.

Q 120.8: A 31-year-old woman is being evaluated for irregular, infrequent menstrual periods. On further questioning, she complains of headaches, fatigue, and breast discharge. She takes ibuprofen only occasionally. Which of the following labs would most likely be elevated in this patient? A BUN and creatinine B luteinizing hormone (LH) and follicle-stimulating hormone (FSH) C oxytocin D prolactin E TSH

The Correct Answer is: D This patient's symptoms are consistent with a pituitary adenoma. Prolactinomas account for about half of all functioning pituitary tumors and may secrete PRL, GH, and ACTH. (Herman-Bonert, 2007, p. 640) Herman-Bonert VS. Hypothalamic-pituitary axis. In: Andreoli TA , Carpenter CC , Griggs RC, et al., eds. Cecil Essentials of Medicine. 7th ed. Philadelphia, PA: WB Saunders; 2007.

Q 119.16: A 55-year-old right-hand dominant man presents with a 4-hour history of weakness and tingling of his right hand and numbness of the right side of his mouth. Mild difficulty was noted with word finding. His symptoms have improved since onset but have not fully resolved. There is no significant medical history. Physical examination revealed flat right nasolabial fold, subjective numbness of the right hand, right pronator drift, clumsiness of finger tapping on the right hand, increased deep tendon reflexes on the right, as well as a present Babinski. What is the most likely etiology for this patient's problem? A migraine headache B peripheral neuropathy C syncope D transient ischemic attack E seizure

The Correct Answer is: D Three key features of a transient ischemic attack include sudden onset and complete reversal of symptoms within 24 hours, usually within 15 minutes. The symptoms are usually in the anatomical distribution of a single blood vessel. This patient's history is not suggestive of migraine or syncope. His physical examination findings do not correlate with peripheral neuropathy or seizure. (Aminoff et al., 2005, p. 286) Aminoff MJ , Greenberg DA , Simon RP. Clinical Neurology. 6th ed. New York, NY: McGraw-Hill; 2005.

Q 59.4: Which of the following findings is consistent with a lower motor neuron deficit? A aphasia B dysdiadochokinesia C sensory loss D weakness E hyperreflexia

The Correct Answer is: D Weakness is one potential finding of a lower motor neuron process. Aphasia results from injury to the speech pathways within the brain. Sensory loss arises from many causes, but it is not a motor issue. Dysdiadochokinesia is consistent with cerebellar pathology. Hyperreflexia is typically a signal of upper motor neuron disease.

Q 86.5: You are examining a 65-year-old male who complains of partial vision loss in his right eye. Besides obtaining visual acuity, what is the most important initial part of the physical exam to perform in order to evaluate his condition? A Cover uncover test B Consensual papillary response C Extraocular movements D Visual fields by confrontation E Intraocular pressure

The Correct Answer is: D When a patient complains of vision loss, all of the choices are important parts of the eye examination; the visual field by confrontation exam is a screen to detect visual field defects. (LeBlond et al., 2009, Chapter 20)

Q 54.12: A patient carrying the diagnosis of asymptomatic multiple myeloma is asking what the best course of therapy and treatment is. Calcium levels are currently at high normal, he is slightly anemic with a hemoglobin of 11.8 (12 to 16), and no new lytic lesions are identified. What is the best treatment at this time? A Allogeneic transplant B Autologous stem cell transplant C Hydration and calcitonin D Observation E Reduced intensity allogeneic transplant

The Correct Answer is: D With patients that are not showing signs of advancing disease or minimal disease, and it is unclear whether paraproteniemia is benign, observation is the preferred therapy. All others are possible treatments with advancing disease, depending on the condition of the patient, advancement of disease, and response to prior therapies.

Q 77.7: A 50-year-old woman presents with constipation and crampy abdominal pain for the past 3 months. She is also undergoing a divorce and has had a 15-lb weight loss in the past 3 months. You note mild tenderness to palpation in the left lower quadrant; no masses are noted. Rectal exam result is negative, but her stool tests positive for fecal occult blood. Which of the following is the most appropriate next step to evaluate her symptoms? A keep a food diary for the next 2 weeks B flexible sigmoidoscopy C increase dietary fiber and increase daily water intake D refer for psychologic evaluation to help with stress of her divorce E colonoscopy

The Correct Answer is: E Any symptomatic adult with a positive fecal occult blood test should undergo colonoscopy to rule out colorectal cancer. A flexible sigmoidoscopy will allow for only partial visualization of the colon. (McQuaid, 2009, p. 507) McQuaid KR. Gastrointestinal disorders. In: McPhee SJ , Papadakis MA, eds. Current Medical Diagnosis and Treatment. 48th ed. New York, NY: McGraw-Hill; 2009.

Q 77.9: A 13-year-old girl is brought to the office by her mother who is having difficulty managing the child's recent angry outbursts. Her grades in schools have gone from A's and B's to C's and D's; she has been sent to the office for "behavior problems" several times in the last two months despite having been an exemplary student in the past. The girl says she "feels nervous" all the time, "just can't sit still" in class, and "sweats like a pig" even when she isn't exercising. Her weight today is 10# less than at her previous well-child visit 6 months ago, but the child denies dieting and her mother reports that she eats "constantly." Vital signs include temperature of 99.0F, pulse of 96, respirations of 26. Her blood pressure is 148/82. Her skin is warm and moist. Which of the following is the most likely diagnosis? A amphetamine abuse B anorexia nervosa C bipolar disorder D bulimia nervosa E hyperthyroidism

The Correct Answer is: E Hyperthyroidism occurs more common in girls than in boys, most frequently in adolescence. Deterioration in school performance is a common finding, along with emotion lability, nervousness, personality changes, warm and moist skin, polyphagia, and weight loss. Systolic hypertension and a widened pulse pressure are often found. Amphetamine abuse (A) is certainly a possibility, but it tends to depress the appetite. Anorexia nervosa (B) is unlikely given the child's appetite. Bipolar disorder (C) in adolescents more typically presents with depressive symptoms, and does not explain the physical findings. Patients with bulimia nervosa (D) are typically normal or overweight. Zeitler PS, et al. Endocrine Disorders, in Current Diagnosis and Treatment, Pediatrics, 21 st ed. 2012.

Q 117.5: What type of disorder develops within 3 months of an identified stressor such as finances, going to school, divorce, or illness in their life. The stressor causes impairment in their job and relationships, but the symptoms resolve within 6 months. What is the most likely diagnosis? A depression B bereavement C post-traumatic stress disorder D personality disorder E adjustment disorder

The Correct Answer is: E A response to a stressor that disturbs the mood of the patient causes impairment in function. The symptoms occur within 3 months of the stressor and last no longer than 6 months. Anxiety, depression, or combination is associated with adjustment disorders. (Eisendrath and Lichtmacher, 2009, p. 912; Sadock and Sadock, 2008, p. 371) Eisendrath SJ , Lichtmacher JE. Psychiatric disorders. In: McPhee SJ , Papdakis MA, eds. Current Medical Diagnosis and Treatment, 48th ed. New York: McGraw-Hill; 2009. Sadock BJ , Sadock VA. Concise Textbook of Clinical Psychiatry, 3rd ed. Philadelphia, PA: Lippincott, Williams & Wilkins; 2008.

Q 91.7: A 78-year-old male presents to the office due to increasing exertional dyspnea and cough for the past week. Physical exam reveals an S3 gallop, mild JVD, and 2+ pitting edema of the bilateral lower extremities. The patient has had mild congestive heart failure in the past. A chest x-ray reveals prominent pulmonary vasculature without any additional complications. The patient's current medications are aspirin and simvastatin daily. He has no drug allergies. Which of the following is the most appropriate initial management for this patient? A Angiotensin II receptor blocker B β blocker C Calcium channel blocker D Nitrate E Thiazide diuretic plus ACE inhibitor

The Correct Answer is: E ACE inhibitors are recommended, as first line treatment for symptomatic congestive heart failure, based upon clinical trials, reveal an approximately 20% reduction in CHF mortality in symptomatic heart failure patients. Diuretics provide CHF symptom improvement and promote water and sodium excretion to decrease intravascular volume. blockers and Angiotensin II receptor blockers are also beneficial for CHF patients. Nitrates are usually reserved for acute or decompensating patients. Calcium channel blockers may accelerate CHF progression and should be used with caution. (McPhee SJ, Papadakis MA. Current Medical Diagnosis & Treatment 2011, Chapter 10, Heart Disease)

Q 65.7: Which of the following lab results reflect acute anxiety neurosis hyperventilation? A Acidotic pH, PCO2 increased, HCO3 decreased B Acidotic pH, PCO2 decreased, HCO3 decreased C Acidotic pH, PCO2 increased, HCO3 normal D Alkalotic pH, PCO2 increased, HCO3 increased E Alkalotic pH, PCO2 decreased, HCO3 normal

The Correct Answer is: E Acute hyperventilation, associated with anxiety and other disorders, results in decreased PCO 2 and leads to acute respiratory alkalosis. Acute respiratory alkalosis is associated with an increased pH and a decreased PCO 2 . Chronic respiratory alkalosis will result in compensatory bicarbonate (HCO 3 ) level changes; in this scenario, the bicarbonate is expected to be normal initially. Respiratory alkalosis has other causes (as shown in Table 21-26). The other answers represent a variety of acid-base disorders including, mixed acid-base disorder (choice A), metabolic acidosis with compensation (choice B), uncompensated respiratory acidosis (choice C), and metabolic alkalosis with compensation (choice D). These disorders, and the respective expected compensation, are reviewed in Table 21-12. Table 21-16. Causes of respiratory alkalosis. Hypoxia Decreased inspired oxygen tension High altitude Ventilation/perfusion inequality Hypotension Severe anemia CNS-mediated disorders Voluntary hyperventilation Anxiety-hyperventilation syndrome Neurologic disease Cerebrovascular accident (infarction, hemorrhage) Infection Trauma Tumor Pharmacologic and hormonal stimulation Salicylates Nicotine Xanthines Pregnancy (progesterone) Hepatic failure Gram-negative septicemia Recovery from metabolic acidosis Heat exposure Pulmonary disease Interstitial lung disease Pneumonia Pulmonary embolism Pulmonary edema Mechanical overventilation (Current Medical Dx & Tx, Chapter 21, Electrolyte & Acid-Base Disorders, Acid-Base Disorders, Respiratory Alkalosis (Hypocapnia)) (McPhee SJ, Papadakis MA. Current Medical Diagnosis & Treatment 2011, Chapter 21, Electrolyte & Acid-Base Disorders)

Q 85.3: A patient that was diagnosed with severe hemophilia B in 1974 and requiring regular factor IX replacement therapy will have a 1:2 chance of also having which of the following? A Hemarthroses B Hemophilia A C Hepatitis B D Hepatitis C E HIV

The Correct Answer is: E Approximately 50% of older and severely affected patients with either hemophilia A or hemophilia B are now HIV positive, due to the pooled nature of therapeutic products prior to 1985. These patients are also at increased risk for Hepatitis B and C, but not at the same ratio. Hemarthroses is a complication of not treating factor IX bleeding. (Lichtman et al., Williams Hematology 8e, Chapter 124, Hemophilia A and Hemophilia B)

Q 105.20: Which of the following is the primary mechanism by which benzodiazepines exert their sedative and anxiolytic effects? A acting as dopamine receptor agonists B acting as NMDA receptor antagonists C acting as serotonin receptor antagonists D decreasing reuptake of serotonin and norepinephrine E increasing GABAA receptor-mediated chloride conductance

The Correct Answer is: E Benzodiazepines bind to GABA A receptors, which consist of many peripheral subunits that form chloride channels at their core. GABA is one of the major inhibitory neurotransmitters in the brain; hence, benzodiazepines enhance this inhibitory influence to produce sedation and calm. (Kirkwood and Melton, 2008, p. 1167) Kirkwood CK , Melton ST. Anxiety disorders I: generalized anxiety, panic and social anxiety disorders. In: DiPiro JT , Talbert RL , Yee GC, et al., eds. Pharmacotherapy: A Pathophysiologic Approach. 7th ed. New York: McGraw-Hill; 2008.

Q 116.20: A 73-year-old male presents to clinic with a history of blood in his urine for the past month. He denies irritation with voiding or any other symptoms. He does state an unintentional weight loss of 20 pounds in the past 6 months. His past medical history includes hyperlipidemia, seasonal allergies, and fibromyalgia. He denies alcohol use and has a 22 pack-year history of tobacco use. On physical examination you note bilateral inguinal lymphadenopathy. Which of the following is the most likely diagnosis? A Benign prostate hypertrophy B Urinary tract infection C Prostatitis D Syphilis E Bladder carcinoma

The Correct Answer is: E Bladder cancer's presenting symptom is hematuria in 85-90% of patients (E). Clinical findings include masses detected on bimanual exam, lymphedema of the lower extremities, or palpable lymphadenopathy with advanced cancers. Urinary tract infections (B) and prostatitis (C) commonly cause discomfort with voiding. BPH (A) does not correlate with a 20-pound weight loss, smoking history, or lymphadenopathy. This history and physical examination do not show any signs of syphilis (D). Papadakis MA, McPhee SJ, "Bladder Cancer." Quick Medical Diagnosis & Treatment: http://www.accessmedicine.com/quickam.aspx

Q 72.6: A 70-year-old man, with a history of HTN and aortic valve replacement 3 months ago, presents with complaints of arthralgia, myalgia, anorexia, fatigue, and weight loss over the last month, with recent dyspnea on exertion and lower extremity edema. Vital signs are as follows: Temperature 38°C, BP 102/64, P 98, RR 20. On physical exam, a new high-pitched, blowing, decrescendo diastolic murmur is noted along the left lower sternal border. Two separate blood cultures are positive for S. aureus. Which of the following is the most appropriate next diagnostic study? A EKG B CXR C TTE D ESR E TEE

The Correct Answer is: E Choice E, TEE or transesophageal echocardiogram, would be most useful in establishing a diagnosis of infective endocarditis, as a positive echocardiogram demonstrating presence of a vegetation would satisfy one of the Duke criteria's major criteria, as well as determine the extent of the prosthetic valvular dysfunction, if present. TEE is more sensitive than TTE, transthoracic echocardiogram, for detecting vegetations, so choice C is incorrect. Choices A and B, EKG and CXR, should be performed as part of this patient's evaluation, but would be less useful than TEE in establishing a diagnosis of infective endocarditis. Choice D, erythrocyte sedimentation rates, are frequently elevated in patients with endocarditis, but are not specific to the diagnosis of endocarditis. (Fauci et al., 2001, pp. 809-811)

Q 108.9: A 70-year-old man with a history of hypertension, DM Type 2, and hyperlipidemia is seen for preoperative evaluation prior to left total knee replacement. On auscultation, a very soft high-frequency decrescendo early diastolic murmur is heard at the upper left sternal border. Utilizing isometric hand grip exercises, the murmur increases in intensity and can be heard radiating to the left sternal border and apex. Given the patient's physical exam findings, which of the following is the most appropriate next step in the management of this patient? A Chest x-ray B Transesophageal echocardiogram C Holter monitor D Treadmill exercise stress test E Transthoracic echocardiogram

The Correct Answer is: E Choice E, transthoracic echocardiogram, is a simple, sensitive and non-invasive diagnostic tool which can evaluate for the presence of valvulopathy in this patient. Choice A might be able to give evidence of cardiomegaly, but would not be sensitive enough to detect valvulopathy. Choice B, transesophageal echocardiogram, would give information regarding valvulopathy, but is a more invasive test; therefore, choice E is more appropriate. Choice C is a useful diagnostic tool for evaluation of patients complaining of palpitations, but incorrect for this patient who has no symptoms. Choice D, although a useful diagnostic tool for the evaluation of exercise tolerance and in patients complaining of chest pain, does not allow direct visualization of the heart valves to evaluate for valvulopathy. In addition, a patient scheduled for left total knee replacement is unlikely to perform very well on a treadmill, thus the more appropriate test for preoperative evaluation, if necessary after transthoracic echocardiogram, would be a nuclear stress test. (Crawford et al., 2009, Chapter 1)

Q 88.5: A 33-year-old female with history of non-insulin dependent diabetes mellitus (NIDDM), hypertension, and obesity comes to see you for treatment of her significantly elevated cholesterol. She intends to have children in the next year and has been off birth control. Which of the following medications is considered to be safest for treatment of hyperlipidemia in pregnancy? A lovastatin B ezetimibe C rosuvastatin D fenofibrate E colesevelam

The Correct Answer is: E Colesevelam (WelChol) is considered a Category B pregnancy risk. Both lovastatin and rosuvastatin (statins) are considered Pregnancy Category X and should be avoided in pregnancy. Ezetimibea and fenofibrate are both Pregnancy Category C. (McPhee and Papadakis, 2011, Chapter 28)

Q 113.6: A 52-year-old man recently underwent surgery for a hip replacement. Upon discharge, he is prescribed oxycodone 7.5 mg and acetaminophen 325 mg and told to take 1 to 2 tablets every 6 hours prn to help manage the pain he is expected to encounter as he recovers at home. Which of the following medications would you also recommend for the patient to help minimize potential side effects associated with his pain medication? A esomeprazole B diphenhydramine C guaifenesin D hydrocortisone 1% cream E senna & docusate

The Correct Answer is: E Constipation is a common adverse effect of opioid therapy. In order to minimize or prevent constipation, the use of stool softeners (docusate) and a stimulant laxative (senna) can be initiated when opioid therapy is begun. (Rabow and Pantilat, 2008, pp. 69, 78) Rabow MW , Pantilat SZ. Palliative care & pain management. In: Tierney LM Jr , McPhee SJ , Papadakis MA, eds. Current Medical Diagnosis & Treatment. 47th ed. New York: McGraw-Hill; 2008.

Q 65.9: During a pre-surgical workup, a patient reports that she had received Desmopressin following the birth of her son. Her CBC is within normal range, as well as her PT and aPTT. What is the most likely cause of her needing this medication? A Allo immunization following vaginal delivery B Hemolytic anemia C Iron deficiency anemia D Thalassemia E Von Willebrand's disease

The Correct Answer is: E Desmopressin is the mainstay of therapy for people with von Willebrand's disease. Hemolytic anemia, iron deficiency anemia, and thalassemia would all have abnormal findings on the CBC. Allo immunization does not require desmopressin therapy. (Lichtman et al., Williams Hematology 8e, Chapter 127, von Willebrand Disease)

Q 83.8: A 23-year-old male presents with uncontrolled mucocutaneous bleeding following a wisdom tooth extraction. Traditional methods of controlling the bleeding have not significantly reduced the bleeding. On history, he reports epistaxis that takes longer than usual to stop bleeding. He also states that he seems to bruise more frequently than his friends. What laboratory test would be the most beneficial given his history? A Bleeding time and platelet count B Bleeding time and VWF antigen C Factor VII assay and factor VIII assay D Factor VII assay and platelet count E Factor VIII assay and VWF antigen

The Correct Answer is: E In a patient suspected of having von Willebrand's disease, routinely performed tests include the assay of factor VIII activity, VWF antigen, and VEW Ristocetin cofactor activity. Bleeding time was previously used as a screening test, but is inconsistent. Factor VII assay would be used for someone suspected of having VII deficiency. Platelet counts are usually normal in a patient with von Willebrand's disease. (Lichtman et al., Williams Hematology 8e, Chapter 127, von Willebrand Disease)

Q 86.9: A patient that has exhibited excessive bleeding tendencies is scheduled for a cholecystectomy. By history, he is noted to be of Ashkenazi Jewish decent. Which coagulation disorder is most likely the cause of his bleeding tendency? A Factor V B Factor VII C Factor VIII D Factor IX E Factor XI

The Correct Answer is: E Most patients with factor XI deficiency are Jewish, with most in the Ashkenazi population. While other factors may affect individual patients, the most common of these coagulation disorders within this population base is factor XI. (Lichtman et al., Williams Hematology 8e, Chapter 125, Inherited Deficiencies of Coagulation Factors II, V, VII, X, XI, and XIII and Combined Deficiencies of Factors V and VIII and of the Vitamin K-Dependent Factors)

Q 104.5: Which of the following antibiotics is the most appropriate treatment for antibiotic-associated colitis? A oral ciprofloxacin B intravenous vancomycin C oral sulfasalazine D intravenous penicillin E oral metronidazole

The Correct Answer is: E Oral metronidazole is the drug of choice. Both vancomycin and metronidazole are effective; however, metronidazole is less expensive and there is less of a concern for vancomycin resistance. (Dipiro, 2008, p. 1863) Dipiro JT , Talbert RL , Yee GC , et al. , Pharmacotherapy: A Pathophysiologic Approach. 7th ed. New York, NY: McGraw-Hill; 2008.

Q 75.2: What is the laboratory test that has the best predictability for diagnosing pneumonia caused by adenovirus? A Nasal washings B Viral DNA test C Complete blood count with differential D Sputum gram stain E Polymerase chain reaction test (PCR)

The Correct Answer is: E PCR testing is the best method for determining adenovirus pneumonia. It has a sensitivity of 90 to 100% and the specificity is >95%. (Lee et al., Current Diagnosis and Treatment in Pulmonary Medicine, Section X, Pulmonary Lung Disease, Chapter 37, Viral and Atypical Pneumonia)

Q 106.9: A 35-year-old male presents complaining of increasingly constant headaches, double vision centrally, and a progressive loss of peripheral vision for two weeks. He has no previous headache history and denies any other medical conditions. Physical examination reveals bitemporal hemianopsia without additional neurologic findings. What is the most likely diagnosis? A Acute ischemic stroke B Circle of Willis ruptured aneurysm C Migraine headache D Multiple sclerosis E Pituitary adenoma

The Correct Answer is: E Pituitary adenomas, benign neoplasms associated with pituitary hormone secretory changes, may enlarge and become symptomatic. Symptoms are based upon the location and size of the tumor, and may include bitemporal hemianopsia, double vision, color desaturation, and visual acuity loss. Headaches may occur, due to associated pressure changes within the intrasellar space. Additional evaluation should include a T1-weighted MRI, screening laboratory tests, and a full ophthalmologic evaluation. These tests will also help evaluate for potential differential diagnoses, such as those listed. The patient's history is not consistent with an acute ischemic stroke or migraine headache. Although an unruptured aneurysm may have very similar findings to a pituitary tumor, ruptured aneurysms present with acute headache, nausea, vomiting, and potential changes in consciousness. Multiple sclerosis (MS) should remain on the differential for this patient and will also be evaluated through MRI (although the current findings are more consistent with a pituitary adenoma), and additional neurologic findings would be likely with MS. (Fauci et al., Harrison's Principles of Internal Medicine, 17e, Chapter 333, Disorders of the Anterior Pituitary and Hypothalamus)

Q 92.3: A 68-year-old female has been having increasing low back pain that radiates into her right hip and down her right leg. She has failed physical therapy and anti-inflammatories. She feels like the bones "are rubbing on each other." Radiographic imaging of the spinal cord, hip, and leg reveal several lytic lesions. The most helpful finding to confirm the diagnosis would include which of the following? A Reed-Sternberg cells in the marrow B Granulocytosis in the marrow C Macrocytosis in the peripheral smear D Monocytosis in the peripheral smear E Plasmacytosis in the marrow

The Correct Answer is: E Plasmacytosis in the marrow is the most prominent and diagnostically important finding. Reed-Sternberg cells are seen in Hodgkin's lymphoma. Granulocytosis is an increased white cell count seen in leukemias. Macrocytosis is enlargement of red cells, and can be seen in vitamin B12 deficiency. Monocytosis is an increase in monocytes, and can be seen in infectious mononucleosis or some leukemias. (Lichtman et al., Williams Hematology 8e, Chapter 109, Myeloma)

Q 64.10: A 57-year-old woman underwent a hemicolectomy for adenocarcinoma of the colon. Which of the following recommendations is part of postoperative monitoring for a potential recurrence? A Annual fecal occult blood testing B Annual chest radiograph C Annual CA 19-9 testing D Annual flexible sigmoidoscopy E Annual colonoscopy

The Correct Answer is: E Routine follow-up after surgical resection of a colon cancer includes annual colonoscopy not sigmoidoscopy, which only assesses the distal colon. The tumor marker for colon cancer is carcinoembryonic antigen (CEA) not carbohydrate antigen 19-9 (CA 19-9), which is used for pancreatic cancer. There is no role for annual chest films or fecal occult blood testing to monitor for a recurrence.

Q 114.6: A 68-year-old man is recently diagnosed with depression associated with the loss of his close sister to an automobile accident. He is currently taking oxybutynin for overactive bladder disease and lisinopril for hypertension. He has no known drug allergies. Which of the following medications would be most appropriate to prescribe for this patient? A alprazolam B amitriptyline C buspirone D desipramine E fluoxetine

The Correct Answer is: E Selective serotonin reuptake inhibitors (SSRIs) such as fluoxetine are usually considered first-line antidepressants due to their relative safeness in overdose and their minimal affinity for muscarinic, α-adrenergic, and histamine receptors, thereby causing fewer side effects. Tricyclic antidepressants such as amitriptyline and desipramine produce several adverse effects associated with their antimuscarinic properties (eg, dry mouth, constipation, blurred vision, urinary retention, etc). The patient is already taking the antimuscarinic agent oxybutynin, so a tricyclic antidepressant could attenuate these adverse effects. Orthostatic hypotension is also common with tricyclic antidepressants, and because the patient is taking lisinopril for hypertension, the risk for a significant drop in blood pressure is high. Buspirone and alprazolam are not indicated for depression. (Teter et al., 2008, pp. 1128-1129) Teter CJ , Kando JC , Wells BG , et al. , Depressive disorders. In: DiPiro JT , Talbert RL , Yee GC, et al., eds. Pharmacotherapy: A Pathophysiologic Approach. 7th ed. New York: McGraw-Hill; 2008.

Q 102.1: What is the most appropriate age to administer the pneumococcal vaccine in healthy individuals with no other comorbid diseases? A Under age 16 B After age 16-34 C > Age 35 D > Age 50 E > Age 65

The Correct Answer is: E The Advisory Committee on Immunization Practices of the Centers for Disease Control and Prevention recommend that the most appropriate age to administer the pneumococcal vaccine in healthy individuals with no other comorbid diseases is over the age of 65. (Musher D. Harrisons Online, Part 6, Infectious Disease, Section 6, Diseases Caused by Gram-Positive Bacteria)

Q 102.2: A 42-year-old female returns for a follow up of her hypothyroidism and admits to increased hair loss and dry skin. Her thyroid-stimulating hormone (TSH) is 20 mU/L. She is currently on levothyroxine 25 mcg daily which she has taken as directed for the past 6 months. What treatment do you recommend for this patient at this time? A change to methimazole B radioactive iodine therapy (131I therapy) C decrease the dose of levothyroxine to 12.5 mcg QD D add propylthiouracil E increase the dose of levothyroxine to 50 mcg QD

The Correct Answer is: E The patient's symptoms and elevated TSH suggests that she requires more thyroid hormone replacement. Increasing the dose to 50 mcg daily (by 25 mcg) is a reasonable treatment plan. Decreasing the dose (already very low) would likely cause worsening symptoms of hypothyroidism. Choices (A), (B), and (D) are treatments for hyperthyroidism. (McPhee and Papadakis, 2011, Chapter 26)

Q 118.3: A 58-year-old male is diagnosed with a gastrin-secreting gut neuroendocrine tumor. What is the medical term for this condition? A Ascending cholangitis B Celiac disease C Clostridium difficile colitis D Irritable bowel syndrome E Zollinger-Ellison syndrome

The Correct Answer is: E The question is referring to a gastrinoma, which is also called Zollinger-Ellison syndrome. Ascending cholangitis results from blockage of the common bile duct, which leads to inflammation and fibrosis. Celiac disease is a small bowel enteropathy due to the ingestion of gluten, which leads to malabsorption of nutrients. C. difficile colitis is an overgrowth of this bacteria, which results in diarrhea. The cause of irritable bowel syndrome may be due to abnormal motility, visceral hypersensitivity, or enteric infection, but is not due to a gastrin secreting tumor. (McPhee SJ, Papadakis MA. Current Medical Diagnosis & Treatment, 2010, p. 556)

Q 79.2: A 43-year-old male farmer is seen in the oncology clinic for his next round of chemotherapy for acute myelogenous leukemia. He complains of a persistent cough over the last three weeks, but has not run a fever at home. Today, his temperature is 37.4°C, HR is 67, RR is 18, and BP is 120/72. Lung examination is clear to auscultation bilaterally, no lymphadenopathy is noted, color is pale, there is good capillary refill, and the rest of the examination is negative. A CT is obtained, and a RUL wedge-shaped infiltrate at the periphery and a 1 cm cavitary lesion is noted. The best course of treatment would include which of the following? A Acyclovir B Gentamicin C Cefepime D Valacyclovir E Voriconazole

The Correct Answer is: E This patient is neutropenic following chemotherapy induction for the acute myelogenous leukemia. Likely infectious etiologies include gram negative and positive organisms, viral and fungal etiologies. A classic finding on CT is a cavitating lesion consistent with a fungal infection. The best treatment is voriconazole, an antifungal treatment. Acyclovir and valacyclovir are used for antiviral treatments. The others are antibiotics for gram negative organisms. (Lichtman et al., Williams Hematology 8e, Chapter 89, Acute Myelogenous Leukemia) (Lichtman et al., Williams Hematology 8e, Chapter 22, Treatment of Infections in the Immunocompromised Host)

Q 70.10: A 29-year-old woman presents in July to your office with symptoms of palpitations, sore neck, and excessive sweating, despite using her air conditioner all the time. No surgical or trauma history is noted. She is currently not taking any medications. Vitals include the following: BP = 124/68, pulse = 110 beats per minute, respirations = 18 per minute, and temperature = 101 o F orally. Upon exam, her thyroid is mildly enlarged without nodules, and severely tender. No local erythema or heat is noted. What is the most likely diagnosis in this patient? A Thyroid abscess B Papillary thyroid cancer C Hashimoto's thyroiditis D Graves' disease E Subacute thyroiditis

The Correct Answer is: E This patient is presenting with signs and symptoms of hyperthyroidism, most likely caused by subacute thyroiditis. The enlarged and tender thyroid, along with the fever, are classic findings in this condition. Choice A, thyroid abscess, is a localized infection of the thyroid. It would present with heat, swelling, and erythema in the area of tenderness. Choice B, papillary thyroid cancer, commonly presents with either a local non-tender thyroid nodule or as lymph node metastasis. Choice C, Hashimoto's thyroiditis, is a cause of chronic thyroiditis and is associated with a clinical presentation of hypothyroidism. Choice D, Graves' disease, is the most common cause of hyperthyroidism, but does not present with thyroid tenderness and fever. (Cooper et al., 2007, Chapter 8)

Q 81.9: A 52-year-old man with hypertension associated with recent unexplained weight loss presents with fever, malaise, and gradual onset of pain and weakness of his leg muscles for the past month. Physical examination reveals a mottled reticular pattern overlying portions of both calves and an area of ulceration with surrounding induration on the left lateral malleolus. Initial laboratory results reveal mild normochromic anemia, leukocytosis, and elevation of C-reactive protein, BUN, and creatinine. Which of the following is the most appropriate diagnostic evaluation to confirm the suspected diagnosis? A HLA-B27 typing B rheumatoid factor C MRI of sacroiliac joints D antinuclear antibodies test E tissue biopsy of area of induration

The Correct Answer is: E This patient most likely has polyarteritis nodosa (PN). A major obstacle in making the diagnosis is the absence of a disease-specific serological test. The diagnosis requires confirmation with either a tissue biopsy or angiogram. HLA-B27 antigens are not associated with the suspected diagnosis. While classic PN will have low titers of rheumatoid factor and antinuclear antibodies, both are nonspecific findings and will not confirm the diagnosis. An MRI of the sacroiliac joints is indicated in evaluation of the early stages of suspected ankylosing spondylitis and plays no role in the evaluation of PN. (Hellmann and Imboden, 2008, pp. 738-739) Hellmann DB , Imboden JB. Arthritis and musculoskeletal disorders. In: Tierney LM , McPhee SJ , Papadakis MA, eds. 2008 Current Medical Diagnosis and Treatment. New York, NY: McGraw-Hill; 2008:703-756.

Q 107.6: A 28-year-old female presents complaining of progressive vision loss and mild eye pain in the right eye. She states that her vision is most impacted centrally. She denies any injury, chemical exposure, contact lens use, recent illness, or history of similar symptoms. She has had some diplopia and dysequilibrium for the past two weeks. Her visual acuity reveals 20/50 in her right eye, 20/20 in her left eye, and 20/30 together. She has no conjunctival, scleral, or fundoscopic changes. Her intraocular pressure is 14 bilaterally. What is the most likely diagnosis? A Conjunctivitis B Glaucoma C Keratitis D Macular degeneration E Optic neuritis

The Correct Answer is: E This patient presents with classic symptoms for optic neuritis. Symptoms may include sudden or progressive unilateral vision loss, often with central field defect (scotoma). Pain may be present with eye movement. Vision changes will vary in severity, based on the level of inflammation. The majority of optic neuritis cases are retrobulbar, resulting in a normal fundoscopic exam. There is often a loss of color differentiation, and the affected eye may have decreased papillary light reactivity. The patient also exhibits additional symptoms that are suggestive of possible demyelinating disease (such as Multiple Sclerosis), which are strongly associated with optic neuritis. Conjunctivitis would have associated conjunctival injection, a clear cornea, no visual acuity changes, and would likely be associated with allergic, viral, or bacterial symptomology. Acute glaucoma would likely be associated with marked vision loss, headache, a fixed pupil, corneal cloudiness, and conjunctival injection. In addition, the intraocular pressure would be increased. Keratitis often presents with vision loss, pain, photophobia, conjunctival injection, stromal infiltrate, and corneal disruption. Macular degeneration develops gradually with increasing age, causing bilateral central visual loss. (McPhee SJ, Papadakis MA. Current Medical Diagnosis & Treatment 2011, Chapter 7, Disorders of the Eyes & Lids)

Q 67.8: A 52-year-old male, who is a known alcoholic, is admitted to the hospital for GI bleed. Librium is ordered, but he must be monitored for early signs of alcohol withdrawal as his total daily intake of alcohol prior to admission is unknown. Which of the following symptoms might occur as an early sign of withdrawal rather than a later sign? A Delusions B Hallucinations C Nausea and vomiting D Seizures E Tremulousness

The Correct Answer is: E Tremulousness (E) and agitation are early signs of alcohol withdrawal, occurring within the first few hours. These symptoms can progress to nausea and vomiting (C) along with hallucinations (B), delusions (A), and seizures (D) after several hours. Seizures generally occur 12-48 hours after the last drink and full delirium tremens occurs after 48-72 hours. Martin, PR. Chapter 15. Substance-Related Disorders. In: Ebert MH, Loosen PT, Nurcombe B, Leckman JF, eds. CURRENT Diagnosis & Treatment: Psychiatry. 2 nd ed. New York: McGraw-Hill; 2008. http://accessmedicine.com/content.aspx?aid=3283393 .

Q 76.6: A patient being treated for Burkitt's lymphoma is found to have hyperkalemia, hyperphosphatemia, hypocalcemia, a metabolic acidosis, hyperuricemia, and appears to be going into renal failure after the first dose of chemotherapy. These are all classic signs of which of the following? A Chemo-reactive disease B Chemotherapeutic anaphylaxis C Pseudohyperkalemia syndrome D Refeeding syndrome E Tumor lysis syndrome

The Correct Answer is: E Tumor lysis syndrome is a serious metabolic complication of rapidly growing tumors. Burkitt's is a classic example of a rapidly growing tumor that can cause this syndrome, as the tumor is broken down by the chemotherapeutic treatment. Chemo-reactive disease means that the tumor responds to chemotherapy. Anaphylaxis is an immune mediated response to any allergen producing substance. There is no diagnosis called Pseudohyperkalemia syndrome. Refeeding syndrome is caused by patients that haven't eaten for an extended period and are then provided parentral nutrition, displacing electrolytes and causing other metabolic complications. (Lichtman et al., Williams Hematology 8e, Chapter 104, Burkitt Lymphoma)

Q 71.9: A 38-year-old woman diagnosed with pancreatic cancer 2 months ago develops jaundice and steatorrhea. Which of the following is most likely to be diminished in the blood? A calcium B iron C vitamin B12 D vitamin C E vitamin K

The Correct Answer is: E Tumors in the pancreatic head region can often block the flow of bile from the gall bladder and liver to the duodenum, resulting in jaundice and steatorrhea. The bile salts are important for micelle formation within the lumen of the small intestine. Micelles provide a mechanism whereby the hydrophobic products of lipid digestion as well as fat-soluble vitamins (eg, A, D, E, and K) can be absorbed in the small intestine. (Costanzo, 2006, pp. 368-369; Rugo, 2008, pp. 1420-1421) Costanzo LS. Physiology. 3rd ed. Philadelphia, PA: Saunders Elsevier; 2006. Rugo HS. Cancer. In: McPhee SJ , Papadakis MA, eds. Current Medical Diagnosis & Treatment. New York, NY: McGraw-Hill Medical; 2008.

Q 92.2: The risk of extrapyramidal side effects (pseudoparkinsonism) and tardive dyskinesia is associated with which class of medications? A amphetamines B benzodiazepines C monoamine oxidase inhibitors (MAOIs) D tricyclic antidepressants (TCAs) E typical (first-generation) antipsychotics

The Correct Answer is: E Typical antipsychotics (eg, haloperidol, chlorpromazine, fluphenazine) can produce pseudoparkinsonism via blockade of dopamine (D 2 ) receptors in the nigrostriatum. Symptoms can include akinesia, bradykinesia, mask-like facial expression, tremor, cogwheel rigidity, and postural abnormalities. Tardive dyskinesia may also occur, as the reported incidence with first generation antipsychotics ranges from 0.5% to 62%. (Carson et al., 2008, pp. 1111-1113) Carson ML , Argo TR , Buckley PF. Schizophrenia. In: DiPiro JT , Talbert RL , Yee GC, et al., eds. Pharmacotherapy: A Pathophysiologic Approach. 7th ed. New York: McGraw-Hill; 2008.

Q 109.6: A patient with chronic cluster headaches is on verapamil for preventive therapy. Which of the following is a potentially harmful complication of this medication? A Cardiovascular thrombotic events B Coronary artery vasospasm C Hyperkalemia D Increased neuromuscular transmission E Slowed conduction at the AV node leading to heart block

The Correct Answer is: E Verapamil, a calcium channel blocker, has been shown to benefit patients with chronic cluster headaches. The patient should have a baseline EKG and EKG monitoring while on this medication, due to an increased risk of developing heart block, which is based on the medication's effect on the atrioventricular node. The other complications are not consistent with this medication. (Fauci et al., Harrison's Principles of Internal Medicine, 17e, Chapter 15, Headache)

Q 89.5: A 24-year-old female patient develops her second deep vein thrombosis (DVT) in two years. After a hypercoaguability work up, you discover that she has a factor V Leiden abnormality. How long should she remain on oral anticoagulation therapy? A She does not need anticoagulation therapy. B You may stop her anticoagulation therapy when her current DVT has resolved. C You may stop her anticoagulation therapy in six months. D You may stop her anticoagulation therapy in one year. E She needs lifetime anticoagulation therapy.

The Correct Answer is: E When patients have a deep vein thrombosis, it usually warrants a hypercoaguability work up to rule out genetic causes. While complicated and sometimes requiring hematology consultation, patients with hypercoaguable genetic disorders such as a factor V Leiden abnormality generally require lifetime anticoagulation. (Lichtman, et al., 2010, Chapter 131)

Q 119.20: A 64-year-old woman with a medical history of rheumatoid arthritis and deteriorating vision presents to the clinic with complaints of painful bilateral swelling of her ankles and hands, morning stiffness, loss of appetite, and fatigue. She is currently taking naproxen sodium 500 mg twice per day. Which medication(s) would be most appropriate for this patient? A acetaminophen B azathioprine C cyclosporine D hydroxychloroquine E methotrexate

The Correct Answer is: E Whereas NSAIDs such as naproxen provide some symptomatic relief in rheumatoid arthritis, they do not alter disease progression like DMARDs (disease-modifying antirheumatic drugs). NSAIDs are best used in conjunction with DMARDs. Methotrexate is usually the DMARD of choice because it is well tolerated by the majority of patients and can produce beneficial effects in 2 to 6 weeks. Hydroxychloroquine is another DMARD that can be used for rheumatoid arthritis but can produce ocular toxicity. Cyclosporine and azathioprine are used less frequently today due to toxicity and lack of long-term benefits. Similar to naproxen, acetaminophen would also fail to alter disease progression. (Schuna, 2008 pp. 1512-1513; Rabow and Pantilat, 2008, pp. 69, 78) Schuna AA. Rheumatoid arthritis. In: DiPiro JT , Talbert RL , Yee GC, et al., eds. Pharmacotherapy: A Pathophysiologic Approach. 7th ed. New York: McGraw-Hill; 2008. Rabow MW , Pantilat SZ. Palliative care & pain management. In: Tierney LM Jr , McPhee SJ , Papadakis MA, eds. Current Medical Diagnosis & Treatment. 47th ed. New York: McGraw-Hill; 2008.

Q 91.8: A 41-year-old male appears pale and mildly icteric. He complains of increasing fatigue lasting several weeks. Laboratory findings note an elevated MCV of 133. What other physical finding would most support a diagnosis of megaloblastic anemia? A Anorexia B Dementia C Diarrhea D Glossitis E Paresthesias

The Correct Answer is: E While all choices may be clinical findings in a vitamin B12 deficiency, paresthesias most supports the diagnosis. Peripheral nerves are usually affected first and patients initially complain of paresthesias. The other findings are later in the disease process or appear with very advanced anemia. (McPhee SJ, Papadakis MA, Tierney LM. Current Medical Diagnosis and Treatment, 2010, Chapter 13, Blood Disorders)

Q 68.6: A 32-year-old male presents with odynophagia, dysphagia, and chest pain. His past medical history consists of him being HIV positive. He is currently not taking any medications, as he cannot afford to pay for them. An endoscopy is ordered, and the results show several shallow, superficial ulcerations. What is the most likely diagnosis? A Cytomegalovirus esophagitis B Gastroesophageal reflux disease C Candidal esophagitis D Barrett's esophagus E Herpetic esophagitis

The correct answer is (A). Cytomegalovirus esophagitis can occur in patients with HIV. Gastroesophageal reflux disease typically presents with heartburn and regurgitation, and an endoscopy may be normal or show erosions in the distal esophagus at the squamocolumnar junction. Candidal esophagitis appears as diffuse, linear, yellow-white plaques adherent to the mucosa on endoscopy. Barrett's esophagus appears as orange, gastric-type epithelium that extends upward from the stomach, into the distal tubular esophagus, in a tongue-like or circumferential fashion on endoscopy. Herpetic esophagitis appears as multiple small, deep ulcerations on endoscopy.

Q 102.9: A patient with severe hemophilia A is brought into the emergency department following an automobile accident. Concern is for hemorrhage. What is the best intervention to raise factor VIII levels to hemostatic levels? A Fresh frozen plasma B Cryoprecipitate C Recombinate D DDAVP (desmopressin) E EACA (aminocaproic acid)

The correct answer is (E). While fresh frozen plasma and cryoprecipitate have been used in the past for hemophilia A hemostatis, volumes may be too large or unable to reach levels to achieve hemostasis in severe hemophilic A patients. Recombinate is a commercial lyophilized factor VIII concentrate that can case factor VIII levels to reach hemostasis in smaller volumes, and do not have the disadvantages of plasma or cryopreciptate. DDAVP is not used in severe hemophilics. EACA is used to enhance hemostasis, but is not able to achieve initial hemostasis seen with recombinate therapy.

Q 70.5: A patient presents with a several year history of developing coarse facial features, prognathism, widely spaced teeth, and sleep apnea. Upon exam, you also note oily skin and high blood pressure. What is the most likely cause of this patient's symptoms? A Pituitary adenoma B Pheochromocytoma C GHRH secreting hypothalamic tumor D Adrenal carcinoma E Ectopic ACTH secretion

The correct choice is A, pituitary adenoma. The patient is presenting with classic signs and symptoms of acromegaly. This disorder is caused most commonly by a growth hormone secreting pituitary adenoma. On rare occasions, it has been caused by choice C, a GHRH secreting hypothalamic tumor. Choice B, pheochromocytoma, is a tumor of the adrenal glands that causes high blood pressure, but does not cause the classic body features of acromegaly. Choice D, adrenal carcinoma, and choice E, ectopic ACTH secretion, may cause Cushing's syndrome, but not acromegaly. (Aron et al., 2007, Chapter 5)

Q 59.1: A 40-year-old female is status post a dilatation and curettage for hydatidiform mole. On week 3 post surgery, her follow-up quantitative hCG level has elevated slightly. What is the most likely diagnosis? A Adenocarcinoma of the ovary B Adenocarcinoma of the uterus C Retained hydatidiform mole D Choriocarcinoma E Corpus luteum cyst

The Correct Answer is: D 5% of hydatidiform mole progress to choriocarcinoma; the longer the mole in intrauterine the higher the risk. Pre-surgical evaluation for mole removal includes a chest x-ray to rule out distant metastasis. hCg that either plateuas or elevates is choriocarcinoma until proven otherwise, and requires prompt evaluation.

Q 62.3: A 37-year-old woman under your care is diagnosed with bipolar I disorder. As part of her drug regimen, you prescribe lithium carbonate as long-term maintenance therapy. Which of the following would be most appropriate to perform or order prior to the initiation of lithium carbonate? A electrocardiogram B fasting plasma glucose C liver function tests D serum creatinine E urine culture

The Correct Answer is: D Patients on chronic lithium carbonate therapy have an approximate 10% to 20% risk of developing renal problems such as glomerulosclerosis, tubular atrophy, or interstitial nephritis. Each of these conditions can lead to filtration problems and a subsequent rise in serum creatinine. Hence, it is advised to obtain a baseline serum creatinine prior to administering lithium carbonate to follow any changes that may occur in renal function during therapy. It is also advised that lithium carbonate be avoided in patients with pre-existing renal disease.

Q 62.9: A 68-year-old male presents with the complaint of palpitations in the center of his chest over the last few hours. The symptoms come and go, and last anywhere from 30 seconds to a few minutes. There is some associated lightheadedness with these episodes, and occasionally there is mild shortness of breath. While examining the patient, he has another episode. During this time, the patient's vital signs reveal a pulse of 170 and a blood pressure of 118/69. Based on this history and the findings on the exam and rhythm strip shown, what is the best treatment for this patient? A Diltiazem B Digoxin C Lisinopril D Metoprolol E Amiodarone

The Correct Answer is: E This patient is having non-sustained ventricular tachycardia (NSVT). Based on the clinical presentation, this patient can benefit from an infusion of amiodarone. The dose is normally a 150mg bolus, followed by a drip of 1mg/min for the first 6 hours, then followed by 0.5mg/min for the next 18 to 24 hours. The other agents would not be able to maintain a rhythm for this type of abnormality.

Q 56.6: A 42-year-old female has an endoscopy performed, which shows a smooth, sessile nodule with normal overlying mucosa present in the esophagus. A biopsy is performed and the results show a benign esophageal neoplasm. What is the most likely diagnosis? A Leiomyoma B Zenker diverticulum C Esophageal web D Leiomyosarcoma E Barrett's esophagus

A The correct answer is leiomyoma, which is the most common benign esophageal neoplasm. Zenker diverticulum is a protrusion of pharyngeal mucosa, which develops at the pharyngoesophageal junction, between the inferior pharyngeal constrictor and the cricopharyngeus. An esophageal web would appear as a thin, diaphragm-like membrane of squamous mucosa. Leiomyosarcoma is a malignant tumor. Barrett's esophagus would appear as orange, gastric-type epithelium that extends upward from the stomach, into the distal tubular esophagus, in a tongue-like or circumferential fashion.

Q 58.8: A 38-year-old male complains of increasing fatigue, weakness, weight loss, and intermittent nausea, vomiting, and diarrhea over the past few months. He has noted some agitation at times. When this first started he thought that this was related to a virus but the GI symptoms have reoccurred on multiple occasions. Labs show a complete blood count (CBC) within the normal reference range. He is noted to have hyponatremia. On examination you note the multiple areas of hyperpigmentation as seen below. His blood pressure in the office is 100/50, P = 66, T = 97.1˚F. What test would you order to confirm your suspected diagnosis? Source: (Wolff and Johnson, 2009, Section 15) A dexamethasone suppression test B vasopressin challenge test C radioactive iodine uptake scan D cosyntropin stimulation test E follicular stimulation test

D The correct answer is (D). The patient's symptoms and examination findings are consistent with a diagnosis of Addison's disease, which is most likely due to an autoimmune process that destroys the adrenal glands resulting in a chronic adrenal insufficiency. The cosyntropin (ACTH) stimulation test should reveal a low am cortisol level and an elevated ACTH level if he has Addison's disease. The dexamethasone suppression test, choice (A), is a laboratory test for Cushing's syndrome. The vasopressin challenge test, choice (B), is a laboratory test for diagnosis of diabetes insipidus. A radioactive iodine uptake scan, choice (C), is used in the diagnosis of thyroid disease (hyperthyroidism and thyroid nodules). A follicular stimulation test, choice (E), is a factitious test.

Q 61.6: What is the most common heart sound that is heard in patients with pulmonary hypertension? A S3 gallop B S2 Split C S4 D Normal S1S2 E Regularly irregular rhythm

The Correct Answer is: B The splitting on the second heart sound is also accompanied by the louder second sound of a split of the pulmonary component. There should not be a change in the rhythm with the patient due to pure pulmonary hypertension.

Q 60.2: Which of the following medications, used for the treatment of Tourette syndrome, has been associated with long-term extrapyramidal side effects? A Clonazepam B Clonidine C Haloperidal D Metoclopramide E Risperidone

The Correct Answer is: C Haldol, a typical antipsychotic agent, has been shown to be effective for symptomatic treatment of patients with Tourette syndrome. Haldol blocks dopaminergic action and decreases psychomotor agitation. It is linked to a high frequency of extrapyramidal side effects, likely due to this action. It also has sedative properties within the limbic system. Other medications, such as atypical psychotics, benzodiazepines, alpha-2 agonists, and dopamine-blockers have been used for Tourette management. Each medication should be monitored for effectiveness and side effects, as other medications can cause extrapyramidal side effects, such as metoclopramide.

Q 54.5: What is the most common site of an acute arterial occlusion due to embolic disease? A Iliac artery B Aortic bifurcation C Mesenteric arteries D Femoral artery E Popliteal artery

The Correct Answer is: D The most common site for an acute embolic occlusion is the femoral artery. Other common sites include the axillary, popliteal, and iliac arteries as well as the aortic bifurcation and mesenteric vessels. The majority (80%) of arterial embolic originate in the heart in patients with atrial fibrillation or from mural thrombi in the left ventricle from an akinetic or dyskinetic portion of the myocardium following a myocardial infarction.

Q 61.4: A patient was recently diagnosed with acromegaly. He was reading on the Internet that acromegaly is associated with diabetes mellitus. He is especially concerned, since his father was recently diagnosed with diabetes as well. How would you reply to the patient's following question: "What percentage of people with acromegaly develop diabetes mellitus?" A 10% B 25% C 50% D 65% E 80%

B The correct choice is B, 25%. Growth hormone is a counter-regulatory hormone of insulin, and therefore acts against insulin. This can lead to hyperglycemia, glucose intolerance, and diabetes mellitus in 25% of patients with acromegaly.

Q 57.6: A 64-year-old male presents with fatigue, pallor, and dyspnea on exertion. He has noted this gradual onset over the last several months. His initial reason for coming in was due to new papules and nodules that he noted on his skin, with some violaceous in color. On exam you also note petechiae to the lower extremities and splenomegaly. On questioning him further, he notes feeling full sooner than he had in the past. On laboratory you note a pancytopenia. What is the most likely diagnosis? A Acute myelogenous leukemia with leukemia cutis B Hereditary Spherocytosis with drug eruption C Lymphoma with erythema nodosum D Pyoderma Gangrenosum E Sarcoidosis

The Correct Answer is: A The most likely diagnosis is AML with leukemia cutis. Leukemia cutis is seen in a subset of AML patients that are nontender, and are infiltrates of leukemic cells into the dermis. Hereditary spherocytosis is a hemolytic anemia, and does not present with papules and nodules or pancytopenia. Lymphoma does not typically present with a pancytopenia or skin rash. Pyoderma gangrenosum is an uncommon ulcerative cutaneous condition of uncertain etiology. Sarcoidosis is a chronic noncaseating granulomatous disease of unknown etiology, which affects many organs and tissues, most commonly the lungs.

Q 60.3: You are monitoring a 30-year-old G2P1 at 40 weeks gestation, who is in an active stage of labor and is 6-cm dilated. The fetal heart tracing has a baseline heart rate of 140, with 7 to 10 beats of variability. With the last five contractions you have noted late decelerations. What would be the next most appropriate course of action? A Close observation of FHR tracing B Assessment of dilatation C Augment contractions with oxytocin D Intravenous analgesic E Surgical intervention

The Correct Answer is: B The presence of recurrent late decelerations should raise the suspicion for fetal distress. Vaginal evaluation for change in dilatation or cord prolapse, and to assess the fetal response to stimulation, are the first steps in evaluating the need for intervention.

Q 62.6: What type of pharmacological agent would be used as a first-line medication to treat obsessive compulsive disorder? A risperidone B lonazepam C fluvoxamine D trazadone E venlafaxine

The Correct Answer is: C Fluvoxamine, paroxetine, and sertraline are all approved for the treatment of obsessive compulsive disorder. Use of an SSRI in combination with behavioral therapy is recommended.

Q 55.2: A patient who is an Ashkenazi Jew and is homozygous for Factor XI deficiency requires abdominal surgery. Which hemostatic laboratory testing outcomes would you expect to find? A Decreased platelets, normal aPTT, prolonged PT B Decreased platelets, prolonged aPTT, normal PT C Normal platelets, normal aPTT, prolonged PT D Normal platelets, prolonged aPTT, normal PT E Normal platelets, prolonged aPTT, prolonged PT

The Correct Answer is: D Patients with factor XI deficiency have a prolonged aPTT, normal PT, and normal platelets count.

Q 57.7: You are evaluating a urinalysis on a 44-year-old female due to complaints of recurrent urinary tract infections (UTIs). Which of the following microscopic findings would be most suggestive of chronic kidney disease? A red blood cell (RBC) casts B hyaline casts C white blood cell (WBC) casts D broad waxy casts E granular casts

D The correct answer is (D). Broad waxy casts are suggestive of chronic kidney disease. RBC casts suggest glomerulonephritis. Hyaline casts may occur with heavy exercise or febrile illness. WBC casts suggest infection or inflammation such as in pyelonephritis. Granular casts, although nonspecific, may suggest acute tubular necrosis.

Q 60.7: A 74-year-old woman presented with the new onset of seizures. An MRI with gadolinium showed a parasagittal mass with homogenous enhancement and a "dural tail." What is the appropriate management of this patient? A Radiation therapy B Chemotherapy C Surgical resection D Surgical resection with chemotherapy

The Correct Answer is: C The clinical presentation is consistent with a meningioma. Meningiomas are commonly located in the parasagittal region, the convexity of the brain, sphenoid ridge, or posterior fossa. Radiographic features on MRI include homogenous enhancement and evidence of a "dural tail" indicating the origin of the tumor. Since meningiomas are a benign tumor, the primary treatment is surgical removal. In the event of a subtotal resection or if the meningioma is found to be malignant, surgical resection is followed by radiation therapy.

Q 56.9: A 22-year-old woman with a history of scoliosis presents to the office with complaints of retrosternal chest discomfort, occurring frequently at rest and lasting for several hours at a time. She is currently experiencing this chest discomfort during the office visit, but has never experienced this while working out three days per week. On physical exam, a mid-systolic click is noted. With standing, the click moves closer to S1. An EKG demonstrates normal sinus rhythm at 76 bpm, with no acute ST or T wave changes. A transthoracic echocardiogram reveals mild prolapse of the posterior leaflet of the mitral valve. Which of the following would be the most appropriate next step in the management of this patient? A Reassurance and monitoring with periodic transthoracic echocardiogram B Cardiac catheterization C Infectious endocarditis prophylaxis D Mitral valve replacement E Transesophageal echocardiogram

The Correct Answer is: A Choice A, reassurance and monitoring with periodic transthoracic echocardiogram, is the most appropriate choice given the patient's findings on echocardiogram. Most patients with mitral valve prolapse are asymptomatic, and do not demonstrate significant progression of their valvulopathy over their lifetime. Periodic transthoracic echocardiogram allows a noninvasive, highly sensitive method of monitoring. Choice B, cardiac catheterization, is useful for evaluation of coronary artery anatomy and for evaluation of valvulopathy; however, it is invasive, and usually reserved for investigation of serious valvular dysfunction, and/or following a stress test suggestive of myocardial ischemia. The chest pain experienced by patients with mitral valve prolapse is varied in presentation, and in this setting, with a young patient with no cardiac risk factors, unlikely to be secondary to coronary artery disease. According to the American Heart Association's most recent guidelines, patients with mitral valve prolapse alone do not require infectious endocarditis prophylaxis, so choice C is inappropriate. Choice D, mitral valve replacement, is only indicated with severe mitral valve prolapse, resulting in severe mitral regurgitation. Choice E, transesophageal echocardiogram, while an excellent diagnostic tool for the evaluation of mitral valve disease, would be more invasive than monitoring via transthoracic echocardiogram, and thus would not be an appropriate choice in this patient with only mild mitral valve prolapse.

Q 56.4: Which of the following etiologies is the most common cause of nontraumatic subarachnoid hemorrhage (SAH)? A Cerebral aneurysm B Poorly controlled hypertension C Anticoagulant use D Arteriovenous malformation

The Correct Answer is: A Congenital cerebral aneurysms or Berry aneurysms account for 75% to 80% of nontraumatic subarachnoid hemorrhages (SAHs). Poorly controlled hypertension and anticoagulant use are more commonly associated with intracerebral hemorrhages (ICH). AVMs can cause either SAH or ICH.

Q 61.1: A 38-year-old thin, HIV+ male is found to have a distended abdomen and is complaining of a feeling of fullness. A CT scan of the abdomen revealed a large abdominal mass with ascites in the retroperitoneal region. Laboratories found an elevated EBV of 5400 copies (normal=0), LDH 554 (100 to 250), and Ig-positive MYC translocation. What is the most likely diagnosis? A Burkitt lymphoma B Cat scratch disease C Hodgkin's lymphoma D Infectious mononucleosis E Sarcoidosis

The Correct Answer is: A In immunosuppressed individuals, including those who are HIV+ and in the presence of highly reactive EBV, the development of an abdominal mass with elevated LDH is most likely Burkitt's lymphoma. Of the masses that are found, 30 to 40% will be positive for EBV, and it appears that this virus drives the conversion of cells to a cancerous state. Infectious mononucleosis would be positive for EBV, but does not develop large abdominal masses.

Q 61.8: A 66-year-old female has a chief complaint of vision loss in her left eye. She denies pain and states that this occurred over the past few hours. Her past medical history includes hypertension, high cholesterol, and peripheral vascular disease. Upon funduscopic exam, you note marked hemorrhages in all quadrants and disc edema. The contralateral eye shows only mild hypertensive vascular changes. What is your diagnosis? A Macular degeneration B Retinal detachment C Central retinal artery occlusion D Cerebrovascular accident E Central retinal vein occlusion

The Correct Answer is: E A central retinal vein occlusion is characterized by a "blood and thunder" fundus, with marked hemorrhages, tortuous vessels, and optic disc edema.

Q 60.4: A G4P4 woman delivers a viable infant at 38 weeks gestation by normal spontaneous vaginal delivery. The infant has apgars of 7 and 8 (at 1 and 5 minutes respectively). What is the most crucial time for maternal and fetal physiologic changes to occur? A 30 minutes prior to birth B 60 minutes prior to birth C Birth D 30 minutes after birth E 60 minutes after birth

The Correct Answer is: E The hour after birth is the most critical time for physiologic changes to occur, including maternal fluid shifts, hemorrhage, retained placenta, and fetal lung cardiovascular transitions.

Q 58.9: A 22-year old female presents to your office complaining of itchy red welts all over her body now fading. She is vacationing from Florida. She has no past medical history and her only medication is an oral contraceptive, but she did take an over-the-counter dipenhydramine four days ago on the flight from Florida to calm her nerves. The welts began after swimming in the ocean in New England three days ago, lasted a few hours, then disappeared spontaneously. They reoccurred Saturday morning again shortly after swimming, lasted a little longer, and again resolved. She relates that they were intensely itchy, red, and raised. She ate out at a restaurant and had seafood Saturday night, and thought that she might be allergic to the seafood, although she ate nothing new or unusual. From the history, which of the following is the likely cause of her urticaria? A atopic dermatitis B cold urticaria C contact dermatitis from jellyfish stings D food allergy E medication allergy

The Correct Answer is: B Cold urticaria is a hypersensitivity to cold exposure (ie, wind, freezer compartments, water) resulting in histamine release. The hypersensitivity usually presents as localized redness, burning, pruritus, and urticaria in the exposed areas, or the response may progress to generalized systemic reaction, shock, and death. This condition may be familial or acquired. Familial cold urticaria is an autosomal dominant inflammatory disorder (including the Muckle-Wells syndrome), manifested as a burning sensation of the skin occurring about 30 minutes after exposure to cold. Acquired cold urticaria may be associated with medication (ie, griseofulvin) or with infection. Cold urticaria may occur secondarily to cryoglobulinemia or as a complication of syphilis. Most cases of acquired cold urticaria are idiopathic. For diagnosis, an ice cube is usually applied to the skin of the forearm for 4 to 5 minutes, then removed, and the area is observed for 10 minutes. As the skin rewarms, an urticarial wheal appears at the site that may be accompanied by itching. Second-generation antihistamines have been used as first-line treatment. Ebastine is also reported to safely and effectively prevent symptoms from acquired cold urticaria. Use of antileukotrienes in cold urticaria is anecdotal.

Q 60.6: Which of the following is an absolute contraindication to acute thrombolysis, with a recombinant tissue plasminogen activator (tPA) in an ischemic stroke patient? A Platelet count of 120,000/mm3 B Ischemic stroke two months ago C Symptoms for three hours D Current daily low dose aspirin regimen E Systolic blood pressure of 170mmHg

The Correct Answer is: B The role of thrombolysis with recombinant tPA (tissue plasminogen activator) during acute ischemic stroke has been clearly defined. In select patients without contraindications, the use of tPA has been associated with decreased mortality, decreased disability, and improved clinical outcome. However, tPA has defined contraindications to use

Q 54.11: Which of the following foodborne illnesses is most likely to be acquired through eating raw oysters? A salmonellosis B shigellosis C cholera D giardia E hookworms

The Correct Answer is: C Cholera infection is most often caused by Vibrio cholerae, Vibrio vulnificus, or Vibrio parahaemolyticus. Although cholera infection can be transmitted through wounds, the most often reported cause is eating undercooked shellfish. The clinical picture most often includes a watery diarrhea that can lead to dehydration. In the immune compromised host, overwhelming sepsis is possible. It is most often treated with doxycycline plus a third-generation cephalosporin or by a fluoroquinolone alone. Salmonella infection is associated with consumption of raw eggs and undercooked chicken or beef. Shigella is transmitted by the fecal-oral route, often because of poor hygiene. Giardia is waterborne and hookworms are found in the soil.

Q 59.5: You are providing care to a woman who is at 33 weeks gestation. Her pregnancy is complicated by gestational diabetes. She is being provided education by the dietician, and has weekly obstetrical appointments. What fasting blood sugar (FBS) readings should necessitate switching from diet control to insulin therapy? A FBS > 70 mg/dL B FBS > 90 mg/dL C FBS > 95 mg/dL D FBS > 110 mg/dL E FBS > 126 mg/dL

The Correct Answer is: C Gestational diabetes has different risks associated with it, in particular for the fetus. Stricter glycemic controls are recommended for pregnant versus non pregnant women by the ACOG and the ADA.

Q 54.14: A 42-year-old woman, with a history of struvite renal calculus, calls the office with a complaint of a urinary tract infection. As part of the interview, she reports intermittent, mild right flank pain for 4 days. Her urine dipstick is positive for microscopic hematuria, and the urine pH is 7.5. The KUB film is positive with two visible stones in the right kidney. Which of the following organisms is most likely to be cultured from the urine specimen? A Escherichia coli B Klebsiella C Proteus D Chlamydia trachomatis

The Correct Answer is: C This patient has struvite stones. They are frequently associated with recurrent urinary tract infections, visible stones, and high urine pH. These stones are formed by urease-producing organisms including Proteus and Pseudomonas while being caused less commonly by Klebsiella. Struvite stones are not typically caused by E. coli and C. trachomatis.

Q 61.3: A 45-year-old man with a history of neck irradiation for Hodgkin lymphoma at the age of 15 is found to have a 1.5-cm, nontender, firm thyroid nodule. Upon laboratory evaluation, the patient is found to be euthyroid, and fine needle biopsy reveals malignancy. What histologic type is most likely? A anaplastic B follicular C medullary D papillary

The Correct Answer is: D Thyroid carcinoma often presents as an asymptomatic thyroid nodule. The most common histologic form is papillary carcinoma, representing more than 80% of cases.

Q 55.7: You have determined that a multiple sclerosis patient with relapsing-remitting disease has not returned to her baseline functional status with her most recent exacerbations, despite her returning to baseline with exacerbations for many years. Her exam also reveals residual neurologic dysfunction. Which of the following multiple sclerosis clinical types would you classify this patient within? A Clinically isolated syndrome B Primary progressive C Progressive/relapsing D Relapsing/remitting E Secondary progressive

The Correct Answer is: E This patient would be classified within the secondary progressive clinical type. Secondary progressive MS follows a relapsing/remitting course initially, during which the patient returns to baseline neurologically and functionally between exacerbations, then evolves into a course of the patient having deterioration in function and neurologic status, which is not associated with acute exacerbations. The patient is also noted to not return to baseline following acute exacerbations. Primary progressive MS is associated with a decline in functional and neurologic status from the onset of disease, and disability is common. Progressive/relapsing MS is associated with progressive neurologic and functional decline from the onset of disease, but with associated exacerbations during the clinical course as well. The designation of clinically isolated syndrome indicates that a patient has had a single clinical event suggestive of MS but does not meet MS diagnostic criteria, thus requiring a waiting period and additional studies.

Q 58.4: Patients prescribed which of the following medications may present with a dorsocervical fat pad, thin extremities, and central obesity, which may mimic Cushing's syndrome? A ACE inhibitors B HIV antiretrovial medications C Loop diuretics D Opioid analgesics E Aminoglycosides

B The correct choice is B, HIV antiretroviral medications. Patients using highly active antiretroviral therapy (HAART) can develop partial lipodystrophy and changes in body fat distribution. This complication occurs more commonly with long-term use, and affects 1/3 to 2/3 of patients on therapy for longer than one year. The other medication classes noted are not associated with lipodystrophy. Opioids, choice D, are associated with a decline of cortisol secretion.

Q 62.4: A 62-year-old man with a history of hypertension, diabetes mellitus type 2, hyperlipidemia, and chronic tobacco use presents to the office with complaints of a retrosternal chest pressure radiating down his left arm, associated with diaphoresis, nausea, and dyspnea, for the last 45 minutes after mowing his lawn. The patient's vital signs are stable, and on physical examination a new systolic murmur is appreciated. According to the most recent American College of Cardiology/American Heart Association Guidelines for the Management of Patients with Unstable Angina/Non-ST-Elevation MI recommendations, an EKG should be performed on patients with a clinical suspicion for acute coronary syndrome within how many minutes of their arrival to the emergency department? A 1 minute B 5 minutes C 10 minutes D 15 minutes E 20 minutes.

C Choice C is correct, as the 2007 American College of Cardiology/American Heart Association Guidelines for Management of Patients with Unstable Angina/Non-ST-Elevation MI recommend that an ECG "be performed and shown to an experienced emergency physician as soon as possible after ED arrival, with a goal of within 10 minutes of ED arrival for all patients with chest discomfort or other symptoms suggestive of ACS." Although choices A and B would be optimal, the question asks for the goal time for which busy emergency departments should aim for in obtaining an EKG in at-risk patients. Choices D and E are less desirable, as times greater than 10 minutes increase both morbidity and mortality rates.

Q 57.8: A young man presents with difficulty breathing at times. Upon exam you note evidence of a firm, fixed thyroid nodule with extension toward the trachea and surrounding muscles. The patient has a family history of thyroid cancer. You are concerned that the patient may have medullary thyroid cancer. After you confirm your suspicion, which of the following genetic tests would you suggest to his family members? A BRCA 1 B APC C CFTR D ret proto-oncogene E CAG triplet expansion

D *The correct choice is D, ret proto-oncogene. Genetic testing to identify this oncogene is available for patients whom you suspect have medullary thyroid cancer and/or pheochromocytoma, and for their asymptomatic families who may be at risk for the same disorders. A mutation in this gene leads to uncontrolled growth of the C cells of the thyroid.* Choice A, BRCA 1, is an oncogene that is associated with risk for breast, ovarian, colon, and prostate cancers. Choice B, APC, is an oncogene noted in the familial colon cancer syndrome known as familial adenomatous polyposis. DNA analysis for CFTR mutations, choice C, can identify the gene mutations in patients with cystic fibrosis in the vast majority of instances. Choice E, CAG triplet expansion, is the result of a gene mutation in Huntington's disease.

Q 62.1: Which of the following medication classes has been shown to improve the short-term rate of cognitive preservation in mild to moderate Alzheimer's dementia patients? A Acetylcholinesterase inhibitors B Decarboxylase inhibitors C Dopamine receptor agonists D Monoamine oxidase inhibitor E N-methyl-D-aspartate receptor antagonist

The Correct Answer is: A Alzheimer's dementia is a chronic, progressive, neurodegenerative disorder. Acetylcholinesterase inhibitors have been associated with a modest decrease of cognitive decline and increased functioning. These medications increase the levels of acetylcholine, a neurotransmitter, and increase cholinergic activity within the affected brain regions. They have been approved for use in mild to moderate Alzheimer's disease. Memantine, an N-methyl-D-aspartate receptor antagonist, is indicated for use in moderate to severe disease, has been associated with decreased destruction of cholinergic neurons, and may slow cognitive decline. It is often added to acetylcholinesterase inhibitor therapy for progressed disease, but may be first line in a patient unable to tolerate the acetylcholinesterase inhibitor medications. Decarboxylase inhibitor medications inhibit the metabolism of dopamine; one example, carbidopa, is used in conjunction with L-dopa in the treatment of Parkinson's disease. Dopamine receptor agonists activate dopamine receptors in the absence of dopamine, and are useful in conditions with low dopamine levels, such as Parkinson's, prolactinomas, and restless leg syndrome. Selegiline, a monoamine oxidase inhibitor, is selective for monoamine oxidase-B and inhibits catabolic dopamine breakdown, with a potential neuroprotective effect. Use in moderately advanced Alzheimer's disease may slow progression, but remains controversial.

Q 56.1: A 70-year-old man with a history of hypertension, DM Type 2, and hyperlipidemia is seen for preoperative evaluation prior to left knee replacement. On auscultation, a very soft high-frequency decrescendo early diastolic murmur is heard at the upper left sternal border. Which of the following maneuvers would be the most appropriate to choose to increase the intensity of the murmur for better identification? A Isometric hand grip exercise B Listening with the bell at the apex with the patient in the left lateral decubitus position C Inspiration, followed by the patient holding his/her breath D The Valsalva maneuver E Having the patient lie flat with the knees bent

The Correct Answer is: A Isometric hand grip exercises will increase the intensity of the murmur of aortic regurgitation, which is usually described as a high-frequency decrescendo early diastolic murmur heard best at the left upper sternal border or at the right upper sternal border. Radiation, if it occurs, is frequently to the lower left sternal border and the apex. Isometric hand exercises increase arterial and left ventricular pressure, which increases the flow across the aortic valve, thereby increasing the murmur's intensity. Choice B is best used when listening to the murmur of mitral stenosis. Choice C will increase the AP diameter, making it more difficult to hear the murmur. Choice D, the Valsalva maneuver, will decrease the intensity of the murmur. Choice E is the best position for the abdominal exam, especially in males.

Q 62.7: A 49-year-old male presents with night sweats, weight loss, coughing, and shortness of breath. A urinalysis demonstrates hematuria. The patient has had a positive PPD skin test six months ago, but did not receive treatment. Which of the following suggests reactivation tuberculosis on chest x-ray? A Fibrocavitary apical disease B Diffuse small nodular densities C Ghon complex D Kerley B lines E Pleural scarring

The Correct Answer is: A Primary tuberculosis, caused by inhalation and infection with M tuberculosis, is often without marked systemic symptoms or lung changes. Radiologic findings may include atelectasis, small infiltrates, and lympadenopathy (greatest in the hilar region), but initial x-rays may be negative. The immune system often contains the infection, and it remains latent until reactivated or appropriately treated. In some cases of primary tuberculosis, but more often in reactivated latent infection, systemic symptoms and lung infection with x-ray findings will occur. X-ray findings may include infiltrates, cavitary lesions, and nodules, with the apices and upper lobes commonly being involved. A Ghon complex represents a calcified focus of previous infection, typically containing viable bacteria. A pleural effusion may be present with active infection, and may lead to pleural scarring. Kerley B lines are associated with congestive heart failure.

Q 59.6: A 62-year-old male presents with complaints of numbness in his hands and feet, with occasional foot drop, memory disturbance, fatigue, paleness, anorexia, nausea, and weight loss. He has a known history of diabetes and hypertension. Which of the following conditions is most likely responsible for these symptoms? A Autonomic neuropathy B Chronic renal failure C Diabetic polyneuropathy D Guillain Barre syndrome E Middle cerebral artery occlusion

The Correct Answer is: B *Chronic renal disease is associated with functional disturbances in all organ systems, including the central nervous system. Renal disease promotes CNS complications including neuropathies and neuromuscular irritability, along with systemic symptoms. The symptoms are typically progressive if the underlying renal disease is not addressed. Although other conditions promote similar neuropathies, such as diabetes, they are differentiated by the level of involvement, progression, and associated symptoms.* With Guillain Barre, an acute polyradiculoneuropathy would be expected to progress, and have associated weakness. Cerebrovascular accidents are not typically accompanied by generalized systemic symptoms, and a middle cerebral artery occlusion would be expected to have contralateral hemiparesis and hemisensory deficit.

Q 54.6: A 66-year-old man with a history of a cardiac murmur since childhood presents with complaints of increasing dyspnea while walking up one flight of stairs and increased lower extremity edema. On physical examination, a late-peaking crescendo-decrescendo murmur, preceded by a systolic ejection click, is noted. An S4 gallop is appreciated. Hepatomegaly and splenomegaly are appreciated. Which of the following is the most likely finding on echocardiogram? A Mitral stenosis B Pulmonic stenosis C Tricuspid stenosis D Atrial septal defect E Aortic regurgitation

The Correct Answer is: B Choice B, pulmonic stenosis, is the most likely finding on echocardiogram, given the patient's physical exam findings. Pulmonic stenosis can present with symptoms of right heart failure in the later stages. Patients may exhibit symptoms similar to aortic stenosis, including dyspnea with exertion, angina, fatigue and syncope, and evaluation through echocardiogram is recommended. Choice A, mitral stenosis, presents with a systolic murmur that is heard best in the left lateral decubitus position, with the bell of the stethoscope at the apex. Choice C, tricuspid stenosis, presents with a diastolic murmur that increases with inspiration. It is heard best at the left lower sternal border. Choice D, atrial septal defect, if large, could present with similar symptoms of exertional dyspnea secondary to a large shunt, but auscultation would reveal a moderately loud systolic ejection murmur that is heard best in the second and third interspaces. This is secondary to increased pulmonary arterial flow. Choice E, aortic regurgitation, presents with a soft diastolic murmur that is heard best at the left sternal border.

Q 58.10: A 48-year-old woman has been taking gentamicin for the last several weeks for a serious Pseudomonas aeruginosa infection. She presents today with a complaint of tinnitus. After further evaluation, it is determined that, in addition, she is also suffering from a bilateral high-frequency sensorineural hearing loss. The patient denies any dizziness or balance disturbances. Which of the following structures is most likely affected? A ear ossicles B organ of Corti C saccule D semicircular canals E tympanic membrane

The Correct Answer is: B Gentamicin belongs to the class of antibiotics known as "aminoglycosides," which are capable of producing ototoxicity as an adverse effect. Ototoxic agents target the hair (receptor) cells of the inner ear. Hair cells are found in organs involved in hearing (eg, cochlea) and balance (eg, semicircular canals, utricle, and saccule). This patient complains only of tinnitus and has sensorineural hearing loss. She denies any symptoms associated with a balance disturbance. The structure within the cochlea that contains the hair cells is the organ of Corti.

Q 56.7: A 30-year-old man presents to the office for follow-up on an endoscopically diagnosed gastric ulcer. At endoscopy, he was found to have a Helicobacter pylori infection and now he has completed appropriate therapy. He has another refill available on the proton-pump inhibitor. He is currently asymptomatic. What is the most appropriate follow-up on the infection? A Because he is asymptomatic, no further testing is required. B Check urea breath test or fecal antigen today. C Repeat endoscopy with histologic testing for H. pylori. D Check H. pylori serology today. E Collect stool specimen for culture.

The Correct Answer is: B Helicobacter pylori is a spiral, Gram-negative rod that resides in the gastric mucosa, where it causes PUD. It may be diagnosed by rapid urease test or by histology when endoscopy is performed. Noninvasive H pylori testing options include the urease breath test, fecal antigen testing, and serology. Serological and fecal antigen tests are the most cost-effective methods. All three noninvasive tests have sensitivities and specificities greater than 90%. Proton-pump inhibitor therapy should be discontinued 1 to 2 weeks prior to the fecal antigen or breath tests because PPIs may increase the number of false negatives. In this case, serology is the least invasive, most cost-effective, and least likely to be invalidated by the proton-pump inhibitor therapy.

Q 54.2: A 72-year-old man is transported via ambulance to the emergency department with severe chest pain and shortness of breath. Electrocardiogram (ECG) reveals ST-segment elevation in leads II, III, and aVF. While in the emergency department, he loses consciousness and is found to be in ventricular fibrillation. Resuscitation is successful, and a pulse is restored within 3 minutes. He is taken to the cardiac catheterization laboratory, where he undergoes two-vessel stenting. Two days later, his creatinine has increased from a baseline of 1.1 to 2.2 mg/dL. The next day, the creatinine is 3.9 mg/dL. Fractional excretion of sodium is ordered. You would expect this to be A <1 B >1 C unchanged from baseline D undetectable E equal to the serum creatinine level

The Correct Answer is: B Intrinsic ARF results in alterations in the kidneys' ability to respond to changes in hemostasis. When the integrity of the kidneys remains intact, sodium is conserved when GFR declines in an attempt to reestablish volume and perfusion, resulting in a fractional excretion of sodium (FENa) of <1. However, when the glomeruli are injured, the kidneys lose the ability to reabsorb sodium as the GFR decreases, and the FENa will be >1. The etiology of this patient's renal failure is most likely contrast-induced acute tubular necrosis following an ischemic episode, which is intrinsic ARF.

Q 58.3: A 66-year-old man with a medical history of aortic stenosis is admitted to the hospital with increasing shortness of breath. Physical examination reveals a regular pulse of 120 beats/min, blood pressure of 95/50 mm Hg, and a respiratory rate of 32 breaths/min. The estimated jugular venous pressure (JVP) is greater than 15 cm, rales are heard halfway up the lung fields bilaterally, and a holosystolic murmur is heard at the apex. There is a tender enlarged liver with hepatojugular reflux and 2+ pretibial and pedal edema. Plain film of the chest reveals cardiomegaly and pulmonary edema. ECG is suggestive of left ventricular hypertrophy. Admission laboratory studies include the following: What type of hyponatremia does this patient most likely have? A hypovolemic hypotonic B hypervolemic hypotonic C hypovolemic isotonic D hypervolemic hypertonic E hypovolemic hypertonic

The Correct Answer is: B Most often, hyponatremia is due to excessive water retention rather than a true sodium deficiency. The first step in evaluating hyponatremia is to determine serum osmolality. Knowing whether the serum is isotonic (normal osmolality), hypotonic (low osmolality), or hypertonic (high osmolality) can help determine the etiology of the hyponatremia, and therefore, treatment. The most common causes of isotonic hyponatremia are hyperproteinemia and hyperlipidemia. The most common causes of hypertonic hyponatremia are hyperglycemia, presence of radiocontrast agents, and the presence of inactive metabolites, that is, mannitol, sorbitol, glycerol, and maltose. Treatment is aimed at correcting the underlying disorder. Most commonly, hyponatremia occurs in the setting of low osmolality (hypotonic). To further evaluate the etiology of the hyponatremia, it must be determined if the patient is hypovolemic, euvolemic, or hypervolemic. Hypovolemic hyponatremia is usually due either to extrarenal or intrarenal sodium losses. Extrarenal losses occur from dehydration, diarrhea, and vomiting. Urinary sodium measures <10 mEq/L (normal, >20 mEq/L), as the kidneys are avidly retaining sodium in an attempt to restore volume. Treatment is directed at restoring volume. Intrarenal sodium losses occur from the use of diuretics and ACE inhibitors, nephropathies, and mineralocorticoid deficiency. Urinary sodium measures >20 mEq/L. Treatment is directed at reversing the underlying cause. The most common causes of euvolemic hyponatremia are SIADH, postoperative hyponatremia, hypothyroidism, psychogenic polydipsia, and endurance exercise. In these cases, electrolyte-free water is retained, which results in a true physiologic hyponatremia. Treatment is directed at correcting the underlying abnormality and replacing sodium losses. Hypervolemic hyponatremia is caused by congestive heart failure, liver disease, nephrotic syndrome, and advanced CKD in general, anything that causes fluid retention. Treatment is directed at treating the underlying disease, restricting water intake, and facilitating excretion of water. The first step in characterizing this patient's hyponatremia is to determine the serum osmolality, which is 276 mOsm/kg. This is low, so we know that this is hypotonic. Second, we have to determine the volume status. Urinary sodium is 12 mEq, which does not indicate either intrarenal or extrarenal losses of sodium and is not consistent with hypovolemia. Given the elevated JVP, extensive edema, and hepatojugular reflux, this patient is presenting with a clinical picture of fluid overload, or hypervolemia. This patient has hypervolemic hypotonic hyponatremia due to CHF.

Q 60.5: A 19-year-old woman presents to the emergency department complaining of headache. The headaches are generalized and increasing in intensity. They have not responded to over-the-counter (OTC) medications. She complains of approximately 1 week of blurred vision, intermittent diplopia, and vague dizziness. Her medical history includes obesity and acne. She takes Accutane and oral contraceptives. She is found to have bilateral papilledema, visual acuity of 20/30 on physical examination, and a normal MRI of the brain. The next most appropriate step would be A CT scan of the head B lumbar puncture C therapy with high-dose prednisone D stat cerebral arteriogram E reassurance and follow-up in the office in 6 months

The Correct Answer is: B The presence of headache associated with papilledema raises the concern for a brain tumor. The MRI excluded a mass lesion, raising a strong suspicion of pseudotumor cerebri. This is also known as benign intracranial hypertension. It is not a benign condition, however, since it causes severe headache and may result in visual loss. It is particularly frequent in obese adolescent girls and young women. The etiology is unknown but may be associated with the use of oral contraceptives, vitamin A, and tetracycline. The presentation consists of headaches caused by an increase in intracranial pressure and blurring of vision. There may be diplopia, but the remainder of the neurologic examination is unremarkable. Papilledema is virtually always part of the presentation. The mental status is normal. The differential diagnosis includes venous sinus thrombosis, sarcoidosis, and tuberculosis or carcinomatous meningitis. The last two are excluded by lumbar puncture. An abnormal cerebrospinal fluid is not consistent with pseudotumor cerebri. The diagnosis is made by excluding mass lesions with CT scan or MRI and demonstrating markedly increased intracranial pressure by lumbar puncture. The treatment involves weight loss, diuretics, and steroids. Repeat lumbar punctures to remove cerebrospinal fluid and decrease intracranial pressure are effective. In cases that are unresponsive to these measures, lumbar-peritoneal shunting is effective, as is unilateral optic nerve sheath fenestration. Effective treatment can improve headaches and prevent vision loss.

Q 55.3: You have just diagnosed Mr. Jones, a 31-year old male, with community-acquired pneumonia. He has no comorbidities and was treated for a strep throat with antibiotics within the last three months. According to the Infectious Disease Society of America (ISDA) and American Thoracic Society (ATS) guidelines, the first-line antibiotic choice would be? A a cephalosporin B a respiratory fluoroquinolone C macrolide D trimethoprim-sulfamethoxazole (TMP-SMZ) E an augmented semi-synthetic penicillin

The Correct Answer is: B With the emergence of other pathogens causing pneumonia and the development of resistance to penicillin and other drugs in S pneumoniae, treatment decisions have become more complex. The 2003 update to ISDA and ATS guidelines for the treatment of community-acquired pneumonia differ depending on the health and age of patients (ie, 65 years or older), whether they have recently been treated with an antibiotic, and whether they are at risk for an aspiration pneumonia or influenza superinfection. For patients with no serious comorbidities, the ISDA/ATS recommends a respiratory quinolone or an advanced macrolide plus high-dose amoxicillin (or amoxicillin-clavulanic acid) as first-line therapy. If an antibiotic has been used recently, a respiratory quinolone or an advanced macrolide, plus a second- or third-generation cephalosporin are recommended options by the ISDA/ATS.

Q 58.6: Which of the following best describes the mechanism of action of angiotensin-converting enzyme (ACE) inhibitors in controlling blood pressure and preventing or slowing kidney damage? A They result in systemic vasodilation. B They increase renal tubular excretion of sodium. C They result in dilation of the efferent arteriole, reducing glomerular pressure. D They block the angiotensin II receptor on the cell membrane. E They reduce production of angiotensinogen, the precursor to angiotensin I.

The Correct Answer is: C *ACE inhibitors prevent the conversion of angiotensin I to angiotensin II, thereby interrupting the renin-angiotensin-aldosterone system, which regulates blood pressure. The glomerular efferent arteriole dilates, given the decreased stimulus from angiotensin II to constrict. This lowers pressure in the glomerulus by lowering resistance to outflow. This effectively results in a decrease in GFR, resulting in increased serum creatinine and potassium levels. However, these changes are not necessarily indications to discontinue the ACE inhibitor. Usually, the creatinine increases 0.2 to 0.4 mg/dL and then levels out.* Monitoring serum creatinine and potassium levels is indicated. If only mild increases occur and stabilize, or if there are no changes, the ACE inhibitor can, and should, be continued so that the patient derives the beneficial effect of the decline in pressure within the glomerulus, which will slow down the progression of CKD.

Q 57.4: A 42-year-old man is prescribed a drug classified as a "bile acid sequestrant." Which of the following will most likely be diminished as a result of administering this drug? A activity of pancreatic amylase B bile flow from the gall bladder to the duodenum C emulsification of triglycerides D secretion of gastric acid E synthesis of plasma proteins

The Correct Answer is: C Bile acids are constituents of the bile that are synthesized into bile salts by liver hepatocytes or by intestinal bacteria. One of the major roles of bile salts is to emulsify dietary lipids within the lumen of the small intestine. Emulsification is an important first step in lipid digestion, as it increases the surface area for lipases to function. Bile salts also help to form micelles, which transport the products of lipid digestion from the lumen and into the cells (enterocytes) lining the small intestine. Without bile salts, digestion and absorption of dietary lipids is incomplete and therefore decreased.

Q 61.2: A 36-year-old male with a history of acute lymphoblastic leukemia (ALL) is treated with high dose methotrexate and glucocorticoids, vincristine, and L-asparaginase at age 10. He has been in remission since induction therapy was completed. He presents with pain to his right hip/back area, which radiates down his leg. He is also having some back pain that keeps him up at night. What is the most likely diagnosis? A Bladder cancer with referred pain B Lymphoma with metastasis to the bone C Osteonecrosis of the right hip D Osteoporosis E Relapsed ALL

The Correct Answer is: C Common treatment sequelae that can present years out from therapy include osteonecrosis, especially following the use of high dose methotrexate and glucocorticoids. Lymphoma can arise as a post transplant complication, but is not typically seen following anti-leukemic therapy. Osteoporosis is also a complication of anti-leukemic therapy, but is more associated with the use of mercaptopurine. Relapsed ALL is typically seen in the first two years following therapy, but can arise more than 10 years out. Typically, this is seen with fever, pancytopenia, and bony pain. The pain is not typically isolated to one side of the body.

Q 55.9: A 55-year-old man with a history of chronic renal failure, 6 months status post renal transplant, presents with chest pain, productive cough, and low-grade fever. He reports generalized malaise as well. Current medications include only those related to the transplant. He has no known allergies. Examination reveals a temperature of 102°F, unremarkable HEENT (head, ears, eyes, nose, throat), and few crackles anteriorly in the upper right lung field. Chest X-ray reveals a solitary nodule in the right upper lobe. The most likely etiology for his symptoms is A Streptococcus pneumoniae B Pneumocystis jiroveci C cryptococcosis D Candida E influenza A

The Correct Answer is: C Cryptococcal species are opportunistic organisms responsible for infections in immune-compromised hosts. With the rise of HIV infections in the past few decades in the United States, cryptococcosis is becoming increasingly prevalent. It is also a common infection in those who have undergone solid organ transplantation. The two most common areas for infection are the lungs and the central nervous system. Pulmonary involvement includes fever, productive cough, chest discomfort, and weight loss. Pleural effusions, lymphadenopathy, and solitary or multiple nodules can all be seen on chest x-ray. Central nervous system manifestations include meningitis and meningoencephalitis. Diagnosis is confirmed with India ink prep of cerebrospinal fluid showing yeast or histologic stains of tissue from the involved organs. Treatment is with oral or parenteral antifungal agents.

Q 54.7: A 29-year-old man who is HIV positive has developed dark purple papular nonblanching lesions between the toes of his right foot. He has no other symptoms. Careful examination shows that this is the only area of involvement. His CD4 count is 150 cells/mL. Of the following, what is the most appropriate treatment? A alpha interferon B chemotherapy with daunorubicin, bleomycin, and vinblastine C intralesional vinblastine D liposomal doxorubicin E radiation

The Correct Answer is: C Kaposi sarcoma that is in a limited area of the skin may be treated with intralesional vinblastine or by simply observing it over time. Liposomal doxorubicin and alpha interferon are used for extensive or aggressive skin disease, while combination chemotherapy is used for visceral disease.

Q 57.2: Jane, a 21-year-old female, was seen in the office 10 days ago and was diagnosed with perennial allergic rhinitis and sent home with instructions for increased fluids, decongestants, and nasal steroids. She returns today with worsened symptoms of malaise, low-grade fever, nasal discharge, cough that is worse at night, mouth breathing, early morning unilateral pain over sinuses, and congestion. Physical examination reveals thick purulent nasal discharge, postnasal discharge visible in the posterior pharynx, periorbital swelling, and tenderness of sinuses upon palpation. She is 36-weeks pregnant and allergic to penicillin. Of the following, what is the most appropriate antibiotic? A amoxicillin B trimethoprim-sulfamethoxazole C clindamycin D levofloxacin

The Correct Answer is: C Most patients with a diagnosis of acute rhinosinusitis based on clinical grounds improve without antibiotic therapy. The preferred initial approach in patients with mild to moderate symptoms of short duration is therapy aimed at facilitating sinus drainage, such as oral and topical decongestants, nasal saline lavage, and—in patients with a history of chronic sinusitis or allergies—nasal glucocorticoids. Adult patients who do not improve after seven days, children who do not improve after 10 to 14 days, and patients with more severe symptoms (regardless of duration) should be treated with antibiotics. Empirical therapy should consist of the narrowest-spectrum agent active against the most common bacterial pathogens, including S. pneumoniae and H. influenzae—e.g., amoxicillin. But amoxicillin is contraindicated in patients with urticarial reactions to penicillins, and quinolones are similarly contraindicated in pregnancy. trimethoprim-sulfamethoxazole is contraindicated in the third trimester of pregnancy. The best choice is clindamycin.

Q 54.8: An 18-year-old female presents to your office with the complaint of palpitations for the last 2 months. The episodes are frequent and are accompanied with lightheadedness and shortness of breath. The patient's mother has taken her pulse when some of the episodes occur and states that the rate gets as high as 170 beats per minute. On exam, she is alert, awake, and oriented. Her resting pulse is 80 and her blood pressure is 122/65. Her lungs are clear throughout, and her cardiac exam revealed a regular rate and rhythm, without murmurs, rubs, or gallops. The patient has been treated with flecanide for several months, and has done well until she started experiencing more episodes of tachycardia. Her blood pressure remains stable in the 125 to 135 systolic range, and her symptoms are mild when the tachycardia occurs. What therapy should be given next for this patient? A Stop flecanide B Ablation therapy C Beta blocker with flecanide D Beta blocker without flecanide E Stop flecanide, add diltiazem

The Correct Answer is: C Patients currently on flecanide with ongoing symptoms can benefit from the addition of a beta-blocker, to help control rate and symptoms. This would be started prior to ablation therapy, provided that the patient's vital signs can tolerate the added medication. A calcium channel blocker would potentially worsen the patients arrhythmia.

Q 59.2: A 20-year-old woman was just told by her new sexual partner that she needed to be checked for a sexually transmitted infection because he has developed dysuria and a profuse urethral discharge. She herself has had a subjective fever for the past two days, some nausea but no vomiting, diffuse lower abdominal pain, and a severe backache. On examination, she has a temperature of 100.5˚F, hypoactive bowel sounds, bilateral lower abdominal quadrant tenderness, a profuse mucopurulent cervical discharge and pronounced cervical motion tenderness. Serum pregnancy testing is negative. She is given an injection of ceftriaxone and a prescription for doxycycline for 14 days and an appointment for follow up the next day. Under which of the following conditions should metronidazole be added to her regimen? A If she does not appear improved by the following morning B If she develops vomiting or diarrhea C If she has a probable tubo-ovarian abscess D If she has an allergic reaction to the doxycycline E If she has had more than one partner in the past month

The Correct Answer is: C Recommended regimens for treatment of pelvic inflammatory disease include ceftriaxone or another parenteral third-generation cephalosporin and doxycycline or cefoxitin, probenecid, and doxycycline. Metronidazole (or clindamycin) should be added to either regimen if a tubo-ovarian abscess is present. If being treated as an outpatient and she has not improved in 3 days (A), or if she develops vomiting (B), she should be hospitalized rather than given metronidazole. Metronidazole is not a satisfactory substitute for doxycycline (D). The patient's condition rather than the number of sexual partners (E) determines the appropriate therapy.

Q 63.1: A 27-year-old African American with sickle cell anemia presents to the emergency department with acute onset intractable pain. She is taking quick, shallow breaths and her oxygen saturation is 84% on room air. She appears desiccated, states she hasn't eaten in the last 24 hours, and says that she "just doesn't feel well." She is also afebrile. What should your next course of action be? A Start morphine, hydrate, and start antibiotics B Start morphine, oxygen, and start antibiotics C Start oxygen, hydrate, and exchange transfusion D Start oxygen, hydrate, and give pneumococcal vaccination E Start oxygen, hydrate, and start antibiotics

The Correct Answer is: C Start oxygen, hydrate, and exchange transfusion Exchange transfusions are primarily indicated for the treatment of intractable pain crises, priapism, and stroke. Patients should be kept well hydrated, and oxygen should be given if the patient is hypoxic. Antibiotics would be used if there was an infection identified, but are not part of initial treatment in a sickle crises.

Q 60.1: A 54-year-old woman has suffered a stroke that has resulted in dramatic changes to her personality, left leg and foot weakness, loss of sensation in the left leg, and apathy. Which of the following arteries was most likely affected by the stroke? A left anterior cerebral B left middle cerebral C right anterior cerebral D right middle cerebral E right posterior cerebral

The Correct Answer is: C The anterior cerebral arteries supply the frontal lobes as well as the medial aspects of the parietal and occipital lobes rostral to the parietooccipital sulcus. The prefrontal cortex of the frontal lobe is concerned with a person's personality, depth of feeling, and initiative. Hence, occlusion of an anterior cerebral artery can cause neuronal injury to this area, leading to feelings of apathy and personality changes. The paracentral lobule represents the medial aspects of the precentral gyrus (frontal lobe) and postcentral gyrus (parietal lobe), which are responsible for motor control and somatosensory perception, respectively, of the leg and foot. Hence, occlusion of an anterior cerebral artery can produce contralateral hemiparesis and hemisensory loss involving the leg and foot. With the 54-year-old patient, the symptoms were occurring on the left side, which points to a right anterior cerebral artery occlusion.

Q 57.5: A 36-year-old female patient is being treated for thyroid disease and returns to your family practice office for a six month exam. She is on 100 ugm of levothyroxine once daily. Her vital signs are a blood pressure of 128/82 mm&thinsp;Hg, pulse 62 bpm, respirations 12 bpm, and she is afebrile. Her physical examination is normal. Her thyroid-stimulating hormone (TSH) level is 11.5 uIU/mL (normal 0.4 to 5.0 uIU/mL). What is your next step in her treatment? A Draw a free T3 and free T4. B Lower her levothyroxine to 88 ugm daily and recheck her TSH in 4 to 6 weeks to adjust if necessary. C Increase her levothyroxine to 112 ugm daily and recheck her TSH in 4 to 6 weeks to adjust if necessary. D She is asymptomatic so do not adjust her levothyroxine, but recheck her level at her next visit in six months.

The Correct Answer is: C The thyroid gland operates on a reverse feedback mechanism. When TSH is high, free T 3 /T 4 is low and need repletion, and vice versa. In this case, the patient has an elevated TSH, indicating that her serum T 3 /T 4 is likely low and she should have her dose of levothyroxine adjusted upwards and levels can be rechecked starting in approximately four weeks and adjusted subsequently as necessary.

Q 55.5: Which of the following diagnostic tests is the imaging study of choice in patients suspected of having Zollinger-Ellison syndrome? A Transabdominal ultrasound B Computed tomography of the abdomen C Magnetic resonance imaging D Somatostatin receptor scintography E Endoscopic ultrasonography

The Correct Answer is: D Although imaging tests such as CT scan, MRI, transabdominal ultrasound, and endoscopic ultrasound can be utilized with varying frequencies of sensitivity and specificity, the test of choice is the somatostatin receptor scintigraphy. This test has close to a 100% sensitivity and specificity.

Q 58.5: Which of the following diagnostic studies is indicated for a patient with amaurosis fugax? A CT of the head B Intraocular pressure C Temporal artery biopsy D Carotid ultrasound E Ocular fluorescein angiogram

The Correct Answer is: D Amaurosis fugax is a monocular vision loss that appears like a curtain passing over the eye, and comes from carotid artery disease. A CT of the head is indicated for lateralizing stroke symptoms. Intraocular pressure is taken for evaluation of chronic or acute glaucoma. A temporal artery biopsy is taken if giant cell arteritis is suspected. An ocular fluorescein angiogram is done to evaluate retinal disorders.

Q 54.1: A 70-year-old man, with a history of pulmonary hypertension and obstructive sleep apnea, presents with complaints of increasing dyspnea while walking his dog. He has also recently noted increased lower extremity edema. On physical examination, jugular venous distension is noted. Auscultation of the chest demonstrates a high-pitched blowing diastolic murmur. The murmur is heard over the second and third left intercostal spaces. An S 3 is appreciated. Abdominal exam reveals hepatomegaly and splenomegaly. Which of the following maneuvers would be the most appropriate to choose for better identification of the murmur? A Left lateral decubitus position listening with the bell of the stethoscope B Standing C Seated leaning forward D Inspiration E Expiration

The Correct Answer is: D Choice D, inspiration, will increase the intensity of the murmur of pulmonic regurgitation/insufficiency. The Valsava maneuver will diminish the intensity of the murmur. Choice B, standing, will cause the mid-systolic click associated with mitral valve prolapsed to move toward S1 or become more audible. Choice A, rolling the patient to the left lateral decubitus position, is most useful in identification of the murmur of mitral stenosis. Choices B, C, and E, have no effect on the Graham Steell murmur of pulmonic regurgitation.

Q 58.1: A 62-year-old man with a history of hypertension, diabetes mellitus type 2, hyperlipidemia, and chronic tobacco use presents to the office with complaints of retrosternal chest pressure radiating down his left arm, associated with diaphoresis, nausea, and dyspnea for the last 45 minutes after mowing his lawn. The patient's vital signs are stable, and on physical examination a new systolic murmur is appreciated. His EKG demonstrates normal sinus rhythm with minor nonspecific changes. Which of the following would be the most appropriate next step in management of this patient? A Discharge patient with anxiolytic therapy B Discharge patient with NSAID therapy C Transesophageal echocardiogram D Repeat EKG with addition of leads V7 through V9 E Transthoracic echocardiogram

The Correct Answer is: D Choice D, repeating the EKG with addition of the posterior leads (leads V 7 through V 9 ), can uncover hidden posterior ST-segment elevation in patients with symptoms suggestive of acute coronary syndrome (Figure 4). These patients should then be treated following acute STEMI guidelines, including acute reperfusion therapy. A new systolic murmur would not occur with anxiety or costochondritis, and in a patient with numerous cardiac risk factors, with symptoms suggestive of acute coronary syndrome, neither anxiolytics nor anti-inflammatory therapy would be appropriate. Choices C and E may be considered later in this patient's treatment plan, once acute myocardial infarction has been ruled out, for further investigation of this patient's new murmur. (Tintinalli et al., 2011, Chapter 52)

Q 55.8: A 25-year-old female presents to your office complaining of intermittent episodes of shortness of breath, especially at night and when she gets a cold. She has smoked one pack per day for 11 years and is asymptomatic on physical examination, with clear breath sounds and stable vital signs. She had "a touch of asthma" as a child, having been to the emergency department on numerous occasions. What would be the first step in your diagnosis and treatment plan for this patient? A Provide smoking-cessation counseling, prescribe a short-acting beta2-agonist inhaler, and give the patient education in utilizing the medication with a follow-up appointment in one month. B Provide smoking cessation counseling, prescribe an anti-cholinergic inhaler, and give the patient education in the use of the medication with a follow-up appointment in one month. C Perform a peak flow and compare it to the patient's predicted peak flow; if normal (or nearly so) reassure the patient and ask her to return if she develops the symptoms again. D Perform spirometry, administer a nebulizer treatment, and after waiting 20 minutes repeat the spirometry; after interpretation to differentiate between asthma and chronic obstructive pulmonary disease, treat accordingly. E Refer the patient to pulmonary medicine.

The Correct Answer is: D Clinical, imaging, and laboratory findings should enable the clinician to distinguish chronic obstructive pulmonary disease (COPD) from other obstructive pulmonary disorders such as bronchial asthma, bronchiectasis, cystic fibrosis, bronchopulmonary mycosis, and central airflow obstruction. Simple asthma is characterized by complete or near-complete reversibility of airflow obstruction. Bronchiectasis is distinguished from COPD by features such as recurrent pneumonia and hemoptysis, digital clubbing, and characteristic imaging abnormalities. Patients with severe α 1 -antiprotease (α 1 -antitrypsin) deficiency are recognized by family history and the appearance of panacinar bibasilar emphysema early in life, usually in the third or fourth decade; hepatic cirrhosis and hepatocellular carcinoma may occur. Cystic fibrosis occurs in children and younger adults. Rarely, mechanical obstruction of the central airways simulates COPD. Flow-volume loops may help separate patients with central airway obstruction from those with diffuse intrathoracic airway obstruction characteristic of COPD. Spirometry provides objective information about pulmonary function and assesses the results of therapy. Pulmonary function tests early in the course of COPD reveal only evidence of abnormal closing volume and reduced midexpiratory flow rate. Reductions in FEV 1 and in the ratio of forced expiratory volume to vital capacity (FEV 1 % or FEV 1 /FVC ratio) occur later. In severe disease, the FVC is markedly reduced. Lung volume measurements reveal a marked increase in residual volume (RV), an increase in total lung capacity (TLC), and an elevation of the RV/TLC ratio, indicative of air trapping, particularly in emphysema. Arterial blood gas measurements characteristically show no abnormalities early in COPD other than an increased A-a-Do 2 . Indeed, they are unnecessary unless (1) hypoxemia or hypercapnia is suspected, (2) the FEV 1 is < 40% of predicted, or (3) there are clinical signs of right heart failure. Hypoxemia occurs in advanced disease, particularly when chronic bronchitis predominates.

Q 55.10: A 46-year-old man with a history of EtOH abuse is brought to the emergency department in the morning by his wife. She has noted that he has developed tremors in both arms, and he seems mildly confused to her. He complains of feeling weak, with some cramping in the legs. On physical examination, his blood pressure is noted to be 162/95 mm Hg, and his heart rate is 108 beats/min. There is no asterixis. Which of the following electrolyte disorders are you likely to find in this patient? A hypercalcemia B hypocalcemia C hypermagnesemia D hypomagnesemia E hyperphosphatemia

The Correct Answer is: D Hypomagnesemia is a common finding in the patient who abuses alcohol. Other leading causes include diarrhea, diuretics, aminoglycosides, and amphotericin B. The etiology of hypomagnesemia in the patient with a history of alcohol abuse is thought to be a combination of malabsorption and inadequate dietary intake, possibly with alcohol exerting an antagonistic effect on absorption. Signs and symptoms are those of neuromuscular and central nervous system hyperirritability, including weakness and muscle cramps, tremors, nystagmus, a positive Babinski response, confusion, and disorientation. Hypertension, tachycardia, and ventricular arrhythmias may develop.

Q 61.5: A 70-year-old man with a history of hypertension, DM Type 2, and hyperlipidemia is seen for pre-operative evaluation prior to left knee replacement. On auscultation, a very soft high-frequency decrescendo early diastolic murmur is heard at the upper left sternal border. Utilizing isometric hand grip exercises, the murmur increases in intensity and can be heard radiating to the left sternal border and apex. Given the patient's physical exam findings, which of the following is the most likely diagnosis? A Aortic stenosis B Ventricular septal defect C Mitral stenosis D Aortic regurgitation/insufficiency E Mitral regurgitation/insufficiency

The Correct Answer is: D Isometric hand grip exercises will increase the intensity of the murmur of aortic regurgitation, which is usually described as a high-frequency decrescendo early diastolic murmur that is heard best at the left upper sternal border or at the right upper sternal border. Radiation, if it occurs, is frequently to the lower left sternal border and the apex. Isometric hand exercises increase arterial and left ventricular pressure, which increases the flow across the aortic valve, thereby increasing the murmur's intensity.

Q 54.15: Tumor necrosis factor (TNF) inhibitors are most often considered for use in patients with rheumatoid arthritis (RA) that does not respond to initial therapy. Which of the following screenings should occur before a patient is placed on this class of medication? A chest x-ray B allergy testing C liver function tests D purified protein derivative (PPD) test E serum BUN (blood urea nitrogen) and creatinine test

The Correct Answer is: D Patients being treated for rheumatoid arthritis with tumor necrosis factor (TNF) inhibitors are at increased risk for developing an opportunistic infection, such as tuberculosis (TB). It is recommended that screening for the presence of latent TB occur before TNF inhibitors are started. There is no specific indication to order a chest x-ray, allergy testing, liver function tests, or serum BUN and creatinine prior to initiation of TNF inhibitors.

Q 56.5: Which of the following U.S. Preventive Services Task Force (USPSTF) recommendations for prostate cancer is correct? A All African-American men over 40 years old should have a digital rectal examination (DRE) and a prostate-specific antigen (PSA) screening annually. B All patients should receive a DRE and PSA screening at 50 years of age. C All high-risk patients should receive a PSA screening annually starting at 50 years of age. D DREs are not recommended and PSA screening is recommended based upon patient risk, but only after discussing the benefits and risks with the patient and the patient expressing the desire to have the examination performed.

The Correct Answer is: D Prostate cancer screening in older men is a particularly challenging topic because significantly more older men die with prostate cancer than from it. Nonetheless, 92% of prostate cancer deaths occur in men who are older than 65 years. Although prostate cancer can be detected earlier via PSA testing, mortality from prostate cancer has not been reduced as a result of screening. The USPSTF has concluded that evidence to recommend for or against screening is lacking. Other organizations such as the American Cancer Society and American Urological Society recommend screening among those with at least a ten-year life expectancy. The matter is complicated by the fact that techniques to treat prostate cancer often greatly affect quality of life. Furthermore, we are not able to reliably predict whether a particular prostate cancer is life-threatening. Given the uncertainties, PSA screening should be reserved for the most robust of older men and only after a thorough discussion of the arguments for and against screening.

Q 56.3: A 22-year-old African American male presents to the emergency department with shortness of breath, which started 2 hours prior to arrival. He does not have a history of pulmonary disease that he is aware of, and he states that in the past at random events he has had similar episodes. He does nothing to get the episodes to stop, and he also states that he feels his chest pounding at the same time of the shortness of breath. He has no medical history that he is aware of, and he takes no medications or any illicit drugs. On examination he is alert, awake, and oriented. His vital signs show T 99.0, P 142, R 18, and BP 132/82. His chest x-ray is negative for any acute cardiopulmonary disease, and his electrocardiogram has an irregularly irregular rate of 142 with visible delta waves. Given the clinical situation above, what is the best medication for managing this patient's condition with a long-term approach? A Amiodarone B Atropine C Hydralazine D Flecainide E Digoxin

The Correct Answer is: D This patient has Wolff-Parkinson-White (WPW) syndrome along with atrial fibrillation and a rapid ventricular response. Of the choices given in managing this patient's tachycardia, oral flecainide (D) will serve to slow the process within the accessory pathway and prolong the refractory period. Amiodarone (A) has been shown to not be effective in managing the patient's tachycardia with respect to WPW, and the other medications (B, C, and E) would not have an effect on the condition and may actually worsen the patient's symptoms.

Q 57.3: A 29-year-old female who is only taking oral contraceptives presents to the emergency department with a 1-day history of worsening shortness of breath. On examination, the patient is afebrile, her pulse rate is 105, respiratory rate is 24, and blood pressure is 122/78. She has wheezing to all lung fields and appears to be in mild to moderate distress. There is no swelling or edema to the lower extremities. Based on these findings, what is the best test to order to determine the diagnosis in this patient? A CT chest B Chest x-ray C Ventilation to perfusion scan D Pulmonary arteriography E MRI chest

The Correct Answer is: D This patient presents with a history and physical exam that is consistent with an acute pulmonary embolus (PE). The prognosis for this type of illness can be serious, and in some cases death can result. The test that is still the gold standard for determination of a PE is the pulmonary arteriogram. CT of the chest with contrast tends to be the more utilized testing modality, but the arteriogram remains the test of choice.

Q 61.9: A 65-year-old man presents to the emergency department with an acute ischemic stroke. His CT scan is normal. His blood pressure is 180/100 mm Hg. What is the most appropriate treatment for his hypertension? A labetalol (Normodyne) 20 mg IV B nifedipine (Procardia) 10 mg po C nitroprusside (Nipride) drip at 1 mg/kg/min D clonidine (Catapres) 0.1 mg po E no antihypertensive at this time

The Correct Answer is: E *Blood pressure is typically elevated at the time of presentation in acute ischemic stroke. It will decline without medication in the first few hours to days. Aggressively lowering blood pressure in an acute ischemic stroke may decrease the blood flow to the ischemic but salvageable brain tissue. This potentially salvageable brain tissue is referred to as the penumbra. Decreasing blood flow to the ischemic penumbra by acutely lowering blood pressure may result in eventual infarction of this brain tissue. Treatment of previously undiagnosed hypertension should be deferred for several days.* Blood pressure should be treated if there are other indications, such as angina or heart failure. Control of blood pressure is appropriate in patients who are receiving tissue plasminogen activator (t-Pa) for their stroke. Blood pressure should be lowered cautiously to a systolic of less than 185 mm Hg and a diastolic of less than 110 mm Hg. This is thought to decrease the incidence of intracerebral hemorrhage in these patients.

Q 59.7: A 20-year-old college student describes his headaches as unilateral, a 5 out of 10 for pain, and with throbbing, associated with photophobia and often accompanied by nausea and vomiting. He states that the headaches occur two to three times a month and are associated with decreased sleep. He has taken OTC medication with relief and is trying to stay on a sleep schedule. He denies any other neurologic symptoms. Which of the following is the most appropriate next step for managing this patient? A Brain MRI B Hydrocodone-acetaminophen tablets C Physical therapy D Topiramate E Sumatriptan nasal spray

The Correct Answer is: E *Classic migraines often present with episodic, unilateral, and throbbing headache pain, associated with photophobia and phonophobia. Nausea and vomiting may also be involved. Management of migraines includes nonpharmacologic therapies, such as healthy eating, sleep pattern stabilization, caffeine avoidance, and stress relief. If nonpharmacologic therapies are not sufficient, or if symptoms impact activities of daily living, pharmacologic management is indicated.* Medications are available for migraine sufferers and are typically catergorized as abortive or preventive. Abortive therapy for this patient may include nonsteroidal anti-inflammatory medications. Additionally, studies have shown that stimulation of the 5-HT receptors can successfully stop a migraine, and the 5-HT1 receptor agonists, with selective agents (the triptans) often used successfully for headache improvement. Nasal spray formulations are useful for nausea and vomiting. Topiramate, an anticonvulsant, has received FDA approval for migraine prevention therapy in patients with increasing migraine frequency or poor response to abortive therapies. Narcotics are typically avoided for migraine management, and MRI is not warranted without additional clinical signs and symptoms, and other differential diagnoses.

Q 62.5: A 35-year-old patient with AIDS has had unintended weight loss of nearly 30 lb over the past 6 months. This loss has been primarily in muscle mass. He has little appetite, but no nausea, diarrhea, or evidence of oral candidiasis. He reports interest in resuming his former weight-training regimen. Which of the following is the most appropriate pharmacologic agent to help him gain muscle mass? A dronabinol B megestrol acetate C odansetron D prochlorperazine E testosterone enanthate

The Correct Answer is: E Anabolic steroids, most commonly testosterone enanthate or cypionate, increase lean body mass in patients with AIDS, particularly those who do weight training. Dronabinol, an antiemetic, and megestrol acetate, a progestational agent, are used to increase appetite and assist in weight gain but have little effect on lean muscle mass. Odansetron and prochlorperazine are both used to treat weight loss caused by nausea of unclear origin in patients with AIDS and are given prior to meals.

Q 54.10: Which of the following statements regarding diabetic medications is/are most correct? A Incretin-mimetics like exenatide commonly result in weight loss. B Thiazolidinediones (TZDs) should be held prior to and for 48 hours after administration of ionidated contrast material. C Sulfonylureas have the highest risk of hypoglycemia of all oral diabetic agents. D Sulfonylureas help preserve beta-cell function. E All of the above.

The Correct Answer is: E Incretin-mimetics act upon the satiety center of the brain and as a result promote weight loss, averaging approximately six pounds. Metformin may cause fatal lactic acidosis when given to patients on concomitantly-administered iodinated contrast dye. Sulfanylureas, in part because of first-pass metabolism, have the highest risk of hypoglycemia of all the oral anti-diabetic agents. TZDs have been found to preserve beta-cell function more so than any other agent.

Q 54.18: A 37-year-old man with a history of Paget disease presents with a deviation of the tongue to the right side upon sticking it out. The right side of the tongue is also observed to have slight atrophy. These symptoms most likely point to a lesion of which of the following cranial nerves? A left hypoglossal nerve B left vagus nerve C left glossopharyngeal nerve D right glossopharyngeal nerve E right hypoglossal nerve

The Correct Answer is: E One of the many clinical manifestations of Paget disease is narrowing of cranial foramina. The 12th cranial nerves (hypoglossal nerves) pass through the hypoglossal canals of the occipital bones. The hypoglossal nerves innervate many of the extrinsic muscles that move the tongue, including the genioglossus, hyoglossus, and styloglossus. Each hypoglossal nerve supplies the ipsilateral extrinsic muscles. Hence, the tongue deviates to the paralyzed side during protrusion because of the actions of the unaffected extrinsic muscles on the other side.

Q 54.9: The organism responsible for most cases of peritonitis in patients on peritoneal dialysis is A Candida albicans B Escherichia coli C Streptococcus pneumoniae D Pseudomonas aeruginosa E Staphylococcus aureus

The Correct Answer is: E S. aureus is the organism responsible for most cases of peritonitis in patients on peritoneal dialysis. Overall, gram-positive organisms are responsible for 50% of cases, and gram-negative organisms cause 15% of cases. Four percent of cases are polymicrobial in nature. Improper technique by the patient in making catheter connections during dialysis exchanges is the reason for bacterial inoculation in most cases. Abdominal pain, fever, and cloudy dialysis fluid are the presenting symptoms and signs.

Q 54.20: A 32-year-old man who is HIV positive has a seizure. On presentation to the emergency department (ED) he is confused and unsure of what happened. His partner reports that he had been complaining of headache in the days preceding the event. CT scanning of the head demonstrates five peripheral contrast-enhancing lesions. What is the most likely diagnosis? A AIDS dementia complex B central nervous system lymphoma C cryptococcal meningitis D progressive multifocal leukoencephalopathy E toxoplasmosis

The Correct Answer is: E The most common space-occupying CNS lesion in patients with HIV is toxoplasmosis. This condition may present with headache, focal neurologic deficits, seizures, and/or mental status changes. The typical appearance on brain imaging is that of multiple contrast-enhancing lesions in the periphery, particularly the basal ganglia. CNS lymphoma is more typically a single lesion. AIDS dementia complex presents a diagnosis of exclusion, without a characteristic appearance on imaging. The diagnosis of cryptococcal meningitis is made by examination of the spinal fluid, while PML imaging shows nonenhancing white matter lesions without mass effect.

Q 54.4: A 24-year-old male has an eight-month history of loose thought associations, social withdrawal, auditory hallucinations, and deterioration in his personal appearance and hygiene. Upon examination, he is noted to have a flat affect, perceptual distortions, and behaves like he is detached from his own actions. Which of the following medications used in treatment of this disease has the risk of prolonging the QTc interval? A Aripiprazole (Abilify) B Chlorpromazine (Thorazine) C Loxapine (Loxitane) D Quetiapine (Seroquel) E Ziprasidone (Geodon)

The Correct Answer is: E Ziprasidone is the only medication of the ones listed that is known to prolong the QTc interval. An ECG is recommended for patients at risk for cardiac sequelae.


संबंधित स्टडी सेट्स

Chapter 22: The Respiratory System

View Set

Chapter 12 Forces and Motion Review

View Set

Chapter 11: Substance- Related Disorders

View Set

medsurg2 exam 2 - general cancer quiz

View Set

unit 1 scale drawings study guide

View Set